Sei sulla pagina 1di 309

FIRST DIVISION

[G.R. No. 113539. March 12, 1998]


CELSO R. HALILI and ARTHUR R. HALILI, petitioners, vs. COURT OF APPEALS, HELEN
MEYERS GUZMAN, DAVID REY GUZMAN and EMILIANO CATANIAG, respondents.
D E C I S I O N
PANGANIBAN, J .:
The factual findings of a trial court, when affirmed by the Court of Appeals, may no longer
be reviewed and reversed by this Court in a petition for review under Rule 45 of the Rules of
Court. The transfer of an interest in a piece of land to an alien may no longer be assailed on
constitutional grounds after the entire parcel has been sold to a qualified citizen.
The Case

These familiar and long-settled doctrines are applied by this Court in denying this petition
under Rule 45 to set aside the Decision
[1]
of the Court of Appeals
[2]
in CA-GR CV No. 37829
promulgated on September 14, 1993, the dispositive portion of which states:
[3]

WHEREFORE, and upon all the foregoing, the Decision of the court below dated March 10,
1992 dismissing the complaint for lack of merit is AFFIRMED without pronouncement as to
costs.
The Facts

The factual antecedents, as narrated by Respondent Court, are not disputed by the
parties. We reproduce them in part, as follows:
Simeon de Guzman, an American citizen, died sometime in 1968, leaving real
properties in the Philippines. His forced heirs were his widow, defendant appellee [herein
private respondent] Helen Meyers Guzman, and his son, defendant appellee [also herein
private respondent] David Rey Guzman, both of whom are also American citizens. On
August 9, 1989, Helen executed a deed of quitclaim (Annex A-Complaint), assigning[,]
transferring and conveying to David Rey all her rights, titles and interests in and over six
parcels of land which the two of them inherited from Simeon.
Among the said parcels of land is that now in litigation, x x x situated in Bagbaguin,
Sta. Maria, Bulacan, containing an area of 6,695 square meters, covered by Transfer
Certificate of Title No. T-170514 of the Registry of Deeds of Bulacan. The quitclaim having
been registered, TCT No. T-170514 was cancelled and TCT No. T-120259 was issued in
the name of appellee David Rey Guzman.
On February 5, 1991, David Rey Guzman sold said parcel of land to defendant-
appellee [also herein private respondent] Emiliano Cataniag, upon which TCT No. T-
120259 was cancelled and TCT No. T-130721(M) was issued in the latters name.
[4]

Petitioners, who are owners of the adjoining lot, filed a complaint before the Regional Trial
Court of Malolos, Bulacan, questioning the constitutionality and validity of the two conveyances -
- between Helen Guzman and David Rey Guzman, and between the latter and Emiliano
Cataniag -- and claiming ownership thereto based on their right of legal redemption under Art.
1621
[5]
of the Civil Code.
In its decision
[6]
dated March 10, 1992,
[7]
the trial court dismissed the complaint. It ruled
that Helen Guzmans waiver of her inheritance in favor of her son was not contrary to the
constitutional prohibition against the sale of land to an alien, since the purpose of the waiver
was simply to authorize David Rey Guzman to dispose of their properties in accordance with the
Constitution and the laws of the Philippines, and not to subvert them. On the second issue, it
held that the subject land was urban; hence, petitioners had no reason to invoke their right of
redemption under Art. 1621 of the Civil Code.
The Halilis sought a reversal from the Court of Appeals which, however, denied their
appeal. Respondent Court affirmed the factual finding of the trial court that the subject land was
urban. Citing Tejido vs. Zamacoma
[8]
and Yap vs. Grageda,
[9]
it further held that, although the
transfer of the land to David Rey may have been invalid for being contrary to the Constitution,
there was no more point in allowing herein petitioners to recover the property, since it has
passed on to and was thus already owned by a qualified person.
Hence, this petition.
[10]

Issues

The petition submits the following assignment of errors:
x x x the Honorable Court of Appeals -
1. Erred in affirming the conclusion of the trial court that the land in question is urban, not rural
2. Erred in denying petitioners right of redemption under Art. 1621 of the Civil Code
3. Having considered the conveyance from Helen Meyers Guzman to her son David Rey
Guzman illegal, erred in not declaring the same null and void[.]
[11]

The Courts Ruling

The petition has no merit.
First Issue: The Land Is Urban;

Thus, No Right of Redemption

The first two errors assigned by petitioners being interrelated -- the determination of the first
being a prerequisite to the resolution of the second -- shall be discussed together.
Subject Land Is Urban
Whether the land in dispute is rural or urban is a factual question which, as a rule, is not
reviewable by this Court.
[12]
Basic and long-settled is the doctrine that findings of fact of a trial
judge, when affirmed by the Court of Appeals, are binding upon the Supreme Court. This
admits of only a few exceptions, such as when the findings are grounded entirely on
speculation, surmises or conjectures; when an inference made by the appellate court from its
factual findings is manifestly mistaken, absurd or impossible; when there is grave abuse of
discretion in the appreciation of facts; when the findings of the appellate court go beyond the
issues of the case, run contrary to the admissions of the parties to the case or fail to notice
certain relevant facts which, if properly considered, will justify a different conclusion; when there
is a misappreciation of facts; when the findings of fact are conclusions without mention of the
specific evidence on which they are based, are premised on the absence of evidence or are
contradicted by evidence on record.
[13]

The instant case does not fall within any of the aforecited exceptions. In fact, the
conclusion of the trial court -- that the subject property is urban land -- is based on clear and
convincing evidence, as shown in its decision which disposed thus:
x x x As observed by the court, almost all the roadsides along the national ghighway [sic] of
Bagbaguin, Sta. Maria, Bulacan, are lined up with residential, commercial or industrial
establishments. Lined up along the Bagbaguin Road are factories of feeds, woodcrafts [sic] and
garments, commercial stores for tires, upholstery materials, feeds supply and spare
parts. Located therein likewise were the Pepsi-Cola Warehouse, the Cruz Hospital, three
gasoline stations, apartment buildings for commercial purposes and construction firms. There is
no doubt, therefore, that the community is a commercial area thriving in business
activities. Only a short portion of said road [is] vacant. It is to be noted that in the Tax
Declaration in the name of Helen Meyers Guzman[,] the subject land is termed agricultural[,]
while in the letter addressed to defendant Emiliano Cataniag, dated October 3, 1991, the Land
Regulatory Board attested that the subject property is commercial and the trend of development
along the road is commercial. The Boards classification is based on the present condition of
the property and the community thereat. Said classification is far more later [sic] than the tax
declaration.
[14]

No Ground to Invoke Right of Redemption
In view of the finding that the subject land is urban in character, petitioners have indeed no
right to invoke Art. 1621 of the Civil Code, which presupposes that the land sought to be
redeemed is rural. The provision is clearly worded and admits of no ambiguity in construction:
ART. 1621. The owners of adjoining lands shall also have the right of redemption
when a piece of rural land, the area of which does not exceed one hectare, is alienated,
unless the grantee does not own any rural land.
xxx xxx xxx
Under this article, both lands -- that sought to be redeemed and the adjacent lot belonging
to the person exercising the right of redemption -- must be rural. If one or both are urban, the
right cannot be invoked.
[15]
The purpose of this provision which is limited in scope to rural lands
not exceeding one hectare, is to favor agricultural development.
[16]
The subject land not being
rural and, therefore, not agricultural, this purpose would not be served if petitioners are granted
the right of redemption under Art. 1621. Plainly, under the circumstances, they cannot invoke it.
Second Issue: Sale to Cataniag Valid

Neither do we find any reversible error in the appellate courts holding that the sale of the
subject land to Private Respondent Cataniag renders moot any question on the constitutionality
of the prior transfer made by Helen Guzman to her son David Rey.
True, Helen Guzmans deed of quitclaim -- in which she assigned, transferred and
conveyed to David Rey all her rights, titles and interests over the property she had inherited
from her husband -- collided with the Constitution, Article XII, Section 7 of which provides:
SEC. 7. Save in cases of hereditary succession, no private lands shall be
transferred or conveyed except to individuals, corporations, or associations qualified to
acquire or hold lands of the public domain.
The landmark case of Krivenko vs. Register of Deeds
[17]
settled the issue as to who are
qualified (and disqualified) to own public as well as private lands in the Philippines. Following a
long discourse maintaining that the public agricultural lands mentioned in Section 1, Article XIII
of the 1935 Constitution, include residential, commercial and industrial lands, the Court then
stated:
Under section 1 of Article XIII [now Sec. 2, Art. XII] of the Constitution, natural resources,
with the exception of public agricultural land, shall not be alienated, and with respect to
public agricultural lands, their alienation is limited to Filipino citizens. But this constitutional
purpose conserving agricultural resources in the hands of Filipino citizens may easily be
defeated by the Filipino citizens themselves who may alienate their agricultural lands in
favor of aliens. It is partly to prevent this result that section 5 is included in Article XIII, and
it reads as follows:
Sec. 5. Save in cases of hereditary succession, no private agricultural land will be
transferred or assigned except to individuals, corporations or associations qualified to
acquire or hold lands of the public domain in the Philippines.
This constitutional provision closes the only remaining avenue through which agricultural
resources may leak into aliens hands. It would certainly be futile to prohibit the alienation
of public agricultural lands to aliens if, after all, they may be freely so alienated upon their
becoming private agricultural lands in the hands of Filipino citizens. Undoubtedly, as above
indicated, section 5 [now Sec. 7] is intended to insure the policy of nationalization contained
in section 1 [now Sec. 2]. Both sections must, therefore, be read together for they have the
same purpose and the same subject matter. It must be noticed that the persons against
whom the prohibition is directed in section 5 [now Sec. 7] are the very same persons who
under section 1 [now Sec. 2] are disqualified to acquire or hold lands of the public domain
in the Philippines. And the subject matter of both sections is the same, namely, the non
transferability of agricultural land to aliens. x x x
[18]

The Krivenko rule was recently reiterated in Ong Ching Po vs. Court of
Appeals,
[19]
which involves a sale of land to a Chinese citizen. The Court said:
The capacity to acquire private land is made dependent upon the capacity to acquire
or hold lands of the public domain. Private land may be transferred or conveyed only to
individuals or entities qualified to acquire lands of the public domain (II Bernas, The
Constitution of the Philippines 439-440 [1988 ed.]).
The 1935 Constitution reserved the right to participate in the disposition, exploitation,
development and utilization of all lands of the public domain and other natural resources of
the Philippines for Filipino citizens or corporations at least sixty percent of the capital of
which was owned by Filipinos. Aliens, whether individuals or corporations, have been
disqualified from acquiring public lands; hence, they have also been disqualified from
acquiring private lands.
[20]

In fine, non-Filipinos cannot acquire or hold title to private lands or to lands of the public
domain, except only by way of legal succession.
[21]

But what is the effect of a subsequent sale by the disqualified alien vendee to a qualified
Filipino citizen? This is not a novel question. Jurisprudence is consistent that if land is invalidly
transferred to an alien who subsequently becomes a citizen or transfers it to a citizen, the flaw in
the original transaction is considered cured and the title of the transferee is rendered valid.
[22]

Thus, in United Church Board of World Ministries vs. Sebastian,
[23]
in which an alien
resident who owned properties in the Philippines devised to an American non-stock corporation
part of his shares of stock in a Filipino corporation that owned a tract of land in Davao del Norte,
the Court sustained the invalidity of such legacy. However, upon proof that ownership of the
American corporation has passed on to a 100 percent Filipino corporation, the Court ruled that
the defect in the will was rectified by the subsequent transfer of the property.
The present case is similar to De Castro vs. Tan.
[24]
In that case, a residential lot was sold
to a Chinese. Upon his death, his widow and children executed an extrajudicial settlement,
whereby said lot was allotted to one of his sons who became a naturalized Filipino. The Court
did not allow the original vendor to have the sale annulled and to recover the property, for the
reason that the land has since become the property of a naturalized Filipino citizen who is
constitutionally qualified to own land.
Likewise, in the cases of Sarsosa vs. Cuenco,
[25]
Godinez vs. Pak Luen,
[26]
Vasquez vs. Li
Seng Giap
[27]
and Herrera vs. Luy Kim Guan,
[28]
which similarly involved the sale of land to an
alien who thereafter sold the same to a Filipino citizen, the Court again applied the rule that the
subsequent sale can no longer be impugned on the basis of the invalidity of the initial transfer.
The rationale of this principle was explained in Vasquez vs. Li Seng Giap thus:
x x x [I]f the ban on aliens from acquiring not only agricultural but also urban lands, as
construed by this Court in the Krivenko case, is to preserve the nations lands for future
generations of Filipinos, that aim or purpose would not be thwarted but achieved by making
lawful the acquisition of real estate by aliens who became Filipino citizens by naturalization.
[29]

Accordingly, since the disputed land is now owned by Private Respondent Cataniag, a
Filipino citizen, the prior invalid transfer can no longer be assailed. The objective of the
constitutional provision -- to keep our land in Filipino hands -- has been served.
WHEREFORE, the petition is hereby DENIED. The challenged Decision is
AFFIRMED. Costs against petitioner.
SO ORDERED.
Davide, Jr. (Chairman), Bellosillo, Vitug and Quisumbing, JJ., concur.



[5]
ART. 1621. The owners of adjoining lands shall also have the right of redemption when a
piece of rural land, the area of which does not exceed one hectare, is alienated, unless the
grantee does not own any rural land.
This right is not applicable to adjacent lands which are separated by brooks, drains, ravines,
roads and other apparent servitudes for the benefit of other estates.
If two or more adjoining owners desire to exercise the right of redemption at the same time, the
owner of the adjoining land of smaller area shall be preferred; and should both lands have the
same area, the one who first requested the redemption.

Republic of the Philippines
SUPREME COURT
Manila
FIRST DIVISION
G.R. No. 149615 August 29, 2006
IN RE: PETITION FOR SEPARATION OF PROPERTY ELENA BUENAVENTURA
MULLER, Petitioner,
vs.
HELMUT MULLER, Respondent.
D E C I S I O N
YNARES-SANTIAGO, J .:
This petition for review on certiorari
1
assails the February 26, 2001 Decision
2
of the Court of
Appeals in CA-G.R. CV No. 59321 affirming with modification the August 12, 1996 Decision
3
of
the Regional Trial Court of Quezon City, Branch 86 in Civil Case No. Q-94-21862, which
terminated the regime of absolute community of property between petitioner and respondent, as
well as the Resolution
4
dated August 13, 2001 denying the motion for reconsideration.
The facts are as follows:
Petitioner Elena Buenaventura Muller and respondent Helmut Muller were married in Hamburg,
Germany on September 22, 1989. The couple resided in Germany at a house owned by
respondents parents but decided to move and reside permanently in the Philippines in 1992. By
this time, respondent had inherited the house in Germany from his parents which he sold and
used the proceeds for the purchase of a parcel of land in Antipolo, Rizal at the cost of
P528,000.00 and the construction of a house amounting to P2,300,000.00. The Antipolo
property was registered in the name of petitioner under Transfer Certificate of Title No.
219438
5
of the Register of Deeds of Marikina, Metro Manila.
Due to incompatibilities and respondents alleged womanizing, drinking, and maltreatment, the
spouses eventually separated. On September 26, 1994, respondent filed a petition
6
for
separation of properties before the Regional Trial Court of Quezon City.
On August 12, 1996, the trial court rendered a decision which terminated the regime of absolute
community of property between the petitioner and respondent. It also decreed the separation of
properties between them and ordered the equal partition of personal properties located within
the country, excluding those acquired by gratuitous title during the marriage. With regard to the
Antipolo property, the court held that it was acquired using paraphernal funds of the respondent.
However, it ruled that respondent cannot recover his funds because the property was
purchased in violation of Section 7, Article XII of the Constitution. Thus
However, pursuant to Article 92 of the Family Code, properties acquired by gratuitous title by
either spouse during the marriage shall be excluded from the community property. The real
property, therefore, inherited by petitioner in Germany is excluded from the absolute community
of property of the herein spouses. Necessarily, the proceeds of the sale of said real property as
well as the personal properties purchased thereby, belong exclusively to the petitioner.
However, the part of that inheritance used by the petitioner for acquiring the house and lot in
this country cannot be recovered by the petitioner, its acquisition being a violation of Section 7,
Article XII of the Constitution which provides that "save in cases of hereditary succession, no
private lands shall be transferred or conveyed except to individuals, corporations or associations
qualified to acquire or hold lands of the public domain." The law will leave the parties in the
situation where they are in without prejudice to a voluntary partition by the parties of the said
real property. x x x
x x x x
As regards the property covered by Transfer Certificate of Title No. 219438 of the Registry of
Deeds of Marikina, Metro Manila, situated in Antipolo, Rizal and the improvements thereon, the
Court shall not make any pronouncement on constitutional grounds.
7

Respondent appealed to the Court of Appeals which rendered the assailed decision modifying
the trial courts Decision. It held that respondent merely prayed for reimbursement for the
purchase of the Antipolo property, and not acquisition or transfer of ownership to him. It also
considered petitioners ownership over the property in trust for the respondent. As regards the
house, the Court of Appeals ruled that there is nothing in the Constitution which prohibits
respondent from acquiring the same. The dispositive portion of the assailed decision reads:
WHEREFORE, in view of the foregoing, the Decision of the lower court dated August 12, 1996
is hereby MODIFIED. Respondent Elena Buenaventura Muller is hereby ordered to
REIMBURSE the petitioner the amount of P528,000.00 for the acquisition of the land and the
amount of P2,300,000.00 for the construction of the house situated in Atnipolo, Rizal, deducting
therefrom the amount respondent spent for the preservation, maintenance and development of
the aforesaid real property including the depreciation cost of the house or in the alternative to
SELL the house and lot in the event respondent does not have the means to reimburse the
petitioner out of her own money and from the proceeds thereof, reimburse the petitioner of the
cost of the land and the house deducting the expenses for its maintenance and preservation
spent by the respondent. Should there be profit, the same shall be divided in proportion to the
equity each has over the property. The case is REMANDED to the lower court for reception of
evidence as to the amount claimed by the respondents for the preservation and maintenance of
the property.
SO ORDERED.
8

Hence, the instant petition for review raising the following issues:
I
THE HONORABLE COURT OF APPEALS GRAVELY ERRED IN HOLDING THAT THE
RESPONDENT HEREIN IS ENTITLED TO REIMBURSEMENT OF THE AMOUNT USED TO
PURCHASE THE LAND AS WELL AS THE COSTS FOR THE CONSTRUCTION OF THE
HOUSE, FOR IN SO RULING, IT INDIRECTLY ALLOWED AN ACT DONE WHICH
OTHERWISE COULD NOT BE DIRECTLY x x x DONE, WITHOUT DOING VIOLENCE TO
THE CONSTITUTIONAL PROSCRIPTION THAT AN ALIEN IS PROHIBITED FROM
ACQUIRING OWNERSHIP OF REAL PROPERTIES LOCATED IN THE PHILIPPINES.
II
THE COURT OF APPEALS GRAVELY ERRED IN SUSTAINING RESPONDENTS CAUSE OF
ACTION WHICH IS ACTUALLY A DESPERATE ATTEMPT TO OBTAIN OWNERSHIP OVER
THE LOT IN QUESTION, CLOTHED UNDER THE GUISE OF CLAIMING REIMBURSEMENT.
Petitioner contends that respondent, being an alien, is disqualified to own private lands in the
Philippines; that respondent was aware of the constitutional prohibition but circumvented the
same; and that respondents purpose for filing an action for separation of property is to obtain
exclusive possession, control and disposition of the Antipolo property.
Respondent claims that he is not praying for transfer of ownership of the Antipolo property but
merely reimbursement; that the funds paid by him for the said property were in consideration of
his marriage to petitioner; that the funds were given to petitioner in trust; and that equity
demands that respondent should be reimbursed of his personal funds.
The issue for resolution is whether respondent is entitled to reimbursement of the funds used for
the acquisition of the Antipolo property.
The petition has merit.
Section 7, Article XII of the 1987 Constitution states:
Save in cases of hereditary succession, no private lands shall be transferred or conveyed
except to individuals, corporations, or associations qualified to acquire or hold lands of the
public domain.
Aliens, whether individuals or corporations, are disqualified from acquiring lands of the public
domain. Hence, they are also disqualified from acquiring private lands.
9
The primary purpose of
the constitutional provision is the conservation of the national patrimony. In the case of Krivenko
v. Register of Deeds,
10
the Court held:
Under section 1 of Article XIII of the Constitution, "natural resources, with the exception of public
agricultural land, shall not be alienated," and with respect to public agricultural lands, their
alienation is limited to Filipino citizens. But this constitutional purpose conserving agricultural
resources in the hands of Filipino citizens may easily be defeated by the Filipino citizens
themselves who may alienate their agricultural lands in favor of aliens. It is partly to prevent this
result that section 5 is included in Article XIII, and it reads as follows:
"Sec. 5. Save in cases of hereditary succession, no private agricultural land will be transferred
or assigned except to individuals, corporations, or associations qualified to acquire or hold lands
of the public domain in the Philippines."
This constitutional provision closes the only remaining avenue through which agricultural
resources may leak into aliens hands. It would certainly be futile to prohibit the alienation of
public agricultural lands to aliens if, after all, they may be freely so alienated upon their
becoming private agricultural lands in the hands of Filipino citizens. x x x
x x x x
If the term "private agricultural lands" is to be construed as not including residential lots or lands
not strictly agricultural, the result would be that "aliens may freely acquire and possess not only
residential lots and houses for themselves but entire subdivisions, and whole towns and cities,"
and that "they may validly buy and hold in their names lands of any area for building homes,
factories, industrial plants, fisheries, hatcheries, schools, health and vacation resorts, markets,
golf courses, playgrounds, airfields, and a host of other uses and purposes that are not, in
appellants words, strictly agricultural." (Solicitor Generals Brief, p. 6.) That this is obnoxious to
the conservative spirit of the Constitution is beyond question.
Respondent was aware of the constitutional prohibition and expressly admitted his knowledge
thereof to this Court.
11
He declared that he had the Antipolo property titled in the name of
petitioner because of the said prohibition.
12
His attempt at subsequently asserting or claiming a
right on the said property cannot be sustained.
The Court of Appeals erred in holding that an implied trust was created and resulted by
operation of law in view of petitioners marriage to respondent. Save for the exception provided
in cases of hereditary succession, respondents disqualification from owning lands in the
Philippines is absolute. Not even an ownership in trust is allowed. Besides, where the purchase
is made in violation of an existing statute and in evasion of its express provision, no trust can
result in favor of the party who is guilty of the fraud.
13
To hold otherwise would allow
circumvention of the constitutional prohibition.
Invoking the principle that a court is not only a court of law but also a court of equity, is likewise
misplaced. It has been held that equity as a rule will follow the law and will not permit that to be
done indirectly which, because of public policy, cannot be done directly.
14
He who seeks equity
must do equity, and he who comes into equity must come with clean hands. The latter is a
frequently stated maxim which is also expressed in the principle that he who has done inequity
shall not have equity. It signifies that a litigant may be denied relief by a court of equity on the
ground that his conduct has been inequitable, unfair and dishonest, or fraudulent, or deceitful as
to the controversy in issue.
15

Thus, in the instant case, respondent cannot seek reimbursement on the ground of equity where
it is clear that he willingly and knowingly bought the property despite the constitutional
prohibition.
Further, the distinction made between transfer of ownership as opposed to recovery of funds is
a futile exercise on respondents part. To allow reimbursement would in effect permit
respondent to enjoy the fruits of a property which he is not allowed to own. Thus, it is likewise
proscribed by law. As expressly held in Cheesman v. Intermediate Appellate Court:
16

Finally, the fundamental law prohibits the sale to aliens of residential land. Section 14, Article
XIV of the 1973 Constitution ordains that, "Save in cases of hereditary succession, no private
land shall be transferred or conveyed except to individuals, corporations, or associations
qualified to acquire or hold lands of the public domain." Petitioner Thomas Cheesman was, of
course, charged with knowledge of this prohibition. Thus, assuming that it was his intention that
the lot in question be purchased by him and his wife, he acquired no right whatever over the
property by virtue of that purchase; and in attempting to acquire a right or interest in land,
vicariously and clandestinely, he knowingly violated the Constitution; the sale as to him was null
and void. In any event, he had and has no capacity or personality to question the subsequent
sale of the same property by his wife on the theory that in so doing he is merely exercising the
prerogative of a husband in respect of conjugal property. To sustain such a theory would permit
indirect controversion of the constitutional prohibition. If the property were to be declared
conjugal, this would accord to the alien husband a not insubstantial interest and right over land,
as he would then have a decisive vote as to its transfer or disposition. This is a right that the
Constitution does not permit him to have.
As already observed, the finding that his wife had used her own money to purchase the property
cannot, and will not, at this stage of the proceedings be reviewed and overturned. But even if it
were a fact that said wife had used conjugal funds to make the acquisition, the considerations
just set out to militate, on high constitutional grounds, against his recovering and holding the
property so acquired, or any part thereof. And whether in such an event, he may recover from
his wife any share of the money used for the purchase or charge her with unauthorized
disposition or expenditure of conjugal funds is not now inquired into; that would be, in the
premises, a purely academic exercise. (Emphasis added)
WHEREFORE, in view of the foregoing, the instant petition is GRANTED. The Decision dated
February 26, 2001 of the Court of Appeals in CA-G.R. CV No. 59321 ordering petitioner Elena
Buenaventura Muller to reimburse respondent Helmut Muller the amount of P528,000 for the
acquisition of the land and the amount of P2,300,000 for the construction of the house in
Antipolo City, and the Resolution dated August 13, 2001 denying reconsideration thereof, are
REVERSED and SET ASIDE. The August 12, 1996 Decision of the Regional Trial Court of
Quezon City, Branch 86 in Civil Case No. Q-94-21862 terminating the regime of absolute
community between the petitioner and respondent, decreeing a separation of property between
them and ordering the partition of the personal properties located in the Philippines equally, is
REINSTATED.
SO ORDERED.
CONSUELO YNARES-SANTIAGO
Associate Justice
WE CONCUR:
ARTEMIO V. PANGANIBAN
Chief Justice
Chairperson
MA. ALICIA AUSTRIA-MARTINEZ, ROMEO J. CALLEJO, SR.
Associate Justice Associate Justice
MINITA V. CHICO-NAZARIO
Associate Justice
C E R T I F I C A T I O N
Pursuant to Section 13, Article VIII of the Constitution, it is hereby certified that the conclusions
in the above Decision were reached in consultation before the case was assigned to the writer
of the opinion of the Courts Division.
ARTEMIO V. PANGANIBAN
Chief Justice
Republic of the Philippines
SUPREME COURT
Manila
EN BANC

G.R. No. 108998 August 24, 1994
REPUBLIC OF THE PHILIPPINES, petitioner,
vs.
THE COURT OF APPEALS AND SPOUSES MARIO B. LAPIA AND FLOR DE
VEGA, respondents.
Byron V. Belarmino and Juan B. Belarmino for private respondents.

BIDIN, J .:
Can a foreign national apply for registration of title over a parcel of land which he acquired by
purchase while still a citizen of the Philippines, from a vendor who has complied with the
requirements for registration under the Public Land Act (CA 141)?
The Republic would have us rule on the negative and asks this Court to nullify the decision of
the appellate court which affirmed the judgment of the court a quo in granting the application of
respondent spouses for registration over the lots in question.
On June 17, 1978, respondent spouses bought Lots 347 and 348, Cad. s38-D, as their
residence with a total area of 91.77 sq. m. situated in San Pablo City, from one Cristeta Dazo
Belen (Rollo, p. 41). At the time of the purchase, respondent spouses where then natural-born
Filipino citizens.
On February 5, 1987, the spouses filed an application for registration of title of the two (2)
parcels of land before the Regional Trial Court of San Pablo City, Branch XXXI. This time,
however, they were no longer Filipino citizens and have opted to embrace Canadian citizenship
through naturalization.
An opposition was filed by the Republic and after the parties have presented their respective
evidence, the court a quorendered a decision confirming private respondents' title to the lots in
question, the dispositive portion of which reads as follows:
WHEREFORE, in view of the foregoing, this Court hereby approves the said
application and confirms the title and possession of herein applicants over Lots
347 and 348, Ap-04-003755 in the names of spouses Mario B. Lapia and Flor
de Vega, all of legal age, Filipino citizens by birth but now Canadian citizens by
naturalization and residing at 14 A. Mabini Street, San Pablo City and/or 201-
1170-124 Street, Edmonton, Alberta T5M-OK9, Canada.
Once this Decision becomes final, let the corresponding decree of registration be
issued. In the certificate of title to be issued, there shall be annotated an
easement of .265 meters road right-of-way.
SO ORDERED. (Rollo, p. 25)
On appeal, respondent court affirmed the decision of the trial court based on the following
ratiocination:
In the present case, it is undisputed that both applicants were still Filipino citizens
when they bought the land in controversy from its former owner. For this reason,
the prohibition against the acquisition of private lands by aliens could not apply.
In justice and equity, they are the rightful owners of the subject realty considering
also that they had paid for it quite a large sum of money. Their purpose in
initiating the instant action is merely to confirm their title over the land, for, as has
been passed upon, they had been the owners of the same since 1978. It ought to
be pointed out that registration is not a mode of acquiring ownership. The
Torrens System was not established as a means for the acquisition of title to
private land. It is intended merely to confirm and register the title which one may
already have (Municipality of Victorias vs. Court of Appeals, G.R. No. L-31189,
March 31, 1987). With particular reference to the main issue at bar, the High
Court has ruled that title and ownership over lands within the meaning and for the
purposes of the constitutional prohibition dates back to the time of their purchase,
not later. The fact that the applicants-appellees are not Filipino citizens now
cannot be taken against them for they were not disqualified from acquiring the
land in question (Bollozos vs. Yu Tieng Su, G.R. No. L-29442, November 11,
1987). (Rollo, pp. 27-28)
Expectedly, respondent court's disposition did not merit petitioner's approval, hence this present
recourse, which was belatedly filed.
Ordinarily, this petition would have been denied outright for having been filed out of time had it
not been for the constitutional issue presented therein.
At the outset, petitioner submits that private respondents have not acquired proprietary rights
over the subject properties before they acquired Canadian citizenship through naturalization to
justify the registration thereof in their favor. It maintains that even privately owned unregistered
lands are presumed to be public lands under the principle that lands of whatever classification
belong to the State under the Regalian doctrine. Thus, before the issuance of the certificate of
title, the occupant is not in the jurisdical sense the true owner of the land since it still pertains to
the State. Petitioner further argued that it is only when the court adjudicates the land to the
applicant for confirmation of title would the land become privately owned land, for in the same
proceeding, the court may declare it public land, depending on the evidence.
As found by the trial court:
The evidence thus presented established that applicants, by themselves and
their predecessors-in-interest, had been in open, public, peaceful, continuous,
exclusive and notorious possession and occupation of the two adjacent parcels
of land applied for registration of title under a bona-fide claim of ownership long
before June 12, 1945. Such being the case, it is conclusively presumed that all
the conditions essential to the confirmation of their title over the two adjacent
parcels of land are sought to be registered have been complied with thereby
entitling them to the issuance of the corresponding certificate of title pursuant to
the provisions of Presidential Decree No. 1529, otherwise known as the Property
Registration Decree. (Rollo, p. 26)
Respondent court echoed the court a quo's observation, thus:
The land sought to be registered has been declared to be within the alienable
and disposable zone established by the Bureau of Forest Development (Exhibit
"P"). The investigation conducted by the Bureau of Lands, Natural Resources
District (IV-2) reveals that the disputed realty had been occupied by the
applicants "whose house of strong materials stands thereon"; that it had been
declared for taxation purposes in the name of applicants-spouses since 1979;
that they acquired the same by means of a public instrument entitled "Kasulatan
ng Bilihang Tuluyan" duly executed by the vendor, Cristeta Dazo Belen, on June
17, 1978 (Exhibits "I" and "J"); and that applicants and their predecessors in
interest had been in possession of the land for more than 30 years prior to the
filing of the application for registration. But what is of great significance in the
instant case is the circumstance that at the time the applicants purchased the
subject lot in 1978, both of them were Filipino citizens such that when they filed
their application for registration in 1987, ownership over the land in dispute had
already passed to them. (Rollo, p., 27)
The Republic disagrees with the appellate court's concept of possession and argues:
17. The Court of Appeals found that the land was declared for taxation purposes
in the name of respondent spouses only since 1979. However, tax declarations
or reality tax payments of property are not conclusive evidence of ownership.
(citing cases)
18. Then again, the appellate court found that "applicants (respondents) and their
predecessors-in-interest had been in possession of the land for more than 30
years prior to the filing of the application for registration." This is not, however,
the same as saying that respondents have been in possession "since June 12,
1945." (PD No. 1073, amending Sec. 48 [b], CA NO. 141; sec. also Sec. 14, PD
No. 1529). So there is a void in respondents' possession. They fall short of the
required possession since June 12, 1945 or prior thereto. And, even if they
needed only to prove thirty (30) years possession prior to the filing of their
application (on February 5, 1987), they would still be short of the required
possession if the starting point is 1979 when, according to the Court of Appeals,
the land was declared for taxation purposes in their name. (Rollo, pp. 14-15)
The argument is myopic, to say the least. Following the logic of petitioner, any transferee is thus
foreclosed to apply for registration of title over a parcel of land notwithstanding the fact that the
transferor, or his predecessor-in-interest has been in open, notorious and exclusive possession
thereof for thirty (30) years or more. This is not, however, what the law provides.
As petitioner itself argues, Section 48 of the Public Land Act (CA 141) reads:
Sec. 48. The following-described citizens of the Philippines, occupying lands of
the public domain or claiming interest therein, but whose titles have not been
perfected or completed, may apply to the Court of First Instance (now Regional
Trial Court) of the province where the land is located for confirmation of their
claims and the issuance of a certificate of title therefor under the Land
Registration Act, to wit:
xxx xxx xxx
(b) Those who by themselves or through their predecessors-in-interest have
been in open, continuous, exclusive, and notorious possession and occupation of
agricultural lands of the public domain, under a bona fide claim of acquisition or
ownership, for at least thirty years immediately preceding the filing of the
application for confirmation of title except when prevented by wars or force
majeure. These shall be conclusively presumed to have performed all the
conditions essential to a Government grant and shall be entitled to a certificate of
title under the provisions of this chapter. (Emphasis supplied)
As amended by PD 1073:
Sec. 4. The provisions of Section 48(b) and Section 48(c), Chapter VIII, of the
Public Land Act are hereby amended in the sense that these provisions shall
apply only to alienable and disposable lands of the public domain which have
been in open, continuous, exclusive and notorious possession and occupation by
the applicant himself or thru his predecessor-in-interest, under a bona fide claim
of acquisition or ownership, since June 12, 1945.
It must be noted that with respect to possession and occupation of the alienable and disposable
lands of the public domain, the law employs the terms "by themselves", "the applicant himself or
through his predecessor-in-interest". Thus, it matters not whether the vendee/applicant has
been in possession of the subject property for only a day so long as the period and/or legal
requirements for confirmation of title has been complied with by his predecessor-in-interest, the
said period is tacked to his possession. In the case at bar, respondents' predecessors-in-
interest have been in open, continuous, exclusive and notorious possession of the disputed land
not only since June 12, 1945, but even as early as 1937. Petitioner does not deny this except
that respondent spouses, in its perception, were in possession of the land sought to be
registered only in 1978 and therefore short of the required length of time. As aforesaid, the
disputed parcels of land were acquired by private respondents through their predecessors-in-
interest, who, in turn, have been in open and continued possession thereof since 1937. Private
respondents stepped into the shoes of their predecessors-in-interest and by virtue thereof,
acquired all the legal rights necessary to confirm what could otherwise be deemed as an
imperfect title.
At this juncture, petitioner's reliance in Republic v. Villanueva (114 SCRA 875 [1982]) deserves
scant consideration. There, it was held that before the issuance of the certificate of title, the
occupant is not in the juridical sense the true owner of the land since it still pertains to the State.
Suffice it to state that the ruling in Republic v. Villanueva (supra), has already been abandoned
in the 1986 case of Director of Lands v. Intermediate Appellate Court (146 SCRA 509; and
reiterated in Director of Lands v. Iglesia ni Cristo, 200 SCRA 606 [1991]) where the Court,
through then Associate Justice, now Chief Justice Narvasa, declared that:
(The weight of authority is) that open, exclusive and undisputed possession of
alienable public land for the period prescribed by law creates the legal fiction
whereby the land, upon completion of the requisite periodipso jure and without
the need of judicial or other sanction, ceases to be public land and becomes
private property. . . .
Herico in particular, appears to be squarely affirmative:
. . . Secondly, under the provisions of Republic Act
No. 1942, which the respondent Court held to be inapplicable to
the petitioner's case, with the latter's proven occupation and
cultivation for more than 30 years since 1914, by himself and by
his predecessors-in-interest, title over the land has vested on
petitioner so as to segregate the land from the mass of public
land. Thereafter, it is no longer disposable under the Public Land
Act as by free patent . . .
xxx xxx xxx
As interpreted in several cases, when the conditions as specified
in the foregoing provision are complied with, the possessor is
deemed to have acquired, by operation of law, a right to a grant, a
government grant, without the necessity of a certificate of title
being issued. The land, therefore, ceases to be of the public
domain and beyond the authority of the Director of Lands to
dispose of. The application for confirmation is mere formality, the
lack of which does not affect the legal sufficiency of the title as
would be evidenced by the patent and the Torrens title to be
issued upon the strength of said patent.
Nothing can more clearly demonstrate the logical inevitability of considering
possession of public land which is of the character and duration prescribed by
the statute as the equivalent of an express grant from the State than the dictum
of the statute itself (Section 48 [b]) that the possessor(s) ". . . shall be
conclusively presumed to have performed all the conditions essential to a
Government grant and shall be entitled to a certificate of title ..." No proof being
admissible to overcome a conclusive presumption, confirmation proceedings
would, in truth be little more than a formality, at the most limited to ascertaining
whether the possession claims is of the required character and length of time;
and registration thereunder would not confer title, but simply recognize a title
already vested. The proceedings would not originally convert the land from public
to private land, but only confirm such a conversion already affected by operation
of law from the moment the required period of possession became complete. As
was so well put in Cario, ". . .(There are indications that registration was
expected from all, but none sufficient to show that, for want of it, ownership
actually gained would be lost. The effect of the proof, wherever made, was not to
confer title, but simply to establish it, as already conferred by the decree, if not by
earlier law. (Emphasis supplied)
Subsequent cases have hewed to the above pronouncement such that open, continuous and
exclusive possession for at least 30 years of alienable public land ipso jure converts the same to
private property (Director of Lands v. IAC, 214 SCRA 604 [1992]; Pineda v. CA, 183 SCRA 602
[1990]). This means that occupation and cultivation for more than 30 years by an applicant and
his predecessors-in-interest, vest title on such applicant so as to segregate the land from the
mass of public and (National Power Corporation v. CA, 218 SCRA 41 [1993]).
The Public Land Act requires that the applicant must prove that (a) the land is alienable public
land and (b) his possession, in the concept above stated, must be either since time immemorial
or for the period prescribed in the Public Land Act (Director of Lands v. Buyco, 216 SCRA 78
[1992]). When the conditions set by law are complied with, the possessor of the land, by
operation of law, acquires a right to a grant, a government grant, without the necessity of a
certificate of title being issued (National Power Corporation v. CA, supra). As such, the land
ceases to be a part of the public domain and goes beyond the authority of the Director of Lands
to dispose of.
In other words, the Torrens system was not established as a means for the acquisition of title to
private land (Municipality of Victorias v. CA, 149 SCRA 32 [1987]). It merely confirms, but does
not confer ownership. As could be gleaned from the evidence adduced, private respondents
were able to establish the nature of possession of their predecessors-in-interest. Evidence was
offered to prove that their predecessors-in-interest had paid taxes on the subject land and
introduced improvements thereon (Exhibits "F" to "F9"). A certified true copy of the affidavit
executed by Cristeta Dazo and her sister Simplicia was also formally offered to prove that the
subject parcels of land were inherited by vendor Cristeta Dazo from her father Pedro Dazo with
the conformity of her only sister Simplicia (Exhibit "G"). Likewise, a report from the Bureau of
Lands was presented in evidence together with a letter from the Bureau of Forest Development,
to prove that the questioned lots were part of the alienable and disposable zone of the
government and that no forestry interest was affected (CA GR No. 28953, Records, p. 33).
In the main, petitioner seeks to defeat respondents' application for registration of title on the
ground of foreign nationality. Accordingly, the ruling in Director of Lands v. Buyco (supra)
supports petitioner's thesis.
We disagree.
In Buyco, the applicants therein were likewise foreign nationals but were natural-born Filipino
citizens at the time of their supposed acquisition of the property. But this is where the similarity
ends. The applicants in Buyco sought to register a large tract of land under the provisions of the
Land Registration Act, and in the alternative, under the provisions of the Public Land Act. The
land registration court decided in favor of the applicants and was affirmed by the appellate court
on appeal. The Director of Lands brought the matter before us on review and we reversed.
This Court, speaking through Justice Davide, Jr., stated:
As could be gleaned from the evidence adduced, the private respondents do not
rely on fee simple ownership based on a Spanish grant or possessory
information title under Section 19 of the Land Registration Act; the private
respondents did not present any proof that they or their predecessors-in-interest
derived title from an old Spanish grant such as (a) the "titulo real" or royal grant
(b) the "concession especial" or especial grant; (c) the "composicion con el
estado" title or adjustment title; (d) the "titulo de compra" or title by purchase; and
(e) the "informacion posesoria" or possessory information title, which could
become a "titulo gratuito" or a gratuitous title (Director of Forestry v. Muoz, 23
SCRA 1183 [1968]). The primary basis of their claim is possession, by
themselves and their predecessors-in-interest, since time immemorial.
If indeed private respondents and their predecessors have been in possession
since time immemorial, the rulings of both courts could be upheld for, as this
Court stated in Oh Cho v. Director of Lands (75 Phil. 890 [1946]):
. . . All lands that were not acquired from the Government, either
by purchase or by grant, belong to the public domain. An
exception to the rule would be any land that should have been in
the possession of an occupant and of his predecessors in interest
since time immemorial, for such possession would justify the
presumption that the land had never been part of the public
domain or that if had been a private property even before the
Spanish conquest (Cario v. Insular Government, 41 Phil 935
[1909]; 212 U.S. 449; 53 Law. Ed., 594) The applicant does not
come under the exception, for the earliest possession of the lot by
his first predecessor in interest began in 1880.
. . . alienable public land held by a possessor, personally or
through his predecessors-in-interest, openly, continuously and
exclusively for the prescribed statutory period (30 years under the
Public Land Act, as amended) is converted to private property by
the mere lapse or completion of said period, ipso jure. (Director of
Lands v. Intermediate Appellate Court, supra)
It is obvious from the foregoing rule that the applicant must prove that (a) the
land is alienable public land and (b) his possession, in the concept above stated,
must be either since time immemorial, as ruled in both Cario and Susi, or for the
period prescribed in the Public Land Act. As to the latter, this Court, in Gutierrez
Hermanos v. Court of Appeals (178 SCRA 37 [1989]), adopted the rule
enunciated by the Court of Appeals, per then Associate Justice Hugo R.
Gutierrez, Jr., . . ., that an applicant for registration under Section 48 of the Public
Land Act must secure a certification from the Government that the lands which
he claims to have possessed as owner for more than thirty (30) years are
alienable and disposable. It is the burden of the applicant to prove its positive
averments.
In the instant case, private respondents offered no evidence at all to prove that
the property subject of the application is an alienable and disposable land. On
the contrary, the entire property . . . was pasture land (and therefore inalienable
under the then 1973 Constitution).
. . . (P)rivate respondents' evidence miserably failed to establish their imperfect
title to the property in question. Their allegation of possession since time
immemorial, . . ., is patently baseless. . . . When referring to possession,
specifically "immemorial possession," it means possession of which no man
living has seen the beginning, and the existence of which he has learned from his
elders (Susi v. Razon, supra). Such possession was never present in the case of
private respondents. . . .
. . ., there does not even exist a reasonable basis for the finding that the private
respondents and their predecessors-in-interest possessed the land for more than
eighty (80) years, . . .
xxx xxx xxx
To this Court's mind, private respondents failed to prove that (their predecessor-
in-interest) had possessed the property allegedly covered by Tax Declaration No.
15853 and made the subject of both his last will and testament and the project of
partition of his estate among his heirs in such manner as to remove the same
from the public domain under the Cario and Susi doctrines. Thus, (when the
predecessor-in-interest) died on 31 May 1937, he transmitted no right
whatsoever, with respect to the said property, to his heirs. This being the case,
his possession cannot be tacked to that of the private respondents for the latter's
benefit pursuant to Section 48(b) of the Public Land Act, the alternative ground
relied upon in their application . . .
xxx xxx xxx
Considering that the private respondents became American citizens before such
filing, it goes without saying that they had acquired no vested right, consisting of
an imperfect title, over the property before they lost their Philippine citizenship.
(Emphasis supplied)
Clearly, the application in Buyco were denied registration of title not merely because they were
American citizens at the time of their application therefor. Respondents therein failed to prove
possession of their predecessor-in-interest since time immemorial or possession in such a
manner that the property has been segregated from public domain; such that at the time of their
application, as American citizens, they have acquired no vested rights over the parcel of land.
In the case at bar, private respondents were undoubtedly natural-born Filipino citizens at the
time of the acquisition of the properties and by virtue thereof, acquired vested rights thereon,
tacking in the process, the possession in the concept of owner and the prescribed period of time
held by their predecessors-in-interest under the Public Land Act. In addition, private
respondents have constructed a house of strong materials on the contested property, now
occupied by respondent Lapias mother.
But what should not be missed in the disposition of this case is the fact that the Constitution
itself allows private respondents to register the contested parcels of land in their favor. Sections
7 and 8 of Article XII of the Constitution contain the following pertinent provisions, to wit:
Sec. 7. Save in cases of hereditary succession, no private lands shall be
transferred or conveyed except to individuals, corporations, or associations
qualified to acquire or hold lands of the public domain.
Sec. 8. Notwithstanding the provisions of Section 7 of this Article, a natural-born
citizen of the Philippines who has lost his Philippine citizenship may be a
transferee of private lands, subject to limitations provided by law. (Emphasis
supplied)
Section 8, Article XII of the 1987 Constitution above quoted is similar to Section 15, Article XIV
of the then 1973 Constitution which reads:
Sec. 15. Notwithstanding the provisions of Section 14 of this Article, a natural-
born citizen of the Philippines who has lost his citizenship may be a transferee of
private land, for use by him as his residence, as the Batasang Pambansa may
provide.
Pursuant thereto, Batas Pambansa Blg. 185 was passed into law, the relevant provision of
which provides:
Sec. 2. Any natural-born citizen of the Philippines who has lost his Philippine
citizenship and who has the legal capacity to enter into a contract under
Philippine laws may be a transferee of a private land up to a maximum area of
one thousand square meters, in the case of urban land, or one hectare in the
case of rural land, to be used by him as his residence. In the case of married
couples, one of them may avail of the privilege herein granted; Provided, That if
both shall avail of the same, the total area acquired shall not exceed the
maximum herein fixed.
In case the transferee already owns urban or rural lands for residential purposes,
he shall still be entitled to be a transferee of an additional urban or rural lands for
residential purposes which, when added to those already owned by him, shall not
exceed the maximum areas herein authorized.
From the adoption of the 1987 Constitution up to the present, no other law has been passed by
the legislature on the same subject. Thus, what governs the disposition of private lands in favor
of a natural-born Filipino citizen who has lost his Philippine citizenship remains to be BP 185.
Even if private respondents were already Canadian citizens at the time they applied for
registration of the properties in question, said properties as discussed above were already
private lands; consequently, there could be no legal impediment for the registration thereof by
respondents in view of what the Constitution ordains. The parcels of land sought to be
registered no longer form part of the public domain. They are already private in character since
private respondents' predecessors-in-interest have been in open, continuous and exclusive
possession and occupation thereof under claim of ownership prior to June 12, 1945 or since
1937. The law provides that a natural-born citizen of the Philippines who has lost his Philippine
citizenship may be a transferee of a private land up to a maximum area of 1,000 sq.m., if urban,
or one (1) hectare in case of rural land, to be used by him as his residence (BP 185).
It is undisputed that private respondents, as vendees of a private land, were natural-born
citizens of the Philippines. For the purpose of transfer and/or acquisition of a parcel of
residential land, it is not significant whether private respondents are no longer Filipino citizens at
the time they purchased or registered the parcels of land in question. What is important is that
private respondents were formerly natural-born citizens of the Philippines, and as transferees of
a private land, they could apply for registration in accordance with the mandate of Section 8,
Article XII of the Constitution. Considering that private respondents were able to prove the
requisite period and character of possession of their predecessors-in-interest over the subject
lots, their application for registration of title must perforce be approved.
The dissenting opinion, however, states that the requirements in BP 185, must also be complied
with by private respondents. Specifically, it refers to Section 6, which provides:
Sec. 6. In addition to the requirements provided for in other laws for the
registration of titles to lands, no private land shall be transferred under this Act,
unless the transferee shall submit to the register of deeds of the province or city
where the property is located a sworn statement showing the date and place of
his birth; the names and addresses of his parents, of his spouse and children, if
any; the area, the location and the mode of acquisition of his landholdings in the
Philippines, if any; his intention to reside permanently in the Philippines; the date
he lost his Philippine citizenship and the country of which he is presently a
citizen; and such other information as may be required under Section 8 of this
Act.
The Court is of the view that the requirements in Sec. 6 of BP 185 do not apply in the instant
case since said requirements are primarily directed to the register of deeds before whom
compliance therewith is to be submitted. Nowhere in the provision is it stated, much less
implied, that the requirements must likewise be submitted before the land registration court prior
to the approval of an application for registration of title. An application for registration of title
before a land registration court should not be confused with the issuance of a certificate of title
by the register of deeds. It is only when the judgment of the land registration court approving the
application for registration has become final that a decree of registration is issued. And that is
the time when the requirements of Sec. 6, BP 185, before the register of deeds should be
complied with by the applicants. This decree of registration is the one that is submitted to the
office of the register of deeds for issuance of the certificate of title in favor of the applicant. Prior
to the issuance of the decree of registration, the register of deeds has no participation in the
approval of the application for registration of title as the decree of registration is yet to be issued.
WHEREFORE, the petition is DISMISSED and the decision appealed from is hereby
AFFIRMED.
SO ORDERED.
Narvasa, C.J., Cruz, Feliciano, Padilla, Regalado, Davide, Jr., Romero, Bellosillo, Melo,
Quiason, Puno, Vitug, Kapunan, and Mendoza, JJ.,concur.



Separate Opinions

CRUZ, J., dissenting:
With all due respect, I have to dissent.
The ponencia begins by posing the issue thus:
Can a foreign national apply for registration of title over a parcel of land which he
acquired by purchase while still a citizen of the Philippines, from a vendor who
has complied with the requirements for registration under the Public Land Act
(CA 141)?
There is no question that the property is private land and thus subject to registration by qualified
persons. It was really needless to elaborate on Buyco, which is clearly inapplicable here. We
can agree that the ruling case is Director of Lands v. Intermediate Appellate Court, which is not
challenged in this petition.
But I think the ponencia misses the point. The finding that the respondent spouses were natural-
born Filpinos at the time they acquired the land does not settle the question posed.
The important point is that the respondent spouses are no longer citizens of the Philippines but
naturalized Canadians. It does not follow that because they were citizens of the Philippines
when they acquired the land, they can register it in their names now even if they are no longer
Filipinos.
Section 7 of Article XII of the Constitution is irrelevant because it is not disputed that the
respondent spouses were qualified to acquire the land in question when it was transferred to
them.
Section 8 of the same article is not applicable either because it speaks of a transfer of private
land to a former natural-born citizen of the Philippines after he became a foreigner.
Thus it states:
Sec. 8. Notwithstanding the provisions of Section 7 of this Article, a natural-born
citizen of the Philippines who has lost his Philippine citizenship may be a
transferee of private lands, subject to limitations provided by law.
Even if it be assumed that the provision is applicable, it does not appear that the private
respondents have observed "the limitations provided by law."
The ponencia finds that all the requisites for the registration of the land in the private
respondents' name have been complied with. I do not believe so for there is no showing that
B.P. 185 has also been enforced.
The view has been expressed that we should confine ourselves to the requirements for
registration under the Public Land Act. I respectfully submit that the requirements in B.P. 185
have been read into the Act and should also be applied.
Strict compliance is necessary because of the special privilege granted to former Filipinos who
have become foreigners by their own choice. If we can be so strict with our own citizens, I see
no reason why we should be less so with those who have renounced our country.
Feliciano, J.: concurring
I agree with the great bulk of the majority opinion written by Mr. Justice Bidin and the result
reached therein.
This separate statement is concerned only with the last two (2) paragraphs, just before the
dispositive portion, of the majority opinion. In my view, it should be stressed that B.P. Blg. 185
which took effect on 16 March 1982, does not purport to cover the set of facts before the Court
in this case: i.e., the respondent spouses became transferees (on 17 June 1978) of the land
here involved while they were natural-born Philippine citizens who happened sometime later to
have been naturalized as citizens of another country. B.P. Blg. 185, as far as I can determine,
addresses itself only to a situation ofpersons who were already foreign nationals at the time they
became transferees of private land in the Philippines, but who were previously natural-born
Philippine citizens. It is difficult, therefore, to see how B.P. Blg. 185 can become applicable to
the present situation even at the subsequent time when the respondent spouses would come
before the Register of Deeds. B.P. Blg. 185, especially Section 6 thereof, imposes certain
requirements, including a specific limitation on the quantity of land (not more than 1,000 square
meters) which may be acquired thereunder, an amount limitation which must not be exceeded
both by the land of which such foreign national becomes transferee and by such land taken
together with other land previously acquired by such foreign national. (2nd paragraph, Section
2, B.P. Blg. 185)
B.P. Blg. 185 would, of course, apply to subsequent purchases of land by the respondent
spouses, that is, purchases made after they were naturalized as Canadian nationals.

# Separate Opinions
CRUZ, J., dissenting:
With all due respect, I have to dissent.
The ponencia begins by posing the issue thus:
Can a foreign national apply for registration of title over a parcel of land which he
acquired by purchase while still a citizen of the Philippines, from a vendor who
has complied with the requirements for registration under the Public Land Act
(CA 141)?
There is no question that the property is private land and thus subject to registration by qualified
persons. It was really needless to elaborate on Buyco, which is clearly inapplicable here. We
can agree that the ruling case is Director of Lands v. Intermediate Appellate Court, which is not
challenged in this petition.
But I think the ponencia misses the point. The finding that the respondent spouses were natural-
born Filpinos at the time they acquired the land does not settle the question posed.
The important point is that the respondent spouses are no longer citizens of the Philippines but
naturalized Canadians. It does not follow that because they were citizens of the Philippines
when they acquired the land, they can register it in their names now even if they are no longer
Filipinos.
Section 7 of Article XII of the Constitution is irrelevant because it is not disputed that the
respondent spouses were qualified to acquire the land in question when it was transferred to
them.
Section 8 of the same article is not applicable either because it speaks of a transfer of private
land to a former natural-born citizen of the Philippines after he became a foreigner.
Thus it states:
Sec. 8. Notwithstanding the provisions of Section 7 of this Article, a natural-born
citizen of the Philippines who has lost his Philippine citizenship may be a
transferee of private lands, subject to limitations provided by law.
Even if it be assumed that the provision is applicable, it does not appear that the private
respondents have observed "the limitations provided by law."
The ponencia finds that all the requisites for the registration of the land in the private
respondents' name have been complied with. I do not believe so for there is no showing that
B.P. 185 has also been enforced.
The view has been expressed that we should confine ourselves to the requirements for
registration under the Public Land Act. I respectfully submit that the requirements in B.P. 185
have been read into the Act and should also be applied.
Strict compliance is necessary because of the special privilege granted to former Filipinos who
have become foreigners by their own choice. If we can be so strict with our own citizens, I see
no reason why we should be less so with those who have renounced our country.
Feliciano, J.: concurring
I agree with the great bulk of the majority opinion written by Mr. Justice Bidin and the result
reached therein.
This separate statement is concerned only with the last two (2) paragraphs, just before the
dispositive portion, of the majority opinion. In my view, it should be stressed that B.P. Blg. 185
which took effect on 16 March 1982, does not purport to cover the set of facts before the Court
in this case: i.e., the respondent spouses became transferees (on 17 June 1978) of the land
here involved while they were natural-born Philippine citizens who happened sometime later to
have been naturalized as citizens of another country. B.P. Blg. 185, as far as I can determine,
addresses itself only to a situation ofpersons who were already foreign nationals at the time they
became transferees of private land in the Philippines, but who were previously natural-born
Philippine citizens. It is difficult, therefore, to see how B.P. Blg. 185 can become applicable to
the present situation even at the subsequent time when the respondent spouses would come
before the Register of Deeds. B.P. Blg. 185, especially Section 6 thereof, imposes certain
requirements, including a specific limitation on the quantity of land (not more than 1,000 square
meters) which may be acquired thereunder, an amount limitation which must not be exceeded
both by the land of which such foreign national becomes transferee and by such land taken
together with other land previously acquired by such foreign national. (2nd paragraph, Section
2, B.P. Blg. 185)
B.P. Blg. 185 would, of course, apply to subsequent purchases of land by the respondent
spouses, that is, purchases made after they were naturalized as Canadian nationals.
Republic of the Philippines
SUPREME COURT
Manila
EN BANC
G.R. No. 133250 July 9, 2002
FRANCISCO I. CHAVEZ, petitioner,
vs.
PUBLIC ESTATES AUTHORITY and AMARI COASTAL BAY DEVELOPMENT
CORPORATION, respondents.
CARPIO, J .:
This is an original Petition for Mandamus with prayer for a writ of preliminary injunction and a
temporary restraining order. The petition seeks to compel the Public Estates Authority ("PEA"
for brevity) to disclose all facts on PEA's then on-going renegotiations with Amari Coastal Bay
and Development Corporation ("AMARI" for brevity) to reclaim portions of Manila Bay. The
petition further seeks to enjoin PEA from signing a new agreement with AMARI involving such
reclamation.
The Facts
On November 20, 1973, the government, through the Commissioner of Public Highways, signed
a contract with the Construction and Development Corporation of the Philippines ("CDCP" for
brevity) to reclaim certain foreshore and offshore areas of Manila Bay. The contract also
included the construction of Phases I and II of the Manila-Cavite Coastal Road. CDCP obligated
itself to carry out all the works in consideration of fifty percent of the total reclaimed land.
On February 4, 1977, then President Ferdinand E. Marcos issued Presidential Decree No. 1084
creating PEA. PD No. 1084 tasked PEA "to reclaim land, including foreshore and submerged
areas," and "to develop, improve, acquire, x x x lease and sell any and all kinds of lands."
1
On
the same date, then President Marcos issued Presidential Decree No. 1085 transferring to PEA
the "lands reclaimed in the foreshore and offshore of the Manila Bay"
2
under the Manila-Cavite
Coastal Road and Reclamation Project (MCCRRP).
On December 29, 1981, then President Marcos issued a memorandum directing PEA to amend
its contract with CDCP, so that "[A]ll future works in MCCRRP x x x shall be funded and owned
by PEA." Accordingly, PEA and CDCP executed a Memorandum of Agreement dated
December 29, 1981, which stated:
"(i) CDCP shall undertake all reclamation, construction, and such other works in the
MCCRRP as may be agreed upon by the parties, to be paid according to progress of
works on a unit price/lump sum basis for items of work to be agreed upon, subject to
price escalation, retention and other terms and conditions provided for in Presidential
Decree No. 1594. All the financing required for such works shall be provided by PEA.
x x x
(iii) x x x CDCP shall give up all its development rights and hereby agrees to cede and
transfer in favor of PEA, all of the rights, title, interest and participation of CDCP in and
to all the areas of land reclaimed by CDCP in the MCCRRP as of December 30, 1981
which have not yet been sold, transferred or otherwise disposed of by CDCP as of said
date, which areas consist of approximately Ninety-Nine Thousand Four Hundred
Seventy Three (99,473) square meters in the Financial Center Area covered by land
pledge No. 5 and approximately Three Million Three Hundred Eighty Two Thousand
Eight Hundred Eighty Eight (3,382,888) square meters of reclaimed areas at varying
elevations above Mean Low Water Level located outside the Financial Center Area and
the First Neighborhood Unit."
3

On January 19, 1988, then President Corazon C. Aquino issued Special Patent No. 3517,
granting and transferring to PEA "the parcels of land so reclaimed under the Manila-Cavite
Coastal Road and Reclamation Project (MCCRRP) containing a total area of one million nine
hundred fifteen thousand eight hundred ninety four (1,915,894) square meters." Subsequently,
on April 9, 1988, the Register of Deeds of the Municipality of Paraaque issued Transfer
Certificates of Title Nos. 7309, 7311, and 7312, in the name of PEA, covering the three
reclaimed islands known as the "Freedom Islands" located at the southern portion of the Manila-
Cavite Coastal Road, Paraaque City. The Freedom Islands have a total land area of One
Million Five Hundred Seventy Eight Thousand Four Hundred and Forty One (1,578,441) square
meters or 157.841 hectares.
On April 25, 1995, PEA entered into a Joint Venture Agreement ("JVA" for brevity) with AMARI,
a private corporation, to develop the Freedom Islands. The JVA also required the reclamation of
an additional 250 hectares of submerged areas surrounding these islands to complete the
configuration in the Master Development Plan of the Southern Reclamation Project-MCCRRP.
PEA and AMARI entered into the JVA through negotiation without public bidding.
4
On April 28,
1995, the Board of Directors of PEA, in its Resolution No. 1245, confirmed the JVA.
5
On June 8,
1995, then President Fidel V. Ramos, through then Executive Secretary Ruben Torres,
approved the JVA.
6

On November 29, 1996, then Senate President Ernesto Maceda delivered a privilege speech in
the Senate and denounced the JVA as the "grandmother of all scams." As a result, the Senate
Committee on Government Corporations and Public Enterprises, and the Committee on
Accountability of Public Officers and Investigations, conducted a joint investigation. The Senate
Committees reported the results of their investigation in Senate Committee Report No. 560
dated September 16, 1997.
7
Among the conclusions of their report are: (1) the reclaimed lands
PEA seeks to transfer to AMARI under the JVA are lands of the public domain which the
government has not classified as alienable lands and therefore PEA cannot alienate these
lands; (2) the certificates of title covering the Freedom Islands are thus void, and (3) the JVA
itself is illegal.
On December 5, 1997, then President Fidel V. Ramos issued Presidential Administrative Order
No. 365 creating a Legal Task Force to conduct a study on the legality of the JVA in view of
Senate Committee Report No. 560. The members of the Legal Task Force were the Secretary
of Justice,
8
the Chief Presidential Legal Counsel,
9
and the Government Corporate
Counsel.
10
The Legal Task Force upheld the legality of the JVA, contrary to the conclusions
reached by the Senate Committees.
11

On April 4 and 5, 1998, the Philippine Daily Inquirer and Today published reports that there
were on-going renegotiations between PEA and AMARI under an order issued by then
President Fidel V. Ramos. According to these reports, PEA Director Nestor Kalaw, PEA
Chairman Arsenio Yulo and retired Navy Officer Sergio Cruz composed the negotiating panel of
PEA.
On April 13, 1998, Antonio M. Zulueta filed before the Court a Petition for Prohibition with
Application for the Issuance of a Temporary Restraining Order and Preliminary
Injunction docketed as G.R. No. 132994 seeking to nullify the JVA. The Court dismissed the
petition "for unwarranted disregard of judicial hierarchy, without prejudice to the refiling of the
case before the proper court."
12

On April 27, 1998, petitioner Frank I. Chavez ("Petitioner" for brevity) as a taxpayer, filed the
instant Petition for Mandamus with Prayer for the Issuance of a Writ of Preliminary Injunction
and Temporary Restraining Order. Petitioner contends the government stands to lose billions of
pesos in the sale by PEA of the reclaimed lands to AMARI. Petitioner prays that PEA publicly
disclose the terms of any renegotiation of the JVA, invoking Section 28, Article II, and Section 7,
Article III, of the 1987 Constitution on the right of the people to information on matters of public
concern. Petitioner assails the sale to AMARI of lands of the public domain as a blatant violation
of Section 3, Article XII of the 1987 Constitution prohibiting the sale of alienable lands of the
public domain to private corporations. Finally, petitioner asserts that he seeks to enjoin the loss
of billions of pesos in properties of the State that are of public dominion.
After several motions for extension of time,
13
PEA and AMARI filed their Comments on October
19, 1998 and June 25, 1998, respectively. Meanwhile, on December 28, 1998, petitioner filed an
Omnibus Motion: (a) to require PEA to submit the terms of the renegotiated PEA-AMARI
contract; (b) for issuance of a temporary restraining order; and (c) to set the case for hearing on
oral argument. Petitioner filed a Reiterative Motion for Issuance of a TRO dated May 26, 1999,
which the Court denied in a Resolution dated June 22, 1999.
In a Resolution dated March 23, 1999, the Court gave due course to the petition and required
the parties to file their respective memoranda.
On March 30, 1999, PEA and AMARI signed the Amended Joint Venture Agreement
("Amended JVA," for brevity). On May 28, 1999, the Office of the President under the
administration of then President Joseph E. Estrada approved the Amended JVA.
Due to the approval of the Amended JVA by the Office of the President, petitioner now prays
that on "constitutional and statutory grounds the renegotiated contract be declared null and
void."
14

The Issues
The issues raised by petitioner, PEA
15
and AMARI
16
are as follows:
I. WHETHER THE PRINCIPAL RELIEFS PRAYED FOR IN THE PETITION ARE MOOT
AND ACADEMIC BECAUSE OF SUBSEQUENT EVENTS;
II. WHETHER THE PETITION MERITS DISMISSAL FOR FAILING TO OBSERVE THE
PRINCIPLE GOVERNING THE HIERARCHY OF COURTS;
III. WHETHER THE PETITION MERITS DISMISSAL FOR NON-EXHAUSTION OF
ADMINISTRATIVE REMEDIES;
IV. WHETHER PETITIONER HAS LOCUS STANDI TO BRING THIS SUIT;
V. WHETHER THE CONSTITUTIONAL RIGHT TO INFORMATION INCLUDES
OFFICIAL INFORMATION ON ON-GOING NEGOTIATIONS BEFORE A FINAL
AGREEMENT;
VI. WHETHER THE STIPULATIONS IN THE AMENDED JOINT VENTURE
AGREEMENT FOR THE TRANSFER TO AMARI OF CERTAIN LANDS, RECLAIMED
AND STILL TO BE RECLAIMED, VIOLATE THE 1987 CONSTITUTION; AND
VII. WHETHER THE COURT IS THE PROPER FORUM FOR RAISING THE ISSUE OF
WHETHER THE AMENDED JOINT VENTURE AGREEMENT IS GROSSLY
DISADVANTAGEOUS TO THE GOVERNMENT.
The Court's Ruling
First issue: whether the principal reliefs prayed for in the petition are moot and academic
because of subsequent events.
The petition prays that PEA publicly disclose the "terms and conditions of the on-going
negotiations for a new agreement." The petition also prays that the Court enjoin PEA from
"privately entering into, perfecting and/or executing any new agreement with AMARI."
PEA and AMARI claim the petition is now moot and academic because AMARI furnished
petitioner on June 21, 1999 a copy of the signed Amended JVA containing the terms and
conditions agreed upon in the renegotiations. Thus, PEA has satisfied petitioner's prayer for a
public disclosure of the renegotiations. Likewise, petitioner's prayer to enjoin the signing of the
Amended JVA is now moot because PEA and AMARI have already signed the Amended JVA
on March 30, 1999. Moreover, the Office of the President has approved the Amended JVA on
May 28, 1999.
Petitioner counters that PEA and AMARI cannot avoid the constitutional issue by simply fast-
tracking the signing and approval of the Amended JVA before the Court could act on the issue.
Presidential approval does not resolve the constitutional issue or remove it from the ambit of
judicial review.
We rule that the signing of the Amended JVA by PEA and AMARI and its approval by the
President cannot operate to moot the petition and divest the Court of its jurisdiction. PEA and
AMARI have still to implement the Amended JVA. The prayer to enjoin the signing of the
Amended JVA on constitutional grounds necessarily includes preventing its implementation if in
the meantime PEA and AMARI have signed one in violation of the Constitution. Petitioner's
principal basis in assailing the renegotiation of the JVA is its violation of Section 3, Article XII of
the Constitution, which prohibits the government from alienating lands of the public domain to
private corporations. If the Amended JVA indeed violates the Constitution, it is the duty of the
Court to enjoin its implementation, and if already implemented, to annul the effects of such
unconstitutional contract.
The Amended JVA is not an ordinary commercial contract but one which seeks to transfer title
and ownership to 367.5 hectares of reclaimed lands and submerged areas of Manila Bay
to a single private corporation. It now becomes more compelling for the Court to resolve the
issue to insure the government itself does not violate a provision of the Constitution intended to
safeguard the national patrimony. Supervening events, whether intended or accidental, cannot
prevent the Court from rendering a decision if there is a grave violation of the Constitution. In
the instant case, if the Amended JVA runs counter to the Constitution, the Court can still prevent
the transfer of title and ownership of alienable lands of the public domain in the name of AMARI.
Even in cases where supervening events had made the cases moot, the Court did not hesitate
to resolve the legal or constitutional issues raised to formulate controlling principles to guide the
bench, bar, and the public.
17

Also, the instant petition is a case of first impression. All previous decisions of the Court
involving Section 3, Article XII of the 1987 Constitution, or its counterpart provision in the 1973
Constitution,
18
covered agricultural lands sold to private corporations which acquired the lands
from private parties. The transferors of the private corporations claimed or could claim the right
to judicial confirmation of their imperfect titles
19
under Title II of Commonwealth Act. 141
("CA No. 141" for brevity). In the instant case, AMARI seeks to acquire from PEA, a public
corporation, reclaimed lands and submerged areas for non-agricultural purposes
by purchase under PD No. 1084 (charter of PEA) and Title III of CA No. 141. Certain
undertakings by AMARI under the Amended JVA constitute the consideration for the purchase.
Neither AMARI nor PEA can claim judicial confirmation of their titles because the lands covered
by the Amended JVA are newly reclaimed or still to be reclaimed. Judicial confirmation of
imperfect title requires open, continuous, exclusive and notorious occupation of agricultural
lands of the public domain for at least thirty years since June 12, 1945 or earlier. Besides, the
deadline for filing applications for judicial confirmation of imperfect title expired on December 31,
1987.
20

Lastly, there is a need to resolve immediately the constitutional issue raised in this petition
because of the possible transfer at any time by PEA to AMARI of title and ownership to portions
of the reclaimed lands. Under the Amended JVA, PEA is obligated to transfer to AMARI the
latter's seventy percent proportionate share in the reclaimed areas as the reclamation
progresses. The Amended JVA even allows AMARI to mortgage at any time
the entire reclaimed area to raise financing for the reclamation project.
21

Second issue: whether the petition merits dismissal for failing to observe the principle
governing the hierarchy of courts.
PEA and AMARI claim petitioner ignored the judicial hierarchy by seeking relief directly from the
Court. The principle of hierarchy of courts applies generally to cases involving factual questions.
As it is not a trier of facts, the Court cannot entertain cases involving factual issues. The instant
case, however, raises constitutional issues of transcendental importance to the public.
22
The
Court can resolve this case without determining any factual issue related to the case. Also, the
instant case is a petition for mandamus which falls under the original jurisdiction of the Court
under Section 5, Article VIII of the Constitution. We resolve to exercise primary jurisdiction over
the instant case.
Third issue: whether the petition merits dismissal for non-exhaustion of administrative
remedies.
PEA faults petitioner for seeking judicial intervention in compelling PEA to disclose publicly
certain information without first asking PEA the needed information. PEA claims petitioner's
direct resort to the Court violates the principle of exhaustion of administrative remedies. It also
violates the rule that mandamus may issue only if there is no other plain, speedy and adequate
remedy in the ordinary course of law.
PEA distinguishes the instant case from Taada v. Tuvera
23
where the Court granted the
petition for mandamus even if the petitioners there did not initially demand from the Office of the
President the publication of the presidential decrees. PEA points out that in Taada, the
Executive Department had an affirmative statutory duty under Article 2 of the Civil Code
24
and
Section 1 of Commonwealth Act No. 638
25
to publish the presidential decrees. There was,
therefore, no need for the petitioners in Taada to make an initial demand from the Office of the
President. In the instant case, PEA claims it has no affirmative statutory duty to disclose publicly
information about its renegotiation of the JVA. Thus, PEA asserts that the Court must apply the
principle of exhaustion of administrative remedies to the instant case in view of the failure of
petitioner here to demand initially from PEA the needed information.
The original JVA sought to dispose to AMARI public lands held by PEA, a government
corporation. Under Section 79 of the Government Auditing Code,
26
the disposition of
government lands to private parties requires public bidding. PEA was under a positive legal
duty to disclose to the public the terms and conditions for the sale of its lands. The law
obligated PEA to make this public disclosure even without demand from petitioner or from
anyone. PEA failed to make this public disclosure because the original JVA, like the Amended
JVA, was the result of a negotiated contract, not of a public bidding. Considering that PEA had
an affirmative statutory duty to make the public disclosure, and was even in breach of this legal
duty, petitioner had the right to seek direct judicial intervention.
Moreover, and this alone is determinative of this issue, the principle of exhaustion of
administrative remedies does not apply when the issue involved is a purely legal or
constitutional question.
27
The principal issue in the instant case is the capacity of AMARI to
acquire lands held by PEA in view of the constitutional ban prohibiting the alienation of lands of
the public domain to private corporations. We rule that the principle of exhaustion of
administrative remedies does not apply in the instant case.
Fourth issue: whether petitioner has locus standi to bring this suit
PEA argues that petitioner has no standing to institute mandamus proceedings to enforce his
constitutional right to information without a showing that PEA refused to perform an affirmative
duty imposed on PEA by the Constitution. PEA also claims that petitioner has not shown that he
will suffer any concrete injury because of the signing or implementation of the Amended JVA.
Thus, there is no actual controversy requiring the exercise of the power of judicial review.
The petitioner has standing to bring this taxpayer's suit because the petition seeks to compel
PEA to comply with its constitutional duties. There are two constitutional issues involved here.
First is the right of citizens to information on matters of public concern. Second is the application
of a constitutional provision intended to insure the equitable distribution of alienable lands of the
public domain among Filipino citizens. The thrust of the first issue is to compel PEA to disclose
publicly information on the sale of government lands worth billions of pesos, information which
the Constitution and statutory law mandate PEA to disclose. The thrust of the second issue is to
prevent PEA from alienating hundreds of hectares of alienable lands of the public domain in
violation of the Constitution, compelling PEA to comply with a constitutional duty to the nation.
Moreover, the petition raises matters of transcendental importance to the public. In Chavez v.
PCGG,
28
the Court upheld the right of a citizen to bring a taxpayer's suit on matters of
transcendental importance to the public, thus -
"Besides, petitioner emphasizes, the matter of recovering the ill-gotten wealth of the
Marcoses is an issue of 'transcendental importance to the public.' He asserts that
ordinary taxpayers have a right to initiate and prosecute actions questioning the validity
of acts or orders of government agencies or instrumentalities, if the issues raised are of
'paramount public interest,' and if they 'immediately affect the social, economic and
moral well being of the people.'
Moreover, the mere fact that he is a citizen satisfies the requirement of personal interest,
when the proceeding involves the assertion of a public right, such as in this case. He
invokes several decisions of this Court which have set aside the procedural matter
of locus standi, when the subject of the case involved public interest.
x x x
In Taada v. Tuvera, the Court asserted that when the issue concerns a public right and
the object of mandamus is to obtain the enforcement of a public duty, the people are
regarded as the real parties in interest; and because it is sufficient that petitioner is a
citizen and as such is interested in the execution of the laws, he need not show that he
has any legal or special interest in the result of the action. In the aforesaid case, the
petitioners sought to enforce their right to be informed on matters of public concern, a
right then recognized in Section 6, Article IV of the 1973 Constitution, in connection with
the rule that laws in order to be valid and enforceable must be published in the Official
Gazette or otherwise effectively promulgated. In ruling for the petitioners' legal standing,
the Court declared that the right they sought to be enforced 'is a public right recognized
by no less than the fundamental law of the land.'
Legaspi v. Civil Service Commission, while reiterating Taada, further declared that
'when a mandamus proceeding involves the assertion of a public right, the requirement
of personal interest is satisfied by the mere fact that petitioner is a citizen and, therefore,
part of the general 'public' which possesses the right.'
Further, in Albano v. Reyes, we said that while expenditure of public funds may not have
been involved under the questioned contract for the development, management and
operation of the Manila International Container Terminal, 'public interest [was] definitely
involved considering the important role [of the subject contract] . . . in the economic
development of the country and the magnitude of the financial consideration involved.'
We concluded that, as a consequence, the disclosure provision in the Constitution would
constitute sufficient authority for upholding the petitioner's standing.
Similarly, the instant petition is anchored on the right of the people to information and
access to official records, documents and papers a right guaranteed under Section 7,
Article III of the 1987 Constitution. Petitioner, a former solicitor general, is a Filipino
citizen. Because of the satisfaction of the two basic requisites laid down by decisional
law to sustain petitioner's legal standing, i.e. (1) the enforcement of a public right (2)
espoused by a Filipino citizen, we rule that the petition at bar should be allowed."
We rule that since the instant petition, brought by a citizen, involves the enforcement of
constitutional rights - to information and to the equitable diffusion of natural resources - matters
of transcendental public importance, the petitioner has the requisite locus standi.
Fifth issue: whether the constitutional right to information includes official information
on on-going negotiations before a final agreement.
Section 7, Article III of the Constitution explains the people's right to information on matters of
public concern in this manner:
"Sec. 7. The right of the people to information on matters of public concern shall be
recognized. Access to official records, and to documents, and papers pertaining to
official acts, transactions, or decisions, as well as to government research data used
as basis for policy development, shall be afforded the citizen, subject to such limitations
as may be provided by law." (Emphasis supplied)
The State policy of full transparency in all transactions involving public interest reinforces the
people's right to information on matters of public concern. This State policy is expressed in
Section 28, Article II of the Constitution, thus:
"Sec. 28. Subject to reasonable conditions prescribed by law, the State adopts and
implements a policy of full public disclosure of all its transactions involving public
interest." (Emphasis supplied)
These twin provisions of the Constitution seek to promote transparency in policy-making and in
the operations of the government, as well as provide the people sufficient information to
exercise effectively other constitutional rights. These twin provisions are essential to the
exercise of freedom of expression. If the government does not disclose its official acts,
transactions and decisions to citizens, whatever citizens say, even if expressed without any
restraint, will be speculative and amount to nothing. These twin provisions are also essential to
hold public officials "at all times x x x accountable to the people,"
29
for unless citizens have the
proper information, they cannot hold public officials accountable for anything. Armed with the
right information, citizens can participate in public discussions leading to the formulation of
government policies and their effective implementation. An informed citizenry is essential to the
existence and proper functioning of any democracy. As explained by the Court in Valmonte v.
Belmonte, J r.
30

"An essential element of these freedoms is to keep open a continuing dialogue or
process of communication between the government and the people. It is in the interest
of the State that the channels for free political discussion be maintained to the end that
the government may perceive and be responsive to the people's will. Yet, this open
dialogue can be effective only to the extent that the citizenry is informed and thus able to
formulate its will intelligently. Only when the participants in the discussion are aware of
the issues and have access to information relating thereto can such bear fruit."
PEA asserts, citing Chavez v. PCGG,
31
that in cases of on-going negotiations the right to
information is limited to "definite propositions of the government." PEA maintains the right does
not include access to "intra-agency or inter-agency recommendations or communications during
the stage when common assertions are still in the process of being formulated or are in the
'exploratory stage'."
Also, AMARI contends that petitioner cannot invoke the right at the pre-decisional stage or
before the closing of the transaction. To support its contention, AMARI cites the following
discussion in the 1986 Constitutional Commission:
"Mr. Suarez. And when we say 'transactions' which should be distinguished from
contracts, agreements, or treaties or whatever, does the Gentleman refer to the steps
leading to the consummation of the contract, or does he refer to the contract itself?
Mr. Ople: The 'transactions' used here, I suppose is generic and therefore, it can
cover both steps leading to a contract and already a consummated contract, Mr.
Presiding Officer.
Mr. Suarez: This contemplates inclusion of negotiations leading to the
consummation of the transaction.
Mr. Ople: Yes, subject only to reasonable safeguards on the national interest.
Mr. Suarez: Thank you."
32
(Emphasis supplied)
AMARI argues there must first be a consummated contract before petitioner can invoke the
right. Requiring government officials to reveal their deliberations at the pre-decisional stage will
degrade the quality of decision-making in government agencies. Government officials will
hesitate to express their real sentiments during deliberations if there is immediate public
dissemination of their discussions, putting them under all kinds of pressure before they decide.
We must first distinguish between information the law on public bidding requires PEA to disclose
publicly, and information the constitutional right to information requires PEA to release to the
public. Before the consummation of the contract, PEA must, on its own and without demand
from anyone, disclose to the public matters relating to the disposition of its property. These
include the size, location, technical description and nature of the property being disposed of, the
terms and conditions of the disposition, the parties qualified to bid, the minimum price and
similar information. PEA must prepare all these data and disclose them to the public at the start
of the disposition process, long before the consummation of the contract, because the
Government Auditing Code requires public bidding. If PEA fails to make this disclosure, any
citizen can demand from PEA this information at any time during the bidding process.
Information, however, on on-going evaluation or review of bids or proposals being undertaken
by the bidding or review committee is not immediately accessible under the right to information.
While the evaluation or review is still on-going, there are no "official acts, transactions, or
decisions" on the bids or proposals. However, once the committee makes itsofficial
recommendation, there arises a "definite proposition" on the part of the government. From
this moment, the public's right to information attaches, and any citizen can access all the non-
proprietary information leading to such definite proposition. In Chavez v. PCGG,
33
the Court
ruled as follows:
"Considering the intent of the framers of the Constitution, we believe that it is incumbent
upon the PCGG and its officers, as well as other government representatives, to
disclose sufficient public information on any proposed settlement they have decided to
take up with the ostensible owners and holders of ill-gotten wealth. Such information,
though, must pertain to definite propositions of the government, not necessarily to
intra-agency or inter-agency recommendations or communications during the stage
when common assertions are still in the process of being formulated or are in the
"exploratory" stage. There is need, of course, to observe the same restrictions on
disclosure of information in general, as discussed earlier such as on matters involving
national security, diplomatic or foreign relations, intelligence and other classified
information." (Emphasis supplied)
Contrary to AMARI's contention, the commissioners of the 1986 Constitutional Commission
understood that the right to information "contemplates inclusion of negotiations leading to
the consummation of the transaction." Certainly, a consummated contract is not a
requirement for the exercise of the right to information. Otherwise, the people can never
exercise the right if no contract is consummated, and if one is consummated, it may be too late
for the public to expose its defects.1wphi1.nt
Requiring a consummated contract will keep the public in the dark until the contract, which may
be grossly disadvantageous to the government or even illegal, becomes a fait accompli. This
negates the State policy of full transparency on matters of public concern, a situation which the
framers of the Constitution could not have intended. Such a requirement will prevent the
citizenry from participating in the public discussion of any proposed contract, effectively
truncating a basic right enshrined in the Bill of Rights. We can allow neither an emasculation of
a constitutional right, nor a retreat by the State of its avowed "policy of full disclosure of all its
transactions involving public interest."
The right covers three categories of information which are "matters of public concern," namely:
(1) official records; (2) documents and papers pertaining to official acts, transactions and
decisions; and (3) government research data used in formulating policies. The first category
refers to any document that is part of the public records in the custody of government agencies
or officials. The second category refers to documents and papers recording, evidencing,
establishing, confirming, supporting, justifying or explaining official acts, transactions or
decisions of government agencies or officials. The third category refers to research data,
whether raw, collated or processed, owned by the government and used in formulating
government policies.
The information that petitioner may access on the renegotiation of the JVA includes evaluation
reports, recommendations, legal and expert opinions, minutes of meetings, terms of reference
and other documents attached to such reports or minutes, all relating to the JVA. However, the
right to information does not compel PEA to prepare lists, abstracts, summaries and the like
relating to the renegotiation of the JVA.
34
The right only affords access to records, documents
and papers, which means the opportunity to inspect and copy them. One who exercises the
right must copy the records, documents and papers at his expense. The exercise of the right is
also subject to reasonable regulations to protect the integrity of the public records and to
minimize disruption to government operations, like rules specifying when and how to conduct
the inspection and copying.
35

The right to information, however, does not extend to matters recognized as privileged
information under the separation of powers.
36
The right does not also apply to information on
military and diplomatic secrets, information affecting national security, and information on
investigations of crimes by law enforcement agencies before the prosecution of the accused,
which courts have long recognized as confidential.
37
The right may also be subject to other
limitations that Congress may impose by law.
There is no claim by PEA that the information demanded by petitioner is privileged information
rooted in the separation of powers. The information does not cover Presidential conversations,
correspondences, or discussions during closed-door Cabinet meetings which, like internal
deliberations of the Supreme Court and other collegiate courts, or executive sessions of either
house of Congress,
38
are recognized as confidential. This kind of information cannot be pried
open by a co-equal branch of government. A frank exchange of exploratory ideas and
assessments, free from the glare of publicity and pressure by interested parties, is essential to
protect the independence of decision-making of those tasked to exercise Presidential,
Legislative and Judicial power.
39
This is not the situation in the instant case.
We rule, therefore, that the constitutional right to information includes official information on on-
going negotiations before a final contract. The information, however, must constitute definite
propositions by the government and should not cover recognized exceptions like privileged
information, military and diplomatic secrets and similar matters affecting national security and
public order.
40
Congress has also prescribed other limitations on the right to information in
several legislations.
41

Sixth issue: whether stipulations in the Amended J VA for the transfer to AMARI of lands,
reclaimed or to be reclaimed, violate the Constitution.
The Regalian Doctrine
The ownership of lands reclaimed from foreshore and submerged areas is rooted in the
Regalian doctrine which holds that the State owns all lands and waters of the public domain.
Upon the Spanish conquest of the Philippines, ownership of all "lands, territories and
possessions" in the Philippines passed to the Spanish Crown.
42
The King, as the sovereign ruler
and representative of the people, acquired and owned all lands and territories in the Philippines
except those he disposed of by grant or sale to private individuals.
The 1935, 1973 and 1987 Constitutions adopted the Regalian doctrine substituting, however,
the State, in lieu of the King, as the owner of all lands and waters of the public domain. The
Regalian doctrine is the foundation of the time-honored principle of land ownership that "all
lands that were not acquired from the Government, either by purchase or by grant, belong to the
public domain."
43
Article 339 of the Civil Code of 1889, which is now Article 420 of the Civil
Code of 1950, incorporated the Regalian doctrine.
Ownership and Disposition of Reclaimed Lands
The Spanish Law of Waters of 1866 was the first statutory law governing the ownership and
disposition of reclaimed lands in the Philippines. On May 18, 1907, the Philippine Commission
enacted Act No. 1654 which provided for the lease, but not the sale, of reclaimed lands of
the government to corporations and individuals. Later, on November 29, 1919, the
Philippine Legislature approved Act No. 2874, the Public Land Act, which authorized the lease,
but not the sale, of reclaimed lands of the government to corporations and individuals.
On November 7, 1936, the National Assembly passed Commonwealth Act No. 141, also known
as the Public Land Act, which authorized the lease, but not the sale, of reclaimed lands of
the government to corporations and individuals. CA No. 141 continues to this day as the
general law governing the classification and disposition of lands of the public domain.
The Spanish Law of Waters of 1866 and the Civil Code of 1889
Under the Spanish Law of Waters of 1866, the shores, bays, coves, inlets and all waters within
the maritime zone of the Spanish territory belonged to the public domain for public use.
44
The
Spanish Law of Waters of 1866 allowed the reclamation of the sea under Article 5, which
provided as follows:
"Article 5. Lands reclaimed from the sea in consequence of works constructed by the
State, or by the provinces, pueblos or private persons, with proper permission, shall
become the property of the party constructing such works, unless otherwise provided by
the terms of the grant of authority."
Under the Spanish Law of Waters, land reclaimed from the sea belonged to the party
undertaking the reclamation, provided the government issued the necessary permit and did not
reserve ownership of the reclaimed land to the State.
Article 339 of the Civil Code of 1889 defined property of public dominion as follows:
"Art. 339. Property of public dominion is
1. That devoted to public use, such as roads, canals, rivers, torrents, ports and bridges
constructed by the State, riverbanks, shores, roadsteads, and that of a similar character;
2. That belonging exclusively to the State which, without being of general public use, is
employed in some public service, or in the development of the national wealth, such as
walls, fortresses, and other works for the defense of the territory, and mines, until
granted to private individuals."
Property devoted to public use referred to property open for use by the public. In contrast,
property devoted to public service referred to property used for some specific public service and
open only to those authorized to use the property.
Property of public dominion referred not only to property devoted to public use, but also to
property not so used but employed to develop the national wealth. This class of property
constituted property of public dominion although employed for some economic or commercial
activity to increase the national wealth.
Article 341 of the Civil Code of 1889 governed the re-classification of property of public
dominion into private property, to wit:
"Art. 341. Property of public dominion, when no longer devoted to public use or to the
defense of the territory, shall become a part of the private property of the State."
This provision, however, was not self-executing. The legislature, or the executive department
pursuant to law, must declare the property no longer needed for public use or territorial defense
before the government could lease or alienate the property to private parties.
45

Act No. 1654 of the Philippine Commission
On May 8, 1907, the Philippine Commission enacted Act No. 1654 which regulated the lease of
reclaimed and foreshore lands. The salient provisions of this law were as follows:
"Section 1. The control and disposition of the foreshore as defined in existing law,
and the title to all Government or public lands made or reclaimed by the
Government by dredging or filling or otherwise throughout the Philippine
Islands, shall be retained by the Government without prejudice to vested rights and
without prejudice to rights conceded to the City of Manila in the Luneta Extension.
Section 2. (a) The Secretary of the Interior shall cause all Government or public lands
made or reclaimed by the Government by dredging or filling or otherwise to be divided
into lots or blocks, with the necessary streets and alleyways located thereon, and shall
cause plats and plans of such surveys to be prepared and filed with the Bureau of
Lands.
(b) Upon completion of such plats and plans the Governor-General shall give notice
to the public that such parts of the lands so made or reclaimed as are not needed
for public purposes will be leased for commercial and business purposes, x x x.
x x x
(e) The leases above provided for shall be disposed of to the highest and best
bidder therefore, subject to such regulations and safeguards as the Governor-General
may by executive order prescribe." (Emphasis supplied)
Act No. 1654 mandated that the government should retain title to all lands reclaimed by the
government. The Act also vested in the government control and disposition of foreshore lands.
Private parties could lease lands reclaimed by the government only if these lands were no
longer needed for public purpose. Act No. 1654 mandated public bidding in the lease of
government reclaimed lands. Act No. 1654 made government reclaimed lands sui generis in
that unlike other public lands which the government could sell to private parties, these reclaimed
lands were available only for lease to private parties.
Act No. 1654, however, did not repeal Section 5 of the Spanish Law of Waters of 1866. Act No.
1654 did not prohibit private parties from reclaiming parts of the sea under Section 5 of the
Spanish Law of Waters. Lands reclaimed from the sea by private parties with government
permission remained private lands.
Act No. 2874 of the Philippine Legislature
On November 29, 1919, the Philippine Legislature enacted Act No. 2874, the Public Land
Act.
46
The salient provisions of Act No. 2874, on reclaimed lands, were as follows:
"Sec. 6. The Governor-General, upon the recommendation of the Secretary of
Agriculture and Natural Resources, shall from time to time classify the lands of
the public domain into
(a) Alienable or disposable,
(b) Timber, and
(c) Mineral lands, x x x.
Sec. 7. For the purposes of the government and disposition of alienable or disposable
public lands, the Governor-General, upon recommendation by the Secretary of
Agriculture and Natural Resources, shall from time to time declare what lands are
open to disposition or concession under this Act."
Sec. 8. Only those lands shall be declared open to disposition or concession
which have been officially delimited or classified x x x.
x x x
Sec. 55. Any tract of land of the public domain which, being neither timber nor mineral
land, shall be classified assuitable for residential purposes or for commercial,
industrial, or other productive purposes other than agricultural purposes, and shall
be open to disposition or concession, shall be disposed of under the provisions of this
chapter, and not otherwise.
Sec. 56. The lands disposable under this title shall be classified as follows:
(a) Lands reclaimed by the Government by dredging, filling, or other
means;
(b) Foreshore;
(c) Marshy lands or lands covered with water bordering upon the shores or
banks of navigable lakes or rivers;
(d) Lands not included in any of the foregoing classes.
x x x.
Sec. 58. The lands comprised in classes (a), (b), and (c) of section fifty-six shall be
disposed of to private parties by lease only and not otherwise, as soon as the
Governor-General, upon recommendation by the Secretary of Agriculture and
Natural Resources, shall declare that the same are not necessary for the public
service and are open to disposition under this chapter. The lands included in class
(d) may be disposed of by sale or lease under the provisions of this Act."
(Emphasis supplied)
Section 6 of Act No. 2874 authorized the Governor-General to "classify lands of the public
domain into x x x alienable or disposable"
47
lands. Section 7 of the Act empowered the
Governor-General to "declare what lands are open to disposition or concession." Section 8 of
the Act limited alienable or disposable lands only to those lands which have been "officially
delimited and classified."
Section 56 of Act No. 2874 stated that lands "disposable under this title
48
shall be classified" as
government reclaimed, foreshore and marshy lands, as well as other lands. All these lands,
however, must be suitable for residential, commercial, industrial or other productive non-
agricultural purposes. These provisions vested upon the Governor-General the power to
classify inalienable lands of the public domain into disposable lands of the public domain. These
provisions also empowered the Governor-General to classify further such disposable lands of
the public domain into government reclaimed, foreshore or marshy lands of the public domain,
as well as other non-agricultural lands.
Section 58 of Act No. 2874 categorically mandated that disposable lands of the public domain
classified as government reclaimed, foreshore and marshy lands "shall be disposed of to
private parties by lease only and not otherwise." The Governor-General, before allowing the
lease of these lands to private parties, must formally declare that the lands were "not necessary
for the public service." Act No. 2874 reiterated the State policy to lease and not to sell
government reclaimed, foreshore and marshy lands of the public domain, a policy first
enunciated in 1907 in Act No. 1654. Government reclaimed, foreshore and marshy lands
remained sui generis, as the only alienable or disposable lands of the public domain that the
government could not sell to private parties.
The rationale behind this State policy is obvious. Government reclaimed, foreshore and marshy
public lands for non-agricultural purposes retain their inherent potential as areas for public
service. This is the reason the government prohibited the sale, and only allowed the lease, of
these lands to private parties. The State always reserved these lands for some future public
service.
Act No. 2874 did not authorize the reclassification of government reclaimed, foreshore and
marshy lands into other non-agricultural lands under Section 56 (d). Lands falling under Section
56 (d) were the only lands for non-agricultural purposes the government could sell to private
parties. Thus, under Act No. 2874, the government could not sell government reclaimed,
foreshore and marshy lands to private parties, unless the legislature passed a law allowing
their sale.
49

Act No. 2874 did not prohibit private parties from reclaiming parts of the sea pursuant to Section
5 of the Spanish Law of Waters of 1866. Lands reclaimed from the sea by private parties with
government permission remained private lands.
Dispositions under the 1935 Constitution
On May 14, 1935, the 1935 Constitution took effect upon its ratification by the Filipino people.
The 1935 Constitution, in adopting the Regalian doctrine, declared in Section 1, Article XIII, that

"Section 1. All agricultural, timber, and mineral lands of the public domain, waters,
minerals, coal, petroleum, and other mineral oils, all forces of potential energy and other
natural resources of the Philippines belong to the State, and their disposition,
exploitation, development, or utilization shall be limited to citizens of the Philippines or to
corporations or associations at least sixty per centum of the capital of which is owned by
such citizens, subject to any existing right, grant, lease, or concession at the time of the
inauguration of the Government established under this Constitution. Natural resources,
with the exception of public agricultural land, shall not be alienated, and no license,
concession, or lease for the exploitation, development, or utilization of any of the natural
resources shall be granted for a period exceeding twenty-five years, renewable for
another twenty-five years, except as to water rights for irrigation, water supply, fisheries,
or industrial uses other than the development of water power, in which cases beneficial
use may be the measure and limit of the grant." (Emphasis supplied)
The 1935 Constitution barred the alienation of all natural resources except public agricultural
lands, which were the only natural resources the State could alienate. Thus, foreshore lands,
considered part of the State's natural resources, became inalienable by constitutional fiat,
available only for lease for 25 years, renewable for another 25 years. The government could
alienate foreshore lands only after these lands were reclaimed and classified as alienable
agricultural lands of the public domain. Government reclaimed and marshy lands of the public
domain, being neither timber nor mineral lands, fell under the classification of public agricultural
lands.
50
However, government reclaimed and marshy lands, although subject to classification as
disposable public agricultural lands, could only be leased and not sold to private parties
because of Act No. 2874.
The prohibition on private parties from acquiring ownership of government reclaimed and
marshy lands of the public domain was only a statutory prohibition and the legislature could
therefore remove such prohibition. The 1935 Constitution did not prohibit individuals and
corporations from acquiring government reclaimed and marshy lands of the public domain that
were classified as agricultural lands under existing public land laws. Section 2, Article XIII of the
1935 Constitution provided as follows:
"Section 2. No private corporation or association may acquire, lease, or hold public
agricultural lands in excess of one thousand and twenty four hectares, nor may
any individual acquire such lands by purchase in excess of one hundred and forty
hectares, or by lease in excess of one thousand and twenty-four hectares, or by
homestead in excess of twenty-four hectares. Lands adapted to grazing, not exceeding
two thousand hectares, may be leased to an individual, private corporation, or
association." (Emphasis supplied)
Still, after the effectivity of the 1935 Constitution, the legislature did not repeal Section 58 of Act
No. 2874 to open for sale to private parties government reclaimed and marshy lands of the
public domain. On the contrary, the legislature continued the long established State policy of
retaining for the government title and ownership of government reclaimed and marshy lands of
the public domain.
Commonwealth Act No. 141 of the Philippine National Assembly
On November 7, 1936, the National Assembly approved Commonwealth Act No. 141, also
known as the Public Land Act, which compiled the then existing laws on lands of the public
domain. CA No. 141, as amended, remains to this day theexisting general law governing the
classification and disposition of lands of the public domain other than timber and mineral
lands.
51

Section 6 of CA No. 141 empowers the President to classify lands of the public domain into
"alienable or disposable"
52
lands of the public domain, which prior to such classification are
inalienable and outside the commerce of man. Section 7 of CA No. 141 authorizes the President
to "declare what lands are open to disposition or concession." Section 8 of CA No. 141 states
that the government can declare open for disposition or concession only lands that are "officially
delimited and classified." Sections 6, 7 and 8 of CA No. 141 read as follows:
"Sec. 6. The President, upon the recommendation of the Secretary of Agriculture
and Commerce, shall from time to time classify the lands of the public domain
into
(a) Alienable or disposable,
(b) Timber, and
(c) Mineral lands,
and may at any time and in like manner transfer such lands from one class to
another,
53
for the purpose of their administration and disposition.
Sec. 7. For the purposes of the administration and disposition of alienable or disposable
public lands, the President, upon recommendation by the Secretary of Agriculture
and Commerce, shall from time to time declare what lands are open to disposition
or concession under this Act.
Sec. 8. Only those lands shall be declared open to disposition or concession
which have been officially delimited and classified and, when practicable,
surveyed, and which have not been reserved for public or quasi-public uses, nor
appropriated by the Government, nor in any manner become private property, nor those
on which a private right authorized and recognized by this Act or any other valid law may
be claimed, or which, having been reserved or appropriated, have ceased to be so. x x
x."
Thus, before the government could alienate or dispose of lands of the public domain, the
President must first officially classify these lands as alienable or disposable, and then declare
them open to disposition or concession. There must be no law reserving these lands for public
or quasi-public uses.
The salient provisions of CA No. 141, on government reclaimed, foreshore and marshy lands of
the public domain, are as follows:
"Sec. 58. Any tract of land of the public domain which, being neither timber nor
mineral land, is intended to be used for residential purposes or for commercial,
industrial, or other productive purposes other than agricultural, and is open to
disposition or concession, shall be disposed of under the provisions of this
chapter and not otherwise.
Sec. 59. The lands disposable under this title shall be classified as follows:
(a) Lands reclaimed by the Government by dredging, filling, or other
means;
(b) Foreshore;
(c) Marshy lands or lands covered with water bordering upon the shores or
banks of navigable lakes or rivers;
(d) Lands not included in any of the foregoing classes.
Sec. 60. Any tract of land comprised under this title may be leased or sold, as the case
may be, to any person, corporation, or association authorized to purchase or lease
public lands for agricultural purposes. x x x.
Sec. 61. The lands comprised in classes (a), (b), and (c) of section fifty-nine shall
be disposed of to private parties by lease only and not otherwise, as soon as the
President, upon recommendation by the Secretary of Agriculture, shall declare that
the same are not necessary for the public service and are open to disposition under
this chapter. The lands included in class (d) may be disposed of by sale or lease
under the provisions of this Act." (Emphasis supplied)
Section 61 of CA No. 141 readopted, after the effectivity of the 1935 Constitution, Section 58 of
Act No. 2874 prohibiting the sale of government reclaimed, foreshore and marshy disposable
lands of the public domain. All these lands are intended for residential, commercial, industrial or
other non-agricultural purposes. As before, Section 61 allowed only the lease of such lands to
private parties. The government could sell to private parties only lands falling under Section 59
(d) of CA No. 141, or those lands for non-agricultural purposes not classified as government
reclaimed, foreshore and marshy disposable lands of the public domain. Foreshore lands,
however, became inalienable under the 1935 Constitution which only allowed the lease of these
lands to qualified private parties.
Section 58 of CA No. 141 expressly states that disposable lands of the public domain intended
for residential, commercial, industrial or other productive purposes other than agricultural "shall
be disposed of under the provisions of this chapter and not otherwise." Under Section 10
of CA No. 141, the term "disposition" includes lease of the land. Any disposition of government
reclaimed, foreshore and marshy disposable lands for non-agricultural purposes must comply
with Chapter IX, Title III of CA No. 141,
54
unless a subsequent law amended or repealed these
provisions.
In his concurring opinion in the landmark case of Republic Real Estate Corporation v. Court
of Appeals,
55
Justice Reynato S. Puno summarized succinctly the law on this matter, as
follows:
"Foreshore lands are lands of public dominion intended for public use. So too are lands
reclaimed by the government by dredging, filling, or other means. Act 1654 mandated
that the control and disposition of the foreshore and lands under water remained in the
national government. Said law allowed only the 'leasing' of reclaimed land. The Public
Land Acts of 1919 and 1936 also declared that the foreshore and lands reclaimed by the
government were to be "disposed of to private parties by lease only and not otherwise."
Before leasing, however, the Governor-General, upon recommendation of the Secretary
of Agriculture and Natural Resources, had first to determine that the land reclaimed was
not necessary for the public service. This requisite must have been met before the land
could be disposed of. But even then, the foreshore and lands under water were not
to be alienated and sold to private parties. The disposition of the reclaimed land
was only by lease. The land remained property of the State." (Emphasis supplied)
As observed by Justice Puno in his concurring opinion, "Commonwealth Act No. 141 has
remained in effect at present."
The State policy prohibiting the sale to private parties of government reclaimed, foreshore and
marshy alienable lands of the public domain, first implemented in 1907 was thus reaffirmed in
CA No. 141 after the 1935 Constitution took effect. The prohibition on the sale of foreshore
lands, however, became a constitutional edict under the 1935 Constitution. Foreshore lands
became inalienable as natural resources of the State, unless reclaimed by the government and
classified as agricultural lands of the public domain, in which case they would fall under the
classification of government reclaimed lands.
After the effectivity of the 1935 Constitution, government reclaimed and marshy disposable
lands of the public domain continued to be only leased and not sold to private parties.
56
These
lands remained sui generis, as the only alienable or disposable lands of the public domain the
government could not sell to private parties.
Since then and until now, the only way the government can sell to private parties government
reclaimed and marshy disposable lands of the public domain is for the legislature to pass a law
authorizing such sale. CA No. 141 does not authorize the President to reclassify government
reclaimed and marshy lands into other non-agricultural lands under Section 59 (d). Lands
classified under Section 59 (d) are the only alienable or disposable lands for non-agricultural
purposes that the government could sell to private parties.
Moreover, Section 60 of CA No. 141 expressly requires congressional authority before lands
under Section 59 that the government previously transferred to government units or entities
could be sold to private parties. Section 60 of CA No. 141 declares that
"Sec. 60. x x x The area so leased or sold shall be such as shall, in the judgment of the
Secretary of Agriculture and Natural Resources, be reasonably necessary for the
purposes for which such sale or lease is requested, and shall not exceed one hundred
and forty-four hectares: Provided, however, That this limitation shall not apply to grants,
donations, or transfers made to a province, municipality or branch or subdivision of the
Government for the purposes deemed by said entities conducive to the public
interest; but the land so granted, donated, or transferred to a province,
municipality or branch or subdivision of the Government shall not be alienated,
encumbered, or otherwise disposed of in a manner affecting its title, except when
authorized by Congress: x x x." (Emphasis supplied)
The congressional authority required in Section 60 of CA No. 141 mirrors the legislative
authority required in Section 56 of Act No. 2874.
One reason for the congressional authority is that Section 60 of CA No. 141 exempted
government units and entities from the maximum area of public lands that could be acquired
from the State. These government units and entities should not just turn around and sell these
lands to private parties in violation of constitutional or statutory limitations. Otherwise, the
transfer of lands for non-agricultural purposes to government units and entities could be used to
circumvent constitutional limitations on ownership of alienable or disposable lands of the public
domain. In the same manner, such transfers could also be used to evade the statutory
prohibition in CA No. 141 on the sale of government reclaimed and marshy lands of the public
domain to private parties. Section 60 of CA No. 141 constitutes by operation of law a lien on
these lands.
57

In case of sale or lease of disposable lands of the public domain falling under Section 59 of CA
No. 141, Sections 63 and 67 require a public bidding. Sections 63 and 67 of CA No. 141 provide
as follows:
"Sec. 63. Whenever it is decided that lands covered by this chapter are not needed for
public purposes, the Director of Lands shall ask the Secretary of Agriculture and
Commerce (now the Secretary of Natural Resources) for authority to dispose of the
same. Upon receipt of such authority, the Director of Lands shall give notice by public
advertisement in the same manner as in the case of leases or sales of agricultural public
land, x x x.
Sec. 67. The lease or sale shall be made by oral bidding; and adjudication shall be
made to the highest bidder. x x x." (Emphasis supplied)
Thus, CA No. 141 mandates the Government to put to public auction all leases or sales of
alienable or disposable lands of the public domain.
58

Like Act No. 1654 and Act No. 2874 before it, CA No. 141 did not repeal Section 5 of the
Spanish Law of Waters of 1866. Private parties could still reclaim portions of the sea with
government permission. However, the reclaimed land could become private land only if
classified as alienable agricultural land of the public domain open to disposition under CA
No. 141. The 1935 Constitution prohibited the alienation of all natural resources except public
agricultural lands.
The Civil Code of 1950
The Civil Code of 1950 readopted substantially the definition of property of public dominion
found in the Civil Code of 1889. Articles 420 and 422 of the Civil Code of 1950 state that
"Art. 420. The following things are property of public dominion:
(1) Those intended for public use, such as roads, canals, rivers, torrents, ports and
bridges constructed by the State, banks, shores, roadsteads, and others of similar
character;
(2) Those which belong to the State, without being for public use, and are intended for
some public service or for the development of the national wealth.
x x x.
Art. 422. Property of public dominion, when no longer intended for public use or for
public service, shall form part of the patrimonial property of the State."
Again, the government must formally declare that the property of public dominion is no longer
needed for public use or public service, before the same could be classified as patrimonial
property of the State.
59
In the case of government reclaimed and marshy lands of the public
domain, the declaration of their being disposable, as well as the manner of their disposition, is
governed by the applicable provisions of CA No. 141.
Like the Civil Code of 1889, the Civil Code of 1950 included as property of public dominion
those properties of the State which, without being for public use, are intended for public service
or the "development of the national wealth." Thus, government reclaimed and marshy lands
of the State, even if not employed for public use or public service, if developed to enhance the
national wealth, are classified as property of public dominion.
Dispositions under the 1973 Constitution
The 1973 Constitution, which took effect on January 17, 1973, likewise adopted the Regalian
doctrine. Section 8, Article XIV of the 1973 Constitution stated that
"Sec. 8. All lands of the public domain, waters, minerals, coal, petroleum and other
mineral oils, all forces of potential energy, fisheries, wildlife, and other natural resources
of the Philippines belong to the State. With the exception of agricultural, industrial or
commercial, residential, and resettlement lands of the public domain, natural
resources shall not be alienated, and no license, concession, or lease for the
exploration, development, exploitation, or utilization of any of the natural resources shall
be granted for a period exceeding twenty-five years, renewable for not more than
twenty-five years, except as to water rights for irrigation, water supply, fisheries, or
industrial uses other than the development of water power, in which cases, beneficial
use may be the measure and the limit of the grant." (Emphasis supplied)
The 1973 Constitution prohibited the alienation of all natural resources with the exception of
"agricultural, industrial or commercial, residential, and resettlement lands of the public domain."
In contrast, the 1935 Constitution barred the alienation of all natural resources except "public
agricultural lands." However, the term "public agricultural lands" in the 1935 Constitution
encompassed industrial, commercial, residential and resettlement lands of the public
domain.
60
If the land of public domain were neither timber nor mineral land, it would fall under
the classification of agricultural land of the public domain. Both the 1935 and 1973
Constitutions, therefore, prohibited the alienation of all natural resources except
agricultural lands of the public domain.
The 1973 Constitution, however, limited the alienation of lands of the public domain to
individuals who were citizens of the Philippines. Private corporations, even if wholly owned by
Philippine citizens, were no longer allowed to acquire alienable lands of the public domain unlike
in the 1935 Constitution. Section 11, Article XIV of the 1973 Constitution declared that
"Sec. 11. The Batasang Pambansa, taking into account conservation, ecological, and
development requirements of the natural resources, shall determine by law the size of
land of the public domain which may be developed, held or acquired by, or leased to,
any qualified individual, corporation, or association, and the conditions therefor. No
private corporation or association may hold alienable lands of the public domain
except by lease not to exceed one thousand hectares in area nor may any citizen hold
such lands by lease in excess of five hundred hectares or acquire by purchase,
homestead or grant, in excess of twenty-four hectares. No private corporation or
association may hold by lease, concession, license or permit, timber or forest lands and
other timber or forest resources in excess of one hundred thousand hectares. However,
such area may be increased by the Batasang Pambansa upon recommendation of the
National Economic and Development Authority." (Emphasis supplied)
Thus, under the 1973 Constitution, private corporations could hold alienable lands of the public
domain only through lease. Only individuals could now acquire alienable lands of the public
domain, and private corporations became absolutely barred from acquiring any kind of
alienable land of the public domain. The constitutional ban extended to all kinds of alienable
lands of the public domain, while the statutory ban under CA No. 141 applied only to
government reclaimed, foreshore and marshy alienable lands of the public domain.
PD No. 1084 Creating the Public Estates Authority
On February 4, 1977, then President Ferdinand Marcos issued Presidential Decree No. 1084
creating PEA, a wholly government owned and controlled corporation with a special charter.
Sections 4 and 8 of PD No. 1084, vests PEA with the following purposes and powers:
"Sec. 4. Purpose. The Authority is hereby created for the following purposes:
(a) To reclaim land, including foreshore and submerged areas, by dredging, filling
or other means, or to acquire reclaimed land;
(b) To develop, improve, acquire, administer, deal in, subdivide, dispose, lease and sell
any and all kinds of lands, buildings, estates and other forms of real property, owned,
managed, controlled and/or operated by the government;
(c) To provide for, operate or administer such service as may be necessary for the
efficient, economical and beneficial utilization of the above properties.
Sec. 5. Powers and functions of the Authority. The Authority shall, in carrying out the
purposes for which it is created, have the following powers and functions:
(a)To prescribe its by-laws.
x x x
(i) To hold lands of the public domain in excess of the area permitted to private
corporations by statute.
(j) To reclaim lands and to construct work across, or otherwise, any stream,
watercourse, canal, ditch, flume x x x.
x x x
(o) To perform such acts and exercise such functions as may be necessary for the
attainment of the purposes and objectives herein specified." (Emphasis supplied)
PD No. 1084 authorizes PEA to reclaim both foreshore and submerged areas of the public
domain. Foreshore areas are those covered and uncovered by the ebb and flow of the
tide.
61
Submerged areas are those permanently under water regardless of the ebb and flow of
the tide.
62
Foreshore and submerged areas indisputably belong to the public domain
63
and are
inalienable unless reclaimed, classified as alienable lands open to disposition, and further
declared no longer needed for public service.
The ban in the 1973 Constitution on private corporations from acquiring alienable lands of the
public domain did not apply to PEA since it was then, and until today, a fully owned government
corporation. The constitutional ban applied then, as it still applies now, only to "private
corporations and associations." PD No. 1084 expressly empowers PEA "to hold lands of the
public domain" even "in excess of the area permitted to private corporations by statute." Thus,
PEA can hold title to private lands, as well as title to lands of the public domain.
In order for PEA to sell its reclaimed foreshore and submerged alienable lands of the public
domain, there must be legislative authority empowering PEA to sell these lands. This legislative
authority is necessary in view of Section 60 of CA No.141, which states
"Sec. 60. x x x; but the land so granted, donated or transferred to a province,
municipality, or branch or subdivision of the Government shall not be alienated,
encumbered or otherwise disposed of in a manner affecting its title,except when
authorized by Congress; x x x." (Emphasis supplied)
Without such legislative authority, PEA could not sell but only lease its reclaimed foreshore and
submerged alienable lands of the public domain. Nevertheless, any legislative authority granted
to PEA to sell its reclaimed alienable lands of the public domain would be subject to the
constitutional ban on private corporations from acquiring alienable lands of the public domain.
Hence, such legislative authority could only benefit private individuals.
Dispositions under the 1987 Constitution
The 1987 Constitution, like the 1935 and 1973 Constitutions before it, has adopted the Regalian
doctrine. The 1987 Constitution declares that all natural resources are "owned by the State,"
and except for alienable agricultural lands of the public domain, natural resources cannot be
alienated. Sections 2 and 3, Article XII of the 1987 Constitution state that
"Section 2. All lands of the public domain, waters, minerals, coal, petroleum and other
mineral oils, all forces of potential energy, fisheries, forests or timber, wildlife, flora and
fauna, and other natural resources are owned by the State. With the exception of
agricultural lands, all other natural resources shall not be alienated. The
exploration, development, and utilization of natural resources shall be under the full
control and supervision of the State. x x x.
Section 3. Lands of the public domain are classified into agricultural, forest or timber,
mineral lands, and national parks. Agricultural lands of the public domain may be further
classified by law according to the uses which they may be devoted. Alienable lands of
the public domain shall be limited to agricultural lands. Private corporations or
associations may not hold such alienable lands of the public domain except by
lease, for a period not exceeding twenty-five years, renewable for not more than
twenty-five years, and not to exceed one thousand hectares in area. Citizens of the
Philippines may lease not more than five hundred hectares, or acquire not more than
twelve hectares thereof by purchase, homestead, or grant.
Taking into account the requirements of conservation, ecology, and development, and
subject to the requirements of agrarian reform, the Congress shall determine, by law, the
size of lands of the public domain which may be acquired, developed, held, or leased
and the conditions therefor." (Emphasis supplied)
The 1987 Constitution continues the State policy in the 1973 Constitution banning private
corporations from acquiring any kind of alienable land of the public domain. Like the 1973
Constitution, the 1987 Constitution allows private corporations to hold alienable lands of the
public domain only through lease. As in the 1935 and 1973 Constitutions, the general law
governing the lease to private corporations of reclaimed, foreshore and marshy alienable lands
of the public domain is still CA No. 141.
The Rationale behind the Constitutional Ban
The rationale behind the constitutional ban on corporations from acquiring, except through
lease, alienable lands of the public domain is not well understood. During the deliberations of
the 1986 Constitutional Commission, the commissioners probed the rationale behind this ban,
thus:
"FR. BERNAS: Mr. Vice-President, my questions have reference to page 3, line 5 which
says:
`No private corporation or association may hold alienable lands of the public domain
except by lease, not to exceed one thousand hectares in area.'
If we recall, this provision did not exist under the 1935 Constitution, but this was
introduced in the 1973 Constitution. In effect, it prohibits private corporations from
acquiring alienable public lands. But it has not been very clear in jurisprudence what
the reason for this is. In some of the cases decided in 1982 and 1983, it was
indicated that the purpose of this is to prevent large landholdings. Is that the intent
of this provision?
MR. VILLEGAS: I think that is the spirit of the provision.
FR. BERNAS: In existing decisions involving the Iglesia ni Cristo, there were instances
where the Iglesia ni Cristo was not allowed to acquire a mere 313-square meter land
where a chapel stood because the Supreme Court said it would be in violation of this."
(Emphasis supplied)
In Ayog v. Cusi,
64
the Court explained the rationale behind this constitutional ban in this way:
"Indeed, one purpose of the constitutional prohibition against purchases of public
agricultural lands by private corporations is to equitably diffuse land ownership or to
encourage 'owner-cultivatorship and the economic family-size farm' and to prevent a
recurrence of cases like the instant case. Huge landholdings by corporations or private
persons had spawned social unrest."
However, if the constitutional intent is to prevent huge landholdings, the Constitution could have
simply limited the size of alienable lands of the public domain that corporations could acquire.
The Constitution could have followed the limitations on individuals, who could acquire not more
than 24 hectares of alienable lands of the public domain under the 1973 Constitution, and not
more than 12 hectares under the 1987 Constitution.
If the constitutional intent is to encourage economic family-size farms, placing the land in the
name of a corporation would be more effective in preventing the break-up of farmlands. If the
farmland is registered in the name of a corporation, upon the death of the owner, his heirs would
inherit shares in the corporation instead of subdivided parcels of the farmland. This would
prevent the continuing break-up of farmlands into smaller and smaller plots from one generation
to the next.
In actual practice, the constitutional ban strengthens the constitutional limitation on individuals
from acquiring more than the allowed area of alienable lands of the public domain. Without the
constitutional ban, individuals who already acquired the maximum area of alienable lands of the
public domain could easily set up corporations to acquire more alienable public lands. An
individual could own as many corporations as his means would allow him. An individual could
even hide his ownership of a corporation by putting his nominees as stockholders of the
corporation. The corporation is a convenient vehicle to circumvent the constitutional limitation on
acquisition by individuals of alienable lands of the public domain.
The constitutional intent, under the 1973 and 1987 Constitutions, is to transfer ownership of only
a limited area of alienable land of the public domain to a qualified individual. This constitutional
intent is safeguarded by the provision prohibiting corporations from acquiring alienable lands of
the public domain, since the vehicle to circumvent the constitutional intent is removed. The
available alienable public lands are gradually decreasing in the face of an ever-growing
population. The most effective way to insure faithful adherence to this constitutional intent is to
grant or sell alienable lands of the public domain only to individuals. This, it would seem, is the
practical benefit arising from the constitutional ban.
The Amended J oint Venture Agreement
The subject matter of the Amended JVA, as stated in its second Whereas clause, consists of
three properties, namely:
1. "[T]hree partially reclaimed and substantially eroded islands along Emilio Aguinaldo
Boulevard in Paranaque and Las Pinas, Metro Manila, with a combined titled area of
1,578,441 square meters;"
2. "[A]nother area of 2,421,559 square meters contiguous to the three islands;" and
3. "[A]t AMARI's option as approved by PEA, an additional 350 hectares more or less to
regularize the configuration of the reclaimed area."
65

PEA confirms that the Amended JVA involves "the development of the Freedom Islands and
further reclamation of about 250 hectares x x x," plus an option "granted to AMARI to
subsequently reclaim another 350 hectares x x x."
66

In short, the Amended JVA covers a reclamation area of 750 hectares. Only 157.84 hectares
of the 750-hectare reclamation project have been reclaimed, and the rest of the 592.15
hectares are still submerged areas forming part of Manila Bay.
Under the Amended JVA, AMARI will reimburse PEA the sum of P1,894,129,200.00 for PEA's
"actual cost" in partially reclaiming the Freedom Islands. AMARI will also complete, at its own
expense, the reclamation of the Freedom Islands. AMARI will further shoulder all the
reclamation costs of all the other areas, totaling 592.15 hectares, still to be reclaimed. AMARI
and PEA will share, in the proportion of 70 percent and 30 percent, respectively, the total net
usable area which is defined in the Amended JVA as the total reclaimed area less 30 percent
earmarked for common areas. Title to AMARI's share in the net usable area, totaling 367.5
hectares, will be issued in the name of AMARI. Section 5.2 (c) of the Amended JVA provides
that
"x x x, PEA shall have the duty to execute without delay the necessary deed of transfer
or conveyance of the title pertaining to AMARI's Land share based on the Land
Allocation Plan. PEA, when requested in writing by AMARI, shall then cause the
issuance and delivery of the proper certificates of title covering AMARI's Land
Share in the name of AMARI, x x x; provided, that if more than seventy percent (70%)
of the titled area at any given time pertains to AMARI, PEA shall deliver to AMARI only
seventy percent (70%) of the titles pertaining to AMARI, until such time when a
corresponding proportionate area of additional land pertaining to PEA has been titled."
(Emphasis supplied)
Indisputably, under the Amended J VA AMARI will acquire and own a maximum of 367.5
hectares of reclaimed land which will be titled in its name.
To implement the Amended JVA, PEA delegated to the unincorporated PEA-AMARI joint
venture PEA's statutory authority, rights and privileges to reclaim foreshore and submerged
areas in Manila Bay. Section 3.2.a of the Amended JVA states that
"PEA hereby contributes to the joint venture its rights and privileges to perform Rawland
Reclamation and Horizontal Development as well as own the Reclamation Area, thereby
granting the Joint Venture the full and exclusive right, authority and privilege to
undertake the Project in accordance with the Master Development Plan."
The Amended JVA is the product of a renegotiation of the original JVA dated April 25, 1995 and
its supplemental agreement dated August 9, 1995.
The Threshold Issue
The threshold issue is whether AMARI, a private corporation, can acquire and own under the
Amended JVA 367.5 hectares of reclaimed foreshore and submerged areas in Manila Bay in
view of Sections 2 and 3, Article XII of the 1987 Constitution which state that:
"Section 2. All lands of the public domain, waters, minerals, coal, petroleum, and other
mineral oils, all forces of potential energy, fisheries, forests or timber, wildlife, flora and
fauna, and other natural resources are owned by the State. With the exception of
agricultural lands, all other natural resources shall not be alienated. x x x.
x x x
Section 3. x x x Alienable lands of the public domain shall be limited to agricultural
lands. Private corporations or associations may not hold such alienable lands of
the public domain except by lease, x x x."(Emphasis supplied)
Classification of Reclaimed Foreshore and Submerged Areas
PEA readily concedes that lands reclaimed from foreshore or submerged areas of Manila Bay
are alienable or disposable lands of the public domain. In its Memorandum,
67
PEA admits that
"Under the Public Land Act (CA 141, as amended), reclaimed lands are classified as
alienable and disposable lands of the public domain:
'Sec. 59. The lands disposable under this title shall be classified as follows:
(a) Lands reclaimed by the government by dredging, filling, or other means;
x x x.'" (Emphasis supplied)
Likewise, the Legal Task Force
68
constituted under Presidential Administrative Order No. 365
admitted in its Report and Recommendation to then President Fidel V. Ramos, "[R]eclaimed
lands are classified as alienable and disposable lands of the public domain."
69
The Legal
Task Force concluded that
"D. Conclusion
Reclaimed lands are lands of the public domain. However, by statutory authority, the
rights of ownership and disposition over reclaimed lands have been transferred to PEA,
by virtue of which PEA, as owner, may validly convey the same to any qualified person
without violating the Constitution or any statute.
The constitutional provision prohibiting private corporations from holding public land,
except by lease (Sec. 3, Art. XVII,
70
1987 Constitution), does not apply to reclaimed
lands whose ownership has passed on to PEA by statutory grant."
Under Section 2, Article XII of the 1987 Constitution, the foreshore and submerged areas of
Manila Bay are part of the "lands of the public domain, waters x x x and other natural resources"
and consequently "owned by the State." As such, foreshore and submerged areas "shall not be
alienated," unless they are classified as "agricultural lands" of the public domain. The mere
reclamation of these areas by PEA does not convert these inalienable natural resources of the
State into alienable or disposable lands of the public domain. There must be a law or
presidential proclamation officially classifying these reclaimed lands as alienable or disposable
and open to disposition or concession. Moreover, these reclaimed lands cannot be classified as
alienable or disposable if the law has reserved them for some public or quasi-public use.
71

Section 8 of CA No. 141 provides that "only those lands shall be declared open to disposition or
concession which have been officially delimited and classified."
72
The President has the
authority to classify inalienable lands of the public domain into alienable or disposable lands of
the public domain, pursuant to Section 6 of CA No. 141. In Laurel vs. Garcia,
73
the Executive
Department attempted to sell the Roppongi property in Tokyo, Japan, which was acquired by
the Philippine Government for use as the Chancery of the Philippine Embassy. Although the
Chancery had transferred to another location thirteen years earlier, the Court still ruled that,
under Article 422
74
of the Civil Code, a property of public dominion retains such character until
formally declared otherwise. The Court ruled that
"The fact that the Roppongi site has not been used for a long time for actual Embassy
service does not automatically convert it to patrimonial property. Any such conversion
happens only if the property is withdrawn from public use (Cebu Oxygen and Acetylene
Co. v. Bercilles, 66 SCRA 481 [1975]. A property continues to be part of the public
domain, not available for private appropriation or ownership 'until there is a formal
declaration on the part of the government to withdraw it from being such' (Ignacio
v. Director of Lands, 108 Phil. 335 [1960]." (Emphasis supplied)
PD No. 1085, issued on February 4, 1977, authorized the issuance of special land patents for
lands reclaimed by PEA from the foreshore or submerged areas of Manila Bay. On January 19,
1988 then President Corazon C. Aquino issued Special Patent No. 3517 in the name of PEA for
the 157.84 hectares comprising the partially reclaimed Freedom Islands. Subsequently, on April
9, 1999 the Register of Deeds of the Municipality of Paranaque issued TCT Nos. 7309, 7311
and 7312 in the name of PEA pursuant to Section 103 of PD No. 1529 authorizing the issuance
of certificates of title corresponding to land patents. To this day, these certificates of title are still
in the name of PEA.
PD No. 1085, coupled with President Aquino's actual issuance of a special patent covering the
Freedom Islands, is equivalent to an official proclamation classifying the Freedom Islands as
alienable or disposable lands of the public domain. PD No. 1085 and President Aquino's
issuance of a land patent also constitute a declaration that the Freedom Islands are no longer
needed for public service. The Freedom Islands are thus alienable or disposable lands of
the public domain, open to disposition or concession to qualified parties.
At the time then President Aquino issued Special Patent No. 3517, PEA had already reclaimed
the Freedom Islands although subsequently there were partial erosions on some areas. The
government had also completed the necessary surveys on these islands. Thus, the Freedom
Islands were no longer part of Manila Bay but part of the land mass. Section 3, Article XII of the
1987 Constitution classifies lands of the public domain into "agricultural, forest or timber, mineral
lands, and national parks." Being neither timber, mineral, nor national park lands, the reclaimed
Freedom Islands necessarily fall under the classification of agricultural lands of the public
domain. Under the 1987 Constitution, agricultural lands of the public domain are the only natural
resources that the State may alienate to qualified private parties. All other natural resources,
such as the seas or bays, are "waters x x x owned by the State" forming part of the public
domain, and are inalienable pursuant to Section 2, Article XII of the 1987 Constitution.
AMARI claims that the Freedom Islands are private lands because CDCP, then a private
corporation, reclaimed the islands under a contract dated November 20, 1973 with the
Commissioner of Public Highways. AMARI, citing Article 5 of the Spanish Law of Waters of
1866, argues that "if the ownership of reclaimed lands may be given to the party constructing
the works, then it cannot be said that reclaimed lands are lands of the public domain which the
State may not alienate."
75
Article 5 of the Spanish Law of Waters reads as follows:
"Article 5. Lands reclaimed from the sea in consequence of works constructed by the
State, or by the provinces, pueblos or private persons, with proper permission, shall
become the property of the party constructing such works,unless otherwise provided
by the terms of the grant of authority." (Emphasis supplied)
Under Article 5 of the Spanish Law of Waters of 1866, private parties could reclaim from the sea
only with "proper permission" from the State. Private parties could own the reclaimed land only if
not "otherwise provided by the terms of the grant of authority." This clearly meant that no one
could reclaim from the sea without permission from the State because the sea is property of
public dominion. It also meant that the State could grant or withhold ownership of the reclaimed
land because any reclaimed land, like the sea from which it emerged, belonged to the State.
Thus, a private person reclaiming from the sea without permission from the State could not
acquire ownership of the reclaimed land which would remain property of public dominion like the
sea it replaced.
76
Article 5 of the Spanish Law of Waters of 1866 adopted the time-honored
principle of land ownership that "all lands that were not acquired from the government, either by
purchase or by grant, belong to the public domain."
77

Article 5 of the Spanish Law of Waters must be read together with laws subsequently enacted
on the disposition of public lands. In particular, CA No. 141 requires that lands of the public
domain must first be classified as alienable or disposable before the government can alienate
them. These lands must not be reserved for public or quasi-public purposes.
78
Moreover, the
contract between CDCP and the government was executed after the effectivity of the 1973
Constitution which barred private corporations from acquiring any kind of alienable land of the
public domain. This contract could not have converted the Freedom Islands into private lands of
a private corporation.
Presidential Decree No. 3-A, issued on January 11, 1973, revoked all laws authorizing the
reclamation of areas under water and revested solely in the National Government the power to
reclaim lands. Section 1 of PD No. 3-A declared that
"The provisions of any law to the contrary notwithstanding, the reclamation of areas
under water, whether foreshore or inland, shall be limited to the National Government
or any person authorized by it under a proper contract. (Emphasis supplied)
x x x."
PD No. 3-A repealed Section 5 of the Spanish Law of Waters of 1866 because reclamation of
areas under water could now be undertaken only by the National Government or by a person
contracted by the National Government. Private parties may reclaim from the sea only under a
contract with the National Government, and no longer by grant or permission as provided in
Section 5 of the Spanish Law of Waters of 1866.
Executive Order No. 525, issued on February 14, 1979, designated PEA as the National
Government's implementing arm to undertake "all reclamation projects of the government,"
which "shall be undertaken by the PEA or through a proper contract executed by it with
any person or entity." Under such contract, a private party receives compensation for
reclamation services rendered to PEA. Payment to the contractor may be in cash, or in kind
consisting of portions of the reclaimed land, subject to the constitutional ban on private
corporations from acquiring alienable lands of the public domain. The reclaimed land can be
used as payment in kind only if the reclaimed land is first classified as alienable or disposable
land open to disposition, and then declared no longer needed for public service.
The Amended JVA covers not only the Freedom Islands, but also an additional 592.15 hectares
which are still submerged and forming part of Manila Bay. There is no legislative or
Presidential act classifying these submerged areas as alienable or disposable lands of
the public domain open to disposition. These submerged areas are not covered by any
patent or certificate of title. There can be no dispute that these submerged areas form part of
the public domain, and in their present state are inalienable and outside the commerce of
man. Until reclaimed from the sea, these submerged areas are, under the Constitution, "waters
x x x owned by the State," forming part of the public domain and consequently inalienable. Only
when actually reclaimed from the sea can these submerged areas be classified as public
agricultural lands, which under the Constitution are the only natural resources that the State
may alienate. Once reclaimed and transformed into public agricultural lands, the government
may then officially classify these lands as alienable or disposable lands open to disposition.
Thereafter, the government may declare these lands no longer needed for public service. Only
then can these reclaimed lands be considered alienable or disposable lands of the public
domain and within the commerce of man.
The classification of PEA's reclaimed foreshore and submerged lands into alienable or
disposable lands open to disposition is necessary because PEA is tasked under its charter to
undertake public services that require the use of lands of the public domain. Under Section 5 of
PD No. 1084, the functions of PEA include the following: "[T]o own or operate railroads,
tramways and other kinds of land transportation, x x x; [T]o construct, maintain and operate
such systems of sanitary sewers as may be necessary; [T]o construct, maintain and operate
such storm drains as may be necessary." PEA is empowered to issue "rules and regulations as
may be necessary for the proper use by private parties of any or all of the highways, roads,
utilities, buildings and/or any of its properties and to impose or collect fees or tolls for their
use." Thus, part of the reclaimed foreshore and submerged lands held by the PEA would
actually be needed for public use or service since many of the functions imposed on PEA by its
charter constitute essential public services.
Moreover, Section 1 of Executive Order No. 525 provides that PEA "shall be primarily
responsible for integrating, directing, and coordinating all reclamation projects for and on behalf
of the National Government." The same section also states that "[A]ll reclamation projects shall
be approved by the President upon recommendation of the PEA, and shall be undertaken by
the PEA or through a proper contract executed by it with any person or entity; x x x." Thus,
under EO No. 525, in relation to PD No. 3-A and PD No.1084, PEA became the primary
implementing agency of the National Government to reclaim foreshore and submerged lands of
the public domain. EO No. 525 recognized PEA as the government entity "to undertake the
reclamation of lands and ensure their maximum utilization in promoting public welfare and
interests."
79
Since large portions of these reclaimed lands would obviously be needed for public
service, there must be a formal declaration segregating reclaimed lands no longer needed for
public service from those still needed for public service.1wphi1.nt
Section 3 of EO No. 525, by declaring that all lands reclaimed by PEA "shall belong to or be
owned by the PEA," could not automatically operate to classify inalienable lands into alienable
or disposable lands of the public domain. Otherwise, reclaimed foreshore and submerged lands
of the public domain would automatically become alienable once reclaimed by PEA, whether or
not classified as alienable or disposable.
The Revised Administrative Code of 1987, a later law than either PD No. 1084 or EO No. 525,
vests in the Department of Environment and Natural Resources ("DENR" for brevity) the
following powers and functions:
"Sec. 4. Powers and Functions. The Department shall:
(1) x x x
x x x
(4) Exercise supervision and control over forest lands, alienable and disposable
public lands, mineral resources and, in the process of exercising such control, impose
appropriate taxes, fees, charges, rentals and any such form of levy and collect such
revenues for the exploration, development, utilization or gathering of such resources;
x x x
(14) Promulgate rules, regulations and guidelines on the issuance of licenses,
permits, concessions, lease agreements and such other privileges concerning the
development, exploration and utilization of the country's marine, freshwater, and
brackish water and over all aquatic resources of the country and shall continue to
oversee, supervise and police our natural resources; cancel or cause to cancel such
privileges upon failure, non-compliance or violations of any regulation, order, and for all
other causes which are in furtherance of the conservation of natural resources and
supportive of the national interest;
(15) Exercise exclusive jurisdiction on the management and disposition of all
lands of the public domain and serve as the sole agency responsible for
classification, sub-classification, surveying and titling of lands in consultation with
appropriate agencies."
80
(Emphasis supplied)
As manager, conservator and overseer of the natural resources of the State, DENR exercises
"supervision and control over alienable and disposable public lands." DENR also exercises
"exclusive jurisdiction on the management and disposition of all lands of the public domain."
Thus, DENR decides whether areas under water, like foreshore or submerged areas of Manila
Bay, should be reclaimed or not. This means that PEA needs authorization from DENR before
PEA can undertake reclamation projects in Manila Bay, or in any part of the country.
DENR also exercises exclusive jurisdiction over the disposition of all lands of the public domain.
Hence, DENR decides whether reclaimed lands of PEA should be classified as alienable under
Sections 6
81
and 7
82
of CA No. 141. Once DENR decides that the reclaimed lands should be so
classified, it then recommends to the President the issuance of a proclamation classifying the
lands as alienable or disposable lands of the public domain open to disposition. We note that
then DENR Secretary Fulgencio S. Factoran, Jr. countersigned Special Patent No. 3517 in
compliance with the Revised Administrative Code and Sections 6 and 7 of CA No. 141.
In short, DENR is vested with the power to authorize the reclamation of areas under water,
while PEA is vested with the power to undertake the physical reclamation of areas under water,
whether directly or through private contractors. DENR is also empowered to classify lands of the
public domain into alienable or disposable lands subject to the approval of the President. On the
other hand, PEA is tasked to develop, sell or lease the reclaimed alienable lands of the public
domain.
Clearly, the mere physical act of reclamation by PEA of foreshore or submerged areas does not
make the reclaimed lands alienable or disposable lands of the public domain, much less
patrimonial lands of PEA. Likewise, the mere transfer by the National Government of lands of
the public domain to PEA does not make the lands alienable or disposable lands of the public
domain, much less patrimonial lands of PEA.
Absent two official acts a classification that these lands are alienable or disposable and open
to disposition and a declaration that these lands are not needed for public service, lands
reclaimed by PEA remain inalienable lands of the public domain. Only such an official
classification and formal declaration can convert reclaimed lands into alienable or disposable
lands of the public domain, open to disposition under the Constitution, Title I and Title III
83
of CA
No. 141 and other applicable laws.
84

PEA's Authority to Sell Reclaimed Lands
PEA, like the Legal Task Force, argues that as alienable or disposable lands of the public
domain, the reclaimed lands shall be disposed of in accordance with CA No. 141, the Public
Land Act. PEA, citing Section 60 of CA No. 141, admits that reclaimed lands transferred to a
branch or subdivision of the government "shall not be alienated, encumbered, or otherwise
disposed of in a manner affecting its title, except when authorized by Congress: x x
x."
85
(Emphasis by PEA)
In Laurel vs. Garcia,
86
the Court cited Section 48 of the Revised Administrative Code of 1987,
which states that
"Sec. 48. Official Authorized to Convey Real Property. Whenever real property of the
Government is authorized by law to be conveyed, the deed of conveyance shall be
executed in behalf of the government by the following: x x x."
Thus, the Court concluded that a law is needed to convey any real property belonging to the
Government. The Court declared that -
"It is not for the President to convey real property of the government on his or her own
sole will. Any such conveyance must be authorized and approved by a law enacted
by the Congress. It requires executive and legislative concurrence." (Emphasis
supplied)
PEA contends that PD No. 1085 and EO No. 525 constitute the legislative authority allowing
PEA to sell its reclaimed lands. PD No. 1085, issued on February 4, 1977, provides that
"The land reclaimed in the foreshore and offshore area of Manila Bay pursuant to
the contract for the reclamation and construction of the Manila-Cavite Coastal Road
Project between the Republic of the Philippines and the Construction and Development
Corporation of the Philippines dated November 20, 1973 and/or any other contract or
reclamation covering the same area is hereby transferred, conveyed and assigned to
the ownership and administration of the Public Estates Authority established
pursuant to PD No. 1084; Provided, however, That the rights and interests of the
Construction and Development Corporation of the Philippines pursuant to the aforesaid
contract shall be recognized and respected.
Henceforth, the Public Estates Authority shall exercise the rights and assume the
obligations of the Republic of the Philippines (Department of Public Highways) arising
from, or incident to, the aforesaid contract between the Republic of the Philippines and
the Construction and Development Corporation of the Philippines.
In consideration of the foregoing transfer and assignment, the Public Estates Authority
shall issue in favor of the Republic of the Philippines the corresponding shares of stock
in said entity with an issued value of said shares of stock (which) shall be deemed fully
paid and non-assessable.
The Secretary of Public Highways and the General Manager of the Public Estates
Authority shall execute such contracts or agreements, including appropriate agreements
with the Construction and Development Corporation of the Philippines, as may be
necessary to implement the above.
Special land patent/patents shall be issued by the Secretary of Natural Resources
in favor of the Public Estates Authority without prejudice to the subsequent
transfer to the contractor or his assignees of such portion or portions of the land
reclaimed or to be reclaimed as provided for in the above-mentioned contract. On
the basis of such patents, the Land Registration Commission shall issue the
corresponding certificate of title." (Emphasis supplied)
On the other hand, Section 3 of EO No. 525, issued on February 14, 1979, provides that -
"Sec. 3. All lands reclaimed by PEA shall belong to or be owned by the PEA which
shall be responsible for its administration, development, utilization or disposition in
accordance with the provisions of Presidential Decree No. 1084. Any and all income that
the PEA may derive from the sale, lease or use of reclaimed lands shall be used in
accordance with the provisions of Presidential Decree No. 1084."
There is no express authority under either PD No. 1085 or EO No. 525 for PEA to sell its
reclaimed lands. PD No. 1085 merely transferred "ownership and administration" of lands
reclaimed from Manila Bay to PEA, while EO No. 525 declared that lands reclaimed by PEA
"shall belong to or be owned by PEA." EO No. 525 expressly states that PEA should dispose of
its reclaimed lands "in accordance with the provisions of Presidential Decree No. 1084," the
charter of PEA.
PEA's charter, however, expressly tasks PEA "to develop, improve, acquire, administer, deal in,
subdivide, dispose, lease and sell any and all kinds of lands x x x owned, managed,
controlled and/or operated by the government."
87
(Emphasis supplied) There is, therefore,
legislative authority granted to PEA to sell its lands, whether patrimonial or alienable
lands of the public domain. PEA may sell to private parties its patrimonial properties in
accordance with the PEA charter free from constitutional limitations. The constitutional ban on
private corporations from acquiring alienable lands of the public domain does not apply to the
sale of PEA's patrimonial lands.
PEA may also sell its alienable or disposable lands of the public domain to private
individuals since, with the legislative authority, there is no longer any statutory prohibition
against such sales and the constitutional ban does not apply to individuals. PEA, however,
cannot sell any of its alienable or disposable lands of the public domain to private corporations
since Section 3, Article XII of the 1987 Constitution expressly prohibits such sales. The
legislative authority benefits only individuals. Private corporations remain barred from acquiring
any kind of alienable land of the public domain, including government reclaimed lands.
The provision in PD No. 1085 stating that portions of the reclaimed lands could be transferred
by PEA to the "contractor or his assignees" (Emphasis supplied) would not apply to private
corporations but only to individuals because of the constitutional ban. Otherwise, the provisions
of PD No. 1085 would violate both the 1973 and 1987 Constitutions.
The requirement of public auction in the sale of reclaimed lands
Assuming the reclaimed lands of PEA are classified as alienable or disposable lands open to
disposition, and further declared no longer needed for public service, PEA would have to
conduct a public bidding in selling or leasing these lands. PEA must observe the provisions of
Sections 63 and 67 of CA No. 141 requiring public auction, in the absence of a law exempting
PEA from holding a public auction.
88
Special Patent No. 3517 expressly states that the patent is
issued by authority of the Constitution and PD No. 1084, "supplemented by Commonwealth Act
No. 141, as amended." This is an acknowledgment that the provisions of CA No. 141 apply to
the disposition of reclaimed alienable lands of the public domain unless otherwise provided by
law. Executive Order No. 654,
89
which authorizes PEA "to determine the kind and manner of
payment for the transfer" of its assets and properties, does not exempt PEA from the
requirement of public auction. EO No. 654 merely authorizes PEA to decide the mode of
payment, whether in kind and in installment, but does not authorize PEA to dispense with public
auction.
Moreover, under Section 79 of PD No. 1445, otherwise known as the Government Auditing
Code, the government is required to sell valuable government property through public bidding.
Section 79 of PD No. 1445 mandates that
"Section 79. When government property has become unserviceable for any cause, or
is no longer needed, it shall, upon application of the officer accountable therefor, be
inspected by the head of the agency or his duly authorized representative in the
presence of the auditor concerned and, if found to be valueless or unsaleable, it may be
destroyed in their presence. If found to be valuable, it may be sold at public auction
to the highest bidder under the supervision of the proper committee on award or
similar body in the presence of the auditor concerned or other authorized representative
of the Commission, after advertising by printed notice in the Official Gazette, or for
not less than three consecutive days in any newspaper of general circulation, or
where the value of the property does not warrant the expense of publication, by notices
posted for a like period in at least three public places in the locality where the property is
to be sold. In the event that the public auction fails, the property may be sold at a
private sale at such price as may be fixed by the same committee or body
concerned and approved by the Commission."
It is only when the public auction fails that a negotiated sale is allowed, in which case the
Commission on Audit must approve the selling price.
90
The Commission on Audit implements
Section 79 of the Government Auditing Code through Circular No. 89-296
91
dated January 27,
1989. This circular emphasizes that government assets must be disposed of only through public
auction, and a negotiated sale can be resorted to only in case of "failure of public auction."
At the public auction sale, only Philippine citizens are qualified to bid for PEA's reclaimed
foreshore and submerged alienable lands of the public domain. Private corporations are barred
from bidding at the auction sale of any kind of alienable land of the public domain.
PEA originally scheduled a public bidding for the Freedom Islands on December 10, 1991. PEA
imposed a condition that the winning bidder should reclaim another 250 hectares of submerged
areas to regularize the shape of the Freedom Islands, under a 60-40 sharing of the additional
reclaimed areas in favor of the winning bidder.
92
No one, however, submitted a bid. On
December 23, 1994, the Government Corporate Counsel advised PEA it could sell the Freedom
Islands through negotiation, without need of another public bidding, because of the failure of the
public bidding on December 10, 1991.
93

However, the original JVA dated April 25, 1995 covered not only the Freedom Islands and the
additional 250 hectares still to be reclaimed, it also granted an option to AMARI to reclaim
another 350 hectares. The original JVA, a negotiated contract, enlarged the reclamation area
to 750 hectares.
94
The failure of public bidding on December 10, 1991, involving only 407.84
hectares,
95
is not a valid justification for a negotiated sale of 750 hectares, almost double the
area publicly auctioned. Besides, the failure of public bidding happened on December 10, 1991,
more than three years before the signing of the original JVA on April 25, 1995. The economic
situation in the country had greatly improved during the intervening period.
Reclamation under the BOT Law and the Local Government Code
The constitutional prohibition in Section 3, Article XII of the 1987 Constitution is absolute and
clear: "Private corporations or associations may not hold such alienable lands of the public
domain except by lease, x x x." Even Republic Act No. 6957 ("BOT Law," for brevity), cited by
PEA and AMARI as legislative authority to sell reclaimed lands to private parties, recognizes the
constitutional ban. Section 6 of RA No. 6957 states
"Sec. 6. Repayment Scheme. - For the financing, construction, operation and
maintenance of any infrastructure projects undertaken through the build-operate-and-
transfer arrangement or any of its variations pursuant to the provisions of this Act, the
project proponent x x x may likewise be repaid in the form of a share in the revenue of
the project or other non-monetary payments, such as, but not limited to, the grant of a
portion or percentage of the reclaimed land, subject to the constitutional
requirements with respect to the ownership of the land: x x x." (Emphasis supplied)
A private corporation, even one that undertakes the physical reclamation of a government BOT
project, cannot acquire reclaimed alienable lands of the public domain in view of the
constitutional ban.
Section 302 of the Local Government Code, also mentioned by PEA and AMARI, authorizes
local governments in land reclamation projects to pay the contractor or developer in kind
consisting of a percentage of the reclaimed land, to wit:
"Section 302. Financing, Construction, Maintenance, Operation, and Management of
Infrastructure Projects by the Private Sector. x x x
x x x
In case of land reclamation or construction of industrial estates, the repayment plan may
consist of the grant of a portion or percentage of the reclaimed land or the industrial
estate constructed."
Although Section 302 of the Local Government Code does not contain a proviso similar to that
of the BOT Law, the constitutional restrictions on land ownership automatically apply even
though not expressly mentioned in the Local Government Code.
Thus, under either the BOT Law or the Local Government Code, the contractor or developer, if
a corporate entity, can only be paid with leaseholds on portions of the reclaimed land. If the
contractor or developer is an individual, portions of the reclaimed land, not exceeding 12
hectares
96
of non-agricultural lands, may be conveyed to him in ownership in view of the
legislative authority allowing such conveyance. This is the only way these provisions of the BOT
Law and the Local Government Code can avoid a direct collision with Section 3, Article XII of
the 1987 Constitution.
Registration of lands of the public domain
Finally, PEA theorizes that the "act of conveying the ownership of the reclaimed lands to public
respondent PEA transformed such lands of the public domain to private lands." This theory is
echoed by AMARI which maintains that the "issuance of the special patent leading to the
eventual issuance of title takes the subject land away from the land of public domain and
converts the property into patrimonial or private property." In short, PEA and AMARI contend
that with the issuance of Special Patent No. 3517 and the corresponding certificates of titles, the
157.84 hectares comprising the Freedom Islands have become private lands of PEA. In support
of their theory, PEA and AMARI cite the following rulings of the Court:
1. Sumail v. Judge of CFI of Cotabato,
97
where the Court held
"Once the patent was granted and the corresponding certificate of title was issued, the
land ceased to be part of the public domain and became private property over which the
Director of Lands has neither control nor jurisdiction."
2. Lee Hong Hok v. David,
98
where the Court declared -
"After the registration and issuance of the certificate and duplicate certificate of title
based on a public land patent, the land covered thereby automatically comes under the
operation of Republic Act 496 subject to all the safeguards provided therein."3. Heirs of
Gregorio Tengco v. Heirs of Jose Aliwalas,
99
where the Court ruled -
"While the Director of Lands has the power to review homestead patents, he may do so
only so long as the land remains part of the public domain and continues to be under his
exclusive control; but once the patent is registered and a certificate of title is issued, the
land ceases to be part of the public domain and becomes private property over which
the Director of Lands has neither control nor jurisdiction."
4. Manalo v. Intermediate Appellate Court,
100
where the Court held
"When the lots in dispute were certified as disposable on May 19, 1971, and free patents
were issued covering the same in favor of the private respondents, the said lots ceased
to be part of the public domain and, therefore, the Director of Lands lost jurisdiction over
the same."
5.Republic v. Court of Appeals,
101
where the Court stated
"Proclamation No. 350, dated October 9, 1956, of President Magsaysay legally effected
a land grant to the Mindanao Medical Center, Bureau of Medical Services, Department
of Health, of the whole lot, validly sufficient for initial registration under the Land
Registration Act. Such land grant is constitutive of a 'fee simple' title or absolute title in
favor of petitioner Mindanao Medical Center. Thus, Section 122 of the Act, which
governs the registration of grants or patents involving public lands, provides that
'Whenever public lands in the Philippine Islands belonging to the Government of the
United States or to the Government of the Philippines are alienated, granted or
conveyed to persons or to public or private corporations, the same shall be brought
forthwith under the operation of this Act (Land Registration Act, Act 496) and shall
become registered lands.'"
The first four cases cited involve petitions to cancel the land patents and the corresponding
certificates of titles issued to private parties. These four cases uniformly hold that the Director
of Lands has no jurisdiction over private lands or that upon issuance of the certificate of title the
land automatically comes under the Torrens System. The fifth case cited involves the
registration under the Torrens System of a 12.8-hectare public land granted by the National
Government to Mindanao Medical Center, a government unit under the Department of Health.
The National Government transferred the 12.8-hectare public land to serve as the site for the
hospital buildings and other facilities of Mindanao Medical Center, which performed a public
service. The Court affirmed the registration of the 12.8-hectare public land in the name of
Mindanao Medical Center under Section 122 of Act No. 496. This fifth case is an example of a
public land being registered under Act No. 496 without the land losing its character as a
property of public dominion.
In the instant case, the only patent and certificates of title issued are those in the name of PEA,
a wholly government owned corporation performing public as well as proprietary functions. No
patent or certificate of title has been issued to any private party. No one is asking the Director of
Lands to cancel PEA's patent or certificates of title. In fact, the thrust of the instant petition is
that PEA's certificates of title should remain with PEA, and the land covered by these
certificates, being alienable lands of the public domain, should not be sold to a private
corporation.
Registration of land under Act No. 496 or PD No. 1529 does not vest in the registrant private or
public ownership of the land. Registration is not a mode of acquiring ownership but is merely
evidence of ownership previously conferred by any of the recognized modes of acquiring
ownership. Registration does not give the registrant a better right than what the registrant had
prior to the registration.
102
The registration of lands of the public domain under the Torrens
system, by itself, cannot convert public lands into private lands.
103

Jurisprudence holding that upon the grant of the patent or issuance of the certificate of title the
alienable land of the public domain automatically becomes private land cannot apply to
government units and entities like PEA. The transfer of the Freedom Islands to PEA was made
subject to the provisions of CA No. 141 as expressly stated in Special Patent No. 3517 issued
by then President Aquino, to wit:
"NOW, THEREFORE, KNOW YE, that by authority of the Constitution of the Philippines
and in conformity with the provisions of Presidential Decree No. 1084, supplemented
by Commonwealth Act No. 141, as amended, there are hereby granted and conveyed
unto the Public Estates Authority the aforesaid tracts of land containing a total area of
one million nine hundred fifteen thousand eight hundred ninety four (1,915,894) square
meters; the technical description of which are hereto attached and made an integral part
hereof." (Emphasis supplied)
Thus, the provisions of CA No. 141 apply to the Freedom Islands on matters not covered by PD
No. 1084. Section 60 of CA No. 141 prohibits, "except when authorized by Congress," the sale
of alienable lands of the public domain that are transferred to government units or entities.
Section 60 of CA No. 141 constitutes, under Section 44 of PD No. 1529, a "statutory lien
affecting title" of the registered land even if not annotated on the certificate of title.
104
Alienable
lands of the public domain held by government entities under Section 60 of CA No. 141 remain
public lands because they cannot be alienated or encumbered unless Congress passes a law
authorizing their disposition. Congress, however, cannot authorize the sale to private
corporations of reclaimed alienable lands of the public domain because of the constitutional
ban. Only individuals can benefit from such law.
The grant of legislative authority to sell public lands in accordance with Section 60 of CA No.
141 does not automatically convert alienable lands of the public domain into private or
patrimonial lands. The alienable lands of the public domain must be transferred to qualified
private parties, or to government entities not tasked to dispose of public lands, before these
lands can become private or patrimonial lands. Otherwise, the constitutional ban will become
illusory if Congress can declare lands of the public domain as private or patrimonial lands in the
hands of a government agency tasked to dispose of public lands. This will allow private
corporations to acquire directly from government agencies limitless areas of lands which, prior
to such law, are concededly public lands.
Under EO No. 525, PEA became the central implementing agency of the National
Government to reclaim foreshore and submerged areas of the public domain. Thus, EO No. 525
declares that
"EXECUTIVE ORDER NO. 525
Designating the Public Estates Authority as the Agency Primarily Responsible for all
Reclamation Projects
Whereas, there are several reclamation projects which are ongoing or being proposed to
be undertaken in various parts of the country which need to be evaluated for consistency
with national programs;
Whereas, there is a need to give further institutional support to the Government's
declared policy to provide for a coordinated, economical and efficient reclamation of
lands;
Whereas, Presidential Decree No. 3-A requires that all reclamation of areas shall be
limited to the National Government or any person authorized by it under proper contract;
Whereas, a central authority is needed to act on behalf of the National
Government which shall ensure a coordinated and integrated approach in the
reclamation of lands;
Whereas, Presidential Decree No. 1084 creates the Public Estates Authority as a
government corporation to undertake reclamation of lands and ensure their
maximum utilization in promoting public welfare and interests; and
Whereas, Presidential Decree No. 1416 provides the President with continuing authority
to reorganize the national government including the transfer, abolition, or merger of
functions and offices.
NOW, THEREFORE, I, FERDINAND E. MARCOS, President of the Philippines, by
virtue of the powers vested in me by the Constitution and pursuant to Presidential
Decree No. 1416, do hereby order and direct the following:
Section 1. The Public Estates Authority (PEA) shall be primarily responsible for
integrating, directing, and coordinating all reclamation projects for and on behalf
of the National Government. All reclamation projects shall be approved by the
President upon recommendation of the PEA, and shall be undertaken by the PEA or
through a proper contract executed by it with any person or entity; Provided, that,
reclamation projects of any national government agency or entity authorized under its
charter shall be undertaken in consultation with the PEA upon approval of the President.
x x x ."
As the central implementing agency tasked to undertake reclamation projects nationwide, with
authority to sell reclaimed lands, PEA took the place of DENR as the government agency
charged with leasing or selling reclaimed lands of the public domain. The reclaimed lands being
leased or sold by PEA are not private lands, in the same manner that DENR, when it disposes
of other alienable lands, does not dispose of private lands but alienable lands of the public
domain. Only when qualified private parties acquire these lands will the lands become private
lands. In the hands of the government agency tasked and authorized to dispose of
alienable of disposable lands of the public domain, these lands are still public, not
private lands.
Furthermore, PEA's charter expressly states that PEA "shall hold lands of the public domain"
as well as "any and all kinds of lands." PEA can hold both lands of the public domain and
private lands. Thus, the mere fact that alienable lands of the public domain like the Freedom
Islands are transferred to PEA and issued land patents or certificates of title in PEA's name
does not automatically make such lands private.
To allow vast areas of reclaimed lands of the public domain to be transferred to PEA as private
lands will sanction a gross violation of the constitutional ban on private corporations from
acquiring any kind of alienable land of the public domain. PEA will simply turn around, as PEA
has now done under the Amended J VA, and transfer several hundreds of hectares of these
reclaimed and still to be reclaimed lands to a single private corporation in only one transaction.
This scheme will effectively nullify the constitutional ban in Section 3, Article XII of the 1987
Constitution which was intended to diffuse equitably the ownership of alienable lands of the
public domain among Filipinos, now numbering over 80 million strong.
This scheme, if allowed, can even be applied to alienable agricultural lands of the public domain
since PEA can "acquire x x x any and all kinds of lands." This will open the floodgates to
corporations and even individuals acquiring hundreds of hectares of alienable lands of the
public domain under the guise that in the hands of PEA these lands are private lands. This will
result in corporations amassing huge landholdings never before seen in this country - creating
the very evil that the constitutional ban was designed to prevent. This will completely reverse
the clear direction of constitutional development in this country. The 1935 Constitution allowed
private corporations to acquire not more than 1,024 hectares of public lands.
105
The 1973
Constitution prohibited private corporations from acquiring any kind of public land, and the 1987
Constitution has unequivocally reiterated this prohibition.
The contention of PEA and AMARI that public lands, once registered under Act No. 496 or PD
No. 1529, automatically become private lands is contrary to existing laws. Several laws
authorize lands of the public domain to be registered under the Torrens System or Act No. 496,
now PD No. 1529, without losing their character as public lands. Section 122 of Act No. 496,
and Section 103 of PD No. 1529, respectively, provide as follows:
Act No. 496
"Sec. 122. Whenever public lands in the Philippine Islands belonging to the x x x
Government of the Philippine Islands are alienated, granted, or conveyed to persons or
the public or private corporations, the same shall be brought forthwith under the
operation of this Act and shall become registered lands."
PD No. 1529
"Sec. 103. Certificate of Title to Patents. Whenever public land is by the Government
alienated, granted or conveyed to any person, the same shall be brought forthwith
under the operation of this Decree." (Emphasis supplied)
Based on its legislative history, the phrase "conveyed to any person" in Section 103 of PD No.
1529 includes conveyances of public lands to public corporations.
Alienable lands of the public domain "granted, donated, or transferred to a province,
municipality, or branch or subdivision of the Government," as provided in Section 60 of CA No.
141, may be registered under the Torrens System pursuant to Section 103 of PD No. 1529.
Such registration, however, is expressly subject to the condition in Section 60 of CA No. 141
that the land "shall not be alienated, encumbered or otherwise disposed of in a manner
affecting its title, except when authorized by Congress." This provision refers to government
reclaimed, foreshore and marshy lands of the public domain that have been titled but still cannot
be alienated or encumbered unless expressly authorized by Congress. The need for legislative
authority prevents the registered land of the public domain from becoming private land that can
be disposed of to qualified private parties.
The Revised Administrative Code of 1987 also recognizes that lands of the public domain may
be registered under the Torrens System. Section 48, Chapter 12, Book I of the Code states
"Sec. 48. Official Authorized to Convey Real Property. Whenever real property of the
Government is authorized by law to be conveyed, the deed of conveyance shall be
executed in behalf of the government by the following:
(1) x x x
(2) For property belonging to the Republic of the Philippines, but titled in the name
of any political subdivision or of any corporate agency or instrumentality, by the
executive head of the agency or instrumentality." (Emphasis supplied)
Thus, private property purchased by the National Government for expansion of a public wharf
may be titled in the name of a government corporation regulating port operations in the country.
Private property purchased by the National Government for expansion of an airport may also be
titled in the name of the government agency tasked to administer the airport. Private property
donated to a municipality for use as a town plaza or public school site may likewise be titled in
the name of the municipality.
106
All these properties become properties of the public domain,
and if already registered under Act No. 496 or PD No. 1529, remain registered land. There is no
requirement or provision in any existing law for the de-registration of land from the Torrens
System.
Private lands taken by the Government for public use under its power of eminent domain
become unquestionably part of the public domain. Nevertheless, Section 85 of PD No. 1529
authorizes the Register of Deeds to issue in the name of the National Government new
certificates of title covering such expropriated lands. Section 85 of PD No. 1529 states
"Sec. 85. Land taken by eminent domain. Whenever any registered land, or interest
therein, is expropriated or taken by eminent domain, the National Government, province,
city or municipality, or any other agency or instrumentality exercising such right shall file
for registration in the proper Registry a certified copy of the judgment which shall state
definitely by an adequate description, the particular property or interest expropriated, the
number of the certificate of title, and the nature of the public use. A memorandum of the
right or interest taken shall be made on each certificate of title by the Register of Deeds,
and where the fee simple is taken, a new certificate shall be issued in favor of the
National Government, province, city, municipality, or any other agency or
instrumentality exercising such right for the land so taken. The legal expenses incident to
the memorandum of registration or issuance of a new certificate of title shall be for the
account of the authority taking the land or interest therein." (Emphasis supplied)
Consequently, lands registered under Act No. 496 or PD No. 1529 are not exclusively private or
patrimonial lands. Lands of the public domain may also be registered pursuant to existing laws.
AMARI makes a parting shot that the Amended JVA is not a sale to AMARI of the Freedom
Islands or of the lands to be reclaimed from submerged areas of Manila Bay. In the words of
AMARI, the Amended JVA "is not a sale but a joint venture with a stipulation for reimbursement
of the original cost incurred by PEA for the earlier reclamation and construction works
performed by the CDCP under its 1973 contract with the Republic." Whether the Amended JVA
is a sale or a joint venture, the fact remains that the Amended JVA requires PEA to "cause the
issuance and delivery of the certificates of title conveying AMARI's Land Share in the name of
AMARI."
107

This stipulation still contravenes Section 3, Article XII of the 1987 Constitution which provides
that private corporations "shall not hold such alienable lands of the public domain except by
lease." The transfer of title and ownership to AMARI clearly means that AMARI will "hold" the
reclaimed lands other than by lease. The transfer of title and ownership is a "disposition" of the
reclaimed lands, a transaction considered a sale or alienation under CA No. 141,
108
the
Government Auditing Code,
109
and Section 3, Article XII of the 1987 Constitution.
The Regalian doctrine is deeply implanted in our legal system. Foreshore and submerged areas
form part of the public domain and are inalienable. Lands reclaimed from foreshore and
submerged areas also form part of the public domain and are also inalienable, unless converted
pursuant to law into alienable or disposable lands of the public domain. Historically, lands
reclaimed by the government are sui generis, not available for sale to private parties unlike
other alienable public lands. Reclaimed lands retain their inherent potential as areas for public
use or public service. Alienable lands of the public domain, increasingly becoming scarce
natural resources, are to be distributed equitably among our ever-growing population. To insure
such equitable distribution, the 1973 and 1987 Constitutions have barred private corporations
from acquiring any kind of alienable land of the public domain. Those who attempt to dispose of
inalienable natural resources of the State, or seek to circumvent the constitutional ban on
alienation of lands of the public domain to private corporations, do so at their own risk.
We can now summarize our conclusions as follows:
1. The 157.84 hectares of reclaimed lands comprising the Freedom Islands, now
covered by certificates of title in the name of PEA, are alienable lands of the public
domain. PEA may lease these lands to private corporations but may not sell or transfer
ownership of these lands to private corporations. PEA may only sell these lands to
Philippine citizens, subject to the ownership limitations in the 1987 Constitution and
existing laws.
2. The 592.15 hectares of submerged areas of Manila Bay remain inalienable natural
resources of the public domain until classified as alienable or disposable lands open to
disposition and declared no longer needed for public service. The government can make
such classification and declaration only after PEA has reclaimed these submerged
areas. Only then can these lands qualify as agricultural lands of the public domain,
which are the only natural resources the government can alienate. In their present state,
the 592.15 hectares of submerged areas are inalienable and outside the commerce
of man.
3. Since the Amended JVA seeks to transfer to AMARI, a private corporation, ownership
of 77.34 hectares
110
of the Freedom Islands, such transfer is void for being contrary to
Section 3, Article XII of the 1987 Constitution which prohibits private corporations from
acquiring any kind of alienable land of the public domain.
4. Since the Amended JVA also seeks to transfer to AMARI ownership of 290.156
hectares
111
of still submerged areas of Manila Bay, such transfer is void for being
contrary to Section 2, Article XII of the 1987 Constitution which prohibits the alienation of
natural resources other than agricultural lands of the public domain. PEA may reclaim
these submerged areas. Thereafter, the government can classify the reclaimed lands as
alienable or disposable, and further declare them no longer needed for public service.
Still, the transfer of such reclaimed alienable lands of the public domain to AMARI will be
void in view of Section 3, Article XII of the 1987 Constitution which prohibits private
corporations from acquiring any kind of alienable land of the public domain.
Clearly, the Amended JVA violates glaringly Sections 2 and 3, Article XII of the 1987
Constitution. Under Article 1409
112
of the Civil Code, contracts whose "object or purpose is
contrary to law," or whose "object is outside the commerce of men," are "inexistent and void
from the beginning." The Court must perform its duty to defend and uphold the Constitution, and
therefore declares the Amended J VA null and void ab initio.
Seventh issue: whether the Court is the proper forum to raise the issue of whether the
Amended J VA is grossly disadvantageous to the government.
Considering that the Amended JVA is null and void ab initio, there is no necessity to rule on this
last issue. Besides, the Court is not a trier of facts, and this last issue involves a determination
of factual matters.
WHEREFORE, the petition is GRANTED. The Public Estates Authority and Amari Coastal Bay
Development Corporation are PERMANENTLY ENJOINED from implementing the Amended
Joint Venture Agreement which is hereby declaredNULL and VOID ab initio.
SO ORDERED.
Davide, Jr., C.J., Bellosillo, Puno, Vitug, Kapunan, Mendoza, Panganiban, Quisumbing, Ynares-
Santiago, Sandoval-Gutierrez, Austria-Martinez, and Corona, JJ., concur.


Footnote
1
Section 4 of PD No. 1084.
2
PEA's Memorandum dated August 4, 1999, p. 3.
3
PEA's Memorandum, supra note 2 at 7. PEA's Memorandum quoted extensively, in its
Statement of Facts and the Case, the Statement of Facts in Senate Committee Report
No. 560 dated September 16, 1997.
4
In Opinion No. 330 dated December 23, 1994, the Government Corporate Counsel,
citing COA Audit Circular No. 89-296, advised PEA that PEA could negotiate the sale of
the 157.84-hectare Freedom Islands in view of the failure of the public bidding held on
December 10, 1991 where there was not a single bidder. See also Senate Committee
Report No. 560, p. 12.
5
PEA's Memorandum, supra note 2 at 9.
6
Ibid.
7
The existence of this report is a matter of judicial notice pursuant to Section 1, Rule
129 of the Rules of Court which provides, "A court shall take judicial notice, without the
introduction of evidence, of x x x the official acts of the legislature x x x."
8
Teofisto Guingona, Jr.
9
Renato Cayetano.
10
Virgilio C. Abejo.
11
Report and Recommendation of the Legal Task Force, Annex "C", AMARI's
Memorandum dated June 19, 1999.
12
AMARI's Comment dated June 24, 1998, p. 3; Rollo, p. 68.
13
AMARI filed three motions for extension of time to file comment (Rollo, pp. 32, 38, 48),
while PEA filed nine motions for extension of time (Rollo, pp. 127, 139).
14
Petitioner's Memorandum dated July 6, 1999, p. 42.
15
Represented by the Office of the Solicitor General, with Solicitor General Ricardo P.
Galvez, Assistant Solicitor General Azucena R. Balanon-Corpuz, and Associate Solicitor
Raymund I. Rigodon signing PEA's Memorandum.
16
Represented by Azcuna Yorac Arroyo & Chua Law Offices, and Romulo Mabanta
Sayoc & De los Angeles Law Offices.
17
Salonga v. Pao, 134 SCRA 438 (1985); Gonzales v. Marcos, 65 SCRA 624 (1975 );
Aquino v. Enrile, 59 SCRA 183 (1974 ); Dela Camara v. Enage, 41 SCRA 1 (1971 ).
18
Section 11, Article XIV.
19
Manila Electric Co. v. Judge F. Castro-Bartolome, 114 SCRA 799 (1982); Republic v.
CA and Iglesia, and Republic v. Cendana and Iglesia ni Cristo, 119 SCRA 449 (1982);
Republic v. Villanueva and Iglesia ni Cristo, 114 SCRA 875 (1982); Director of Lands v.
Lood, 124 SCRA 460 (1983); Republic v. Iglesia ni Cristo, 128 SCRA 44 (1984); Director
of Lands v. Hermanos y Hermanas de Sta. Cruz de Mayo, Inc., 141 SCRA 21 (1986);
Director of Lands v. IAC and Acme Plywood & Veneer Co., 146 SCRA 509 (1986);
Republic v. IAC and Roman Catholic Bishop of Lucena, 168 SCRA 165 (1988);
Natividad v. CA, 202 SCRA 493 (1991); Villaflor v. CA and Nasipit Lumber Co., 280
SCRA 297 (1997). In Ayog v. Cusi, 118 SCRA 492 (1982), the Court did not apply the
constitutional ban in the 1973 Constitution because the applicant corporation, Bian
Development Co., Inc., had fully complied with all its obligations and even paid the full
purchase price before the effectivity of the 1973 Constitution, although the sales patent
was issued after the 1973 Constitution took effect.
20
PD No. 1073.
21
Annex "B", AMARI's Memorandum dated June 19, 1999, Section 5.2 (c) and (e) of the
Amended JVA, pp. 16-17.
22
Chavez v. PCGG, 299 SCRA 744 (1998).
23
136 SCRA 27 (1985).
24
Article 2 of the Civil Code (prior to its amendment by EO No. 200) provided as follows:
"Laws shall take effect after fifteen days following the completion of their publication in
the Official Gazette, unless it is provided otherwise, x x x."
25
Section 1 of CA No. 638 provides as follows: "There shall be published in the Official
Gazette all important legislative acts and resolutions of the Congress of the Philippines;
all executive and administrative orders and proclamations, except such as have no
general applicability; x x x."
26
Section 79 of the Government Auditing Codes provides as follows: "When
government property has become unserviceable for any cause, or is no longer
needed, it shall, upon application of the officer accountable therefor, be inspected by the
head of the agency or his duly authorized representative in the presence of the auditor
concerned and, if found to be valueless or unsaleable, it may be destroyed in their
presence. If found to be valuable, it may be sold at public auction to the highest
bidder under the supervision of the proper committee on award or similar body in the
presence of the auditor concerned or other authorized representative of the
Commission, after advertising by printed notice in the Official Gazette, or for not
less than three consecutive days in any newspaper of general circulation, or where
the value of the property does not warrant the expense of publication, by notices posted
for a like period in at least three public places in the locality where the property is to be
sold. In the event that the public auction fails, the property may be sold at a private
sale at such price as may be fixed by the same committee or body concerned and
approved by the Commission."
27
Paat v. Court of Appeals, 266 SCRA 167 (1997); Quisumbing v. Judge Gumban, 193
SCRA 520 (1991); Valmonte v. Belmonte, Jr., 170 SCRA 256 (1989).
28
See note 22.
29
Section 1, Article XI of the 1987 Constitution states as follows: "Public office is a public
trust. Public officers and employees must at all times be accountable to the people,
serve them with utmost responsibility, integrity, loyalty, and efficiency, act with patriotism
and justice, and lead modest lives."
30
170 SCRA 256 (1989).
31
See note 22.
32
Record of the Constitutional Commission, Vol. V, pp. 24-25, (1986).
33
Supra, Note 22.
34
Ibid.
35
Legaspi v. Civil Service Commission, 150 SCRA 530 (1987).
36
Almonte v. Vasquez, 244 SCRA 286 (1995).
37
See Note 22.
38
Chavez v. PCGG, see note 22; Aquino-Sarmiento v. Morato, 203 SCRA 515 (1991).
39
Almonte v. Vasquez, see note 36.
40
People's Movement for Press Freedom, et al. v. Hon. Raul Manglapus, G.R. No.
84642, En Banc Resolution dated April 13, 1988; Chavez v. PCGG, see note 22.
41
Section 270 of the National Internal Revenue Code punishes any officer or employee
of the Bureau of Internal Revenue who divulges to any person, except as allowed by
law, information regarding the business, income, or estate of any taxpayer, the secrets,
operation, style of work, or apparatus of any manufacturer or producer, or confidential
information regarding the business of any taxpayer, knowledge of which was acquired by
him in the discharge of his official duties. Section 14 of R.A. No. 8800 (Safeguard
Measures Act) prohibits the release to the public of confidential information submitted in
evidence to the Tariff Commission. Section 3 (n) of R.A. No. 8504 (Philippine AIDS
Prevention and Control Act) classifies as confidential the medical records of HIV
patients. Section 6 (j) of R.A. No. 8043 (Inter-Country Adoption Act) classifies as
confidential the records of the adopted child, adopting parents, and natural parents.
Section 94 (f) of R.A. No. 7942 (Philippine Mining Act) requires the Department of
Environment and Natural Resources to maintain the confidentiality of confidential
information supplied by contractors who are parties to mineral agreements or financial
and technical assistance agreements.
42
The Recopilacion de Leyes de las Indias declared that: "We, having acquired full
sovereignty over the Indies, and all lands, territories, and possessions not heretofore
ceded away by our royal predecessors, or by us, or in our name, still pertaining to the
royal crown and patrimony, it is our will that all lands which are held without proper and
true deeds of grant be restored to us according as they belong to us, in order that after
reserving before all what to us or to our viceroys, audiencias, and governors may seem
necessary for public squares, ways, pastures, and commons in those places which are
peopled, taking into consideration not only their present condition, but also their future
and their probable increase, and after distributing to the natives what may be necessary
for tillage and pasturage, confirming them in what they now have and giving them more if
necessary, all the rest of said lands may remain free and unencumbered for us to
dispose of as we may wish." See concurring opinion of Justice Reynato S. Puno in
Republic Real Estate Corporation v. Court of Appeals, 299 SCRA 199 (1998).
43
Cario v. Insular Government, 41 Phil. 935 (1909). The exception mentioned
in Cario, referring to lands in the possession of an occupant and of his predecessors-in-
interest, since time immemorial, is actually a species of a grant by the State. The United
States Supreme Court, speaking through Justice Oliver Wendell Holmes, Jr., declared
in Cario: "Prescription is mentioned again in the royal cedula of October 15, 1754, cited
in 3 Philippine, 546; 'Where such possessors shall not be able to produce title deeds, it
shall be sufficient if they shall show that ancient possession, as a valid title by
prescription.' It may be that this means possession from before 1700; but, at all events,
the principle is admitted. As prescription, even against the Crown lands, was recognized
by the laws of Spain, we see no sufficient reason for hesitating to admit that it was
recognized in the Philippines in regard to lands over which Spain had only a paper
sovereignty." See also Republic v. Lee, 197 SCRA 13 (1991).
44
Article 1 of the Spanish Law of Waters of 1866.
45
Ignacio v. Director of Lands, 108 Phil. 335 (1960); Joven v. Director of Lands, 93 Phil.
134 (1953); Laurel v. Garcia, 187 SCRA 797 (1990). See concurring opinion of Justice
Reynato S. Puno in Republic Real Estate Corporation v. Court of Appeals, 299 SCRA
199 (1998).
46
Act No. 926, enacted on October 7, 1903, was also titled the Public Land Act. This
Act, however, did not cover reclaimed lands. Nevertheless, Section 23 of this Act
provided as follows: "x x x In no case may lands leased under the provisions of this
chapter be taken so as to gain control of adjacent land, water, stream, shore line, way,
roadstead, or other valuable right which in the opinion of the Chief of the Bureau of
Public Lands would be prejudicial to the interests of the public."
47
Section 10 of Act No. 2874 provided as follows: "The words "alienation," "disposition,"
or "concession" as used in this Act, shall mean any of the methods authorized by this
Act for the acquisition, lease, use, or benefit of the lands of the public domain other than
timber or mineral lands."
48
Title II of Act No. 2874 governed alienable lands of the public domain for agricultural
purposes, while Title III of the same Act governed alienable lands of the public domain
for non-agricultural purposes.
49
Section 57 of Act No. 2874 provided as follows: "x x x; but the land so granted,
donated, or transferred to a province, municipality, or branch or subdivision of the
Government shall not be alienated, encumbered, or otherwise disposed of in a manner
affecting its title, except when authorized by the legislature; x x x."
50
Krivenko v. Register of Deeds, 79 Phil. 461 (1947).
51
Section 2 of CA No. 141 states as follows: "The provisions of this Act shall apply to the
lands of the public domain; but timber and mineral lands shall be governed by special
laws and nothing in this Act provided shall be understood or construed to change or
modify the administration and disposition of the lands commonly called "friar lands" and
those which, being privately owned, have reverted to or become the property of the
Commonwealth of the Philippines, which administration and disposition shall be
governed by the laws at present in force or which may hereafter be enacted."
52
Like Act No. 2874, Section 10 of CA No. 141 defined the terms "alienation" and
"disposition" as follows: "The words "alienation," "disposition," or "concession" as used in
this Act, shall mean any of the methods authorized by this Act for the acquisition, lease,
use, or benefit of the lands of the public domain other than timber or mineral lands."
53
R.A. No. 6657 has suspended the authority of the President to reclassify forest or
mineral lands into agricultural lands. Section 4 (a) of RA No. 6657 (Comprehensive
Agrarian Reform Law of 1988) states, "No reclassification of forest or mineral lands to
agricultural lands shall be undertaken after the approval of this Act until Congress, taking
into account ecological, developmental and equity considerations, shall have delimited
by law, the specific limits of the public domain."
54
Covering Sections 58 to 68 of CA No. 141.
55
299 SCRA 199 (1998).
56
Section 1, Article XIII of the 1935 Constitution limited the disposition and utilization of
public agricultural lands to Philippine citizens or to corporations at least sixty percent
owned by Philippine citizens. This was, however, subject to the original Ordinance
appended to the 1935 Constitution stating, among others, that until the withdrawal of
United States sovereignty in the Philippines, "Citizens and corporations of the United
States shall enjoy in the Commonwealth of the Philippines all the civil rights of the
citizens and corporations, respectively, thereof."
57
Section 44 of PD No. 1529 (previously Section 39 of Act No. 496) provides that "liens,
claims or rights arising or existing under the laws and the Constitution of the Philippines
which are not by law required to appear of record in the Registry of Deeds in order to be
valid against subsequent purchasers or encumbrancers of record" constitute statutory
liens affecting the title.1wphi1.nt
58
RA No. 730, which took effect on June 18, 1952, authorized the private sale of home
lots to actual occupants of public lands not needed for public service. Section 1 of RA
No. 730 provided as follows: "Notwithstanding the provisions of Sections 61 and 67 of
Commonwealth Act No. 141, as amended by RA No. 293, any Filipino citizen of legal
age who is not the owner of a home lot in the municipality or city in which he resides and
who had in good faith established his residence on a parcel of land of the Republic of the
Philippines which is not needed for public service, shall be given preference to purchase
at a private sale of which reasonable notice shall be given to him, not more than one
thousand square meters at a price to be fixed by the Director of Lands with the approval
of the Secretary of Agriculture and Natural Resources. x x x." In addition, on June 16,
1948, Congress enacted R.A. No. 293 allowing the private sale of marshy alienable or
disposable lands of the public domain to lessees who have improved and utilized the
same as farms, fishponds or other similar purposes for at least five years from the date
of the lease contract with the government. R.A. No. 293, however, did not apply to
marshy lands under Section 56 (c), Title III of CA No. 141 which refers to marshy lands
leased for residential, commercial, industrial or other non-agricultural purposes.
59
See note 49.
60
See note 60.
61
Republic Real Estate Corporation v. Court of Appeals, see note 56.
62
Ibid.
63
Insular Government v. Aldecoa, 19 Phil. 505 (1911); Government v. Cabangis, 53 Phil.
112 (1929).
64
118 SCRA 492 (1982).
65
Annex "B", AMARI's Memorandum, see note 2 at 1 & 2.
66
PEA's Memorandum, see note 6.
67
Ibid., p. 44.
68
See notes 9, 10 & 11.
69
Annex "C", p. 3, AMARI's Memorandum, see note 12 at 3.
70
This should read Article XII.
71
Section 8 of CA No. 141.
72
Emphasis supplied.
73
187 SCRA 797 (1990).
74
Article 422 of the Civil Code states as follows: "Property of public dominion, when no
longer needed for public use or public service, shall form part of the patrimonial property
of the State."
75
AMARI's Comment dated June 24, 1998, p. 20; Rollo, p. 85.
76
Dizon v. Rodriguez, 13 SCRA 705 (1965); Republic v. Lat Vda. de Castillo, 163 SCRA
286 (1988).
77
Cario v. Insular Government, 41 Phil. 935 (1909).
78
Proclamation No. 41, issued by President Ramon Magsaysay on July 5, 1954,
reserved for "National Park purposes" 464.66 hectares of the public domain in Manila
Bay "situated in the cities of Manila and Pasay and the municipality of Paranaque,
Province of Rizal, Island of Luzon," which area, as described in detail in the
Proclamation, is "B]ounded on the North, by Manila Bay; on the East, by Dewey
Boulevard; and on the south and west, by Manila Bay." See concurring opinion of Justice
Reynato S. Puno in Republic Real Estate Corporation v. Court of Appeals, 299 SCRA
1999 (1998). Under Sections 2 and 3, Article XII of the 1987 Constitution, "national
parks" are inalienable natural resources of the State.
79
Fifth Whereas clause of EO No. 525.
80
Section 4, Chapter I, Title XIV, Book IV.
81
Section 6 of CA No 141 provides as follows: "The President, upon the
recommendation of the Secretary of Agriculture and Commerce, shall from time to
time classify the lands of the public domain into (a) Alienable or disposable, x x x."
82
Section 7 of CA No. 141 provides as follows: "For purposes of the administration and
disposition of alienable or disposable public lands, the President, upon
recommendation by the Secretary of Agriculture and Commerce, shall from time to
time declare what lands are open to disposition or concession under this Act."
83
On "Lands for Residential, Commercial, or Industrial and other Similar Purposes."
84
RA No. 293, enacted on June 16, 1948, authorized the sale of marshy lands under
certain conditions. Section 1 of RA No. 293 provided as follows: "The provisions of
section sixty-one of Commonwealth Act Numbered One hundred and forty-one to the
contrary notwithstanding, marshy lands and lands under water bordering on shores or
banks or navigable lakes or rivers which are covered by subsisting leases or leases
which may hereafter be duly granted under the provisions of the said Act and are
already improved and have been utilized for farming, fishpond, or similar purposes for at
least five years from the date of the contract of lease, may be sold to the lessees thereof
under the provisions of Chapter Five of the said Act as soon as the President, upon
recommendation of the Secretary of Agriculture and Natural Resources, shall declare
that the same are not necessary for the public service."
85
PEA's Memorandum, see note 2 at 45.
86
See note 73.
87
Section 4 (b) of PD No. 1084
88
R.A. No. 730 allows the private sale of home lots to actual occupants of public
lands. See note 63.
89
Issued on February 26, 1981.
90
While PEA claims there was a failure of public bidding on December 10, 1991, there is
no showing that the Commission on Audit approved the price or consideration stipulated
in the negotiated Amended JVA as required by Section 79 of the Government Auditing
Code. Senate Committee Report No. 560 did not discuss this issue.
91
Paragraph 2 (a) of COA Circular No. 89-296, on "Sale Thru Negotiation," states that
disposal through negotiated sale may be resorted to if "[T]here was a failure of public
auction."
92
Senate Committee Report No. 560, Statement of Facts, p. 7, citing PEA Board
Resolution No. 835, as appearing in the Minutes of the PEA Board of Directors Meeting
held on May 30, 1991, per Certification of Jaime T. De Veyra, Corporate Secretary,
dated June 11, 1991.
93
Opinion No. 330, citing COA Audit Circular No. 89-296. See note 5.
94
PEA's Memorandum, see note 2.
95
Senate Committee Report No. 560, pp. 7-8, citing the Minutes of Meeting of the PEA
Board of Directors held on December 19, 1991.
96
Section 3, Article XII of the 1987 Constitution provides as follows: "x x x Citizens of the
Philippines may x x x acquire not more than twelve hectares thereof by purchase,
homestead or grant." However, Section 6 of R.A. No. 6657 (Comprehensive Agrarian
Reform Law) limits the ownership of "public or private agricultural land" to a maximum of
five hectares per person.
97
96 Phil. 946 (1955).
98
48 SCRA 372 (1977).
99
168 SCRA 198 (1988).
100
172 SCRA 795 (1989).
101
73 SCRA 146 (1976).
102
Avila v. Tapucar, 201 SCRA 148 (1991).
103
Republic v. Ayala Cia, et al., 14 SCRA 259 (1965); Dizon v. Rodriguez, 13 SCRA 705
(1965).
104
Section 44 of PD No. 1529 states as follows: "Every registered owner receiving a
certificate of title in pursuance of a decree of registration, and every subsequent
purchaser of registered land taking a certificate of title for value and in good faith, shall
hold the same free from all encumbrances except those noted on said certificate and
any of the following encumbrances which may be subsisting, namely: First. Liens,
claims or rights arising or existing under the laws and Constitution of the
Philippines which are not by law required to appear of record in the Registry of
Deeds in order to be valid against subsequent purchasers or encumbrancers of
record. x x x." Under Section 103 of PD No. 1529, Section 44 applies to certificates of
title issued pursuant to a land patent granted by the government.
105
Section 2, Article XIII of the 1935 Constitution.
106
Harty v. Municipality of Victoria, 13 Phil. 152 (1909).
107
Annex "B", AMARI's Memorandum, see note 21 at 16, Section 5.2 (c) of the
Amended JVA.
108
Section 10 of CA No. 141 provides as follows: "Sec. 10. The words "alienation,"
"disposition," or "concession" as used in this Act, shall mean any of the methods
authorized by this Act for the acquisition, lease, use, or benefit of the lands of the
public domain other than timber or mineral lands."
109
Section 79 of the Government Auditing Code, which requires public auction in the
sale of government assets, includes all kinds of disposal or divestment of government
assets. Thus, COA Audit Circular No. 86-264 dated October 16, 1986 speaks of
"guidelines (which) shall govern the general procedures on the divestment or disposal
of assets of government-owned and/or controlled corporations and their
subsidiaries." Likewise, COA Audit Circular No. 89-296 dated January 27, speaks of
"guidelines (which) shall be observed and adhered to in the divestment or disposal of
property and other assets of all government entities/instrumentalities" and that
"divestment shall refer to the manner or scheme of taking away, depriving, withdrawing
of an authority, power or title." These COA Circulars implement Section 79 of the
Government Auditing Code.
110
The share of AMARI in the Freedom Islands is 77.34 hectares, which is 70 percent of
the net usable area of 110.49 hectares. The net usable area is the total land area of the
Freedom Islands less 30 percent allocated for common areas.
111
The share of AMARI in the submerged areas for reclamation is 290.129 hectares,
which is 70 percent of the net usable area of 414.47 hectares.
112
Article 1409 of the Civil Code provides as follows: "The following contracts are
inexistent and void from the beginning: (1) Those whose cause, object or purpose is
contrary to law; x x x; (4) Those whose object is outside the commerce of men; x x x."
Republic of the Philippines
SUPREME COURT
Manila
EN BANC
G.R. No. 73002 December 29, 1986
THE DIRECTOR OF LANDS, petitioner,
vs.
INTERMEDIATE APPELLATE COURT and ACME PLYWOOD & VENEER CO. INC.,
ETC., respondents.
D. Nacion Law Office for private respondent.

NARVASA, J .:
The Director of Lands has brought this appeal by certiorari from a judgment of the Intermediate
Appellate Court affirming a decision of the Court of First Instance of Isabela, which ordered
registration in favor of Acme Plywood & Veneer Co., Inc. of five parcels of land measuring 481,
390 square meters, more or less, acquired by it from Mariano and Acer Infiel, members of the
Dumagat tribe.
The registration proceedings were for confirmation of title under Section 48 of Commonwealth
Act No. 141 (The Public Land Act). as amended: and the appealed judgment sums up the
findings of the trial court in said proceedings in this wise:
1. That Acme Plywood & Veneer Co. Inc., represented by Mr. Rodolfo Nazario is a
corporation duly organized in accordance with the laws of the Republic of the Philippines
and registered with the Securities and Exchange Commission on December 23, 1959;
2. That Acme Plywood & Veneer Co. Inc., represented by Mr. Rodolfo Nazario can
acquire real properties pursuant to the provisions of the Articles of Incorporation
particularly on the provision of its secondary purposes (paragraph (9), Exhibit 'M-l');
3. That the land subject of the Land Registration proceeding was ancestrally acquired by
Acme Plywood & Veneer Co., Inc., on October 29, 1962, from Mariano Infiel and Acer
Infiel, both members of the Dumagat tribe and as such are cultural minorities;
4. That the constitution of the Republic of the Philippines of 1935 is applicable as the
sale took place on October 29, 1962;
5. That the possession of the Infiels over the land relinquished or sold to Acme Plywood
& Veneer Co., Inc., dates back before the Philippines was discovered by Magellan as
the ancestors of the Infiels have possessed and occupied the land from generation to
generation until the same came into the possession of Mariano Infiel and Acer Infiel;
6. That the possession of the applicant Acme Plywood & Veneer Co., Inc., is continuous,
adverse and public from 1962 to the present and tacking the possession of the Infiels
who were granted from whom the applicant bought said land on October 29, 1962,
hence the possession is already considered from time immemorial.
7. That the land sought to be registered is a private land pursuant to the provisions of
Republic Act No. 3872 granting absolute ownership to members of the non-Christian
Tribes on land occupied by them or their ancestral lands, whether with the alienable or
disposable public land or within the public domain;
8. That applicant Acme Plywood & Veneer Co. Inc., has introduced more than Forty-Five
Million (P45,000,000.00) Pesos worth of improvements, said improvements were seen
by the Court during its ocular investigation of the land sought to be registered on
September 18, 1982;
9. That the ownership and possession of the land sought to be registered by the
applicant was duly recognized by the government when the Municipal Officials of
Maconacon, Isabela, have negotiated for the donation of the townsite from Acme
Plywood & Veneer Co., Inc., and this negotiation came to reality when the Board of
Directors of the Acme Plywood & Veneer Co., Inc., had donated a part of the land
bought by the Company from the Infiels for the townsite of Maconacon Isabela (Exh. 'N')
on November 15, 1979, and which donation was accepted by the Municipal Government
of Maconacon, Isabela (Exh. 'N-l'), during their special session on November 22, 1979.
The Director of Lands takes no issue with any of these findings except as to the applicability of
the 1935 Constitution to the matter at hand. Concerning this, he asserts that, the registration
proceedings have been commenced only on July 17, 1981, or long after the 1973 Constitution
had gone into effect, the latter is the correctly applicable law; and since section 11 of its Article
XIV prohibits private corporations or associations from holding alienable lands of the public
domain, except by lease not to exceed 1,000 hectares (a prohibition not found in the 1935
Constitution which was in force in 1962 when Acme purchased the lands in question from the
Infiels), it was reversible error to decree registration in favor of Acme Section 48, paragraphs (b)
and (c), of Commonwealth Act No. 141, as amended, reads:
SEC. 48. The following described citizens of the Philippines, occupying lands of the
public domain or claiming to own any such lands or an interest therein, but whose titles
have not been perfected or completed, may apply to the Court of First Instance of the
province where the land is located for confirmation of their claims, and the issuance of a
certificate of title therefor, under the Land Registration Act, to wit:
xxx xxx xxx
(b) Those who by themselves or through their predecessors-in-interest have been in
open, continuous, exclusive and notorious possession and occupation of agricultural
lands of the public domain, under a bona fide claim of acquisition or ownership, for at
least thirty years immediately preceding the filing of the application for confirmation of
title except when prevented by war or force majeure. These shall be conclusively
presumed to have performed all the conditions essential to a Government grant and
shall be entitled to a certificate of title under the provisions of this chapter.
(c) Members of the National Cultural minorities who by themselves or through their
predecessors-in-interest have been in open. continuous, exclusive and notorious
possession and occupation of lands of the public domain suitable to agriculture, whether
disposable or not, under a bona fide claim of ownership for at least 30 years shall be
entitled to the rights granted in subsection (b) hereof.
The Petition for Review does not dispute-indeed, in view of the quoted findings of the trial court
which were cited and affirmed by the Intermediate Appellate Court, it can no longer controvert
before this Court-the fact that Mariano and Acer Infiel, from whom Acme purchased the lands in
question on October 29, 1962, are members of the national cultural minorities who had, by
themselves and through their progenitors, possessed and occupied those lands since time
immemorial, or for more than the required 30-year period and were, by reason thereof, entitled
to exercise the right granted in Section 48 of the Public Land Act to have their title judicially
confirmed. Nor is there any pretension that Acme, as the successor-in-interest of the Infiels, is
disqualified to acquire and register ownership of said lands under any provisions of the 1973
Constitution other than Section 11 of its Article XIV already referred to.
Given the foregoing, the question before this Court is whether or not the title that the Infiels had
transferred to Acme in 1962 could be confirmed in favor of the latter in proceedings instituted by
it in 1981 when the 1973 Constitution was already in effect, having in mind the prohibition
therein against private corporations holding lands of the public domain except in lease not
exceeding 1,000 hectares.
The question turns upon a determination of the character of the lands at the time of institution of
the registration proceedings in 1981. If they were then still part of the public domain, it must be
answered in the negative. If, on the other hand, they were then already private lands, the
constitutional prohibition against their acquisition by private corporations or associations
obviously does not apply.
In this regard, attention has been invited to Manila Electric Company vs. Castro-Bartolome, et
al,
1
where a similar set of facts prevailed. In that case, Manila Electric Company, a domestic
corporation more than 60% of the capital stock of which is Filipino-owned, had purchased in
1947 two lots in Tanay, Rizal from the Piguing spouses. The lots had been possessed by the
vendors and, before them, by their predecessor-in-interest, Olimpia Ramos, since prior to the
outbreak of the Pacific War in 1941. On December 1, 1976, Meralco applied to the Court of First
Instance of Rizal, Makati Branch, for confirmation of title to said lots. The court, assuming that
the lots were public land, dismissed the application on the ground that Meralco, a juridical
person, was not qualified to apply for registration under Section 48(b) of the Public Land Act
which allows only Filipino citizens or natural persons to apply for judicial confirmation of
imperfect titles to public land. Meralco appealed, and a majority of this Court upheld the
dismissal. It was held that:
..., the said land is still public land. It would cease to be public land only upon the
issuance of the certificate of title to any Filipino citizen claiming it under section 48(b).
Because it is still public land and the Meralco, as a juridical person, is disqualified to
apply for its registration under section 48(b), Meralco's application cannot be given due
course or has to be dismissed.
Finally, it may be observed that the constitutional prohibition makes no distinction
between (on the one hand) alienable agricultural public lands as to which no occupant
has an imperfect title and (on the other hand) alienable lands of the public domain as to
which an occupant has on imperfect title subject to judicial confirmation.
Since section 11 of Article XIV does not distinguish, we should not make any distinction
or qualification. The prohibition applies to alienable public lands as to which a Torrens
title may be secured under section 48(b). The proceeding under section 48(b)
'presupposes that the land is public' (Mindanao vs. Director of Lands, L-19535, July 30,
1967, 20 SCRA 641, 644).
The present Chief Justice entered a vigorous dissent, tracing the line of cases beginning
with Carino in 1909
2
thru Susi in 1925
3
down to Herico in 1980,
4
which developed, affirmed
and reaffirmed the doctrine that open, exclusive and undisputed possession of alienable public
land for the period prescribed by law creates the legal fiction whereby the land, upon completion
of the requisite period ipso jure and without the need of judicial or other sanction, ceases to be
public land and becomes private property. That said dissent expressed what is the better
and, indeed, the correct, view-becomes evident from a consideration of some of the principal
rulings cited therein,
The main theme was given birth, so to speak, in Carino involving the Decree/Regulations of
June 25, 1880 for adjustment of royal lands wrongfully occupied by private individuals in the
Philippine Islands. It was ruled that:
It is true that the language of articles 4 and 5
5
attributes title to those 'who may prove'
possession for the necessary time and we do not overlook the argument that this means
may prove in registration proceedings. It may be that an English conveyancer would
have recommended an application under the foregoing decree, but certainly it was not
calculated to convey to the mind of an Igorot chief the notion that ancient family
possessions were in danger, if he had read every word of it. The words 'may prove'
(acrediten) as well or better, in view of the other provisions, might be taken to mean
when called upon to do so in any litigation. There are indications that registration was
expected from all but none sufficient to show that, for want of it, ownership actually
gained would be lost. The effect of the proof, wherever made, was not to confer title, but
simply to establish it, as already conferred by the decree, if not by earlier law. ...
That ruling assumed a more doctrinal character because expressed in more categorical
language, in Susi:
.... In favor of Valentin Susi, there is, moreover, the presumption juris et de
jure established in paragraph (b) of section 45 of Act No. 2874, amending Act No. 926,
that all the necessary requirements for a grant by the Government were complied with,
for he has been in actual and physical possession, personally and through his
predecessors, of an agricultural land of the public domain openly, continuously,
exclusively and publicly since July 26, 1984, with a right to a certificate of title to said
land under the provisions of Chapter VIII of said Act. So that when Angela Razon
applied for the grant in her favor, Valentin Susi had already acquired, by operation of law
not only a right to a grant, but a grant of the Government, for it is not necessary that a
certificate of title should be issued in order that said grant may be sanctioned by the
courts, an application therefore is sufficient, under the provisions of section 47 of Act No.
2874. If by a legal fiction, Valentin Susi had acquired the land in question by a grant of
the State, it had already ceased to be of the public domain and had become private
property, at least by presumption,of Valentin Susi, beyond the control of the Director of
Lands. Consequently, in selling the land in question of Angela Razon, the Director of
Lands disposed of a land over which he had no longer any title or control, and the sale
thus made was void and of no effect, and Angela Razon did not thereby acquire any
right.
6

Succeeding cases, of which only some need be mentioned, likeof Lacaste vs. Director of
Lands,
7
Mesina vs. Vda. de Sonza,
8
Manarpac vs. Cabanatuan,
9
Miguel vs. Court of
Appeals
10
and Herico vs. Dar, supra, by invoking and affirming the Susi doctrine have firmly
rooted it in jurisprudence.
Herico, in particular, appears to be squarely affirmative:
11

.... Secondly, under the provisions of Republic Act No. 1942, which the respondent Court
held to be inapplicable to the petitioner's case, with the latter's proven occupation and
cultivation for more than 30 years since 1914, by himself and by his predecessors-in-
interest, title over the land has vested on petitioner so as to segregate the land from the
mass of public land. Thereafter, it is no longer disposable under the Public Land Act as
by free patent. ....
xxx xxx xxx
As interpreted in several cases, when the conditions as specified in the foregoing
provision are complied with, the possessor is deemed to have acquired, by operation of
law, a right to a grant, a government grant, without the necessity of a certificate of title
being issued. The land, therefore, ceases to be of the public domain and beyond the
authority of the Director of Lands to dispose of. The application for confirmation is mere
formality, the lack of which does not affect the legal sufficiency of the title as would be
evidenced by the patent and the Torrens title to be issued upon the strength of said
patent.
12

Nothing can more clearly demonstrate the logical inevitability of considering possession of
public land which is of the character and duration prescribed by statute as the equivalent of an
express grant from the State than the dictum of the statute itself
13
that the possessor(s) "... shall
be conclusively presumed to have performed all the conditions essential to a Government grant
and shall be entitled to a certificate of title .... " No proof being admissible to overcome a
conclusive presumption, confirmation proceedings would, in truth be little more than a formality,
at the most limited to ascertaining whether the possession claimed is of the required character
and length of time; and registration thereunder would not confer title, but simply recognize a title
already vested. The proceedings would not originally convert the land from public to private
land, but only confirm such a conversion already affected by operation of law from the moment
the required period of possession became complete. As was so well put in Carino, "... (T)here
are indications that registration was expected from all, but none sufficient to show that, for want
of it, ownership actually gained would be lost. The effect of the proof, wherever made, was not
to confer title, but simply to establish it, as already conferred by the decree, if not by earlier law."
If it is accepted-as it must be-that the land was already private land to which the Infiels had a
legally sufficient and transferable title on October 29, 1962 when Acme acquired it from said
owners, it must also be conceded that Acme had a perfect right to make such acquisition, there
being nothing in the 1935 Constitution then in force (or, for that matter, in the 1973 Constitution
which came into effect later) prohibiting corporations from acquiring and owning private lands.
Even on the proposition that the land remained technically "public" land, despite immemorial
possession of the Infiels and their ancestors, until title in their favor was actually confirmed in
appropriate proceedings under the Public Land Act, there can be no serious question of Acmes
right to acquire the land at the time it did, there also being nothing in the 1935 Constitution that
might be construed to prohibit corporations from purchasing or acquiring interests in public land
to which the vendor had already acquired that type of so-called "incomplete" or "imperfect" title.
The only limitation then extant was that corporations could not acquire, hold or lease public
agricultural lands in excess of 1,024 hectares. The purely accidental circumstance that
confirmation proceedings were brought under the aegis of the 1973 Constitution which forbids
corporations from owning lands of the public domain cannot defeat a right already vested before
that law came into effect, or invalidate transactions then perfectly valid and proper. This Court
has already held, in analogous circumstances, that the Constitution cannot impair vested rights.
We hold that the said constitutional prohibition
14
has no retroactive application to the
sales application of Binan Development Co., Inc. because it had already acquired a
vested right to the land applied for at the time the 1973 Constitution took effect.
That vested right has to be respected. It could not be abrogated by the new Constitution.
Section 2, Article XIII of the 1935 Constitution allows private corporations to purchase
public agricultural lands not exceeding one thousand and twenty-four hectares.
Petitioner' prohibition action is barred by the doctrine of vested rights in constitutional
law.
xxx xxx xxx
The due process clause prohibits the annihilation of vested rights. 'A state may not
impair vested rights by legislative enactment, by the enactment or by the subsequent
repeal of a municipal ordinance, or by a change in the constitution of the State, except in
a legitimate exercise of the police power'(16 C.J.S. 1177-78).
xxx xxx xxx
In the instant case, it is incontestable that prior to the effectivity of the 1973 Constitution
the right of the corporation to purchase the land in question had become fixed and
established and was no longer open to doubt or controversy.
Its compliance with the requirements of the Public Land Law for the issuance of a patent
had the effect of segregating the said land from the public domain. The corporation's
right to obtain a patent for the land is protected by law. It cannot be deprived of that right
without due process (Director of Lands vs. CA, 123 Phil. 919).<re||an1w>
15

The fact, therefore, that the confirmation proceedings were instituted by Acme in its own name
must be regarded as simply another accidental circumstance, productive of a defect hardly
more than procedural and in nowise affecting the substance and merits of the right of ownership
sought to be confirmed in said proceedings, there being no doubt of Acme's entitlement to the
land. As it is unquestionable that in the light of the undisputed facts, the Infiels, under either the
1935 or the 1973 Constitution, could have had title in themselves confirmed and registered, only
a rigid subservience to the letter of the law would deny the same benefit to their lawful
successor-in-interest by valid conveyance which violates no constitutional mandate.
The Court, in the light of the foregoing, is of the view, and so holds, that the majority ruling
in Meralco must be reconsidered and no longer deemed to be binding precedent. The correct
rule, as enunciated in the line of cases already referred to, is that alienable public land held by a
possessor, personally or through his predecessors-in-interest, openly, continuously and
exclusively for the prescribed statutory period (30 years under The Public Land Act, as
amended) is converted to private property by the mere lapse or completion of said period, ipso
jure. Following that rule and on the basis of the undisputed facts, the land subject of this appeal
was already private property at the time it was acquired from the Infiels by Acme. Acme thereby
acquired a registrable title, there being at the time no prohibition against said corporation's
holding or owning private land. The objection that, as a juridical person, Acme is not qualified to
apply for judicial confirmation of title under section 48(b) of the Public Land Act is technical,
rather than substantial and, again, finds its answer in the dissent in Meralco:
6. To uphold respondent judge's denial of Meralco's application on the technicality that
the Public Land Act allows only citizens of the Philippines who are natural persons to
apply for confirmation of their title would be impractical and would just give rise to
multiplicity of court actions. Assuming that there was a technical error not having filed
the application for registration in the name of the Piguing spouses as the original owners
and vendors, still it is conceded that there is no prohibition against their sale of the land
to the applicant Meralco and neither is there any prohibition against the application being
refiled with retroactive effect in the name of the original owners and vendors (as such
natural persons) with the end result of their application being granted, because of their
indisputable acquisition of ownership by operation of law and the conclusive
presumption therein provided in their favor. It should not be necessary to go through all
the rituals at the great cost of refiling of all such applications in their names and adding
to the overcrowded court dockets when the Court can after all these years dispose of it
here and now. (See Francisco vs. City of Davao)
The ends of justice would best be served, therefore, by considering the applications for
confirmation as amended to conform to the evidence, i.e. as filed in the names of the
original persons who as natural persons are duly qualified to apply for formal
confirmation of the title that they had acquired by conclusive presumption and mandate
of the Public Land Act and who thereafter duly sold to the herein corporations (both
admittedly Filipino corporations duly qualified to hold and own private lands) and
granting the applications for confirmation of title to the private lands so acquired and sold
or exchanged.
There is also nothing to prevent Acme from reconveying the lands to the Infiels and the latter
from themselves applying for confirmation of title and, after issuance of the certificate/s of title in
their names, deeding the lands back to Acme. But this would be merely indulging in empty
charades, whereas the same result is more efficaciously and speedily obtained, with no
prejudice to anyone, by a liberal application of the rule on amendment to conform to the
evidence suggested in the dissent in Meralco.
While this opinion seemingly reverses an earlier ruling of comparatively recent vintage, in a real
sense, it breaks no precedent, but only reaffirms and re-established, as it were, doctrines the
soundness of which has passed the test of searching examination and inquiry in many past
cases. Indeed, it is worth noting that the majority opinion, as well as the concurring opinions of
Chief Justice Fernando and Justice Abad Santos, in Meralco rested chiefly on the proposition
that the petitioner therein, a juridical person, was disqualified from applying for confirmation of
an imperfect title to public land under Section 48(b) of the Public Land Act. Reference to the
1973 Constitution and its Article XIV, Section 11, was only tangential limited to a brief paragraph
in the main opinion, and may, in that context, be considered as essentially obiter. Meralco, in
short, decided no constitutional question.
WHEREFORE, there being no reversible error in the appealed judgment of the Intermediate
Appellate Court, the same is hereby affirmed, without costs in this instance.
SO ORDERED.
Feria, Yap, Fernan, Alampay, Cruz, Paras and Feliciano, JJ., concur.


Separate Opinions
GUTIERREZ, JR., J ., concurring:
I reiterate my concurrence in Meralco v. Castro-Bartolome, and, therefore, dissent here.

TEEHANKEE, C.J ., concurring:
I am honored by my brethren's judgment at bar that my dissenting opinion in the June,
1982 Meralco and Iglesia ni Cristocases,
1
which is herein upheld, "expressed what is the better.
. . . and indeed the correct view." My dissent was anchored on the landmark 1909 case
of Carino
2
through the 1925 case of Susi
3
and the long line of cases cited therein to the latest
1980 case of Herico
4
that "it is established doctrine....... that an open, continuous, adverse and
public possession of a land of the public domain for the period provided in the Public Land Act
provision in force at the time (from July 26, 1894 in Susi under the old law [this period was
reduced to 'at least thirty years immediately preceding the filing of the application for
confirmation of title' by amendment of Commonwealth Act No. 141, equivalent to the period of
acquisitive prescription
5
]) by a private individual personally and through his predecessors
confers an effective title on said possessor, whereby the land ceases to be land of the public
domain and becomes private property." I hereby reproduce the same by reference for brevity's
sake. But since we are reverting to the old above-cited established doctrine and precedents and
discarding the Meralco and Iglesia ni Cristo cases which departed therefrom in the recent past, I
feel constrained to write this concurrence in amplification of my views and ratio decidendi.
Under the express text and mandate of the cited Act, such possessors "shall be conclusively
presumed to have performed all the conditions essential to a Government grant and shall be
entitled to a certificate of title under the provisions of this chapter. "
The Court thus held in Susi that under the presumption juris et de jure established in the Act,
the rightful possessor of the public land for the statutory period "already acquired, by operation
of law, not only a right to a grant, but a grant of the Government, for it is not necessary that
certificate of title should be issued an order that said grant may be sanctioned by the courts, an
application therefore is sufficient . . . . If by a legal fiction, Valentin Susi had acquired the land in
question by a grant of the State, it had already ceased to be of the public domain and
had become private property, at least by presumption, of Valentin Susi, beyond the control of
the Director of Lands [and beyond his authority to sell to any other person]. "
6

The root of the doctrine goes back to the pronouncement of Justice Oliver Wendell Holmes for
the U.S. Supreme Court in the 1909 case of Carino (the Igorot chief who would have been
deprived of ancestral family lands by the dismissal of his application for registration) which
reversed the dismissal of the registration court (as affirmed by the Supreme Court) and adopted
the liberal view that under the decree and regulations of June 25, 1880, "The words 'may prove'
(acrediten), as well, or better, in view of the other provisions, might be taken to mean when
called upon to do so in any litigation. There are indications that registration was expected from
all, but none sufficient to show that, for want of it, ownership actually gained would be lost.
The effect of the proof, whenever made, was not to confer title, but simply to establish it, as
already conferred by the decree, if not by earlier law."
The Court's decision at bar now expressly overturns the Meralco and related cases subsequent
thereto which failed to adhere to the aforecited established doctrine dating back to 1909 and
was consistently applied up to June 29, 1982 (when the Meralco decision was promulgated).
We reaffirm the established doctrine that such acquisitive prescription of alienable public lands
takes place ipso jure or by operation of law without the necessity of a prior issuance of a
certificate of title. The land ipso jure ceases to be of the public domain and becomes private
property, which may be lawfully sold to and acquired by qualified corporations such as
respondent corporation. (As stressed in Herico supra, "the application for confirmation is a mere
formality, the lack of which does not affect the legal sufficiency of the title.")
Such ipso jure conversion into private property of public lands publicly held under a bona
fide claim of acquisition or ownership is the public policy of the Act and is so expressly stated
therein. By virtue of such conversion into private property, qualified corporations may lawfully
acquire them and there is no "alteration or defeating" of the 1973 Constitution's prohibition
against corporations holding or acquiring title to lands of the public domain, as claimed in the
dissenting opinion, for the simple reason that no public lands are involved.
It should be noted that respondent corporation purchased the land from the Infiels on October
16, 1962 under the aegis of the 1935 Constitution which contained no prohibition against
corporations holding public lands (except a limit of 1,024 hectares) unlike the later 1973
Constitution which imposed an absolute prohibition. Even on the erroneous assumption that the
land remained public land despite the Infiels' open possession thereof as owners from time
immemorial, respondent corporation's lawful purchase from them of the land in 1962 and P
45million investments redounding presumably to the welfare and progress of the community,
particularly the municipality of Maconacon, Isabela to which it donated part of the land for the
townsite created a vested right which could not be impaired by the prohibition adopted eleven
years later. But as sufficiently stressed, the land of the Infiels had been ipso jure converted
into private land and they had a legally sufficient and transferable title conferred by the
conclusive presumption of the Public Land Act (which needed only to be establishedin
confirmation of title proceedings for formalization and issuance of the certificate of title) which
they lawfully and validly transferred to respondent corporation.
In fact, the many amendments to the Act extending the period for the filing of such applications
for judicial confirmation of imperfect and incomplete titles to alienable and disposable public
lands expressly reiterate that it has always been the "policy of the State to hasten the
settlement, adjudication and quieting of titles to [such] unregistered lands," i.e. to recognize that
such lands publicly and notoriously occupied and cultivated under bona fide claim of acquisition
or ownership have ipso jure been converted into private property and grant the possessors the
opportunity to establish and record such fact. Thus, the deadline for the filing of such application
which would have originally expired first on December 31, 1938 was successively extended to
December 31, 1941, then extended to December 31, 1957, then to December 31, 1968, further
extended to December 31, 1976 and lastly extended to December 31, 1987.
7

The cited Act's provision that only natural persons may apply thereunder for confirmation of title
is in effect a technicality of procedure and not of substance. My submittal in Meralco, mutatis
mutandis, is properly applicable: "The ends of justice would best be served, therefore, by
considering the applications for confirmation as amended to conform to the evidence, i.e. as
filed in the names of the original persons who as natural persons are duly qualified to apply for
formal confirmation of the title that they had acquired by conclusive presumption and mandate
of the Public Land Act and who thereafter duly sold to the herein corporations (both admittedly
Filipino corporations duly qualified to hold and own private lands) and granting the applications
for confirmation of title to the private lands so acquired and sold or exchanged."
8
Indeed, then
Chief Justice Enrique M. Fernando likewise dissented along the same line from the majority
ruling therein and held: "I dissent insofar as the opinion of the Court would characterize such
jurisdictional defect that the applicant was Meralco, a juridical person rather than the natural
persons-transferors, under the particular circumstances of this case, as an insurmountable
obstacle to the relief sought. I would apply by analogy, although the facts could be
distinguished, the approach followed by us in Francisco v. City of Davao, where the legal
question raised, instead of being deferred and possibly taken up in another case, was resolved.
By legal fiction and in the exercise of our equitable jurisdiction, I feel that the realistic solution
would be to decide the matter as if the application under Section 48(b) were filed by the Piguing
spouses, who I assume suffer from no such disability."
9
Justice Vicente Abad Santos, now
retired, while concurring in the procedural result, likewise, in effect dissented from the therein
majority ruling on the question of substance, and stated his opinion that "the lots which are
sought to be registered have ceased to be lands of the public domain at the time they were
acquired by the petitioner corporation. They are already private lands because of acquisitive
prescription by the predecessors of the petitioner and all that is needed is the confirmation of
the title. Accordingly, the constitutional provision that no private corporation or association may
hold alienable lands of the public domain is inapplicable. "
10

To my mind, the reason why the Act limits the filing of such applications to natural citizens who
may prove their undisputed and open possession of public lands for the required statutory thirty-
year period, tacking on their predecessors'-in-interest possession is that only natural persons, to
the exclusion of juridical persons such as corporations, can actually, physically and in reality
possess public lands for the required statutory 30-year period. That juridical persons or
corporations cannot do so is obvious. But when the natural persons have fulfilled the required
statutory period of possession, the Act confers on them a legally sufficient and transferable title.
It is preferable to follow the letter of the law that they file the applications for confirmation of their
title, although they have lawfully transferred their title to the land. But such procedural failure
cannot and should not defeat the substance of the law, as stressed in the above-cited opinions,
that the lands are already privatelands because of acquisitive prescription by the corporation's
predecessors and the realistic solution would be to consider the application for confirmation as
filed by the natural persons-transferors, and in accordance with the evidence, confirm their title
to the private lands so converted by operation of law and lawfully transferred by them to the
corporation. The law, after all, recognizes the validity of the transfer and sale of the private land
to the corporation. It should not be necessary to go in a round-about way and have the
corporation reassign its rights to the private land to natural persons-(as I understand), was done
after the decision in the Meralco and Iglesia ni Cristo cases) just for the purpose of complying
on paper with the technicality of having natural persons file the application for confirmation of
title to the private land.

MELENCIO-HERRERA, J ., dissenting:
Section 48 of the Public Land Act, in part, provides:
SEC. 48. The following described citizens of the Philippines, occupying lands of the
public domain or claiming to own any such lands or an interest therein, but whose titles
have not been perfected or completed, may apply to the Court of First Instance of the
province where the land is located for confirmation of their claims and the issuance of a
certificate of title therefor, under the Land Registration Act, to wit:
(a) ...
(b) Those who by themselves or through their predecessors in interest have been in
open, continuous, exclusive, and notorious possession and occupation of agricultural
lands of the public domain, under a bona fide claim of acquisition of ownership, for at
least thirty years immediately preceding the filing of the application for confirmation of
title except when prevented by war or force majeure. These shall be conclusively
presumed to have performed are the conditions essential to a Government grant and
shall be entitled to a certificate of title under the provisions of this chapter.
(c) ...
Article XIV, Section 11, of the 1973 Constitution, in part, provides:
SEC. 11. .... No private corporation or association may hold alienable lands of the public
domain except by lease not to exceed one thousand hectares in area; nor may any
citizen hold such lands by lease in excess of five hundred hectares ....
It has to be conceded that, literally, statutory law and constitutional provision prevent a
corporation from directly applying to the Courts for the issuance of Original Certificates of Title
to lands of the public domain (Manila Electric Company vs. Castro-Bartolome, 114 SCRA 799;
Republic vs. Villanueva, 114 SCRA 875; Republic vs. Court of Appeals, 119 SCRA 449; Iglesia
ni Cristo vs. Hon. Judge, CFI of Nueva Ecija, Br. 1). It is my opinion that the literalism should be
adhered to in this case.
The reasoning of the majority can be restated in simple terms as follows:
(a) The INFIELS can successfully file an application for a certificate of title over the land
involved in the case.
(b) After the INFIELS secure a certificate of title, they can sell the land to ACME.
(c) As ACME can eventually own the certificate of title, it should be allowed to directly apply to
the Courts for the Certificate of Title, thus avoiding the circuituous "literal" requirement that the
INFIELS should first apply to the courts for the titles, and afterwards transfer the title to ACME.
The majority opinion, in effect, adopted the following excerpt from a dissent in Manila Electric
Company vs. Castro-Bartolome (114 SCRA 799, 823 [1982]).
To uphold respondent judge's denial of Meralco's application on the technicality that the
Public Land Act allows only citizens of the Philippines who are natural persons to apply
for confirmation of their title would be impractical and would just give rise to multiplicity of
court actions. Assuming that there was a technical error in not having filed the
application for registration in the name of the Piguing spouses as the original owners
and vendors,
still it is conceded that there is no prohibition against their sale of the land to the
applicant Meralco
and neither is there any prohibition against the application being refiled with retroactive
effect in the name of the original owners and vendors (as such natural persons) with the
end result of their application being granted, because of their indisputable acquisition of
ownership by operation of law and the conclusive presumption therein provided in their
favor.
It should not be necessary to go through all the rituals at the great cost of refiling of all such
applications in their names and adding to the overcrowded court dockets when the Court can
after all these years dispose of it here and now." (Paragraphing supplied)
The effect is that the majority opinion now nullifies the statutory provision that only citizens
(natural persons) can apply for certificates of title under Section 48(b) of the Public Land Act, as
well as the constitutional provision (Article XIV, Section 11) which prohibits corporations from
acquiring title to lands of the public domain. That interpretation or construction adopted by the
majority cannot be justified. "A construction adopted should not be such as to nullify, destroy or
defeat the intention of the legislature" (New York State Dept. of Social Services v. Dublino [UST
37 L. Ed 2d 688, 93 S Ct 2507; United States v. Alpers 338 US 680, 94 L Ed 457, 70 S Ct 352;
cited in 73 Am Jur. 2nd., p. 351).
It has also been said that:
In the construction of statutes, the courts start with the assumption that the legislature
intended to enact an effective law, and the legislature is not to be presumed to have
done a vain thing in the enactment of a statute. Hence, it is a general principle that the
courts should, if reasonably possible to do so interpret the statute, or the provision being
construed, so as to give it efficient operation and effect as a whole. An interpretation
should, if possible, be avoided, under which the statute or provision being construed is
defeated, or as otherwise expressed, nullified, destroyed, emasculated, repealed,
explained away, or rendered insignificant, meaningless, inoperative, or nugatory. If a
statute is fairly susceptible of two constructions, one of which will give effect to the act,
while the other will defeat it, the former construction is preferred. One part of a statute
may not be construed so as to render another part nugatory or of no effect. Moreover,
notwithstanding the general rule against the enlargement of extension of a statute by
construction, the meaning of a statute may be extended beyond the precise words used
in the law, and words or phrases may be altered or supplied, where this is necessary to
prevent a law from becoming a nullity. Wherever the provision of a statute is general
everything which is necessary to make such provision effectual is supplied by
implication. (Pliakos vs. Illinois Liquor Control Com. 11 III 2d 456, 143 NE2d 47; cited in
73 AM Jur. 2d pp. 422-423)
The statutory provision and the constitutional prohibition express a public policy. The proper
course for the Court to take is to promote in the fullest manner the policy thus laid down and to
avoid a construction which would alter or defeat that policy.
In fine, I confirm my adherence to the ruling of this Court in Meralco vs. Hon. Castro-Bartolome,
114 SCRA 799 [1982] and related cases.


Separate Opinions
GUTIERREZ, JR., J ., concurring:
I reiterate my concurrence in Meralco v. Castro-Bartolome, and, therefore, dissent here.

TEEHANKEE, C.J ., concurring:
I am honored by my brethren's judgment at bar that my dissenting opinion in the June,
1982 Meralco and Iglesia ni Cristocases,
1
which is herein upheld, "expressed what is the better.
. . . and indeed the correct view." My dissent was anchored on the landmark 1909 case
of Carino
2
through the 1925 case of Susi
3
and the long line of cases cited therein to the latest
1980 case of Herico
4
that "it is established doctrine....... that an open, continuous, adverse and
public possession of a land of the public domain for the period provided in the Public Land Act
provision in force at the time (from July 26, 1894 in Susi under the old law [this period was
reduced to 'at least thirty years immediately preceding the filing of the application for
confirmation of title' by amendment of Commonwealth Act No. 141, equivalent to the period of
acquisitive prescription
5
]) by a private individual personally and through his predecessors
confers an effective title on said possessor, whereby the land ceases to be land of the public
domain and becomes private property." I hereby reproduce the same by reference for brevity's
sake. But since we are reverting to the old above-cited established doctrine and precedents and
discarding the Meralco and Iglesia ni Cristo cases which departed therefrom in the recent past, I
feel constrained to write this concurrence in amplification of my views and ratio decidendi.
Under the express text and mandate of the cited Act, such possessors "shall be conclusively
presumed to have performed all the conditions essential to a Government grant and shall be
entitled to a certificate of title under the provisions of this chapter. "
The Court thus held in Susi that under the presumption juris et de jure established in the Act,
the rightful possessor of the public land for the statutory period "already acquired, by operation
of law, not only a right to a grant, but a grant of the Government, for it is not necessary that
certificate of title should be issued an order that said grant may be sanctioned by the courts, an
application therefore is sufficient . . . . If by a legal fiction, Valentin Susi had acquired the land in
question by a grant of the State, it had already ceased to be of the public domain and
had become private property, at least by presumption, of Valentin Susi, beyond the control of
the Director of Lands [and beyond his authority to sell to any other person]. "
6

The root of the doctrine goes back to the pronouncement of Justice Oliver Wendell Holmes for
the U.S. Supreme Court in the 1909 case of Carino (the Igorot chief who would have been
deprived of ancestral family lands by the dismissal of his application for registration) which
reversed the dismissal of the registration court (as affirmed by the Supreme Court) and adopted
the liberal view that under the decree and regulations of June 25, 1880, "The words 'may prove'
(acrediten), as well, or better, in view of the other provisions, might be taken to mean when
called upon to do so in any litigation. There are indications that registration was expected from
all, but none sufficient to show that, for want of it, ownership actually gained would be lost.
The effect of the proof, whenever made, was not to confer title, but simply to establish it, as
already conferred by the decree, if not by earlier law."
The Court's decision at bar now expressly overturns the Meralco and related cases subsequent
thereto which failed to adhere to the aforecited established doctrine dating back to 1909 and
was consistently applied up to June 29, 1982 (when the Meralco decision was
promulgated).<re||an1w> We reaffirm the established doctrine that such acquisitive
prescription of alienable public lands takes place ipso jure or by operation of law without the
necessity of a prior issuance of a certificate of title. The land ipso jure ceases to be of the public
domain and becomes private property, which may be lawfully sold to and acquired by qualified
corporations such as respondent corporation. (As stressed in Herico supra, "the application for
confirmation is a mere formality, the lack of which does not affect the legal sufficiency of the
title.")
Such ipso jure conversion into private property of public lands publicly held under a bona
fide claim of acquisition or ownership is the public policy of the Act and is so expressly stated
therein. By virtue of such conversion into private property, qualified corporations may lawfully
acquire them and there is no "alteration or defeating" of the 1973 Constitution's prohibition
against corporations holding or acquiring title to lands of the public domain, as claimed in the
dissenting opinion, for the simple reason that no public lands are involved.
It should be noted that respondent corporation purchased the land from the Infiels on October
16, 1962 under the aegis of the 1935 Constitution which contained no prohibition against
corporations holding public lands (except a limit of 1,024 hectares) unlike the later 1973
Constitution which imposed an absolute prohibition. Even on the erroneous assumption that the
land remained public land despite the Infiels' open possession thereof as owners from time
immemorial, respondent corporation's lawful purchase from them of the land in 1962 and P
45million investments redounding presumably to the welfare and progress of the community,
particularly the municipality of Maconacon, Isabela to which it donated part of the land for the
townsite created a vested right which could not be impaired by the prohibition adopted eleven
years later. But as sufficiently stressed, the land of the Infiels had been ipso jure converted
into private land and they had a legally sufficient and transferable title conferred by the
conclusive presumption of the Public Land Act (which needed only to be establishedin
confirmation of title proceedings for formalization and issuance of the certificate of title) which
they lawfully and validly transferred to respondent corporation.
In fact, the many amendments to the Act extending the period for the filing of such applications
for judicial confirmation of imperfect and incomplete titles to alienable and disposable public
lands expressly reiterate that it has always been the "policy of the State to hasten the
settlement, adjudication and quieting of titles to [such] unregistered lands," i.e. to recognize that
such lands publicly and notoriously occupied and cultivated under bona fide claim of acquisition
or ownership have ipso jure been converted into private property and grant the possessors the
opportunity to establish and record such fact. Thus, the deadline for the filing of such application
which would have originally expired first on December 31, 1938 was successively extended to
December 31, 1941, then extended to December 31, 1957, then to December 31, 1968, further
extended to December 31, 1976 and lastly extended to December 31, 1987.
7

The cited Act's provision that only natural persons may apply thereunder for confirmation of title
is in effect a technicality of procedure and not of substance. My submittal in Meralco, mutatis
mutandis, is properly applicable: "The ends of justice would best be served, therefore, by
considering the applications for confirmation as amended to conform to the evidence, i.e. as
filed in the names of the original persons who as natural persons are duly qualified to apply for
formal confirmation of the title that they had acquired by conclusive presumption and mandate
of the Public Land Act and who thereafter duly sold to the herein corporations (both admittedly
Filipino corporations duly qualified to hold and own private lands) and granting the applications
for confirmation of title to the private lands so acquired and sold or exchanged."
8
Indeed, then
Chief Justice Enrique M. Fernando likewise dissented along the same line from the majority
ruling therein and held: "I dissent insofar as the opinion of the Court would characterize such
jurisdictional defect that the applicant was Meralco, a juridical person rather than the natural
persons-transferors, under the particular circumstances of this case, as an insurmountable
obstacle to the relief sought. I would apply by analogy, although the facts could be
distinguished, the approach followed by us in Francisco v. City of Davao, where the legal
question raised, instead of being deferred and possibly taken up in another case, was resolved.
By legal fiction and in the exercise of our equitable jurisdiction, I feel that the realistic solution
would be to decide the matter as if the application under Section 48(b) were filed by the Piguing
spouses, who I assume suffer from no such disability."
9
Justice Vicente Abad Santos, now
retired, while concurring in the procedural result, likewise, in effect dissented from the therein
majority ruling on the question of substance, and stated his opinion that "the lots which are
sought to be registered have ceased to be lands of the public domain at the time they were
acquired by the petitioner corporation. They are already private lands because of acquisitive
prescription by the predecessors of the petitioner and all that is needed is the confirmation of
the title. Accordingly, the constitutional provision that no private corporation or association may
hold alienable lands of the public domain is inapplicable. "
10

To my mind, the reason why the Act limits the filing of such applications to natural citizens who
may prove their undisputed and open possession of public lands for the required statutory thirty-
year period, tacking on their predecessors'-in-interest possession is that only natural persons, to
the exclusion of juridical persons such as corporations, can actually, physically and in reality
possess public lands for the required statutory 30-year period. That juridical persons or
corporations cannot do so is obvious. But when the natural persons have fulfilled the required
statutory period of possession, the Act confers on them a legally sufficient and transferable title.
It is preferable to follow the letter of the law that they file the applications for confirmation of their
title, although they have lawfully transferred their title to the land. But such procedural failure
cannot and should not defeat the substance of the law, as stressed in the above-cited opinions,
that the lands are already privatelands because of acquisitive prescription by the corporation's
predecessors and the realistic solution would be to consider the application for confirmation as
filed by the natural persons-transferors, and in accordance with the evidence, confirm their title
to the private lands so converted by operation of law and lawfully transferred by them to the
corporation. The law, after all, recognizes the validity of the transfer and sale of the private land
to the corporation. It should not be necessary to go in a round-about way and have the
corporation reassign its rights to the private land to natural persons-(as I understand), was done
after the decision in the Meralco and Iglesia ni Cristo cases) just for the purpose of complying
on paper with the technicality of having natural persons file the application for confirmation of
title to the private land.

MELENCIO-HERRERA, J ., dissenting:
Section 48 of the Public Land Act, in part, provides:
SEC. 48. The following described citizens of the Philippines, occupying lands of the
public domain or claiming to own any such lands or an interest therein, but whose titles
have not been perfected or completed, may apply to the Court of First Instance of the
province where the land is located for confirmation of their claims and the issuance of a
certificate of title therefor, under the Land Registration Act, to wit:
(a) ...
(b) Those who by themselves or through their predecessors in interest have been in
open, continuous, exclusive, and notorious possession and occupation of agricultural
lands of the public domain, under a bona fide claim of acquisition of ownership, for at
least thirty years immediately preceding the filing of the application for confirmation of
title except when prevented by war or force majeure. These shall be conclusively
presumed to have performed are the conditions essential to a Government grant and
shall be entitled to a certificate of title under the provisions of this chapter.
(c) ...
Article XIV, Section 11, of the 1973 Constitution, in part, provides:
SEC. 11. .... No private corporation or association may hold alienable lands of the public
domain except by lease not to exceed one thousand hectares in area; nor may any
citizen hold such lands by lease in excess of five hundred hectares ....
It has to be conceded that, literally, statutory law and constitutional provision prevent a
corporation from directly applying to the Courts for the issuance of Original Certificates of Title
to lands of the public domain (Manila Electric Company vs. Castro-Bartolome, 114 SCRA 799;
Republic vs. Villanueva, 114 SCRA 875; Republic vs. Court of Appeals, 119 SCRA 449; Iglesia
ni Cristo vs. Hon. Judge, CFI of Nueva Ecija, Br. 1). It is my opinion that the literalism should be
adhered to in this case.
The reasoning of the majority can be restated in simple terms as follows:
(a) The INFIELS can successfully file an application for a certificate of title over the land
involved in the case.
(b) After the INFIELS secure a certificate of title, they can sell the land to ACME.
(c) As ACME can eventually own the certificate of title, it should be allowed to directly apply to
the Courts for the Certificate of Title, thus avoiding the circuituous "literal" requirement that the
INFIELS should first apply to the courts for the titles, and afterwards transfer the title to ACME.
The majority opinion, in effect, adopted the following excerpt from a dissent in Manila Electric
Company vs. Castro-Bartolome (114 SCRA 799, 823 [1982]).
To uphold respondent judge's denial of Meralco's application on the technicality that the
Public Land Act allows only citizens of the Philippines who are natural persons to apply
for confirmation of their title would be impractical and would just give rise to multiplicity of
court actions. Assuming that there was a technical error in not having filed the
application for registration in the name of the Piguing spouses as the original owners
and vendors,
still it is conceded that there is no prohibition against their sale of the land to the
applicant Meralco
and neither is there any prohibition against the application being refiled with retroactive
effect in the name of the original owners and vendors (as such natural persons) with the
end result of their application being granted, because of their indisputable acquisition of
ownership by operation of law and the conclusive presumption therein provided in their
favor.
It should not be necessary to go through all the rituals at the great cost of refiling of all such
applications in their names and adding to the overcrowded court dockets when the Court can
after all these years dispose of it here and now." (Emphasis supplied)
The effect is that the majority opinion now nullifies the statutory provision that only citizens
(natural persons) can apply for certificates of title under Section 48(b) of the Public Land Act, as
well as the constitutional provision (Article XIV, Section 11) which prohibits corporations from
acquiring title to lands of the public domain. That interpretation or construction adopted by the
majority cannot be justified. "A construction adopted should not be such as to nullify, destroy or
defeat the intention of the legislature" (New York State Dept. of Social Services v. Dublino [UST
37 L. Ed 2d 688, 93 S Ct 2507; United States v. Alpers 338 US 680, 94 L Ed 457, 70 S Ct 352;
cited in 73 Am Jur. 2nd., p. 351).
It has also been said that:
In the construction of statutes, the courts start with the assumption that the legislature
intended to enact an effective law, and the legislature is not to be presumed to have
done a vain thing in the enactment of a statute. Hence, it is a general principle that the
courts should, if reasonably possible to do so interpret the statute, or the provision being
construed, so as to give it efficient operation and effect as a whole. An interpretation
should, if possible, be avoided, under which the statute or provision being construed is
defeated, or as otherwise expressed, nullified, destroyed, emasculated, repealed,
explained away, or rendered insignificant, meaningless, inoperative, or nugatory. If a
statute is fairly susceptible of two constructions, one of which will give effect to the act,
while the other will defeat it, the former construction is preferred. One part of a statute
may not be construed so as to render another part nugatory or of no effect. Moreover,
notwithstanding the general rule against the enlargement of extension of a statute by
construction, the meaning of a statute may be extended beyond the precise words used
in the law, and words or phrases may be altered or supplied, where this is necessary to
prevent a law from becoming a nullity. Wherever the provision of a statute is general
everything which is necessary to make such provision effectual is supplied by
implication. (Pliakos vs. Illinois Liquor Control Com. 11 III 2d 456, 143 NE2d 47; cited in
73 AM Jur. 2d pp. 422-423)
The statutory provision and the constitutional prohibition express a public policy. The proper
course for the Court to take is to promote in the fullest manner the policy thus laid down and to
avoid a construction which would alter or defeat that policy.
In fine, I confirm my adherence to the ruling of this Court in Meralco vs. Hon. Castro-Bartolome,
114 SCRA 799 [1982] and related cases.
Footnotes
1 114 SCRA 799.
2 Carino vs. Insular Government, 41 Phil. 935, 944.
3 Susi vs. Razon, 48 Phil. 424.
4 Herico vs. Dar 95 SCRA 437.
5 Of said Decree/Regulations of June 25, 1880.
6 emphasis supplied.
7 63 Phil. 654.
8 Phil. 251.
9 21 SCRA 743.
10 29 SCRA 760.
11 There was withal a later attempt by the ponente in Herico (Castro, J.) to somewhat
soften the import of the doctrine, in his concurrence in Meralco (114 SCRA 799, 810-
813)
12 Emphasis supplied; the provision referred to is Section 48(b) of C.A. No. 141. "
13 Sec. 48(b).
14 Referring, precisely, to Article XIV, Section 11, of the 1973 Constitution.
15 Ayog vs. Cusi, Jr., 118 SCRA 492.
Teehankee, C.J.,
1 Meralco vs. Castro-Bartolome, 114 SCRA 799, and Republic vs. Villanueva and Iglesia
ni Cristo, 114 SCRA 875, respectively.
2 Carino vs. Insular Government, 212 U.S. 449, 53 L. ed. 594, 41 Phil. 935 and 7 Phil.
132.
3 Susi vs. Razon, 48 Phil. 424.
4 Herico vs. Dar 95 SCRA 437.
5 For the text of the Act, as amended, see page 3 of the opinion.
6 Emphasis supplied.
7 Under CA 292 approved June 9, 1938; R.A. 107, approved June 2, 1947; R. A 2061,
approved June 13, 1958; R.A. 6236, approved June 19, 1971; and P.D. 1073 issued
January 25, 1977.
8 114 SCRA at pp. 823-824.
9 Idem, at pp. 809-810.
10 Idem, at p. 810.
Republic of the Philippines
Supreme Court
Baguio City



SECOND DIVISION

JEAN TAN, ROSELLER C. ANACINTO, CARLO
LOILO ESPINEDA and DAISY ALIADO
MANAOIS, represented in this act by their
Attorney-in-Fact,
MA. WILHELMINA E. TOBIAS,
Petitioners,


- versus


REPUBLIC OF THE PHILIPPINES,
Respondent.
G.R. No. 193443

Present:

CARPIO, J.,
Chairperson,
BRION,
PEREZ,
SERENO, and
REYES, JJ.

Promulgated:

April 16, 2012

x----------------------------------------------------------------------------------------x

RESOLUTION

REYES, J .:

This is a petition for review under Rule 45 of the Decision
[1]
dated July 6, 2009 and
Resolution
[2]
dated August 12, 2010 Resolution of the Court of Appeals (CA) in CA-G.R. CV No.
88995. The facts leading to its filing are as follows:

On June 14, 2001, the petitioners filed with the Regional Trial Court (RTC) of Naic, Cavite, an
application for land registration covering a parcel of land identified as Lot 9972, Cad-459-D of Indang
Cadastre, situated in Barangay Bancod, Indang, Cavite and with an area of 6,920 square meters.
[3]
The
petitioners alleged that they acquired the subject property from Gregonio Gatdula pursuant to a Deed of
Absolute Sale dated April 25, 1996; and they and their predecessors-in-interest have been in open,
continuous and exclusive possession of the subject property in the concept of an owner for more than 30
years.
[4]


After trial and hearing, the RTC issued a Decision on July 29, 2006, granting the petitioners
application, thus:

WHEREFORE, in view of the foregoing, this Court confirming its previous
Order of general default, decrees and adjudges Lot No. 9972 consisting of 6,920 square
meters, Cad. 459-D, Indang Cadastre and its technical description as herein above-
described situated in Brgy. Bancod, Indang, Cavite, pursuant to the provisions of Act 496
as amended by P.D. 1529, as it is hereby decreed and adjudged to be confirmed and
registered in the names of Jean Tan, of legal age, Filipino, single, with postal address at
Room 54 T. Pinpin St., Binondo, Manila; Roseller C. Anaci[n]to, of legal age, Filipino,
single, with postal address at Moncario Villag[e], Ampid-1, San Mateo, Rizal; Carlo
Loilo Espineda, of legal age, Filipino, with postal address at Cluster F. Cogeo, Antipolo,
Rizal and Daisy Aliado Manaois, of legal age, Filipino and resident of Panghulo Road,
Malabon, Metro Manila.

Once this decision becomes final, let the corresponding decree of registration be
issued by the Administrator, Land Registration Authority.

SO ORDERED.
[5]



The CA gave due course to the appeal filed by the Republic of the Philippines. By way of the
assailed Decision, the CA ruled that the petitioners failed to prove that they and their predecessors-in-
interest have been in possession of the subject property for the requisite period of 30 years. The CA posit:

We now determine if appellees have the right to register their title on such land
despite the fact that their possession commenced only after 12 June 1945. Records show
that the appellees possession over the subject property can be reckoned only from 21
June 1983, the date when according to evidence, the subject property became alienable
and disposable. From said date up to the filing of the application for registration of title
over the subject property on 14 June 2001, only eighteen (18) years had lapsed. Thus,
appellees possession of the subject property fell short of the requirement of open,
continuous and exclusive possession of at least 30 years.

Moreover, there was no adequate evidence which would show that appellees and
their predecessors-in-interest exercised acts of dominion over the subject land as to
indicate possession in the concept of owner. The testimonies of appellees witnesses
regarding actual possession are belied by the absence of evidence on actual use of or
improvements on the subject property. Appellees presented only various tax declarations
to prove possession. However, except for the Certification, showing payment of tax due
on tax declaration for the year 2003, there are no other evidence showing that all the
taxes due corresponding to the rest of the tax declarations were in fact paid by appellees
or their predecessors-in-interest.

In sum, appellees were unable to prove that they or their predecessors-in-interest
have been in possession of the subject property for more than 30 years, which possession
is characterized as open, continuous, exclusive, and notorious, in the concept of an
owner. Appellees failed to discharge their duty of substantiating possession and title to
the subject land.

WHEREFORE, the appeal is hereby GRANTED and the Decision dated 29 July
2006 of the Regional Trial Court (RTC) of Naic, Cavite, Branch 15
isREVERSED and SET ASIDE.

SO ORDERED.
[6]
(citation omitted)


The petitioners moved for reconsideration but this was denied by the CA in its August 12, 2010
Resolution.
[7]


The petitioners question the conclusion arrived at by the CA, alleging that the evidence they
presented prove that they and their predecessors-in-interest have been in possession and occupation of the
subject property for more than 30 years. The petitioners claim that the following sufficed to demonstrate
that they acquired title over the subject property by prescription:

a. the testimony of their attorney-in-fact, Ma. Wilhelmina Tobias, stating that:

i. the petitioners have been in actual, notorious and open
possession of the subject property since the time they purchased the same in
1996;
ii. the petitioners have regularly paid the taxes due on the subject
property;
iii. the petitioners predecessors-in-interest, Victorio Garcia, Felipe
Gatdula and Gregonio Gatdula, had been in possession of the subject property for
more than 30 years and had religiously paid the taxes due thereon; and
iv. the subject property is agricultural, alienable and disposable;

b. the testimony of the caretaker of the subject property, Margarito Pena, stating that:

i. he resides near the subject property;
ii. he witnessed the execution of the deed of sale that petitioners
entered into with Gregonio Gatdula; and
iii. the petitioners and predecessors-in-interest have been in
possession of the subject property for more than 30 years;

c. the testimony of Ferdinand Encarnacion, a clerk in the Docket Division of the Land
Registration Authority (LRA), stating that:

i. no opposition to the petitioners application was filed before
the LRA;
ii. an examiner of the LRA found nothing wrong with the
petitioners application; and
iii. no title covering the subject property was previously issued;

d. Tax Declaration Nos. 2935, 2405 and 1823 for the years 1961, 1967 and 1974 in the
name of Victorio Garcia;
[8]


e. Tax Declaration Nos. 1534 and 3850 for the years 1980 and 1985 in the name of
Felipe Gatdula;
[9]


f. Tax Declaration Nos. 22453-A and 2925 for the years 1991 and 1994 in the name of
Gregonio Gatdula;
[10]


g. Tax Declaration Nos. 21956-A, 22096-A, 22097-A and 97-05078 in the name of the
petitioners;
[11]


h. Resolution No. 69, Series of 1998, of the Sangguniang Bayan of Indang, Cavite,
which approved the reclassification of several lots, including the subject property, from
agricultural to residential/commercial;
[12]


i. DARCO Conversion Order No. 040210005-(340)-99, Series of 2000, issued by the
Department of Agrarian Reform on July 13, 2000, which converted several parcels of
land, including the subject property, from agricultural to residential/commercial;
[13]


j. Certification issued by the Department of Environment and Natural Resources
(DENR) CALABARZON dated October 29, 2002, stating that the subject area falls
within the Alienable and Disposable Land Project No. 13-A of Indang, Cavite per LC
Map 3091 certified on June 21, 1983.
[14]



Issue

This Court is faced with the lone issue of whether the petitioners have proven themselves
qualified to the benefits under the relevant laws on the confirmation of imperfect or incomplete titles.

Our Ruling

Commonwealth Act No. 141, otherwise known as the Public Land Act governs the
classification and disposition of lands forming part of the public domain. Section 11 thereof provides that
one of the modes of disposing public lands suitable for agricultural purposes is by confirmation of
imperfect or incomplete titles. Section 48 thereof enumerates those who are considered to have acquired
an imperfect or incomplete title over an alienable and disposable public land.

Presidential Decree No. 1529 (P.D. No. 1529), otherwise known as the Property Registration
Decree, is a codification of all the laws relative to the registration of property and Section 14 thereof
specifies those who are qualified to register their incomplete title over an alienable and disposable public
land under the Torrens system. Particularly:

Section 14. Who may apply. The following persons may file in the proper Court
of First Instance an application for registration of title to land, whether personally or
through their authorized representatives:

(1) Those who by themselves or through their predecessors-in-interest have
been in open, continuous, exclusive and notorious possession and occupation of alienable
and disposable lands of the public domain under a bona fide claim of ownership since
June 12, 1945, or earlier.
(2) Those who have acquired ownership of private lands by prescription
under the provision of existing laws.
(3) Those who have acquired ownership of private lands or abandoned river
beds by right of accession or accretion under the existing laws.
(4) Those who have acquired ownership of land in any other manner
provided for by law.



As this Court clarified in Heirs of Malabanan v. Republic of the Philippines,
[15]
and Republic of the
Philippines v. East Silverlane Realty Development Corporation,
[16]
Section 14(1) covers alienable and
disposable lands while Section 14(2) covers private property. Thus, for ones possession and
occupation of an alienable and disposable public land to give rise to an imperfect title, the same should
have commenced on June 12, 1945 or earlier. On the other, for one to claim that his possession and
occupation of private property has ripened to imperfect title, the same should have been for the
prescriptive period provided under the Civil Code. Without need for an extensive extrapolation, the
private property contemplated in Section 14(2) is patrimonial property as defined in Article 421 in
relation to Articles 420 and 422 of the Civil Code.

Going further, it was explained in Heirs of Malabanan and East Silverlane, that possession and
occupation of an alienable and disposable public land for the periods provided under the Civil Code will
not convert it to patrimonial or private property. There must be an express declaration that the property is
no longer intended for public service or the development of national wealth. In the absence thereof, the
property remains to be alienable and disposable and may not be acquired by prescription under Section
14(2) of P.D. No. 1529. Thus:

In Heirs of Malabanan, this Court ruled that possession and occupation of an
alienable and disposable public land for the periods provided under the Civil Code do not
automatically convert said property into private property or release it from the public
domain. There must be an express declaration that the property is no longer intended for
public service or development of national wealth. Without such express declaration, the
property, even if classified as alienable or disposable, remains property of the State, and
thus, may not be acquired by prescription.

Nonetheless, Article 422 of the Civil Code states that [p]roperty
of public dominion, when no longer intended for public use or for public
service, shall form part of the patrimonial property of the State. It is this
provision that controls how public dominion property may be converted
into patrimonial property susceptible to acquisition by prescription. After
all, Article 420 (2) makes clear that those property which belong to the
State, without being for public use, and are intended for some public
service or for the development of the national wealth are public
dominion property. For as long as the property belongs to the State,
although already classified as alienable or disposable, it remains
property of the public dominion if when it is intended for some
public service or for the development of the national
wealth. (emphasis supplied)

Accordingly, there must be an express declaration by the
State that the public dominion property is no longer intended for
public service or the development of the national wealth or that the
property has been converted into patrimonial. Without such express
declaration, the property, even if classified as alienable or
disposable, remains property of the public dominion, pursuant to
Article 420(2), and thus incapable of acquisition by prescription. It is
only when such alienable and disposable lands are expressly
declared by the State to be no longer intended for public service or
for the development of the national wealth that the period of
acquisitive prescription can begin to run. Such declaration shall be
in the form of a law duly enacted by Congress or a Presidential
Proclamation in cases where the President is duly authorized by law.

In other words, for one to invoke the provisions of Section 14(2) and set up
acquisitive prescription against the State, it is primordial that the status of the property as
patrimonial be first established. Furthermore, the period of possession preceding the
classification of the property as patrimonial cannot be considered in determining the
completion of the prescriptive period.
[17]



The petitioners application is obviously anchored on Section 14(2) of P.D. No. 1529 as they do not
claim to have possessed, by themselves or their predecessors-in-interest, the subject property since June
12, 1945 or earlier. That it was thru prescription that they had acquired an imperfect title over the subject
property is the foundation upon which the petitioners rest their application.

Unfortunately, this Court finds the evidence presented by the petitioners to be wanting. The
petitioners failed to demonstrate that they and their predecessors-in-interest possessed the property in the
requisite manner, which this Court explained as follows:

It is concerned with lapse of time in the manner and under conditions laid down
by law, namely, that the possession should be in the concept of an owner, public,
peaceful, uninterrupted and adverse. Possession is open when it is patent, visible,
apparent, notorious and not clandestine. It is continuous when uninterrupted, unbroken
and not intermittent or occasional; exclusive when the adverse possessor can show
exclusive dominion over the land and an appropriation of it to his own use and
benefit; and notorious when it is so conspicuous that it is generally known and talked of
by the public or the people in the neighborhood. The party who asserts ownership by
adverse possession must prove the presence of the essential elements of acquisitive
prescription.
[18]



Tax declarations per se do not qualify as competent evidence of actual possession for purposes of
prescription. More so, if the payment of the taxes due on the property is episodic, irregular and random
such as in this case. Indeed, how can the petitioners claim of possession for the entire prescriptive period
be ascribed any ounce of credibility when taxes were paid only on eleven (11) occasions within the 40-
year period from 1961 to 2001? In Wee v. Republic of the Philippines,
[19]
this Court stated that:

It bears stressing that petitioner presented only five tax declarations (for the years 1957,
1961, 1967, 1980 and 1985) for a claimed possession and occupation of more than 45
years (1945-1993). This type of intermittent and sporadic assertion of alleged
ownership does not prove open, continuous, exclusive and notorious possession and
occupation. In any event, in the absence of other competent evidence, tax declarations do
not conclusively establish either possession or declarants right to registration of
title.
[20]
(emphasis supplied and citation omitted)


In East Silverlane, it was emphasized that adverse, continuous, open, public possession in the
concept of an owner is a conclusion of law and the burden to prove it by clear, positive and convincing
evidence is on the applicant. A claim of ownership will not proper on the basis of tax declarations if
unaccompanied by proof of actual possession.
[21]


While there was an attempt to supplement the tax declaration by testimonial evidence, the same is
futile and frivolous. The testimonies of Margarito Pena and Ma. Wilhelmina Tobias do not merit
consideration and do not make up for the inherent inadequacy of the eleven (11) tax declarations
submitted by the petitioners. Such witnesses did not state what specific acts of ownership or dominion
were performed by the petitioners and predecessors-in-interest and simply made that general assertion
that the latter possessed and occupied the subject property for more than thirty (30) years, which, by all
means, is a mere conclusion of law. The RTC should have tackled evidence of such nature with a
disposition to incredulity, if not with an outright rejection.

Furthermore, the petitioners application was filed after only (1) year from the time the subject
property may be considered patrimonial. DARCO Conversion Order No. 040210005-(340)-99, Series of
2000, was issued by the DAR only on July 13, 2000, which means that the counting of the thirty (30)-year
prescriptive period for purposes of acquiring ownership of a public land under Section 14(2) can only
start from such date. Before the property was declared patrimonial by virtue of such conversion order, it
cannot be acquired by prescription. This is clear from the pronouncements of this Court inHeirs of
Malabanan quoted above and in Republic of the Philippines v. Rizalvo,
[22]
which states:

On this basis, respondent would have been eligible for application for registration
because his claim of ownership and possession over the subject property even exceeds
thirty (30) years. However, it is jurisprudentially clear that the thirty (30)-year period of
prescription for purposes of acquiring ownership and registration of public land under
Section 14 (2) of P.D. No. 1529 only begins from the moment the State expressly
declares that the public dominion property is no longer intended for public service or the
development of the national wealth or that the property has been converted into
patrimonial.
[23]



WHEREFORE, premises considered, the instant petition is DENIED for lack of merit. The July 6,
2009 Decision and August 12, 2010 Resolution of the Court of Appeals are AFFIRMED.

SO ORDERED.


BIENVENIDO L. REYES
Associate Justice



WE CONCUR:




ANTONIO T. CARPIO
Associate Justice




ARTURO D. BRION
Associate Justice
JOSE P. PEREZ
Associate Justice




MARIA LOURDES P. A. SERENO
Associate Justice



A T T E S T A T I O N

I attest that the conclusions in the above Resolution had been reached in consultation before the
case was assigned to the writer of the opinion of the Courts Division.



ANTONIO T. CARPIO
Associate Justice
Chairperson, Second Division



C E R T I F I C A T I O N

Pursuant to Section 13, Article VIII of the Constitution and the Division Chairperson's Attestation,
I certify that the conclusions in the above Resolution had been reached in consultation before the case was
assigned to the writer of the opinion of the Courts Division.



RENATO C. CORONA
Chief Justice






[1]
Penned by Associate Justice Monina Arevalo-Zenarosa, with Associate Justices Mariano C.
Del Castillo (now a member of this Court) and Priscilla J. Baltazar-Padilla, concurring; rollo, pp. 52-65.
[2]
Penned by Priscilla J. Baltazar-Padilla, with Associate Justices Magdangal M. De Leon and
Michael P. Elbinias, concurring; id. at 66-68.
[3]
LRC Case No. NC-2001-1205.
[4]
Rollo, p. 53.
[5]
Id. at 57.
[6]
Id. at 63-64.
[7]
Supra note 2.
[8]
Id. at 20.
[9]
Id.
[10]
Id.
[11]
Id. at 21.
[12]
Id.
[13]
Id. at 22.
[14]
Id. at 60.
[15]
G.R. No. 179987, April 29, 2009, 587 SCRA 172.
[16]
G.R. No. 186961, February 20, 2012.
[17]
Supra note at 16.
[18]
Heirs of Marcelina Arzadon-Crisologo v. Raon, G.R. No. 171068, September 5, 2007, 391
SCRA 411, 404.
[19]
G.R. No. 177384, December 8, 2009, 608 SCRA 72
[20]
Id. at 83.
[21]
Supra note at 16.
[22]
G.R. No. 172011, March 7, 2011.
[23]
Id. at.
Republic of the Philippines
Supreme Court
Manila



SECOND DIVISION

REPUBLIC OF THE PHILIPPINES,
Petitioner,




- versus -




EAST SILVERLANE REALTY
DEVELOPMENT CORPORATION,
G.R. No. 186961

Present:

CARPIO, J.,
Chairperson,
VILLARAMA, JR.,
*

PEREZ,
SERENO, and
REYES, JJ.

Promulgated:

Respondent. February 20, 2012

x----------------------------------------------------------------------------------------x

DECISION

REYES, J .:

This Court is urged to review and set aside the July 31, 2008 Decision
[1]
and February 20, 2009
Resolution
[2]
of the Court of Appeals (CA) in CA-G.R. CV No. 00143. In its July 31, 2008 Decision, the
CA affirmed the August 27, 2004 Decision of the Regional Trial Court (RTC), Branch 40 of Cagayan De
Oro City. The dispositive portion thereof states:

WHEREFORE, premises foregoing, the instant appeal is hereby DISMISSED for
lack of merit. The assailed Decision dated August 27, 2004 is herebyAFFIRMED in
toto.

SO ORDERED.
[3]



In its February 20, 2009 Resolution, the CA denied the petitioners August 29, 2008 Motion for
Reconsideration.
[4]


The Factual Antecedents

The respondent filed with the RTC an application for land registration, covering a parcel of land
identified as Lot 9039 of Cagayan Cadastre, situated in El Salvador, Misamis Oriental and with an area of
9,794 square meters. The respondent purchased the portion of the subject property consisting of 4,708
square meters (Area A) from Francisca Oco pursuant to a Deed of Absolute Sale dated November 27,
1990 and the remaining portion consisting of 5,086 square meters (Area B) from Rosario U. Tan Lim,
Nemesia Tan and Mariano U. Tan pursuant to a Deed of Partial Partition with Deed of Absolute Sale
dated April 11, 1991. It was claimed that the respondents predecessors-in-interest had been in open,
notorious, continuous and exclusive possession of the subject property since June 12, 1945.

After hearing the same on the merits, the RTC issued on August 27, 2004 a Decision, granting the
respondents petition for registration of the land in question, thus:

ACCORDINGLY, finding the application meritorious, and pursuant to applicable
law and jurisprudence on the matter, particularly the provisions of P.D. 1529, judgment is
hereby rendered granting the instant application. The Land Registration Authority is
hereby ordered to issue a decree in the name of the applicant EAST SILVERLANE
REALTY DEVELOPMENT CORPORATION covering the parcel of
land, Lot 9039, Cad 237, having an area of 9,794 square meters covered by the two (2)
tax declarations subject of this petition. Based on the
decree, the Register of Deeds for the Province of Misamis Oriental is hereby directed to
issue an original certificate of title in the name of the applicant covering the land subject
matter of this application.
[5]



On appeal by the petitioner, the CA affirmed the RTCs August 27, 2004 Decision. In its July 31,
2008 Decision,
[6]
the CA found no merit in the petitioners appeal, holding that:

It is a settled rule that an application for land registration must conform to three
requisites: (1) the land is alienable public land; (2) the applicants open, continuous,
exclusive and notorious possession and occupation thereof must be since June 12, 1945,
or earlier; and (3) it is a bona fide claim of ownership.

In the case at bench, petitioner-appellee has met all the requirements. Anent the
first requirement, both the report and certification issued by the Department of
Environment and Natural Resources (DENR) shows that the subject land was within the
alienable and disposable zone classified under BF Project [N]o. 8 Blk. I, L.C. Map [N]o.
585 and was released and certified as such on December 31, 1925.

Indubitably, both the DENR certification and report constitute a positive
government act, an administrative action, validly classifying the land in question. It is a
settled rule that the classification or re-classification of public lands into alienable or
disposable, mineral or forest land is now a prerogative of the Executive Department of
the government. Accordingly, the certification enjoys a presumption of regularity in the
absence of contradictory evidence. As it is, the said certification remains uncontested and
even oppositor-appellant Republic itself did not present any evidence to refute the
contents of the said certification. Thus, the alienable and disposable character of the
subject land certified as such as early as December 31, 1925 has been clearly established
by the evidence of the petitioner-appellee.

Anent the second and third requirements, the applicant is required to prove his
open, continuous, exclusive and notorious possession and occupation of the subject land
under a bona fide claim of ownership either since time immemorial or since June 12,
1945.

x x x x

In the case at bench, ESRDC tacked its possession and occupation over the
subject land to that of its predecessors-in-interest. Copies of the tax declarations and real
property historical ownership pertaining thereto were presented in court. A perusal of the
records shows that in 1948, a portion of the subject land was declared under the name of
Agapito Claudel. Subsequently, in 1957 until 1991 the same was declared under the name
of Francisca Oco. Thereafter, the same was declared under the name of ESRDC. A
certification was likewise issued by the Provincial Assessor of Misamis Oriental that
previous tax declarations pertaining to the said portion under the name of Agapita
Claudel could no longer be located as the files were deemed lost or destroyed before
World War II.

On the other hand, the remaining portion of the said land was previously declared
in 1948 under the name of Jacinto Tan Lay Cho. Subsequently, in 1969 until 1990, the
same was declared under the name of Jacinto Tan. Thereafter, the same was declared
under the name of ESRDC. A certification was likewise issued by the Provincial
Assessor that the files of previous tax declarations under the name of Jacinto Tan Lay
Cho were deemed lost or destroyed again before World War II.

In 1991 or upon ESRDCs acquisition of the subject property, the latter took
possession thereto. Albeit it has presently leased the said land to Asia Brewery, Inc.,
where the latter built its brewery plant, nonetheless, ESRDC has its branch office located
at the plant compound of Asia Brewery, Inc.

Corollarily, oppositor-appellants contentions that the court a quo erred in
considering the tax declarations as evidence of ESRDCs possession of the subject land
as the latters predecessors-in-interest declared the same sporadically, is untenable.

It is a settled rule that albeit tax declarations and realty tax payment of property
are not conclusive evidence of ownership, nevertheless, they are good indicia of the
possession in the concept of owner for no one in his right mind would be paying taxes for
a property that is not in his actual or at least constructive possession. They constitute at
least proof that the holder has a claim of title over the property. The voluntary declaration
of a piece of property for taxation purposes manifests not only ones sincere and honest
desire to obtain title to the property and announces his adverse claim against the State and
all other interested parties, but also the intention to contribute needed revenues to the
Government. Such an act strengthens ones bona fide claim of acquisition of ownership.

Finally, it bears stressing that the pieces of evidence submitted by petitioner-
appellee are incontrovertible. Not one, not even oppositor-appellant Republic, presented
any countervailing evidence to contradict the claims of the petitioners that they are in
possession of the subject property and their possession of the same is open, continuous
and exclusive in the concept of an owner for over 30 years.

Verily, from 1948 when the subject land was declared for taxation purposes until
ESRDC filed an application for land registration in 1995, ESRDC have been in
possession over the subject land in the concept of an owner tacking its possession to that
its predecessors-in-interest for forty seven (47) years already. Thus, ESRDC was able to
prove sufficiently that it has been in possession of the subject property for more than 30
years, which possession is characterized as open, continuous, exclusive, and notorious in
the concept of an owner.
[7]
(citations omitted)



The petitioner assails the foregoing, alleging that the respondent failed to prove that its
predecessors-in-interest possessed the subject property in the manner and for the length of time required
under Section 48 (b) of Commonwealth Act No. 141, otherwise known as the Public Land Act (PLA),
and Section 14 of Presidential Decree No. 1529, otherwise known as the Property Registration Decree
(P.D. No. 1529). According to the petitioner, the respondent did not present a credible and competent
witness to testify on the specific acts of ownership performed by its predecessors-in-interest on the
subject property. The respondents sole witness, Vicente Oco, can hardly be considered a credible and
competent witness as he is the respondents liaison officer and he is not related in any way to the
respondents predecessors-in-interest. That coconut trees were planted on the subject property only shows
casual or occasional cultivation and does not qualify as possession under a claim of ownership.

Issue

This Court is confronted with the sole issue of whether the respondent has proven itself entitled to
the benefits of the PLA and P.D. No. 1529 on confirmation of imperfect or incomplete titles.

Our Ruling

This Court resolves to GRANT the petition.

Preliminarily, with respect to the infirmity suffered by this petition from the standpoint of Rule 45,
this Court agrees with the respondent that the issue of whether the respondent had presented sufficient
proof of the required possession under a bona fide claim of ownership raises a question of fact,
considering that it invites an evaluation of the evidentiary record.
[8]
However, that a petition for review
should be confined to questions of law and that this Court is not a trier of facts and bound by the factual
findings of the CA are not without exceptions. Among these exceptions, which obtain in this case, are: (a)
when the judgment of the CA is based on a misapprehension of facts or (b) when its findings are not
sustained by the evidence on record.

This Courts review of the records of this case reveals that the evidence submitted by the
respondent fell short of proving that it has acquired an imperfect title over the subject property under
Section 48 (b) of the PLA. The respondent cannot register the subject property in its name on the basis of
either Section 14 (1) or Section 14 (2) of P.D. No. 1529. It was not established by the required quantum
of evidence that the respondent and its predecessors-in-interest had been in open, continuous, exclusive
and notorious possession of the subject property for the prescribed statutory period.

The PLA governs the classification and disposition of lands of the public domain. Under Section 11
thereof, one of the modes of disposing public lands suitable for agricultural purposes is by confirmation
of imperfect or incomplete titles.
[9]
On the other hand, Section 48 provides the grant to the qualified
possessor of an alienable and disposable public land. Thus:

SEC. 48. The following-described citizens of the Philippines, occupying lands of the
public domain or claiming to own any such lands or an interest therein, but whose titles
have not been perfected or completed, may apply to the Court of First Instance of the
province where the land is located for confirmation of their claims and the issuance of a
certificate of title therefor, under the Land Registration Act, to wit:

(a) Those who prior to the transfer of sovereignty from Spain to the United States
have applied for the purchase, composition or other form of grant of lands of the public
domain under the laws and royal decrees then in force and have instituted and prosecuted
the proceedings in connection therewith, but have with or without default upon their part,
or for any other cause, not received title therefor, if such applicants or grantees and their
heirs have occupied and cultivated said lands continuously since the filing of their
applications.

(b) Those who by themselves or through their predecessors in interest have been
in open, continuous, exclusive, and notorious possession and occupation of agricultural
lands of the public domain, under a bona fide claim of acquisition or ownership, for at
least thirty years immediately preceding the filing of the application for confirmation of
title except when prevented by war or force majeure. These shall be conclusively
presumed to have performed all the conditions essential to a Government grant and shall
be entitled to a certificate of title under the provisions of this chapter.

(c) Members of the national cultural minorities who by themselves or through
their predecessors-in-interest have been in open, continuous, exclusive and notorious
possession and occupation of lands of the public domain suitable to agriculture, whether
disposable or not, under a bona fide claim of ownership for at least 30 years shall be
entitled to the rights granted in sub-section (b) hereof.


Presidential Decree No. 1073 (P.D. No. 1073), which was issued on January 25, 1977, deleted
subsection (a) and amended subsection (b) as follows:

SECTION 4. The provisions of Section 48 (b) and Section 48 (c), Chapter VIII of the
Public Land Act are hereby amended in the sense that these provisions shall apply only to
alienable and disposable lands of the public domain which have been in open,
continuous, exclusive and notorious possession and occupation by the applicant thru
himself or thru his predecessor-in-interest under a bona fide claim of ownership since
June 12, 1945.


Notably, the first PLA, or Act No. 926, required a possession and occupation for a period of ten
(10) years prior to the effectivity of Act No. 2096 on July 26, 1904 or on July 26, 1894. This was adopted
in the PLA until it was amended by Republic Act No. 1942 on June 22, 1957, which provided for a period
of thirty (30) years. It was only with the enactment of P.D. No. 1073 on January 25, 1977 that it was
required that possession and occupation should commence on June 12, 1945.

P.D. No. 1529, which was enacted on June 11, 1978, codified all the laws relative to the
registration of property. Section 14 thereof partially provides:

Section 14. Who may apply. The following persons may file in the proper Court
of First Instance an application for registration of title to land, whether personally or
through their duly authorized representatives:

(1) Those who by themselves or through their predecessors-in-interest have been
in open, continuous, exclusive and notorious possession and occupation of alienable and
disposable lands of the public domain under a bona fide claim of ownership since June
12, 1945, or earlier.

(2) Those who have acquired ownership of private lands by prescription under
the provision of existing laws.

(3) Those who have acquired ownership of private lands or abandoned river beds
by right of accession or accretion under the existing laws.

(4) Those who have acquired ownership of land in any other manner provided for
by law.


Section 14 (1) and Section 14 (2) are clearly different. Section 14 (1) covers alienable and
disposable land while Section 14 (2) covers private property. As this Court categorically stated
in Heirs of Malabanan v. Republic of the Philippines,
[10]
the distinction between the two provisions lies
with the inapplicability of prescription to alienable and disposable lands. Specifically:

At the same time, Section 14 (2) puts into operation the entire regime of
prescription under the Civil Code, a fact which does not hold true with respect to Section
14 (1).
[11]



Property is either part of the public domain or privately owned.
[12]
Under Article 420 of the Civil
Code, the following properties are of public dominion:

(a) Those intended for public use, such as roads, canals, rivers, torrents,
ports and bridges constructed by the State, banks, shores, roadsteads and others of similar
character;

(b) Those which belong to the State, without being for public use, and
are intended for some public service or for the development of the national wealth.


All other properties of the State, which is not of the character mentioned in Article 420 is
patrimonial property,
[13]
hence, susceptible to acquisitive prescription.
[14]


In Heirs of Malabanan, this Court ruled that possession and occupation of an alienable and
disposable public land for the periods provided under the Civil Code do not automatically convert said
property into private property or release it from the public domain. There must be an express declaration
that the property is no longer intended for public service or development of national wealth. Without such
express declaration, the property, even if classified as alienable or disposable, remains property of the
State, and thus, may not be acquired by prescription.

Nonetheless, Article 422 of the Civil Code states that [p]roperty of public
dominion, when no longer intended for public use or for public service, shall form part of
the patrimonial property of the State. It is this provision that controls how public
dominion property may be converted into patrimonial property susceptible to acquisition
by prescription. After all, Article 420 (2) makes clear that those property which belong
to the State, without being for public use, and are intended for some public service or for
the development of the national wealth are public dominion property. For as long as the
property belongs to the State, although already classified as alienable or disposable,
it remains property of the public dominion if when it is intended for some public
service or for the development of the national wealth. (emphasis supplied)

Accordingly, there must be an express declaration by the State that the
public dominion property is no longer intended for public service or the
development of the national wealth or that the property has been converted into
patrimonial. Without such express declaration, the property, even if classified as
alienable or disposable, remains property of the public dominion, pursuant to
Article 420(2), and thus incapable of acquisition by prescription. It is only when
such alienable and disposable lands are expressly declared by the State to be no
longer intended for public service or for the development of the national wealth that
the period of acquisitive prescription can begin to run. Such declaration shall be in
the form of a law duly enacted by Congress or a Presidential Proclamation in cases
where the President is duly authorized by law.
[15]



In other words, for one to invoke the provisions of Section 14 (2) and set up acquisitive
prescription against the State, it is primordial that the status of the property as patrimonial be first
established. Furthermore, the period of possession preceding the classification of the property as
patrimonial cannot be considered in determining the completion of the prescriptive period.

To prove that its predecessors-in-interest were in possession of the subject property on or prior to
June 12, 1945 or had completed the prescriptive period of thirty (30) years, the respondent submitted the
following tax declarations:

a) Tax Declaration in the name of Agapita Claudel for the year 1948;

b) Tax Declarations in the name of Francisca Oco for the years 1957,
1963, 1969, 1973, 1974, 1980, 1987, 1989 and 1991;

c) Tax Declarations in the respondents name for the years 1991, 1992
and 1994;

d) Tax Declarations in the name of Jacinto Tan Lay Cho for the years
1948 and 1952;

e) Tax Declarations in the name of Jacinto Tan for the years 1969,
1973, 1974, 1980, 1989 and 1990; and

f) Tax Declarations in the respondents name for the years 1991, 1992
and 1994.

Pursuant to Agapita Claudels 1948 Tax Declaration, there were nineteen (19) coconut and ten
(10) banana trees planted on Area A. The coconut trees were supposedly four years old, hence, the
reasonable presumption that she had been in possession even before June 12, 1945.
[16]


The respondent also offered the following testimony of Vicente Oco:

Q Mr. Witness, If you know about what period your predecessor has started to
possess this land subject matter of this application?

A Per my personal knowledge, it was before the second world war but the
Municipality of El Salvador was created on June 15, 1948 by virtue of RA 268 and its
started to officially function only on August 2, 1948[.]

Q From whom did you acquire this information?

A From the seller and the adjoining lot owners.
[17]



To prove that its predecessors-in-interest exercised acts of dominion over the subject property, the
respondent claimed that per Francisca Ocos Tax Declarations, the following improvements were
introduced in Area A: nineteen (19) coconut and ten (10) banana trees in Area A in 1957 and 1963; thirty-
three (33) coconut trees in 1969 and 1973; thirty-three (33) coconut trees, one (1) mango tree and three
(3) seguidillas vines in 1974; thirty-three (33) coconut trees in 1980; eighty-seven (87) coconut trees in
1987; and fifteen (15) coconut trees in 1989. Per Jacinto Tans Tax Declarations, there were fifty-seven
(57) coconut trees in Area B in 1973, 1974, 1980, 1989 and 1990.
[18]


A reading of the CAs July 31, 2008 Decision shows that it affirmed the grant of the respondents
application given its supposed compliance with Section 14 (2) of P.D. No. 1529. It ruled that based on the
evidence submitted, the respondent is not qualified to register the subject property in its name under
Section 14 (1) as the possession and occupation of its predecessors-in-interest commenced after June 12,
1945. Nonetheless, as the CA ruled, the respondent acquired title to the subject property by prescription
as its predecessors-in-interest had possessed the subject property for more than thirty (30) years.
Citing Buenaventura v. Republic of the Philippines,
[19]
the CA held that even if possession commenced
after June 12, 1945, registration is still possible under Section 14 (2) and possession in the concept of an
owner effectively converts an alienable and disposable public land into private property.

This Court, however, disagrees on the conclusion arrived at by the CA. On the premise that the
application for registration, which was filed in 1995, is based on Section 14 (2), it was not proven that the
respondent and its predecessors-in-interest had been in possession of the subject property in the manner
prescribed by law and for the period necessary before acquisitive prescription may apply.

While the subject land was supposedly declared alienable and disposable on December 31, 1925
per the April 18, 1997 Certification and July 1, 1997 Report of the Community Environment and Natural
Resources Office (CENRO),
[20]
the Department of Agrarian Reform (DAR) converted the same from
agricultural to industrial only on October 16, 1990.
[21]
Also, it was only in 2000 that
the Municipality of El Salvador passed a Zoning Ordinance, including the subject property in the
industrial zone.
[22]
Therefore, it was only in 1990 that the subject property had been declared patrimonial
and it is only then that the prescriptive period began to run. The respondent cannot benefit from the
alleged possession of its predecessors-in-interest because prior to the withdrawal of the subject property
from the public domain, it may not be acquired by prescription.

On the premise that the application of the respondent is predicated on Section 14 (1), the same
would likewise not prosper. As shown by the tax declarations of the respondents predecessors-in-interest,
the earliest that the respondent can trace back the possession of its predecessors-in-interest is in 1948.
That there were four-year old coconut trees in Area A as stated in Agapita Claudels 1948 Tax
Declaration cannot be considered a well-nigh controvertible evidence that she was in possession prior
to June 12, 1945 without any evidence that she planted and cultivated them. In the case of Jacinto Tan
Lay Cho, the earliest tax declaration in his name is dated 1948 and there is no evidence that he occupied
and possessed Area B on or prior to June 12, 1945. Furthermore, the testimony of the respondents lone
witness that the respondents predecessors-in-interest were already in possession of the subject property
as of June 12, 1945 lacks probative value for being hearsay.

It is explicit under Section 14 (1) that the possession and occupation required to acquire an
imperfect title over an alienable and disposable public land must be open, continuous, exclusive and
notorious in character. In
Republic of the Philippines v. Alconaba,
[23]
this Court explained that the intent behind the use of
possession in conjunction with occupation is to
emphasize the need for actual and not just constructive or fictional possession.

The law speaks of possession and occupation. Since these words are separated by
the conjunction and, the clear intention of the law is not to make one synonymous with
the other. Possession is broader than occupation because it includes constructive
possession. When, therefore, the law adds the word occupation, it seeks to delimit the all
encompassing effect of constructive possession. Taken together with the words open,
continuous, exclusive and notorious, the word occupation serves to highlight the fact that
for an applicant to qualify, his possession must not be a mere fiction. Actual possession
of a land consists in the manifestation of acts of dominion over it of such a nature as a
party would naturally exercise over his own property.
[24]
(citations omitted)


On the other hand, Section 14 (2) is silent as to the required nature of possession and occupation,
thus, requiring a reference to the relevant provisions of the Civil Code on prescription. And under Article
1118 thereof, possession for purposes of prescription must be in the concept of an owner, public,
peaceful and uninterrupted. In Heirs of Marcelina Arzadon-Crisologo v. Raon,
[25]
this Court expounded
on the nature of possession required for purposes of prescription:

It is concerned with lapse of time in the manner and under conditions laid down by law,
namely, that the possession should be in the concept of an owner, public, peaceful,
uninterrupted and adverse. Possession is open when it is patent, visible, apparent,
notorious and not clandestine. It is continuous when uninterrupted, unbroken and not
intermittent or occasional; exclusive when the adverse possessor can show exclusive
dominion over the land and an appropriation of it to his own use and benefit; and
notorious when it is so conspicuous that it is generally known and talked of by the public
or the people in the neighborhood. The party who asserts ownership by adverse
possession must prove the presence of the essential elements of acquisitive
prescription.
[26]
(citations omitted)


This Court is not satisfied with the evidence presented by the respondent to prove compliance with
the possession required either under Section 14 (1) or Section 14 (2).

First, the twelve (12) Tax Declarations covering Area A and the eleven (11) Tax Declarations
covering Area B for a claimed possession of more than forty-six (46) years (1948-1994) do not qualify as
competent evidence of actual possession and occupation. As this Court ruled in Wee v. Republic of the
Philippines:
[27]



It bears stressing that petitioner presented only five tax declarations (for the years 1957,
1961, 1967, 1980 and 1985) for a claimed possession and occupation of more than 45
years (1945-1993). This type of intermittent and sporadic assertion of alleged
ownership does not prove open, continuous, exclusive and notorious possession and
occupation. In any event, in the absence of other competent evidence, tax declarations do
not conclusively establish either possession or declarants right to registration of
title.
[28]
(emphasis supplied and citation omitted)


The phrase adverse, continuous, open, public, and in concept of owner, by which the
respondent describes its possession and that of its predecessors-in-interest is a conclusion of law. The
burden of proof is on the respondent to prove by clear, positive and convincing evidence that the alleged
possession of its predecessors-in-interest was of the nature and duration required by law.
[29]
It is therefore
inconsequential if the petitioner failed to present evidence that would controvert the allegations of the
respondent. A person who seeks the registration of title to a piece of land on the basis of possession by
himself and his predecessors-in-interest must prove his claim by clear and convincing evidence, i.e., he
must prove his
title and should not rely on the absence or weakness of the evidence of the oppositors.
[30]


The respondents claim of ownership will not prosper on the basis of the tax declarations alone.
In Cequea v. Bolante,
[31]
this Court ruled that it is only when these tax declarations are coupled with
proof of actual possession of the property that they may become the basis of a claim of ownership.
[32]
In
the absence of actual public and adverse possession, the declaration of the land for tax purposes does not
prove ownership.
[33]


Second, that the nineteen (19) coconut trees supposedly found on Area A were four years old at
the time Agapita Claudel filed a Tax Declaration in 1948 will not suffice as evidence that her possession
commenced prior to June 12, 1945, in the absence of evidence that she planted and cultivated them.
Alternatively, assuming that Agapita Claudel planted and maintained these trees, such can only be
considered casual cultivation considering the size of Area A. On the other hand, that Jacinto Tan Lay
Cho possessed Area B in the concept of an owner on or prior to June 12, 1945 cannot be assumed from
his 1948 Tax Declaration.

Third, that plants were on the subject property without any evidence that it was the respondents
predecessors-in-interest who planted them and that actual cultivation or harvesting was made does not
constitute well-nigh incontrovertible evidence of actual possession and occupation. As this Court ruled
in Wee:

We are, therefore, constrained to conclude that the mere existence of an
unspecified number of coffee plants, sans any evidence as to who planted them, when
they were planted, whether cultivation or harvesting was made or what other acts of
occupation and ownership were undertaken, is not sufficient to demonstrate petitioners
right to the registration of title in her favor.
[34]



Fourth, Vicente Ocos testimony deserves scant consideration and will not supplement the
inherent inadequacy of the tax declarations. Apart from being self-serving, it is undoubtedly hearsay.
Vicente Oco lacks
personal knowledge as to when the predecessors-in-interest of the respondent started to occupy the
subject property and admitted that his testimony was based on what he allegedly gathered from the
respondents predecessors-in-interest and the owners of adjoining lot. Moreover, Vicente Oco did not
testify as to what specific acts of dominion or ownership were performed by the respondents
predecessors-in-interest and if indeed they did. He merely made a general claim that they came into
possession before World War II, which is a mere conclusion of law and not factual proof of possession,
and therefore unavailing and cannot suffice.
[35]
Evidence of this nature should have been received with
suspicion, if not dismissed as tenuous and unreliable.

Finally, that the respondents application was filed after only four years from the time the subject
property may be considered patrimonial by reason of the DARs October 26, 1990 Order shows lack of
possession whether for ordinary or extraordinary prescriptive period. The principle enunciated in Heirs of
Malabanan cited above was reiterated and applied in Republic of the Philippines v. Rizalvo:
[36]


On this basis, respondent would have been eligible for application for registration
because his claim of ownership and possession over the subject property even exceeds
thirty (30) years. However, it is jurisprudentially clear that the thirty (30)-year period of
prescription for purposes of acquiring ownership and registration of public land under
Section 14 (2) of P.D. No. 1529 only begins from the moment the State expressly
declares that the public dominion property is no longer intended
for public service or the development of the national wealth or that the property has been
converted into patrimonial.
[37]



WHEREFORE, premises considered, the instant petition is GRANTED. The July 31, 2008
Decision and February 20, 2009 Resolution of the Court of Appeals in CA-G.R. CV No. 00143
are REVERSED and
SET ASIDE and the respondents application for registration of title over Lot 9039 of Cagayan Cadastre
is hereby DENIED for lack of merit.

SO ORDERED.



BIENVENIDO L. REYES
Associate Justice







WE CONCUR:




ANTONIO T. CARPIO
Associate Justice




MARTIN S. VILLARAMA, JR.
Associate Justice
JOSE PORTUGAL PEREZ
Associate Justice




MARIA LOURDES P. A. SERENO
Associate Justice


A T T E S T A T I O N

I attest that the conclusions in the above Decision had been reached in consultation before the case
was assigned to the writer of the opinion of the Courts Division.




ANTONIO T. CARPIO
Associate Justice
Chairperson, Second Division


C E R T I F I C A T I O N

Pursuant to Section 13, Article VIII of the Constitution and the Division Chairperson's Attestation,
I certify that the conclusions in the above Decision had been reached in consultation before the case was
assigned to the writer of the opinion of the Courts Division.




RENATO C. CORONA
Chief Justice



*
Additional Member in lieu of Associate Justice Arturo D. Brion per Special Order No. 1195
dated February 15, 2012.
[1]
Penned by Associate Justice Rodrigo F. Lim, Jr., with Associate Justices Michael P. Elbinias
and Ruben C. Ayson, concurring; rollo, pp. 43-54.
[2]
Id. at 56.
[3]
Id. at 54.
[4]
Id. at 57-61.
[5]
Id. at 108-109.
[6]
Supra note 1.
[7]
Rollo, pp. 48-54.
[8]
Republic of the Philippines v. Manna Properties, Inc., 490 Phil. 654, 665 (2005).
[9]
Sec. 11. Public lands suitable for agricultural purposes can be disposed of only as follows, and
not otherwise:
(1) For homestead settlement;
(2) By sale;
(3) By lease;
(4) By confirmation of imperfect or incomplete titles;
(a) By judicial legalization;
(b) By administrative legalization (free patent).
[10]
G.R. No. 179987, April 29, 2009, 587 SCRA 172.
[11]
Id. at 201.
[12]
Article 419, Civil Code.
[13]
Article 421, Civil Code.
[14]
Supra note 10, at 202.
[15]
Id. at 203.
[16]
Rollo, p. 102.
[17]
Id. at 102-103.
[18]
Id. at 99-101.
[19]
G.R. No. 166865, March 2, 2007, 517 SCRA 271.
[20]
Rollo, p. 142.
[21]
Id. at 84, 133.
[22]
Id. at 89-90, 138-140.
[23]
471 Phil. 607 (2004).
[24]
Id. at 620.
[25]
G.R. No. 171068, September 5, 2007, 532 SCRA 391.
[26]
Id. at 404.
[27]
G.R. No. 177384, December 8, 2009, 608 SCRA 72.
[28]
Id. at 83.
[29]
See The Director, Lands Mgt. Bureau v. Court of Appeals, 381 Phil. 761, 772 (2000).
[30]
Arbias v. Republic of the Philippines, G.R. No. 173808, September 17, 2008, 565 SCRA 582,
597.
[31]
386 Phil. 419 (2000).
[32]
Id. at 430.
[33]
Id. at 431.
[34]
Supra note 27, at 84.
[35]
Supra note 29, at 770.
[36]
G.R. No. 172011, March 7, 2011.
Republic of the Philippines
SUPREME COURT
Manila
FIRST DIVISION
G.R. No. 163766 June 22, 2006
REPUBLIC OF THE PHILIPPINES, Petitioner,
vs.
CANDY MAKER, INC., as represented by its President, ONG YEE SEE,
*
Respondent
D E C I S I O N
CALLEJO, SR., J .:
At bar is a Petition for Review under Rule 45 of the Rules of Court seeking to set aside the May
21, 2004 Decision
1
of the Court of Appeals (CA) in CA-G.R. CV No. 73287, which affirmed in
toto the October 12, 2001 Decision
2
of the Municipal Trial Court (MTC) of Taytay, Rizal in Land
Registration Case No. 99-0031 declaring respondent the owner of the parcels of land
designated as Lots 3138-A and 3138-B in Plan CSD. 04-018302, Cainta-Taytay Cadastre.
Sometime in 1998, Candy Maker, Inc. decided to purchase Lot No. 3138 Cad. 688 of the
Cainta-Taytay Cadastre, a parcel of land located below the reglementary lake elevation of 12.50
meters, about 900 meters away from the Laguna de Bay, and bounded on the southwest by the
Manggahan Floodway, and on the southeast by a legal easement.
On April 1, 1998, Geodetic Engineer Potenciano H. Fernandez, prepared and signed a
Subdivision Plan of the property for Apolonio Cruz. The property was subdivided into two lots:
Lot No. 3138-A with an area of 10,971 square meters, and Lot No. 3138-B with an area of 239
square meters.
3
The technical description of Lot No. 3138 was also prepared by Fernandez, and
was approved by the Regional Technical Director of the Bureau of Lands on April 14, 1998.
4

On April 29, 1999, Antonio, Eladia, and Felisa, all surnamed Cruz, executed a Deed of Absolute
Sale in favor of Candy Maker, Inc.
5
The buyer declared Lot No. 3138 for taxation purposes in
1999 under Tax Declaration Nos. 004-18929, 004-18930 and 004-18931.
6

On June 16, 1999, Candy Maker, Inc., as applicant, filed an application with the MTC of Taytay,
Rizal, for the registration of its alleged title over Lot No. 3138-A and Lot No. 3138-B under
Presidential Decree (P.D.) No. 1529.
Acting thereon, the MTC issued an Order
7
on June 18, 1999 directing the applicant to cause the
publication of the notice of initial hearing and for the Deputy Sheriff to post the same. The
Administrator of the Land Registration Authority (LRA) and the Directors of the Land
Management Bureau (LMB) and Forest Management Bureau (FMB) were also instructed to
submit their respective reports on the status of the parcels of land before the initial hearing
scheduled on October 29, 1999.
The Community Environment and Natural Resources Officer (CENRO) of Antipolo City filed on
August 18, 1999 his Report
8
declaring that "[t]he land falls within the Alienable and Disposable
Zone, under Land Classification Project No. 5-A, per L.C. Map No. 639 certified released on
March 11, 1927" and that the property is the subject of CENRO Case No. 520(97) entitled
Perpetua San Jose v. Almario Cruz. On the other hand, the LRA, in its September 21, 1999
Report,
9
recommended the exclusion of Lot No. 3138-B on the ground that it is a legal easement
and intended for public use, hence, inalienable and indisposable.
On September 30, 1999, the Laguna Lake Development Authority (LLDA) approved Resolution
No. 113, Series of 1993, providing that untitled shoreland areas may be leased subject to
conditions enumerated therein.
The applicant filed its Amended Application
10
on December 15, 1999 for the confirmation of its
alleged title on Lot No. 3138, alleging therein that:
1. x x x the applicant is the President of CANDYMAKER[,] INC. and registered owner of a parcel
of land located at Panghulo Brgy. San Juan, Taytay, Rizal with an area of TEN THOUSAND
NINE HUNDRED SEVENTY ONE (10,971) square meters and as fully described and bounded
under Lot 3138-A plan CSD-04-018302[,] copy of which and the corresponding technical
descriptions are hereto attached to form parts hereof;
x x x x
8. That for Lot 3138-A the applicant hereby prays for the benefit granted under the Land
Registration Act and/or under the benefits provided for by P.D. No. 1529, as applicant and their
predecessors-in-interest have been in open, public, continuous, and peaceful occupation and
possession of the said land since time immemorial in [the] concept of true owners and [adverse]
to the whole world; x x x
11

On March 27, 2000, the MTC issued an Order
12
admitting the Amended Application and
resetting the initial hearing to June 23, 2000. However, upon the requests of the LRA for the
timely publication of the Notice of Initial Hearing in the Official Gazette,
13
the court moved the
hearing date to September 22, 2000,
14
then on January 26, 2001
15
and until finally, to June 15,
2001.
16

On July 20, 2001, the Republic of the Philippines, the LLDA filed its Opposition
17
to the
Amended Application in which it alleged that the lot subject of the application for registration
may not be alienated and disposed since it is considered part of the Laguna Lake bed, a public
land within its jurisdiction pursuant to Republic Act (R.A.) No. 4850, as amended. According to
the LLDA, the projection of Lot No. 3138-A, Cad-688-D Csd-04-018302 in its topographic map
based on the Memorandum
18
of Engineer Christopher Pedrezuela of the Engineering and
Construction Division of the LLDA indicated that it is "located below the reglementary lake
elevation of 12.50 meters referred to datum 10.00 meters below mean lower water" and under
Section 41(11) of R.A. No. 4850, the property is a public land which forms part of the bed of the
Laguna Lake. This Memorandum was appended to the application.
At the hearing conducted on August 31, 2001, the applicant marked in evidence the
complementary copies of the Official Gazette and the Peoples Tonight as Exhibits "E-1" and "F-
1," respectively.
19

Except as to the LLDA and the Office of the Solicitor General (OSG), which was represented by
the duly deputized provincial prosecutor,
20
the court, upon motion of the applicant, issued an
Order of general default.
21

The applicant presented as witnesses its Treasurer, Fernando Co Siy, and Antonio Cruz, one of
the vendees.
Cruz testified that his grandparents owned the property,
22
and after their demise, his parents,
the spouses Apolonio Cruz and Aquilina Atanacio Cruz, inherited the lot;
23
he and his father had
cultivated the property since 1937, planting palay during the rainy season and vegetables during
the dry season; his father paid the realty taxes on the property,
24
and he (Cruz) continued
paying the taxes after his fathers death.
25
Cruz insisted that he was the rightful claimant and
owner of the property.
Sometime in the 1980s, Apolonio Cruz executed an extrajudicial deed of partition in which the
property was adjudicated to Antonio Cruz and his sisters, Felisa and Eladia, to the exclusion of
their five (5) other siblings who were given other properties as their shares.
26
He did not know
why his ancestors failed to have the property titled under the Torrens system of
registration.
27
He left the Philippines and stayed in Saudi Arabia from 1973 to 1983.
28
Aside
from this, he hired the services of an "upahan" to cultivate the property.
29
The property is about
3 kilometers from the Laguna de Bay, and is usually flooded when it rains.
30

Fernando Co Siy testified that the applicant acquired Lot No. 3138 from siblings Antonio, Eladia
and Felisa,
31
who had possessed it since 1945;
32
that after paying the real estate taxes due
thereon,
33
it caused the survey of the lot;
34
that possession thereof has been peaceful
35
and
none of the former owners claims any right against it;
36
neither the applicant nor its
predecessors-in-interest received information from any government agency that the lot is a
public land;
37
the subject lot is 3 kms. away from Laguna de Bay,
38
above its elevation and that
of the nearby road;
39
the property is habitable
40
and was utilized as a riceland at the time it was
sold by the former owners;
41
and that he was aware that a legal easement is affecting the lot
and is willing to annotate it in the land title.
42

On cross-examination by the LLDA counsel, Siy admitted that his knowledge as to the distance
of the lot with respect to the Laguna de Bay came from "somebody residing in Taytay" and also
from an adjacent owner of the lot;
43
that the lot is submerged in water since there is no land fill
yet;
44
and that no improvements had been introduced to the property.
45

The LLDA moved for a joint ocular inspection of the parcels of land in order to determine its
exact elevation.
46
On September 14, 2001, a Survey Team of the Engineering and Construction
Division of the LLDA, composed of Ramon D. Magalonga, Virgilio M. Polanco, and Renato Q.
Medenilla, conducted an actual ground survey of the property. The team used a total station and
digital survey instrument to measure the elevation of the ground in reference to the elevation of
the lake water. A representative of the applicant witnessed the survey. The team found that the
lot is below the prescribed elevation of 12.50 m. and thus part of the bed of the lake; as such, it
could not be titled to the applicant. The team also reported that the property is adjacent to the
highway from the Manggahan Floodway to Angono, Rizal. The LLDA moved that the application
be withdrawn, appending thereto a copy of the Survey Report.
47

The LLDA did not offer any testimonial and documentary evidence and agreed to submit the
case for decision based on its Opposition.
On October 12, 2001, the MTC rendered a Decision granting the application for registration over
the lots. The dispositive portion of the decision reads:
WHEREFORE, premises considered[,] the court hereby rendered judgment confirming title of
the applicants over the real property denominated as Lot 3138-A Csd-04-018302 of Cad-688-D
Cainta-Taytay Cadastre; Lot 3138-B Csd-04-018302 of Cad 688-D Cainta-Taytay Cadastre.
48

On appeal to the CA, the petitioner contended that the MTC did not acquire jurisdiction over the
application for registration since the actual copies of the Official Gazette (O.G.) where the notice
of hearing was published were not adduced in evidence; the applicant likewise failed to
establish exclusive ownership over the subject property in the manner prescribed by law. The
petitioner argued further that the requirements of Section 23, par. 1 of P.D. No. 1529,
49
as
amended, are mandatory and jurisdictional, and that failure to observe such requirements has a
fatal effect on the whole proceedings. Citing Republic of the Philippines v. Court of
Appeals
50
and Register of Deeds of Malabon v. RTC, Malabon, MM, Br. 170,
51
the Republic
averred that a mere certificate of publication is inadequate proof of the jurisdictional fact of
publication because the actual copies of the O.G. must be presented at the initial hearing of the
case. Moreover, witnesses were not presented to prove specific acts to show that the applicant
and his predecessors-in-interest have been in exclusive, open, continuous, and adverse
possession of the subject lots in the concept of the owner since June 12, 1945 or earlier, in
accordance with Sec. 14, par. 1 of P.D. No. 1529.
52
It noted that the testimonies of the
applicants witnesses are more of conclusions of law rather than factual evidence of ownership.
Other than the general statement that they planted rice and vegetables on the subject lots, their
possession could properly be characterized as mere casual cultivation since they failed to
account for its exclusive utilization since 1945 or earlier. After stressing that tax declarations are
not conclusive proof of ownership, it concluded that the subject lots rightfully belong to the State
under the Regalian doctrine.
53

The applicant averred in its Appellees Brief
54
that it had marked in evidence the actual copy of
the O.G. where the notice of initial hearing was published; in fact, the MTC Decision stated that
the copy of the O.G. containing the notice was referred to as Exhibit "E-1." Moreover, Sec. 14,
par. 1 of P.D. 1529 is inapplicable since it speaks of possession and occupation of alienable
and disposable lands of the public domain. Instead, par. 4 of the same section
55
should govern
because the subject parcels of land are lands of private ownership, having being acquired
through purchase from its predecessors-in-interest, who, in turn, inherited the same from their
parents. It pointed out that there were no adverse claims of interest or right by other private
persons and even government agencies like the Province of Rizal. Lastly, while tax declarations
and tax receipts do not constitute evidence of ownership, they are nonetheless prima facie
evidence of possession.
On May 21, 2004, the appellate court rendered judgment which dismissed the appeal and
affirmed in toto the Decision of the MTC,
56
holding that the copy of the O.G., where the notice
was published, was marked as Exhibit "E-1" during the initial hearing. On the issue of ownership
over the subject lots, the CA upheld the applicants claim that the parcels of land were alienable
and not part of the public domain, and that it had adduced preponderant evidence to prove that
its predecessors had been tilling the land since 1937, during which palay and vegetables were
planted. In fact, before the lots were purchased, the applicant verified their ownership with the
assessors office, and thereafter caused the property to be surveyed; after the lots were
acquired in 1999 and a survey was caused by the applicant, no adverse claims were filed by
third persons. Further, the CA ruled that tax declarations or tax receipts are good indicia of
possession in the concept of the owner, which constitute at least positive and strong indication
that the taxpayer concerned has made a claim either to the title or to the possession of the
property.
The Republic, now petitioner, filed the instant Petition for Review on the following issues:
A.
WHETHER THE LAND IN QUESTION MAYBE THE SUBJECT OF REGISTRATION.
B.
WHETHER THE COURT A QUO ACQUIRED JURISDICTION OVER
THE RES CONSIDERING ITS INALIENABLE CHARACTER.
C.
WHETHER THE COURT OF APPEALS ERRED IN AFFIRMING THE TRIAL COURTS
FINDING THAT RESPONDENT COMPLIED WITH THE LEGAL REQUIREMENTS ON
POSSESSION AS MANDATED BY SECTION 14 OF P.D. NO. 1529.
57

Petitioner asserts that the Engineers Survey Report
58
and the Laguna de Bay Shoreland
Survey
59
both show that Lot No. 3138-A is located below the reglementary lake elevation,
hence, forms part of the Laguna Lake bed. It insists that the property belongs to the public
domain as classified under Article 502 of the Civil Code.
60
Citing the ruling of this Court in
Bernardo v. Tiamson,
61
petitioner avers that the subject lot is incapable of private appropriation
since it is a public land owned by the State under the Regalian doctrine. On this premise,
petitioner avers that the MTC did not acquire jurisdiction over the subject matter, and as a
consequence, its decision is null and void.
Petitioner maintains that respondent failed to present incontrovertible evidence to warrant the
registration of the property in its name as owner. The testimonies of the two witnesses only
proved that the possession of the land may be characterized as mere casual cultivation; they
failed to prove that its predecessors occupied the land openly, continuously, exclusively,
notoriously and adversely in the concept of owner since June 12, 1945 or earlier.
On the other hand, respondent argues that the Engineers Survey Report and the Laguna de
Bay Shoreland Survey have no probative value because they were neither offered nor admitted
in evidence by the MTC. It points out that petitioner failed to invoke these reports in the
appellate court.
It was only when the petition was filed with this Court that the respondent learned of its
existence. Petitioners reliance on the reports/survey is merely an afterthought. The case of
Bernardo v. Tiamson is irrelevant because the factual issues are different from those of this
case.
On April 28, 2005, respondent filed a Manifestation
62
with this Court, appending thereto the
report
63
conducted by the survey team of the LLDA Engineering and Construction Division on
April 12, 2005. It stated that the 10,971 sq m property subject of the case is below the 12.5
elevation, and that the profile distance of the property from the actual lake waters is about 900
m. to 1 km.
The issues in this case are the following: (1) whether the MTC had jurisdiction over the
amended application; (2) whether the property subject of the amended application is alienable
and disposable property of the State, and, if so, (3) whether respondent adduced the requisite
quantum of evidence to prove its ownership over the property under Section 14 of P.D. 1529.
The petition is meritorious.
On the first issue, we find and so rule that the MTC acquired jurisdiction over respondents
application for registration since a copy of the O.G. containing the notice of hearing was marked
and adduced in evidence as Exhibit "E-1." The representative of the OSG was present during
the hearing and interposed his objection thereto.
On the second and third issues, we find and so rule that the property subject of this application
was alienable and disposable public agricultural land until July 18, 1966. However, respondent
failed to prove that it possesses registerable title over the property.
Section 48(b) of Commonwealth Act No. 141, as amended by R.A. No. 1942, reads:
Section 48. The following described citizens of the Philippines, occupying lands of the public
domain or claiming to own any such lands or an interest therein, but whose titles have not been
perfected or completed, nay apply to the Court of First Instance of the province where the land
is located for confirmation of their claims and the issuance of a certificate of title therefor, under
the Land Registration Act, to wit:
(b) Those who by themselves or through their predecessors in-interest have been in open,
continuous, exclusive, and notorious possession and occupation of agricultural lands of the
public domain, under a bona fide claim of acquisition of ownership, for at least thirty years
immediately preceding the filing of the application for confirmation of title except when
prevented by war or force majeure. These shall be conclusively presumed to have performed all
the conditions essential to a Government grant and shall be entitled to a certificate of title under
the provisions of this chapter.
This provision was further amended by P.D. No. 1073 by substituting the phrase "for at least
thirty years" with "since June 12, 1945;" thus:
Sec. 4. The provisions of Section 48(b) and Section 48(c), Chapter VIII, of the Public Land Act
are hereby amended in the sense that these provisions shall apply only to alienable and
disposable lands of the public domain which have been in open, continuous, exclusive and
notorious possession, and occupation by the applicant himself or through his predecessor-in-
interest, under a bona fide claim of acquisition of ownership, since June 12, 1945.
Section 14(1) of P.D. No. 1529, otherwise known as the Property Registration Decree, provides:
SEC. 14. Who may apply. The following persons may file in the proper Court of First Instance
[now Regional Trial Court] an application for registration of title to land, whether personally or
through their duly authorized representatives:
(1) Those who by themselves or through their predecessors-in-interest have been in open,
continuous, exclusive and notorious possession and occupation of alienable and disposable
lands of the public domain under a bona fide claim of ownership since June 12, 1945, or earlier
(emphasis supplied).
Applicants for confirmation of imperfect title must, therefore, prove the following: (a) that the
land forms part of the disposable and alienable agricultural lands of the public domain; and (b)
that they have been in open, continuous, exclusive, and notorious possession and occupation of
the same under a bona fide claim of ownership either since time immemorial or since June 12,
1945.
64

Under the Regalian doctrine, all lands not otherwise appearing to be clearly within private
ownership are presumed to belong to the State. The presumption is that lands of whatever
classification belong to the State.
65
Unless public land is shown to have been reclassified as
alienable or disposable to a private person by the State, it remains part of the inalienable public
domain. Property of the public domain is beyond the commerce of man and not susceptible of
private appropriation and acquisitive prescription. Occupation thereof in the concept of owner no
matter how long cannot ripen into ownership and be registered as a title.
66
The statute of
limitations with regard to public agricultural lands does not operate against the State unless the
occupant proves possession and occupation of the same after a claim of ownership for the
required number of years to constitute a grant from the State.
67

No public land can be acquired by private persons without any grant from the government,
whether express or implied. It is indispensable that there be a showing of a title from the
State.
68
The rationale for the period "since time immemorial or since June 12, 1945" lies in the
presumption that the land applied for pertains to the State, and that the occupants or possessor
claim an interest thereon only by virtue of their imperfect title as continuous, open and notorious
possession.
A possessor of real property may acquire ownership thereof through acquisitive prescription. In
Alba Vda. de Raz v. Court of Appeals,
69
the Court declared that:
x x x [W]hile Art. 1134 of the Civil Code provides that (o)wnership and other real rights over
immovable property are acquired by ordinary prescription through possession of ten years, this
provision of law must be read in conjunction with Art. 1117 of the same Code. This article states
that x x x (o)rdinary acquisitive prescription of things requires possession in good faith and with
just title for the time fixed by law. Hence, a prescriptive title to real estate is not acquired by
mere possession thereof under claim of ownership for a period of ten years unless such
possession was acquired con justo titulo y buena fe (with color of title and good faith). The good
faith of the possessor consists in the reasonable belief that the person from whom he received
the thing was the owner thereof, and could transmit his ownership. For purposes of prescription,
there is just title when the adverse claimant came into possession of the property through one of
the recognized modes of acquisition of ownership or other real rights but the grantor was not the
owner or could not transmit any right.
70

To prove that the land subject of an application for registration is alienable, an applicant must
conclusively establish the existence of a positive act of the government such as a presidential
proclamation or an executive order, or administrative action, investigation reports of the Bureau
of Lands investigator or a legislative act or statute.
71
Until then, the rules on confirmation of
imperfect title do not apply. A certification of the Community Environment and Natural
Resources Officer in the Department of Environment and Natural Resources stating that the
land subject of an application is found to be within the alienable and disposable site per a land
classification project map is sufficient evidence to show the real character of the land subject of
the application.
72

The applicant is burdened to offer proof of specific acts of ownership to substantiate the claim
over the land.
73
Actual possession consists in the manifestation of acts of dominion over it of
such a nature as a party would actually exercise over his own property.
74
A mere casual
cultivation of portions of the land by the claimant does not constitute sufficient basis for a claim
of ownership; such possession is not exclusive and notorious as to give rise to a presumptive
grant from the State.
75

In this case, the evidence on record shows that the property is alienable agricultural land.
Romeo Cadano of the Community Environment and Natural Resources Office, Antipolo Rizal,
certified that the property "falls within the Alienable and Disposable zone, under Land
Classification Project No. 5-A, per L.C. Map No. 639 certified released on March 11,
1927."
76
However, under R.A. No. 4850 which was approved on July 18, 1966, lands located at
and below the maximum lake level of elevation of the Laguna de Bay are public lands which
form part of the bed of said lake. Such lands denominated as lakeshore areas are linear strips
of open space designed to separate incompatible element or uses, or to control
pollution/nuisance, and for identifying and defining development areas or zone. Such areas of
the lake with an approximate total area of 14,000 hectares form a strip of the lakebed along its
shores alternately submerged or exposed by the annual rising and lowering of the lake water.
They have environmental ecological significance and actual potential economic benefits.
Under Section 1 of the law, the national policy of the State is to promote and accelerate the
development and balanced growth of the Laguna Lake area and the surrounding provinces,
cities and towns within the context of the national and regional plans and policies for social and
economic development and to carry out the development of the Laguna Lake region with due
regard and adequate provisions for environmental management and control, preservation of the
quality of human life and ecological systems, and the prevention of undue ecological
disturbances, deterioration and pollution.
The rapid expansion of Metropolitan Manila, the suburbs and the lakeshore town of Laguna de
Bay, combined with current and prospective uses of the lake for municipal-industrial water
supply, irrigation, fisheries, and the like, created deep concern on the part of the Government
and the general public over the environmental impact of such development, on the water quality
and ecology of the lake and its related river systems. The inflow of polluted water from the Pasig
River, industrial, domestic and agricultural wastes from developed areas around the lake and
the increasing urbanization have induced the deterioration of the lake, and that water quality
studies have shown that the lake will deteriorate further if steps are not taken to check the
same. The floods in the Metropolitan Manila area and the lakeshore towns are also influenced
by the hydraulic system of the Laguna de Bay, and any scheme of controlling the floods will
necessarily involve the lake and its river systems.
This prompted then President Ferdinand E. Marcos to issue on October 17, 1978 P.D. 813
amending Rep. Act No. 4850. Under Section 6 of the law, the LLDA is empowered to issue such
rules and regulations as may be necessary to effectively carry out the policies and programs
therein provided including the policies and projects of the LLDA, subject to the approval of the
National Economic Development Authority.
In 1996, the Board of Directors of LLDA approved Resolution No. 113, series of 1996 relating to
the Environmental Uses Fee Systems and Approval of the Work and Financial Plan for its
operationalization in the Laguna de Bay Basin. Section 5 of the Resolution provides that the
LLDA as a matter of policy is to maintain all shoreland areas lying below elevation 12.50 meters
as buffer zone in consonance with the LLDA policies, plans programs for the improvement of
the water quality and pollution and conservation of the water resources of the Laguna de Bay.
As gleaned from the Survey Report of Magalonga, Polanco and Medenilla of the LLDA based
on the ocular inspection dated September 14, 2001 as well as the Memorandum of Engineer
Christopher Pedrezuela, the property is located below the reglementary level of 12.50 m.;
hence, part of the bed of the Laguna de Bay, and, as such, is public land. Although the Report
and Memorandum were not offered as evidence in the MTC, the respondent admitted in its
Manifestation in this Court that the property is situated below the 12.50 elevation based on the
survey of Magalonga, Polanco and Medenilla, the same survey team who conducted an ocular
inspection of the property on April 12, 2005, which thus confirmed the September 14, 2001
survey report. This is a judicial admission in the course of judicial proceedings which is binding
on it.
77

Under R.A. No. 4850 and the issuances of LLDA, registerable rights acquired by occupants
before the effectivity of the law are recognized. However, the respondent failed to adduce proof
that its predecessors-in-interest had acquired registerable title over the property before July 18,
1966:
First. Cruz failed to prove how his parents acquired ownership of the property, and even
failed to mention the names of his grandparents. He likewise failed to present his fathers
death certificate to support his claim that the latter died in 1980. There is likewise no
evidence when his mother died.
Second. Cruz also failed to adduce in evidence the extrajudicial partition allegedly
executed by his parents in 1980 where the property was supposedly deeded to him and
his sisters, Felisa and Eladia, to the exclusion of their five siblings.
Third. Cruz claimed that he and his parents cultivated the property and planted palay
and vegetables, and that they had been paying the realty taxes over the property before
his parents died. However, no tax declarations under the names of the spouses Apolonio
Cruz and/or Eladia Cruz and his siblings were presented, or realty tax receipts
evidencing payment of such taxes. Indeed, while tax receipts and tax payment receipts
themselves do not convincingly prove title to the land,
78
these are good indicia of
possession in the concept of an owner, for no one in his right mind would pay taxes for a
property that is not in his actual or, at least, constructive possession.
79
While tax receipts
and declarations are not incontrovertible evidence of ownership, they constitute, at the
least, proof that the holder has a claim of title over the property, particularly when
accompanied by proof of actual possession of property.
80
The voluntary declaration of a
piece of property for taxation purposes not only manifests ones sincere and honest
desire to obtain title to the property, but also announces an adverse claim against the
State and all other interested parties with an intention to contribute needed revenues to
the government. Such an act strengthens ones bona fide claim of acquisition of
ownership.
81

Fourth. When he testified on October 5, 2001, Antonio Cruz declared that he was "74
years old."
82
He must have been born in 1927, and was thus merely 10 years old in
1937. It is incredible that, at that age, he was already cultivating the property with his
father. Moreover, no evidence was presented to prove how many cavans of palay were
planted on the property, as well as the extent of such cultivation, in order to support the
claim of possession with a bona fide claim of ownership.
Fifth. Cruz testified that he hired a worker "upahan" to help him cultivate the property.
He, however, failed to state the name of the worker or to even present him as witness for
the respondent.
IN LIGHT OF ALL THE FOREGOING, the petition is GRANTED. The decision of the Court of
Appeals in CA-G.R. CV No. 73278 is SET ASIDE. The Municipal Trial Court of Taytay, Rizal is
DIRECTED to dismiss the application for registration of respondent Candymaker, Inc. in Land
Registration Case No. 99-0031. No costs.
SO ORDERED.
ROMEO J. CALLEJO, SR.
Associate Justice
WE CONCUR:
ARTEMIO V. PANGANIBAN
Chief Justice
Chairperson
CONSUELO YNARES-SANTIAGO
Associate Justice
MA. ALICIA AUSTRIA-MARTINEZ
Asscociate Justice
MINITA V. CHICO-NAZARIO
Associate Justice
C E R T I F I C A T I O N
Pursuant to Section 13, Article VIII of the Constitution, it is hereby certified that the conclusions
in the above decision were reached in consultation before the case was assigned to the writer of
the opinion of the Courts Division.
ARTEMIO V. PANGANIBAN
Chief Justice
SECOND DIVISION
[G.R. No. 144057. January 17, 2005]
REPUBLIC OF THE PHILIPPINES, petitioner, vs. THE HONORABLE COURT OF APPEALS
and CORAZON NAGUIT,respondents.
D E C I S I O N
TINGA, J .:
This is a Petition for Review on Certiorari under Rule 45 of the 1997 Rules of Civil
Procedure, seeking to review the Decision
[1]
of the Sixth Division of the Court of Appeals dated
July 12, 2000 in CA-G.R. SP No. 51921. The appellate court affirmed the decisions of both the
Regional Trial Court (RTC),
[2]
Branch 8, of Kalibo, Aklan dated February 26, 1999, and the
7
th
Municipal Circuit Trial Court (MCTC)
[3]
of Ibajay-Nabas, Aklan dated February 18, 1998,
which granted the application for registration of a parcel of land of Corazon Naguit (Naguit), the
respondent herein.
The facts are as follows:
On January 5, 1993, Naguit, a Filipino citizen, of legal age and married to Manolito S.
Naguit, filed with the MCTC of Ibajay-Nabas, Aklan, a petition for registration of title of a parcel
of land situated in Brgy. Union, Nabas, Aklan. The parcel of land is designated as Lot No.
10049, Cad. 758-D, Nabas Cadastre, AP 060414-014779, and contains an area of 31,374
square meters. The application seeks judicial confirmation of respondents imperfect title over
the aforesaid land.
On February 20, 1995, the court held initial hearing on the application. The public
prosecutor, appearing for the government, and Jose Angeles, representing the heirs of Rustico
Angeles, opposed the petition. On a later date, however, the heirs of Rustico Angeles filed a
formal opposition to the petition. Also on February 20, 1995, the court issued an order of
general default against the whole world except as to the heirs of Rustico Angeles and the
government.
The evidence on record reveals that the subject parcel of land was originally declared for
taxation purposes in the name of Ramon Urbano (Urbano) in 1945 under Tax Declaration No.
3888 until 1991.
[4]
On July 9, 1992, Urbano executed a Deed of Quitclaim in favor of the heirs of
Honorato Maming (Maming), wherein he renounced all his rights to the subject property and
confirmed the sale made by his father to Maming sometime in 1955 or 1956.
[5]
Subsequently,
the heirs of Maming executed a deed of absolute sale in favor of respondent Naguit who
thereupon started occupying the same. She constituted Manuel Blanco, Jr. as her attorney-in-
fact and administrator. The administrator introduced improvements, planted trees, such as
mahogany, coconut and gemelina trees in addition to existing coconut trees which were then 50
to 60 years old, and paid the corresponding taxes due on the subject land. At present, there are
parcels of land surrounding the subject land which have been issued titles by virtue of judicial
decrees. Naguit and her predecessors-in-interest have occupied the land openly and in the
concept of owner without any objection from any private person or even the government until
she filed her application for registration.
After the presentation of evidence for Naguit, the public prosecutor manifested that the
government did not intend to present any evidence while oppositor Jose Angeles, as
representative of the heirs of Rustico Angeles, failed to appear during the trial despite notice.
On September 27, 1997, the MCTC rendered a decision ordering that the subject parcel be
brought under the operation of the Property Registration Decree or Presidential Decree (P.D.)
No. 1529 and that the title thereto registered and confirmed in the name of Naguit.
[6]

The Republic of the Philippines (Republic), thru the Office of the Solicitor General (OSG),
filed a motion for reconsideration. The OSG stressed that the land applied for was declared
alienable and disposable only on October 15, 1980, per the certification from Regional
Executive Director Raoul T. Geollegue of the Department of Environment and Natural
Resources, Region VI.
[7]
However, the court denied the motion for reconsideration in an order
dated February 18, 1998.
[8]

Thereafter, the Republic appealed the decision and the order of the MCTC to the RTC,
Kalibo, Aklan, Branch 8. On February 26, 1999, the RTC rendered its decision, dismissing the
appeal.
[9]

Undaunted, the Republic elevated the case to the Court of Appeals via Rule 42 of the 1997
Rules of Civil Procedure. On July 12, 2000, the appellate court rendered a decision dismissing
the petition filed by the Republic and affirmed in toto the assailed decision of the RTC.
Hence, the present petition for review raising a pure question of law was filed by the
Republic on September 4, 2000.
[10]

The OSG assails the decision of the Court of Appeals contending that the appellate court
gravely erred in holding that there is no need for the governments prior release of the subject
lot from the public domain before it can be considered alienable or disposable within the
meaning of P.D. No. 1529, and that Naguit had been in possession of Lot No. 10049 in the
concept of owner for the required period.
[11]

Hence, the central question for resolution is whether is necessary under Section 14(1) of
the Property Registration Decree that the subject land be first classified as alienable and
disposable before the applicants possession under a bona fide claim of ownership could even
start.
The OSG invokes our holding in Director of Lands v. Intermediate Appellate Court
[12]
in
arguing that the property which is in open, continuous and exclusive possession must first be
alienable. Since the subject land was declared alienable only on October 15, 1980, Naguit could
not have maintained a bona fide claim of ownership since June 12, 1945, as required by Section
14 of the Property Registration Decree, since prior to 1980, the land was not alienable or
disposable, the OSG argues.
Section 14 of the Property Registration Decree, governing original registration proceedings,
bears close examination. It expressly provides:
SECTION 14. Who may apply. The following persons may file in the proper Court of First Instance an
application for registration of title to land, whether personally or through their duly authorized
representatives:
(1) those who by themselves or through their predecessors-in-interest have been in open,
continuous, exclusive and notorious possession and occupation of alienable and
disposable lands of the public domain under a bona fide claim of ownership since June
12, 1945, or earlier.
(2) Those who have acquired ownership over private lands by prescription under the
provisions of existing laws.
. . . .
There are three obvious requisites for the filing of an application for registration of title
under Section 14(1) that the property in question is alienable and disposable land of the public
domain; that the applicants by themselves or through their predecessors-in-interest have been
in open, continuous, exclusive and notorious possession and occupation, and; that such
possession is under a bona fide claim of ownership since June 12, 1945 or earlier.
Petitioner suggests an interpretation that the alienable and disposable character of the land
should have already been established since June 12, 1945 or earlier. This is not borne out by
the plain meaning of Section 14(1). Since June 12, 1945, as used in the provision, qualifies its
antecedent phrase under a bonafide claim of ownership. Generally speaking, qualifying words
restrict or modify only the words or phrases to which they are immediately associated, and not
those distantly or remotely located.
[13]
Ad proximum antecedents fiat relation nisi impediatur
sentencia.
Besides, we are mindful of the absurdity that would result if we adopt petitioners position.
Absent a legislative amendment, the rule would be, adopting the OSGs view, that all lands of
the public domain which were not declared alienable or disposable before June 12, 1945 would
not be susceptible to original registration, no matter the length of unchallenged possession by
the occupant. Such interpretation renders paragraph (1) of Section 14 virtually inoperative and
even precludes the government from giving it effect even as it decides to reclassify public
agricultural lands as alienable and disposable. The unreasonableness of the situation would
even be aggravated considering that before June 12, 1945, the Philippines was not yet even
considered an independent state.
Instead, the more reasonable interpretation of Section 14(1) is that it merely requires the
property sought to be registered as already alienable and disposable at the time the application
for registration of title is filed. If the State, at the time the application is made, has not yet
deemed it proper to release the property for alienation or disposition, the presumption is that the
government is still reserving the right to utilize the property; hence, the need to preserve its
ownership in the State irrespective of the length of adverse possession even if in good faith.
However, if the property has already been classified as alienable and disposable, as it is in this
case, then there is already an intention on the part of the State to abdicate its exclusive
prerogative over the property.
This reading aligns conformably with our holding in Republic v. Court of
Appeals.
[14]
Therein, the Court noted that to prove that the land subject of an application for
registration is alienable, an applicant must establish the existence of a positive act of the
government such as a presidential proclamation or an executive order; an administrative action;
investigation reports of Bureau of Lands investigators; and a legislative act or a statute.
[15]
In
that case, the subject land had been certified by the DENR as alienable and disposable in 1980,
thus the Court concluded that the alienable status of the land, compounded by the established
fact that therein respondents had occupied the land even before 1927, sufficed to allow the
application for registration of the said property. In the case at bar, even the petitioner admits that
the subject property was released and certified as within alienable and disposable zone in 1980
by the DENR.
[16]

This case is distinguishable from Bracewell v. Court of Appeals,
[17]
wherein the Court noted
that while the claimant had been in possession since 1908, it was only in 1972 that the lands in
question were classified as alienable and disposable. Thus, the bid at registration therein did
not succeed. In Bracewell, the claimant had filed his application in 1963, or nine (9) years
before the property was declared alienable and disposable. Thus, in this case, where the
application was made years after the property had been certified as alienable and disposable,
the Bracewell ruling does not apply.
A different rule obtains for forest lands,
[18]
such as those which form part of a reservation for
provincial park purposes
[19]
the possession of which cannot ripen into ownership.
[20]
It is
elementary in the law governing natural resources that forest land cannot be owned by private
persons. As held in Palomo v. Court of Appeals,
[21]
forest land is not registrable and possession
thereof, no matter how lengthy, cannot convert it into private property, unless such lands are
reclassified and considered disposable and alienable.
[22]
In the case at bar, the property in
question was undisputedly classified as disposable and alienable; hence, the ruling in Palomo is
inapplicable, as correctly held by the Court of Appeals.
[23]

It must be noted that the present case was decided by the lower courts on the basis of
Section 14(1) of the Property Registration Decree, which pertains to original registration through
ordinary registration proceedings. The right to file the application for registration derives from
a bona fide claim of ownership going back to June 12, 1945 or earlier, by reason of the
claimants open, continuous, exclusive and notorious possession of alienable and disposable
lands of the public domain.
A similar right is given under Section 48(b) of the Public Land Act, which reads:
Sec. 48. The following described citizens of the Philippines, occupying lands of the public domain or
claiming to own any such land or an interest therein, but those titles have not been perfected or
completed, may apply to the Court of First Instance of the province where the land is located for
confirmation of their claims and the issuance of a certificate of title therefor, under the Land Registration
Act, to wit:
xxx xxx xxx
(b) Those who by themselves or through their predecessors in interest have been in open, continuous,
exclusive, and notorious possession and occupation of agricultural lands of the public domain, under a
bona fide claim of acquisition of ownership, for at least thirty years immediately preceding the filing of
the application for confirmation of title except when prevented by war or force majeure. These shall be
conclusively presumed to have performed all the conditions essential to a Government grant and shall be
entitled to a certificate of title under the provisions of this chapter.
When the Public Land Act was first promulgated in 1936, the period of possession deemed
necessary to vest the right to register their title to agricultural lands of the public domain
commenced from July 26, 1894. However, this period was amended by R.A. No. 1942, which
provided that the bona fide claim of ownership must have been for at least thirty (30) years.
Then in 1977, Section 48(b) of the Public Land Act was again amended, this time by P.D. No.
1073, which pegged the reckoning date at June 12, 1945. This new starting point is concordant
with Section 14(1) of the Property Registration Decree.
Indeed, there are no material differences between Section 14(1) of the Property
Registration Decree and Section 48(b) of the Public Land Act, as amended. True, the Public
Land Act does refer to agricultural lands of the public domain, while the Property Registration
Decree uses the term alienable and disposable lands of the public domain. It must be noted
though that the Constitution declares that alienable lands of the public domain shall be limited
to agricultural lands.
[24]
Clearly, the subject lands under Section 48(b) of the Public Land Act
and Section 14(1) of the Property Registration Decree are of the same type.
Did the enactment of the Property Registration Decree and the amendatory P.D. No. 1073
preclude the application for registration of alienable lands of the public domain, possession over
which commenced only after June 12, 1945? It did not, considering Section 14(2) of the
Property Registration Decree, which governs and authorizes the application of those who have
acquired ownership of private lands by prescription under the provisions of existing laws.
Prescription is one of the modes of acquiring ownership under the Civil Code.
[25]
There is a
consistent jurisprudential rule that properties classified as alienable public land may be
converted into private property by reason of open, continuous and exclusive possession of at
least thirty (30) years.
[26]
With such conversion, such property may now fall within the
contemplation of private lands under Section 14(2), and thus susceptible to registration by
those who have acquired ownership through prescription. Thus, even if possession of the
alienable public land commenced on a date later than June 12, 1945, and such possession
being been open, continuous and exclusive, then the possessor may have the right to register
the land by virtue of Section 14(2) of the Property Registration Decree.
The land in question was found to be cocal in nature, it having been planted with coconut
trees now over fifty years old.
[27]
The inherent nature of the land but confirms its certification in
1980 as alienable, hence agricultural. There is no impediment to the application of Section 14(1)
of the Property Registration Decree, as correctly accomplished by the lower courts.
The OSG posits that the Court of Appeals erred in holding that Naguit had been in
possession in the concept of owner for the required period. The argument begs the question. It
is again hinged on the assertionshown earlier to be unfoundedthat there could have been
no bona fide claim of ownership prior to 1980, when the subject land was declared alienable or
disposable.
We find no reason to disturb the conclusion of both the RTC and the Court of Appeals that
Naguit had the right to apply for registration owing to the continuous possession by her and her
predecessors-in-interest of the land since 1945. The basis of such conclusion is primarily
factual, and the Court generally respects the factual findings made by lower courts. Notably,
possession since 1945 was established through proof of the existence of 50 to 60-year old trees
at the time Naguit purchased the property as well as tax declarations executed by Urbano in
1945. Although tax declarations and realty tax payment of property are not conclusive evidence
of ownership, nevertheless, they are good indicia of the possession in the concept of owner for
no one in his right mind would be paying taxes for a property that is not in his actual or at least
constructive possession. They constitute at least proof that the holder has a claim of title over
the property. The voluntary declaration of a piece of property for taxation purposes manifests
not only ones sincere and honest desire to obtain title to the property and announces his
adverse claim against the State and all other interested parties, but also the intention to
contribute needed revenues to the Government. Such an act strengthens ones bona fide claim
of acquisition of ownership.
[28]

Considering that the possession of the subject parcel of land by the respondent can be
traced back to that of her predecessors-in-interest which commenced since 1945 or for almost
fifty (50) years, it is indeed beyond any cloud of doubt that she has acquired title thereto which
may be properly brought under the operation of the Torrens system. That she has been in
possession of the land in the concept of an owner, open, continuous, peaceful and without any
opposition from any private person and the government itself makes her right thereto
undoubtedly settled and deserving of protection under the law.
WHEREFORE, foregoing premises considered, the assailed Decision of the Court of
Appeals dated July 12, 2000 is hereby AFFIRMED. No costs.
SO ORDERED.
Puno, (Chairman), Austria-Martinez, Callejo, Sr., and Chico-Nazario, JJ., concur.



[1]
Penned by Justice P. Alio-Hormachuelos, concurred in by Justices A. Austria-Martinez and
E.J. Asuncion.
[2]
Penned by Judge E. Terencio.
[3]
Penned by Judge R. Barrios.
[4]
Rollo, p. 31.
[5]
Ibid.
[6]
Id. at 50.
[7]
Id. at 40.
[8]
Id. at 16; but see 103.
[9]
Id. at 77.
[10]
Id. at 10.
[11]
Id. at 19.
[12]
G.R. No. 65663, 16 October 1992, 214 SCRA 604.
[13]
R. AGPALO, STATUTORY CONSTRUCTION, 3
rd
ed., 1995 at 182.
[14]
G.R. No. 127060, 19 November 2002, 392 SCRA 190.
[15]
Id. at 201.
[16]
Rollo, p. 21.
[17]
380 Phil. 156 (2000).
[18]
See e.g., Almeda v. Court of Appeals, G.R. No. 85322, 30 April 1991, 196 SCRA 476, 480;
Director of Lands v. Court of Appeals, 218 Phil. 666,674 (1984); Heirs of Amunategui v.
Director of Forestry, 211 Phil 260 (1983); Pagkatipunan v. Court of Appeals 429 Phil.
377 (2002).
[19]
See Palomo v. Court of Appeals, 334 Phil 357 (1997).
[20]
Director of Lands v. Court of Appeals, supra note 12 citing Director of Forestry v. Muoz,
G.R. No. 24796, 28 June 1968, 23 SCRA 1183.
[21]
Supra note 19.
[22]
Id. citing Vano v. Government of P.I., 41 Phil. 161 [1920]; Li Seng Giap y CIAA v. Director, 55
Phil. 693 [1931]; Fernandez Hermanos v. Director, 57 Phil. 929 [1931]; Military
Reservations v. Marcos, 52 SCRA 238 [1973]; Republic v. Court of Appeals, 154 SCRA
476; Vallarta v. IAC, 152 SCRA 679; Director of Forest Administration v. Fernandez, 192
SCRA 121.
[23]
See Rollo, at 35.
[24]
Section 3, Article XII, Constitution.
[25]
See Article 1113, Civil Code, which states: All things which are within the commerce of men
are susceptible of prescription, unless otherwise provided. Property of the State or any
of its subdivisions not patrimonial in character shall not be the object of prescription.
[26]
See e.g., Director of Lands v. IAC, G.R. No. 65663, 16 October 1992, 214 SCRA 604, 611;
Republic v. Court of Appeals, G.R. No. 108998, 24 August 1994, 235 SCRA 567, 576;
Group Commander, Intelligence and Security Group v. Dr. Malvar, 438 Phil. 252, 275
(2002).
[27]
Rollo, p. 35.
[28]
Director of Lands v. Court of Appeals, 367 Phil. 597 (1999); Director of Lands v. Intermediate
Appellate Court, G.R. No. 70825, March 11, 1991, 195 SCRA 38; Rivera v. Court of
Appeals, G.R. No. 130876, January 31, 2002, 244 SCRA 218; Republic v. Court of
Appeals, 325 Phil. 674 (1996); Heirs of Placido Miranda v. Court of Appeals, 255 SCRA
368; Alonso v. Cebu Country Club, Inc., 375 SCRA 390.
SECOND DIVISION

CRISOLOGO C. DOMINGO,
Petitioner,




- versus -




SEVERINO AND RAYMUNDO
LANDICHO, JULIAN ABELLO,
MARTA DE SAGUN AND EDITHA G.
SARMIENTO,
Respondents.

G.R. No. 170015

Present:

QUISUMBING, J., Chairperson,
CARPIO,
CARPIO MORALES,
TINGA, and
VELASCO, JR., JJ.


Promulgated:

August 29, 2007

x - - - - - - - - - - - - - - - - - - - - - - - - - - - - - - - - - - - - - - - - - - - -x

D E C I S I O N

CARPIO MORALES, J .:
Crisologo C. Domingo (Domingo) filed on April 20, 1993 with the Regional Trial Court (RTC)
of Tagaytay City an application for registration,
[1]
docketed as LRC No. TG-451, of five parcels of land
delineated as Lot Nos. 7513, 7515, 7516, 7517 and 7518, Cad. 355 under Approved Survey Plan AS-04-
002475
[2]
(the lots).

The lots, which are located at Barangay Tolentino, Tagaytay, have a total land area of 38,975
square meters.

In his application, Domingo claimed that he bought the lots from Genoveva Manlapit (Genoveva)
in 1948 and has since been in continuous, open, public, adverse and uninterrupted possession thereof in
the concept of an owner.

Domingo further claimed that prior to his purchase of the lots, Genoveva had been in possession
thereof in the concept of an owner for more than 30 years.
[3]


To Domingos application the following documents were attached:
1. Tracing Cloth of Approved Plan AS-04-002475 (surveyed from September 24,
1963 to February 13, 1964 and approved on December 12, 1990).
[4]


2. Photocopy of the Technical Description of Lot Nos. 7513, 7515, 7516, 7517,
and 7518.
[5]


3. Photocopy of the Geodetic Engineers Certificate.
[6]


4. Owners Copy of Tax Declaration Nos. GR-019-0893-R (covering Lot 7513),
GR-019-0894-R (covering Lot 7515), GR-019-0895-R (covering Lot 7516), GR-
019-0896-R (covering Lot 7517), GR-019-0897-R (covering Lot 7518), all
dated January 7, 1993 and in the name of Crisologo C. Domingo.
[7]


5. Land Management Inspectors 2
nd
Indorsement dated October 22,
1990 recommending approval of AS-Plan.
[8]



The Land Registration Authority (LRA), which filed before the RTC its
Report
[9]
dated September 27, 1993, stated that after plotting Plan AS-04-002475 in the Municipal Index
Sheet thru its tie lines, a discrepancy was noted. The RTC thus referred the matter to the Lands
Management Sector, Region IV for verification and correction.

Acting on the directive of the RTC, the Director of Lands filed a Report that per records of the
Lands Management Bureau in Manila, the land involved in said case was not covered by any land patent
or by public land application pending issuance of patent.
[10]


The LRA later filed a Supplementary Report
[11]
dated December 22, 1993 stating that:

x x x x
2. The Regional Technical Director, Region Office IV, thru the Chief, Surveys
Division, Robert C. Pangyarihan in his letter dated November 22, 1993, a copy is
attached hereto as Annex A, informed that per records on file in that Office, the
correct adjoining survey along line 8-9 of Lot 7516 and along lines 3-4-5 of Lot
7515 should be Lot 9237 Cad-355, covered by As-04-000091 and that the parcel of
land covered by As-04-002475 are not portions of or identical to any previously
approved isolated survey; and

3. When the above-furnished correction was applied on plan As-04-002475 no more
discrepancy exists.

x x x x

On November 26, 1993, herein respondents Severino and Raymundo Landicho, Julian Abello,
Marta de Sagun, and Editha G. Sarmiento filed an Answer/Opposition
[12]
to Domingos application,
claiming that they have been in open, continuous, adverse and actual possession and cultivation of the lots
in the concept of an owner and have been paying real estate taxes thereon;
[13]
and that Survey Plan AS-04-
002475 was lifted from the cadastral survey of the government which was surveyed for them and other
individual owners.
[14]


During the pendency of his application or on March 9, 1996, Domingo died. His counsel,
Atty. Irineo Anarna, did not, however, inform the RTC of his death.
By Decision
[15]
of December 22, 1997, the RTC approved Domingos application for registration,
the dispositive portion of which reads:

WHEREFORE, in the light of the foregoing premises and considerations, this
Court hereby renders judgment approving the instant application for registration and thus
places under the operation of Act 141, Act 496 and/or P.D. 1529, otherwise known as the
Property Registration Law, the lands described in Plan AS-04-002475 as Lots 7513,
7515, 7516, 7517 and 7518, Cad-355, Tagaytay Cadastre, containing an area of 10,519
square meters, 3, 956 square meters, 18, 921 square meters, 3, 985 square meters and 1,
594 square meters, respectively, as supported by their technical descriptions now forming
parts of the records of this case, in addition to other proofs adduced, in the name of
CRISOLOGO C. DOMINGO, Filipino, of legal age, married to Corazon A. Domingo,
and with residence at No. 34 Dao St., Project 3, Quezon City, Metro Manila.

Once this decision becomes final and executory, the corresponding decree of
registration shall forthwith issue.

SO ORDERED.
[16]



Respondents appealed to the Court of Appeals, contending that contrary to Domingos claim that
he and his predecessors-in-interest have been in actual, continuous and uninterrupted possession of the
lots, Domingo has always been a resident of No. 34 Dao St., Project 3, Quezon City; that despite
Domingos claim that he has a caretaker overseeing the lots, he could not even give the name of the
caretaker; and that Domingo admittedly declared the lots in his name only in 1993.

By Decision
[17]
of June 30, 2005, the Court of Appeals reversed and set aside the RTC decision
and dismissed Domingos application for registration of land title.

The appellate court ruled that while Domingo sought judicial confirmation of his imperfect title
under the Public Land Act and Section 14 (1) of Presidential Decree (P.D.) No. 1529, THE PROPERTY
REGISTRATION DECREE, he failed to prove that he and his predecessors-in-interest had been in
possession and occupation of the lots under a bona fide claim of ownership since June 12, 1945 or
earlier.
[18]


And the appellate court noted that Domingo failed to present the alleged deed of sale executed
by Genoveva
[19]
and could only prove through his Tax Declaration No. 0298 (new) that his possession in
the concept of an owner started only in 1948 (Exhibit L, Records, p. 117).

Domingos Motion for Reconsideration having been denied by the appellate court, the present
petition was lodged, faulting the appellate court:

I

. . . x x x x WHEN IT LIMITED CONSIDERATION OF THE MATTERS
ESTABLISHED IN THE APPLICATION TO SECTION 48 (B) OF THE PUBLIC
LAND ACT AND SECTION 14 (1) OF PD 1529.

II

. . . x x x WHEN IT HELD THAT PETITIONER IS NOT ENTITLED FOR
REGISTRATION OF TITLE OVER THE SUBJECT LAND, NOTWITHSTANDING
THE FACT THAT THE EVIDENCE ON RECORD CLEARLY ESTABLISHED HIS
ENTITLEMENT [TO] REGISTRATION OF TITLE OVER THE LAND UNDER
SECTION 14 (1) AND (4) OF PD 1529.
[20]
(Underscoring supplied)


Domingos present counsel argues that assuming that Domingo failed to establish his possession
from June 12, 1945 or earlier in accordance with Section 14(1) of P.D. No. 1529, he is still entitled to
registration of title under Article 1113
[21]
in relation to Article 1137
[22]
of the Civil Code.
[23]

In their Comment
[24]
to the present petition, respondents pray for its denial for being substantially
defective, Domingos death not having been alleged, albeit the Verification and Certification against
Forum Shopping was signed by Domingos alleged Surviving Spouse and Heirs.
[25]


To respondents Comment, Domingos counsel filed a Reply
[26]
stating that there is no clearer
manifestation of the death of Domingo than the statement under oath of his surviving spouse and heirs in
substitution of deceased CRISOLOGO C. DOMINGO contained in the Verification and Certification
against Forum Shopping which forms part of the present petition.
[27]
Nonetheless, the counsel presented a
certified true copy of Domingos death certificate
[28]
showing that he died on March 9, 1996 (during
the pendency of his application before the RTC as earlier stated).

The petition is bereft of merit.

Section 14 (1) of P.D. No. 1529 provides:

Sec. 14. Who may apply. The following persons may file in the proper Court of First
Instance an application for registration of title to land, whether personally or through their
duly authorized representatives:

(1) Those who by themselves or through their predecessors-in-interest have been in
open, continuous, exclusive and notorious possession and occupation of alienable and
disposable lands of the public domain under a bona fide claim of ownership since
June 12, 1945, or earlier.
[29]
(Underscoring supplied)
To thus be entitled to registration of a land, the applicant must prove that (a) the land applied for
forms part of the disposable and alienable agricultural lands of the public domain; and (b) he has been in
open, continuous, exclusive, and notorious possession and occupation of the same under abona fide claim
of ownership either since time immemorial or since June 12, 1945.
[30]


All lands not otherwise appearing to be clearly within private ownership are presumed to belong
to the State, and unless it has been shown that they have been reclassified by the State as alienable or
disposable to a private person, they remain part of the inalienable public domain.
[31]


To prove that a land is alienable, an applicant must conclusively establish the existence of a
positive act of the government, such as a presidential proclamation or an executive order, or
administrative action, investigation reports of the Bureau of Lands investigator or a legislative act or
statute.
[32]


While petitioner presented a document denominated as 2
nd
Indorsement
[33]
issued by Land
Management Inspector Amadeo Mediran that the lots are within the alienable and disposable zone under
Project No. 3 LSC-3113 issued on April 5, 1978 as certified by the Director of the Forest Development,
the genuineness of the document cannot be ascertained, it being a mere photocopy. Besides, the truth of
its contents cannot be ascertained, Mediran having failed to take the witness stand to identify and testify
thereon.

In fine, Domingo failed to adduce incontrovertible evidence
[34]
showing that the lots have been
declared alienable. They are thus presumed to belong to the public domain, beyond the commerce of
man, and are not susceptible of private appropriation and acquisitive prescription.

But even assuming arguendo that the lots are alienable, Domingo failed to comply with the
requirement on the period of possession. While he alleged in his petition that he bought the lots
from Genoveva in 1948, he failed, as the appellate court correctly noted, to adduce the deed of sale
executed for the purpose, or to explain the reason behind the failure and to present sufficient evidence to
prove the fact of sale.

Again, even assuming arguendo that the lots were indeed sold to him by Genoveva, Domingo
failed to adduce proof that Genoveva, from whom he seeks to tack his possession,
acquired registrable title over them on June 12, 1945 or earlier. Under the same assumption, Domingos
claim that he has been in actual, continuous, adverse and open possession of the lots in the concept of an
owner since 1948 is a conclusion of law which must be substantiated with proof of specific acts of
ownership and factual evidence of possession.
[35]


An examination of the tax receipts
[36]
presented by Domingo shows that they are of recent
vintage, the earliest being dated January 8, 1993.

Tax Declaration Nos. 0298, GR-019-0884, and GR-019-0885,
[37]
which appear to have been
issued in 1947 [sic], 1964, and 1968, respectively, contain the declaration Filed under Presidential
Decree No. 464 below the title Declaration of Real Property. P.D. No. 464, THE REAL PROPERTY
TAX CODE, took effect, however, only on June 1, 1974. Specifically with respect to the first tax
declaration, it even shows that Domingo subscribed and swore to it on August 1, 1947 at which time he
had not bought the lot yet, in 1948 by his claim.

A note on Domingos death during the pendency of his application at the RTC. Indeed, the
records do not show that his death on March 9, 1996was brought to the RTCs attention, which is not in
accordance with Sections 16 and 17, Rule 3 of the 1994 Rules of Court, viz:

SEC. 16. Duty of attorney upon death, incapacity, or incompetency of party.
Whenever a party to a pending case dies, becomes incapacitated or incompetent, it
shallbe the duty of his attorney to inform the court promptly of such death, incapacity or
incompetency, and to give the name and residence of his executor, administrator,
guardian or other legal representative. (Italics in the original; underscoring supplied)

SEC. 17. Death of party. After a party dies and the claim is not thereby extinguished,
the court shall order, upon proper notice, the legal representative of the deceased to
appear and to be substituted for the deceased, within a period of thirty (30) days, or
within such time as may be granted. If the legal representative fails to appear within said
time, the court may order the opposing party to procure the appointment of a legal
representative of the deceased within a time to be specified by the court, and the
representative shall immediately appear for and on behalf of the interest of the
deceased. The court charges involved in procuring such appointment, if defrayed by the
opposing party, may be recovered as costs. The heirs of the deceased may be allowed to
be substituted for the deceased, without requiring the appointment of an executor or
administrator and the court may appoint guardian ad litem for the minor heirs. (Italics in
the original; underscoring supplied)


When a party dies in an action that survives and no order is issued by the court for the appearance
of the legal representative or of the heirs of the deceased in substitution of the deceased, and as a matter
of fact no substitution has ever been effected, the proceedings held by the court without such legal
representatives or heirs and the judgment rendered after such trial are null and void because the court
acquired no jurisdiction over the person of the legal representative or of the heirs upon whom the trial and
judgment would be binding.
[38]


Unlike, however, jurisdiction over the subject matter which is conferred by law, jurisdiction over
the person of the parties to the case may, however, be waived either expressly or impliedly.
[39]
In the case
at bar, the surviving heirs voluntarily submitted themselves to the jurisdiction of this Court, albeit
belatedly, by participating in the present petition.

Under the now amended Section 16, Rule 3 of the 1997 Rules of Court, failure of a counsel to
comply with the provision thereof is a ground for disciplinary action, viz:

SEC. 16. Death of party; duty of counsel. Whenever a party to a pending action dies,
and the claim is not thereby extinguished, it shall be the duty of his counsel to inform the
court within thirty (30) days after such death of the fact thereof, and to give the name and
address of his legal representative or representatives. Failure of counsel to comply with
this duty shall be a ground for disciplinary action.

The heirs of the deceased may be allowed to be substituted for the deceased,
without requiring the appointment of an executor or administrator and the court may
appoint a guardian ad litem for the minor heirs.

The court shall forthwith order said legal representative or representatives to
appear and be substituted within a period of thirty (30) days from notice.

If no legal representative is named by the counsel for the deceased party, or if the
one so named shall fail to appear within the specified period, the court may order the
opposing party, within a specified time, to procure the appointment of an executor or
administrator for the estate of the deceased and the latter shall immediately appear for
and on behalf of the deceased. The court charges in procuring such appointment, if
defrayed by the opposing party, may be recovered as costs. (Italics in the
original; underscoring supplied)


The failure of Domingos former counsel, Atty. Irineo A. Anarna of No.
4 Madlansacay St., Poblacion Lilang 4118 Cavite, to comply with the immediately quoted provisions of
the Rules, is compounded by his misrepresentation, before the CA, that Domingo was well and alive
when he stated in his Motion to Withdraw Appearance as Counsel
[40]
dated July 8, 2004 that the motion
for withdrawal [was] conformed to by Mrs. Rosemarie Manlapit Zamora, representative of the
applicant as shown by her signature . . . and that Mrs. Rosemarie Zamora also undertakes to personally
seek the conformity of the Applicant (Underscoring supplied); and by his retaining of the name of
Domingo in the title of his pleadings before the appellate court.

Canon 10 of the Code of Professional Responsibility provides that a lawyer owes candor, fairness
and good faith to the court. Rule 10.01 likewise provides that a lawyer shall do no falsehood, nor
consent to the doing of any in court; nor shall he mislead, or allow the court to be mislead by any
artifice. And Rule 10.03 provides that a lawyer shall observe the rules of procedure and shall not
misuse them to defeat the ends of justice.

This Court thus takes this occasion to warn Atty. Anarna that a repetition of a similar violation of
the Rules of Court and the Code of Professional Responsibility will be dealt with strictly.

WHEREFORE, the petition is, in light of the foregoing discussion, DENIED.

Let a copy of this Decision be furnished Atty. Irineo A. Anarna of No.
4 Madlansacay St., Poblacion Lilang, 4118 Cavite.

SO ORDERED.

CONCHITA CARPIO MORALES
Associate Justice


WE CONCUR:





LEONARDO A. QUISUMBING
Associate Justice
Chairperson





ANTONIO T. CARPIO
Associate Justice
DANTE O. TINGA
Associate Justice

Republic of the Philippines
SUPREME COURT
Manila
FIRST DIVISION
G.R. No. L-61647 October 12, 1984
REPUBLIC OF THE PHILIPPINES (DIRECTOR OF LANDS), petitioner,
vs.
THE HON. COURT OF APPEALS, BENJAMIN TANCINCO, AZUCENA TANCINCO REYES,
MARINA TANCINCO IMPERIAL and MARIO C. TANCINCO, respondents.
The Solicitor General for petitioner.
Martin B. Laurea for respondents.

GUTIERREZ, JR., J .:+.wph!1
This is a petition for certiorari to set aside the decision of the respondent Court of Appeals (now
Intermediate Appellate Court) affirming the decision of the Court of First Instance of Bulacan,
Fifth Judicial District, Branch VIII, which found that Lots 1 and 2 of Plan Psu-131892 are
accretion to the land covered by Transfer Certificate of Title No. 89709 and ordered their
registration in the names of the private respondents.
Respondents Benjamin Tancinco, Azucena Tancinco Reyes, Marina (should be "Maria")
Tancinco Imperial and Mario C. Tancinco are registered owners of a parcel of land covered by
Transfer Certificate of Title No. T-89709 situated at Barrio Ubihan, Meycauayan, Bulacan
bordering on the Meycauayan and Bocaue rivers.
On June 24, 1973, the private respondents filed an application for the registration of three lots
adjacent to their fishpond property and particularly described as follows: t.hqw
Lot 1-Psu-131892
(Maria C. Tancinco)
A parcel of land (lot 1 as shown on plan Psu-131892), situated in the Barrio of
Ubihan, Municipality of Meycauayan, Province of Bulacan. Bounded on the NE.,
along line 1-2, by Lot 3 of plan Psu-131892; on the SE., along lines 2-3-4, by
Meycauayan River; on the S.W., along fines 4-5-6-7-8-9, by Bocaue River; on the
NE., along line 9-10, by property of Joaquina Santiago; on the E., NE., and NW.,
along lines 10-11-12-1, by property of Mariano Tancinco (Lot 2, Psu-111877). ...
containing an area of THIRTY THREE THOUSAND NINE HUNDRED THIRTY
SEVEN (33,937) SQUARE METERS. ...
Lot 2-Psu-131892
(Maria C. Tancinco)
A parcel of land (Lot 2 as shown on plan Psu-131892), situated in the Barrio of
Ubihan, Municipality of Meycauayan, Province of Bulacan. Bounded on the E.,
along line 1-2, by property of Rafael Singson; on the S., along line 2-3, by
Meycauayan River; on the SW., along line 3-4, by Lot 3 of plan Psu-131892; and
on the N., along line 4-1, by property of Mariano Tancinco (Lot 1, Psu-111877).
... containing an area of FIVE THOUSAND FOUR HUNDRED FIFTY THREE
(5,453) SQUARE METERS. ...
Lot 3-Psu-131892
(Maria C. Tancinco)
A parcel of land (Lot 3 as shown on plan Psu-131892), situated in the Barrio of
Ubihan, Municipality of Meycauayan, Province of Bulacan. Bounded on the NE.,
along line 1-2, by property of Mariano Tancinco (Lot 1, Psu-111877); and along
line 2-3, by Lot 2 of plan Psu-131892; on the S., along line 3-4, by Meycauayan
River, on the SW., along line 4-5, by Lot 1 of plan Psu-131892; and along line 5-6
by property of Mariano Tancinco (Lot 2, Psu-111877), and on the NW., along line
6-1, by property of Joaquina Santiago. ... containing an area of ONE
THOUSAND NINE HUNDRED EIGHTY FIVE (1,985) SQUARE METERS. ...
On April 5, 1974, Assistant Provincial Fiscal Amando C. Vicente, in representation of the Bureau
of Lands filed a written opposition to the application for registration.
On March 6, 1975, the private respondents filed a partial withdrawal of the application for
registration with respect to Lot 3 of Plan Psu-131892 in line with the recommendation of the
Commissioner appointed by the Court.
On March 7, 1975, Lot 3 was ordered withdrawn from the application and trial proceeded only
with respect to Lots 1 and 2 covered by Plan Psu-131892.
On June 26, 1976, the lower court rendered a decision granting the application on the finding
that the lands in question are accretions to the private respondents' fishponds covered by
Transfer Certificate of Title No. 89709. The dispositive portion of the decision
reads: t.hqw
WHEREFORE, it appearing that Lots 1 & 2 of plan Psu-131892 (Exh. H) are
accretions to the land covered by Transfer Certificate of Title No. 89709 of the
Register of Deeds of Bulacan, they belong to the owner of said property. The
Court, therefore, orders the registration of lots 1 & 2 situated in the barrio of
Ubihan, municipality of Meycauayan, province of Bulacan, and more particularly
described in plan Psu-131892 (Exh. H) and their accompanying technical
descriptions (Exhs. E, E-1) in favor of Benjamin Tancinco, married to Alma
Fernandez and residing at 3662 Heatherdown, Toledo, Ohio 43614 U.S.A.;
Azucena Tancinco Reyes, married to Alex Reyes, Jr., residing at 4th St., New
Manila, Quezon City; Marina Tancinco Imperial, married to Juan Imperial,
residing at Pasay Road, Dasmarias Village, Makati, Rizal; and Mario C.
Tancinco, married to Leticia Regidor, residing at 1616 Cypress St., Dasmarias
Village, Makati, Rizal, all of legal age, all Filipino citizens.
On July 30, 1976, the petitioner Republic appealed to the respondent Court of Appeals.
On August, 19, 1982, the respondent Court rendered a decision affirming in toto the decision of
the lower court. The dispositive portion of the decision reads: t.hqw
DAHIL DITO, ang hatol na iniakyat ay sinasangayunan at pinagtitibay sa
kanyang kabuuan nang walang bayad.
The rule that the findings of fact of the trial court and the Court of Appeals are binding upon this
Court admits of certain exceptions. Thus in Carolina Industries Inc. v. CMS Stock Brokerage,
Inc. (97 SCRA 734) we held that this Court retains the power to review and rectify the findings of
fact of said courts when (1) the conclusion is a finding grounded entirely on speculations,
surmises and conjectures; (2) when the inference made is manifestly mistaken, absurd, and
impossible; (3) where there is grave abuse of discretion, (4) when the judgment is based on a
misapprehension of facts; and (5) when the court, in making its findings, went beyond the issues
of the case and the same are contrary to the admissions of both appellant and appellee.
There are facts and circumstances in the record which render untenable the findings of the trial
court and the Court of Appeals that the lands in question are accretions to the private
respondents' fishponds.
The petitioner submits that there is no accretion to speak of under Article 457 of the New Civil
Code because what actually happened is that the private respondents simply transferred their
dikes further down the river bed of the Meycauayan River, and thus, if there is any accretion to
speak of, it is man-made and artificial and not the result of the gradual and imperceptible
sedimentation by the waters of the river.
On the other hand, the private respondents rely on the testimony of Mrs. Virginia Acua to the
effect that: t.hqw
xxx xxx xxx
... when witness first saw the land, namely, Lots 1 & 2, they were already dry
almost at the level of the Pilapil of the property of Dr. Tancinco, and that from the
boundaries of the lots, for about two (2) arms length the land was still dry up to
the edge of the river; that sometime in 1951, a new Pilapil was established on the
boundaries of Lots 1 & 2 and soil from the old Pilapil was transferred to the new
Pilapil and this was done sometime in 1951; that the new lots were then
converted into fishpond, and water in this fishpond was two (2) meters deep on
the side of the Pilapil facing the fishpond ... .
The private respondents submit that the foregoing evidence establishes the fact of accretion
without human intervention because the transfer of the dike occurred after the accretion was
complete.
We agree with the petitioner.
Article 457 of the New Civil Code provides: t.hqw
To the owners of lands adjoining the banks of rivers belong the accretion which
they gradually receive from the effects of the current of the waters.
The above-quoted article requires the concurrence of three requisites before an accretion
covered by this particular provision is said to have taken place. They are (1) that the deposit be
gradual and imperceptible; (2) that it be made through the effects of the current of the water;
and (3) that the land where accretion takes place is adjacent to the banks of rivers.
The requirement that the deposit should be due to the effect of the current of the river is
indispensable. This excludes from Art. 457 of the New Civil Code all deposits caused by human
intervention. Alluvion must be the exclusive work of nature. In the instant case, there is no
evidence whatsoever to prove that the addition to the said property was made gradually through
the effects of the current of the Meycauayan and Bocaue rivers. We agree with the observation
of the Solicitor General that it is preposterous to believe that almost four (4) hectares of land
came into being because of the effects of the Meycauayan and Bocaue rivers. The lone witness
of the private respondents who happens to be their overseer and whose husband was first
cousin of their father noticed the four hectare accretion to the twelve hectare fishpond only in
1939. The respondents claim that at this point in time, accretion had already taken place. If so,
their witness was incompetent to testify to a gradual and imperceptible increase to their land in
the years before 1939. However, the witness testified that in that year, she observed an
increase in the area of the original fishpond which is now the land in question. If she was telling
the truth, the accretion was sudden. However, there is evidence that the alleged alluvial
deposits were artificial and man-made and not the exclusive result of the current of the
Meycauayan and Bocaue rivers. The alleged alluvial deposits came into being not because of
the sole effect of the current of the rivers but as a result of the transfer of the dike towards the
river and encroaching upon it. The land sought to be registered is not even dry land cast
imperceptibly and gradually by the river's current on the fishpond adjoining it. It is under two
meters of water. The private respondents' own evidence shows that the water in the fishpond is
two meters deep on the side of the pilapil facing the fishpond and only one meter deep on the
side of the pilapil facing the river
The reason behind the law giving the riparian owner the right to any land or alluvion deposited
by a river is to compensate him for the danger of loss that he suffers because of the location of
his land. If estates bordering on rivers are exposed to floods and other evils produced by the
destructive force of the waters and if by virtue of lawful provisions, said estates are subject to
incumbrances and various kinds of easements, it is proper that the risk or danger which may
prejudice the owners thereof should be compensated by the right of accretion. (Cortes v. City of
Manila, 10 Phil. 567). Hence, the riparian owner does not acquire the additions to his land
caused by special works expressly intended or designed to bring about accretion. When the
private respondents transferred their dikes towards the river bed, the dikes were meant for
reclamation purposes and not to protect their property from the destructive force of the waters of
the river.
We agree with the submission of the Solicitor General that the testimony of the private
respondents' lone witness to the effect that as early as 1939 there already existed such alleged
alluvial deposits, deserves no merit. It should be noted that the lots in question were not
included in the survey of their adjacent property conducted on May 10, 1940 and in the
Cadastral Survey of the entire Municipality of Meycauayan conducted between the years 1958
to 1960. The alleged accretion was declared for taxation purposes only in 1972 or 33 years after
it had supposedly permanently formed. The only valid conclusion therefore is that the said areas
could not have been there in 1939. They existed only after the private respondents transferred
their dikes towards the bed of the Meycauayan river in 1951. What private respondents claim as
accretion is really an encroachment of a portion of the Meycauayan river by reclamation.
The lower court cannot validly order the registration of Lots 1 & 2 in the names of the private
respondents. These lots were portions of the bed of the Meycauayan river and are therefore
classified as property of the public domain under Article 420 paragraph 1 and Article 502,
paragraph 1 of the Civil Code of the Philippines. They are not open to registration under the
Land Registration Act. The adjudication of the lands in question as private property in the
names of the private respondents is null and void.
WHEREFORE, the instant petition is GRANTED. The decision appealed from is hereby
REVERSED and SET ASIDE. The private respondents are ordered to move back the dikes of
their fishponds to their original location and return the disputed property to the river to which it
belongs.
SO ORDERED.1wph1.t
Teehankee, Actg. C.J., Melencio-Herrera, Plana, Relova and De la Fuente, JJ., concur.


EN BANC

HEIRS OF MARIO MALABANAN, G.R. No. 179987
Petitioner,

Present:

PUNO, C.J.,
QUISUMBING,
YNARES-SANTIAGO,
CARPIO,
- versus - AUSTRIA-MARTINEZ,
CORONA,
CARPIO MORALES,
TINGA,
CHICO-NAZARIO,
VELASCO, JR.,
NACHURA,
LEONARDO DE CASTRO,
BRION,
REPUBLIC OF THE PHILIPPINES, PERALTA, and
Respondent. BERSAMIN, JJ.

Promulgated:

April 29, 2009

x--------------------------------------------------------------------------- x


D E C I S I O N

TINGA, J.:

One main reason why the informal sector has not become formal is that
from Indonesia to Brazil, 90 percent of the informal lands are not titled and registered.
This is a generalized phenomenon in the so-called Third World. And it has many
consequences.


xxx

The question is: How is it that so many governments, from Suharto's
in Indonesia to Fujimori's in Peru, have wanted to title these people and have not been able
to do so effectively? One reason is that none of the state systems in Asia or Latin
America can gather proof of informal titles. In Peru, the informals have means of proving
property ownership to each other which are not the same means developed by the Spanish
legal system. The informals have their own papers, their own forms of agreements, and
their own systems of registration, all of which are very clearly stated in the maps which
they use for their own informal business transactions.
If you take a walk through the countryside, from Indonesia to Peru, and you walk
by field after field--in each field a different dog is going to bark at you. Even dogs know
what private property is all about. The only one who does not know it is the government.
The issue is that there exists a "common law" and an "informal law" which the Latin
American formal legal system does not know how to recognize.
- Hernando De Soto
[1]


This decision inevitably affects all untitled lands currently in possession of persons and entities
other than the Philippine government. The petition, while unremarkable as to the facts, was accepted by
the Court en banc in order to provide definitive clarity to the applicability and scope of original
registration proceedings under Sections 14(1) and 14(2) of the Property Registration Decree. In doing so,
the Court confronts not only the relevant provisions of the Public Land Act and the Civil Code, but also
the reality on the ground. The countrywide phenomenon of untitled lands, as well as the problem of
informal settlement it has spawned, has unfortunately been treated with benign neglect. Yet our current
laws are hemmed in by their own circumscriptions in addressing the phenomenon. Still, the duty on our
part is primarily to decide cases before us in accord with the Constitution and the legal principles that
have developed our public land law, though our social obligations dissuade us from casting a blind eye
on the endemic problems.

I.

On 20 February 1998, Mario Malabanan filed an application for land registration covering a parcel
of land identified as Lot 9864-A, Cad-452-D, Silang Cadastre,
[2]
situated in Barangay Tibig, Silang Cavite,
and consisting of 71,324 square meters. Malabanan claimed that he had purchased the property from
Eduardo Velazco,
[3]
and that he and his predecessors-in-interest had been in open, notorious, and
continuous adverse and peaceful possession of the land for more than thirty (30) years.

The application was raffled to the Regional Trial Court of (RTC) Cavite-Tagaytay City, Branch 18.
The Office of the Solicitor General (OSG) duly designated the Assistant Provincial Prosecutor of Cavite,
Jose Velazco, Jr., to appear on behalf of the State.
[4]
Apart from presenting documentary evidence,
Malabanan himself and his witness, Aristedes Velazco, testified at the hearing. Velazco testified that the
property was originally belonged to a twenty-two hectare property owned by his great-grandfather, Lino
Velazco. Lino had four sons Benedicto, Gregorio, Eduardo and Estebanthe fourth being Aristedess
grandfather. Upon Linos death, his four sons inherited the property and divided it among themselves.
But by 1966, Estebans wife,Magdalena, had become the administrator of all the properties inherited by
the Velazco sons from their father, Lino. After the death of Esteban andMagdalena, their son Virgilio
succeeded them in administering the properties, including Lot 9864-A, which originally belonged to his
uncle, Eduardo Velazco. It was this property that was sold by Eduardo Velazco to Malabanan.
[5]


Assistant Provincial Prosecutor Jose Velazco, Jr. did not cross-examine Aristedes Velazco. He
further manifested that he also *knew+ the property and I affirm the truth of the testimony given by
Mr. Velazco.
[6]
The Republic of the Philippines likewise did not present any evidence to controvert the
application.

Among the evidence presented by Malabanan during trial was a Certification dated 11 June
2001, issued by the Community Environment & Natural Resources Office, Department of Environment
and Natural Resources (CENRO-DENR), which stated that the subject property was verified to be within
the Alienable or Disposable land per Land Classification Map No. 3013 established under Project No. 20-
A and approved as such under FAO 4-1656 on March 15, 1982.
[7]


On 3 December 2002, the RTC rendered judgment in favor of Malabanan, the dispositive portion
of which reads:

WHEREFORE, this Court hereby approves this application for registration and
thus places under the operation of Act 141, Act 496 and/or P.D. 1529, otherwise known
as Property Registration Law, the lands described in Plan Csd-04-0173123-D, Lot 9864-A
and containing an area of Seventy One Thousand Three Hundred Twenty Four (71,324)
Square Meters, as supported by its technical description now forming part of the record
of this case, in addition to other proofs adduced in the name of MARIO MALABANAN,
who is of legal age, Filipino, widower, and with residence at Munting Ilog, Silang, Cavite.

Once this Decision becomes final and executory, the corresponding decree of
registration shall forthwith issue.

SO ORDERED.


The Republic interposed an appeal to the Court of Appeals, arguing that Malabanan had failed
to prove that the property belonged to the alienable and disposable land of the public domain, and that
the RTC had erred in finding that he had been in possession of the property in the manner and for the
length of time required by law for confirmation of imperfect title.

On 23 February 2007, the Court of Appeals rendered a Decision
[8]
reversing the RTC and
dismissing the application of Malabanan. The appellate court held that under Section 14(1) of the
Property Registration Decree any period of possession prior to the classification of the lots as alienable
and disposable was inconsequential and should be excluded from the computation of the period of
possession. Thus, the appellate court noted that since the CENRO-DENR certification had verified that
the property was declared alienable and disposable only on 15 March 1982, the Velazcos
possession prior to that date could not be factored in the computation of the period of possession. This
interpretation of the Court of Appeals of Section 14(1) of the Property Registration Decree was based on
the Courts ruling in Republic v. Herbieto.
[9]


Malabanan died while the case was pending with the Court of Appeals;
[10]
hence, it was his heirs
who appealed the decision of the appellate court. Petitioners, before this Court, rely on our ruling
in Republic v. Naguit,
[11]
which was handed down just four months prior to Herbieto. Petitioners suggest
that the discussion in Herbieto cited by the Court of Appeals is actually obiter dictum since the
Metropolitan Trial Court therein which had directed the registration of the property had no jurisdiction
in the first place since the requisite notice of hearing was published only after the hearing had already
begun. Naguit, petitioners argue, remains the controlling doctrine, especially when the property in
question is agricultural land. Therefore, with respect to agricultural lands, any possession prior to the
declaration of the alienable property as disposable may be counted in reckoning the period of
possession to perfect title under the Public Land Act and the Property Registration Decree.



The petition was referred to the Court en banc,
[12]
and on 11 November 2008, the case was
heard on oral arguments. The Court formulated the principal issues for the oral arguments, to wit:

1. In order that an alienable and disposable land of the public domain may
be registered under Section 14(1) of Presidential Decree No. 1529, otherwise known as
the Property Registration Decree, should the land be classified as alienable and
disposable as of June 12, 1945 or is it sufficient that such classification occur at any time
prior to the filing of the applicant for registration provided that it is established that the
applicant has been in open, continuous, exclusive and notorious possession of the land
under abona fide claim of ownership since June 12, 1945 or earlier?

2. For purposes of Section 14(2) of the Property Registration Decree may a
parcel of land classified as alienable and disposable be deemed private land and
therefore susceptible to acquisition by prescription in accordance with the Civil Code?

3. May a parcel of land established as agricultural in character either
because of its use or because its slope is below that of forest lands be registrable under
Section 14(2) of the Property Registration Decree in relation to the provisions of the Civil
Code on acquisitive prescription?

4. Are petitioners entitled to the registration of the subject land in their
names under Section 14(1) or Section 14(2) of the Property Registration Decree or
both?
[13]


Based on these issues, the parties formulated their respective positions.

With respect to Section 14(1), petitioners reiterate that the analysis of the Court in Naguit is the
correct interpretation of the provision. The seemingly contradictory pronouncement in Herbieto, it is
submitted, should be considered obiter dictum, since the land registration proceedings therein was
void ab initio due to lack of publication of the notice of initial hearing. Petitioners further point out that
in Republic v. Bibonia,
[14]
promulgated in June of 2007, the Court applied Naguit and adopted the same
observation that the preferred interpretation by the OSG of Section 14(1) was patently absurd. For its
part, the OSG remains insistent that for Section 14(1) to apply, the land should have been classified as
alienable and disposable as of 12 June 1945. Apart from Herbieto, the OSG also cites the subsequent
rulings in Buenaventura v. Republic,
[15]
Fieldman Agricultural Trading v. Republic
[16]
and Republic v.
Imperial Credit Corporation,
[17]
as well as the earlier case of Director of Lands v. Court of Appeals.
[18]


With respect to Section 14(2), petitioners submit that open, continuous, exclusive and notorious
possession of an alienable land of the public domain for more than 30 years ipso jure converts the land
into private property, thus placing it under the coverage of Section 14(2). According to them, it would
not matter whether the land sought to be registered was previously classified as agricultural land of the
public domain so long as, at the time of the application, the property had already been converted into
private property through prescription. To bolster their argument, petitioners cite extensively from our
2008 ruling in Republic v. T.A.N. Properties.
[19]



The arguments submitted by the OSG with respect to Section 14(2) are more extensive. The OSG
notes that under Article 1113 of the Civil Code, the acquisitive prescription of properties of the State
refers to patrimonial property, while Section 14(2) speaks of private lands. It observes that the Court
has yet to decide a case that presented Section 14(2) as a ground for application for registration, and
that the 30-year possession period refers to the period of possession under Section 48(b) of the Public
Land Act, and not the concept of prescription under the Civil Code. The OSG further submits that,
assuming that the 30-year prescriptive period can run against public lands, said period should be
reckoned from the time the public land was declared alienable and disposable.

Both sides likewise offer special arguments with respect to the particular factual circumstances
surrounding the subject property and the ownership thereof.

II.

First, we discuss Section 14(1) of the Property Registration Decree. For a full understanding of the
provision, reference has to be made to the Public Land Act.

A.

Commonwealth Act No. 141, also known as the Public Land Act, has, since its enactment,
governed the classification and disposition of lands of the public domain. The President is authorized,
from time to time, to classify the lands of the public domain into alienable and disposable, timber, or
mineral lands.
[20]
Alienable and disposable lands of the public domain are further classified according to
their uses into (a) agricultural; (b) residential, commercial, industrial, or for similar productive purposes;
(c) educational, charitable, or other similar purposes; or (d) reservations for town sites and for public
and quasi-public uses.
[21]


May a private person validly seek the registration in his/her name of alienable and disposable
lands of the public domain? Section 11 of the Public Land Act acknowledges that public lands suitable for
agricultural purposes may be disposed of by confirmation of imperfect or incomplete titles through
judicial legalization.
[22]
Section 48(b) of the Public Land Act, as amended by P.D. No. 1073, supplies the
details and unmistakably grants that right, subject to the requisites stated therein:

Sec. 48. The following described citizens of the Philippines, occupying lands of
the public domain or claiming to own any such land or an interest therein, but whose
titles have not been perfected or completed, may apply to the Court of First Instance of
the province where the land is located for confirmation of their claims and the issuance
of a certificate of title therefor, under the Land Registration Act, to wit:

xxx

(b) Those who by themselves or through their predecessors in interest have
been in open, continuous, exclusive, and notorious possession and occupation of
alienable and disposable lands of the public domain, under a bona fide claim of
acquisition of ownership, since June 12, 1945, or earlier, immediately preceding the
filing of the application for confirmation of title except when prevented by war or force
majeure. These shall be conclusively presumed to have performed all the conditions
essential to a Government grant and shall be entitled to a certificate of title under the
provisions of this chapter.


Section 48(b) of Com. Act No. 141 received its present wording in 1977 when the law was
amended by P.D. No. 1073. Two significant amendments were introduced by P.D. No. 1073. First, the
term agricultural lands was changed to alienable and disposable lands of the public domain. The
OSG submits that this amendment restricted the scope of the lands that may be registered.
[23]
This is not
actually the case. Under Section 9 of the Public Land Act, agricultural lands are a mere subset of lands
of the public domain alienable or open to disposition. Evidently, alienable and disposable lands of the
public domain are a larger class than only agricultural lands.

Second, the length of the requisite possession was changed from possession for thirty (30) years
immediately preceding the filing of the application to possession since June 12, 1945 or earlier. The
Court in Naguit explained:

When the Public Land Act was first promulgated in 1936, the period of
possession deemed necessary to vest the right to register their title to agricultural lands
of the public domain commenced from July 26, 1894. However, this period was amended
by R.A. No. 1942, which provided that the bona fide claim of ownership must have been
for at least thirty (30) years. Then in 1977, Section 48(b) of the Public Land Act was again
amended, this time by P.D. No. 1073, which pegged the reckoning date at June 12, 1945.
xxx


It bears further observation that Section 48(b) of Com. Act No, 141 is virtually the same as Section
14(1) of the Property Registration Decree. Said Decree codified the various laws relative to the
registration of property, including lands of the public domain. It is Section 14(1) that operationalizes the
registration of such lands of the public domain. The provision reads:


SECTION 14. Who may apply. The following persons may file in the proper
Court of First Instance an application for registration of title to land, whether personally
or through their duly authorized representatives:

(1) those who by themselves or through their predecessors-in-
interest have been in open, continuous, exclusive and notorious
possession and occupation of alienable and disposable lands of the
public domain under a bona fide claim of ownership since June 12,
1945, or earlier.


Notwithstanding the passage of the Property Registration Decree and the inclusion of Section
14(1) therein, the Public Land Act has remained in effect. Both laws commonly refer to persons or their
predecessors-in-interest who have been in open, continuous, exclusive and notorious possession and
occupation of alienable and disposable lands of the public domain under a bona fide claim of ownership
since June 12, 1945, or earlier. That circumstance may have led to the impression that one or the other
is a redundancy, or that Section 48(b) of the Public Land Act has somehow been repealed or mooted.
That is not the case.

The opening clauses of Section 48 of the Public Land Act and Section 14 of the Property
Registration Decree warrant comparison:

Sec. 48 [of the Public Land Act]. The following described citizens of the
Philippines, occupying lands of the public domain or claiming to own any such land or
an interest therein, but whose titles have not been perfected or completed, may apply
to the Court of First Instance of the province where the land is located for confirmation
of their claims and the issuance of a certificate of title therefor, under the Land
Registration Act, to wit:

xxx

Sec. 14 [of the Property Registration Decree]. Who may apply. The following
persons may file in the proper Court of First Instance an application for registration of
title to land, whether personally or through their duly authorized representatives:

xxx

It is clear that Section 48 of the Public Land Act is more descriptive of the nature of the right
enjoyed by the possessor than Section 14 of the Property Registration Decree, which seems to presume
the pre-existence of the right, rather than establishing the right itself for the first time. It is proper to
assert that it is the Public Land Act, as amended by P.D. No. 1073 effective 25 January 1977, that has
primarily established the right of a Filipino citizen who has been in open, continuous, exclusive, and
notorious possession and occupation of alienable and disposable lands of the public domain, under a
bona fide claim of acquisition of ownership, since June 12, 1945 to perfect or complete his title by
applying with the proper court for the confirmation of his ownership claim and the issuance of the
corresponding certificate of title.


Section 48 can be viewed in conjunction with the afore-quoted Section 11 of the Public Land Act,
which provides that public lands suitable for agricultural purposes may be disposed of by confirmation of
imperfect or incomplete titles, and given the notion that both provisions declare that it is indeed the
Public Land Act that primarily establishes the substantive ownership of the possessor who has been in
possession of the property since 12 June 1945. In turn, Section 14(a) of the Property Registration Decree
recognizes the substantive right granted under Section 48(b) of the Public Land Act, as well provides the
corresponding original registration procedure for the judicial confirmation of an imperfect or incomplete
title.

There is another limitation to the right granted under Section 48(b). Section 47 of the Public Land
Act limits the period within which one may exercise the right to seek registration under Section 48. The
provision has been amended several times, most recently by Rep. Act No. 9176 in 2002. It currently
reads thus:

Section 47. The persons specified in the next following section are hereby
granted time, not to extend beyond December 31, 2020 within which to avail of the
benefits of this Chapter: Provided, That this period shall apply only where the area
applied for does not exceed twelve (12) hectares: Provided, further, That the several
periods of time designated by the President in accordance with Section Forty-Five of this
Act shall apply also to the lands comprised in the provisions of this Chapter, but this
Section shall not be construed as prohibiting any said persons from acting under this
Chapter at any time prior to the period fixed by the President.
[24]


Accordingly under the current state of the law, the substantive right granted under Section 48(b)
may be availed of only until 31 December 2020.

B.

Despite the clear text of Section 48(b) of the Public Land Act, as amended and Section 14(a) of
the Property Registration Decree, the OSG has adopted the position that for one to acquire the right to
seek registration of an alienable and disposable land of the public domain, it is not enough that the
applicant and his/her predecessors-in-interest be in possession under a bona fide claim of ownership
since 12 June 1945; the alienable and disposable character of the property must have been declared also
as of 12 June 1945. Following the OSGs approach, all lands certified as alienable and disposable after 12
June 1945 cannot be registered either under Section 14(1) of the Property Registration Decree or Section
48(b) of the Public Land Act as amended. The absurdity of such an implication was discussed in Naguit.

Petitioner suggests an interpretation that the alienable and disposable character
of the land should have already been established since June 12, 1945 or earlier. This is
not borne out by the plain meaning of Section 14(1). Since June 12, 1945, as used in
the provision, qualifies its antecedent phrase under a bonafide claim of ownership.
Generally speaking, qualifying words restrict or modify only the words or
phrases to which they are immediately associated, and not those distantly or remotely
located.
[25]
Ad proximum antecedents fiat relation nisi impediatur sentencia.

Besides, we are mindful of the absurdity that would result if we adopt
petitioners position. Absent a legislative amendment, the rule would be, adopting the
OSGs view, that all lands of the public domain which were not declared alienable or
disposable before June 12, 1945 would not be susceptible to original registration, no
matter the length of unchallenged possession by the occupant. Such interpretation
renders paragraph (1) of Section 14 virtually inoperative and even precludes the
government from giving it effect even as it decides to reclassify public agricultural lands
as alienable and disposable. The unreasonableness of the situation would even be
aggravated considering that before June 12, 1945, the Philippines was not yet even
considered an independent state.

Accordingly, the Court in Naguit explained:

[T]he more reasonable interpretation of Section 14(1) is that it merely requires
the property sought to be registered as already alienable and disposable at the time the
application for registration of title is filed. If the State, at the time the application is
made, has not yet deemed it proper to release the property for alienation or disposition,
the presumption is that the government is still reserving the right to utilize the property;
hence, the need to preserve its ownership in the State irrespective of the length of
adverse possession even if in good faith. However, if the property has already been
classified as alienable and disposable, as it is in this case, then there is already an
intention on the part of the State to abdicate its exclusive prerogative over the property.


The Court declares that the correct interpretation of Section 14(1) is that which was adopted
in Naguit. The contrary pronouncement in Herbieto, as pointed out in Naguit, absurdly limits the
application of the provision to the point of virtual inutility since it would only cover lands actually
declared alienable and disposable prior to 12 June 1945, even if the current possessor is able to
establish open, continuous, exclusive and notorious possession under a bona fide claim of ownership
long before that date.

Moreover, the Naguit interpretation allows more possessors under a bona fide claim of
ownership to avail of judicial confirmation of their imperfect titles than what would be feasible
under Herbieto. This balancing fact is significant, especially considering our forthcoming discussion on
the scope and reach of Section 14(2) of the Property Registration Decree.

Petitioners make the salient observation that the contradictory passages
from Herbieto are obiter dicta since the land registration proceedings therein is void ab initio in the first
place due to lack of the requisite publication of the notice of initial hearing. There is no need to explicitly
overturnHerbieto, as it suffices that the Courts acknowledgment that the particular line of argument
used therein concerning Section 14(1) is indeed obiter.
It may be noted that in the subsequent case of Buenaventura,
[26]
the Court, citing Herbieto, again
stated that [a]ny period of possession prior to the date when the [s]ubject [property was] classified as
alienable and disposable is inconsequential and should be excluded from the computation of the period of
possession That statement, in the context of Section 14(1), is certainly erroneous. Nonetheless, the
passage as cited in Buenaventura should again be considered as obiter. The application therein was
ultimately granted, citing Section 14(2). The evidence submitted by petitioners therein did not establish
any mode of possession on their part prior to 1948, thereby precluding the application of Section 14(1). It
is not even apparent from the decision whether petitioners therein had claimed entitlement to original
registration following Section 14(1), their position being that they had been in exclusive possession under
a bona fide claim of ownership for over fifty (50) years, but not before 12 June 1945.
Thus, neither Herbieto nor its principal discipular ruling Buenaventura has any precedental value
with respect to Section 14(1). On the other hand, the ratio of Naguit is embedded in Section 14(1), since
it precisely involved situation wherein the applicant had been in exclusive possession under a bona
fide claim of ownership prior to 12 June 1945. The Courts interpretation of Section 14(1) therein was
decisive to the resolution of the case. Any doubt as to which between Naguit or Herbieto provides the
final word of the Court on Section 14(1) is now settled in favor of Naguit.

We noted in Naguit that it should be distinguished from Bracewell v. Court of Appeals
[27]
since in
the latter, the application for registration had been filed before the land was declared alienable or
disposable. The dissent though pronounces Bracewell as the better rule between the two. Yet two years
after Bracewell, its ponente, the esteemed Justice Consuelo Ynares-Santiago, penned the ruling
in Republic v. Ceniza,
[28]
which involved a claim of possession that extended back to 1927 over a public
domain land that was declared alienable and disposable only in 1980. Ceniza cited Bracewell, quoted
extensively from it, and following the mindset of the dissent, the attempt at registration
in Ceniza should have failed. Not so.

To prove that the land subject of an application for registration is alienable, an
applicant must establish the existence of a positive act of the government such as a
presidential proclamation or an executive order; an administrative action; investigation
reports of Bureau of Lands investigators; and a legislative act or a statute.

In this case, private respondents presented a certification dated November 25,
1994, issued by Eduardo M. Inting, the Community Environment and Natural Resources
Officer in the Department of Environment and Natural Resources Office in Cebu City,
stating that the lots involved were "found to be within the alienable and disposable (sic)
Block-I, Land Classification Project No. 32-A, per map 2962 4-I555 dated December 9,
1980." This is sufficient evidence to show the real character of the land subject of
private respondents application. Further, the certification enjoys a presumption of
regularity in the absence of contradictory evidence, which is true in this case. Worth
noting also was the observation of the Court of Appeals stating that:

[n]o opposition was filed by the Bureaus of Lands and Forestry
to contest the application of appellees on the ground that the property
still forms part of the public domain. Nor is there any showing that the
lots in question are forestal land....

Thus, while the Court of Appeals erred in ruling that mere possession of public
land for the period required by law would entitle its occupant to a confirmation of
imperfect title, it did not err in ruling in favor of private respondents as far as the first
requirement in Section 48(b) of the Public Land Act is concerned, for they were able to
overcome the burden of proving the alienability of the land subject of their application.

As correctly found by the Court of Appeals, private respondents were able to
prove their open, continuous, exclusive and notorious possession of the subject land
even before the year 1927. As a rule, we are bound by the factual findings of the Court
of Appeals. Although there are exceptions, petitioner did not show that this is one of
them.
[29]



Why did the Court in Ceniza, through the same eminent member who authored Bracewell,
sanction the registration under Section 48(b) of public domain lands declared alienable or disposable
thirty-five (35) years and 180 days after 12 June 1945? The telling difference is that in Ceniza, the
application for registration was filed nearly six (6) years after the land had been declared alienable or
disposable, while in Bracewell, the application was filed nine (9) years before the land was declared
alienable or disposable. That crucial difference was also stressed in Naguit to contradistinguish it
from Bracewell, a difference which the dissent seeks to belittle.

III.

We next ascertain the correct framework of analysis with respect to Section 14(2). The provision
reads:



SECTION 14. Who may apply. The following persons may file in the proper
Court of First Instance an application for registration of title to land, whether personally
or through their duly authorized representatives:

xxx

(2) Those who have acquired ownership over private lands by
prescription under the provisions of existing laws.


The Court in Naguit offered the following discussion concerning Section 14(2), which we did
even then recognize, and still do, to be an obiter dictum, but we nonetheless refer to it as material for
further discussion, thus:

Did the enactment of the Property Registration Decree and the amendatory P.D.
No. 1073 preclude the application for registration of alienable lands of the public
domain, possession over which commenced only after June 12, 1945? It did not,
considering Section 14(2) of the Property Registration Decree, which governs and
authorizes the application of those who have acquired ownership of private lands by
prescription under the provisions of existing laws.

Prescription is one of the modes of acquiring ownership under the Civil Code.[
[30]
]
There is a consistent jurisprudential rule that properties classified as alienable public land
may be converted into private property by reason of open, continuous and exclusive
possession of at least thirty (30) years.[
[31]
] With such conversion, such property may now
fall within the contemplation of private lands under Section 14(2), and thus susceptible
to registration by those who have acquired ownership through prescription. Thus, even if
possession of the alienable public land commenced on a date later than June 12, 1945,
and such possession being been open, continuous and exclusive, then the possessor may
have the right to register the land by virtue of Section 14(2) of the Property Registration
Decree.

Naguit did not involve the application of Section 14(2), unlike in this case where petitioners have
based their registration bid primarily on that provision, and where the evidence definitively establishes
their claim of possession only as far back as 1948. It is in this case that we can properly appreciate the
nuances of the provision.

A.

The obiter in Naguit cited the Civil Code provisions on prescription as the possible basis for
application for original registration under Section 14(2). Specifically, it is Article 1113 which provides
legal foundation for the application. It reads:

All things which are within the commerce of men are susceptible of prescription,
unless otherwise provided. Property of the State or any of its subdivisions not patrimonial
in character shall not be the object of prescription.


It is clear under the Civil Code that where lands of the public domain are patrimonial in character,
they are susceptible to acquisitive prescription. On the other hand, among the public domain lands that
are not susceptible to acquisitive prescription are timber lands and mineral lands. The Constitution itself
proscribes private ownership of timber or mineral lands.

There are in fact several provisions in the Civil Code concerning the acquisition of real property
through prescription. Ownership of real property may be acquired by ordinary prescription of ten (10)
years,
[32]
or through extraordinary prescription of thirty (30) years.
[33]
Ordinary acquisitive prescription
requires possession in good faith,
[34]
as well as just title.
[35]


When Section 14(2) of the Property Registration Decree explicitly provides that persons who
have acquired ownership over private lands by prescription under the provisions of existing laws, it
unmistakably refers to the Civil Code as a valid basis for the registration of lands. The Civil Code is the
only existing law that specifically allows the acquisition by prescription of private lands, including
patrimonial property belonging to the State. Thus, the critical question that needs affirmation is whether
Section 14(2) does encompass original registration proceedings over patrimonial property of the State,
which a private person has acquired through prescription.

The Naguit obiter had adverted to a frequently reiterated jurisprudence holding that properties
classified as alienable public land may be converted into private property by reason of open, continuous
and exclusive possession of at least thirty (30) years.
[36]
Yet if we ascertain the source of the thirty-year
period, additional complexities relating to Section 14(2) and to how exactly it operates would emerge.
For there are in fact two distinct origins of the thirty (30)-year rule.

The first source is Rep. Act No. 1942, enacted in 1957, which amended Section 48(b) of the
Public Land Act by granting the right to seek original registration of alienable public lands through
possession in the concept of an owner for at least thirty years.

The following-described citizens of the Philippines, occupying lands of the
public domain or claiming to own any such lands or an interest therein, but whose titles
have not been perfected or completed, may apply to the Court of First Instance of the
province where the land is located for confirmation of their claims and the issuance of
a certificate of title therefor, under the Land Registration Act, to wit:

x x x x x x x x x

(b) Those who by themselves or through their predecessors in interest have
been in open, continuous, exclusive and notorious possession and occupation of
agricultural lands of the public domain, under a bona fide claim of acquisition of
ownership, for at least thirty years immediately preceding the filing of the application
for confirmation of title, except when prevented by war or force majeure. These shall
be conclusively presumed to have performed all the conditions essential to a
Government grant and shall be entitled to a certificate of title under the provisions of
this Chapter. (emphasis supplied)
[37]



This provision was repealed in 1977 with the enactment of P.D. 1073, which made the date 12
June 1945 the reckoning point for the first time. Nonetheless, applications for registration filed prior to
1977 could have invoked the 30-year rule introduced by Rep. Act No. 1942.

The second source is Section 14(2) of P.D. 1529 itself, at least by implication, as it applies the
rules on prescription under the Civil Code, particularly Article 1113 in relation to Article 1137. Note that
there are two kinds of prescription under the Civil Codeordinary acquisitive prescription and
extraordinary acquisitive prescription, which, under Article 1137, is completed through uninterrupted
adverse possession for thirty years, without need of title or of good faith.

Obviously, the first source of the thirty (30)-year period rule, Rep. Act No. 1942, became
unavailable after 1977. At present, the only legal basis for the thirty (30)-year period is the law on
prescription under the Civil Code, as mandated under Section 14(2). However, there is a material
difference between how the thirty (30)-year rule operated under Rep. Act No. 1942 and how it did under
the Civil Code.

Section 48(b) of the Public Land Act, as amended by Rep. Act No. 1942, did not refer to or call
into application the Civil Code provisions on prescription. It merely set forth a requisite thirty-year
possession period immediately preceding the application for confirmation of title, without any
qualification as to whether the property should be declared alienable at the beginning of, and continue
as such, throughout the entire thirty-(30) years. There is neither statutory nor jurisprudential basis to
assert Rep. Act No. 1942 had mandated such a requirement,
[38]
similar to our earlier finding with respect
to the present language of Section 48(b), which now sets 12 June 1945 as the point of reference.

Then, with the repeal of Rep. Act No. 1942, the thirty-year possession period as basis for original
registration became Section 14(2) of the Property Registration Decree, which entitled those who have
acquired ownership over private lands by prescription under the provisions of existing laws to apply for
original registration. Again, the thirty-year period is derived from the rule on extraordinary prescription
under Article 1137 of the Civil Code. At the same time, Section 14(2) puts into operation the entire
regime of prescription under the Civil Code, a fact which does not hold true with respect to Section
14(1).

B.

Unlike Section 14(1), Section 14(2) explicitly refers to the principles on prescription under
existing laws. Accordingly, we are impelled to apply the civil law concept of prescription, as set forth in
the Civil Code, in our interpretation of Section 14(2). There is no similar demand on our part in the case
of Section 14(1).

The critical qualification under Article 1113 of the Civil Code is thus: *p+roperty of the State or
any of its subdivisions not patrimonial in character shall not be the object of prescription. The
identification what consists of patrimonial property is provided by Articles 420 and 421, which we quote
in full:

Art. 420. The following things are property of public dominion:

(1) Those intended for public use, such as roads, canals, rivers, torrents, ports and
bridges constructed by the State, banks, shores, roadsteads, and others of similar
character;

(2) Those which belong to the State, without being for public use, and are intended
for some public service or for the development of the national wealth.

Art. 421. All other property of the State, which is not of the character stated in
the preceding article, is patrimonial property

It is clear that property of public dominion, which generally includes property belonging to the
State, cannot be the object of prescription or, indeed, be subject of the commerce of man.
[39]
Lands of
the public domain, whether declared alienable and disposable or not, are property of public dominion
and thus insusceptible to acquisition by prescription.

Let us now explore the effects under the Civil Code of a declaration by the President or any duly
authorized government officer of alienability and disposability of lands of the public domain. Would
such lands so declared alienable and disposable be converted, under the Civil Code, from property of the
public dominion into patrimonial property? After all, by connotative definition, alienable and disposable
lands may be the object of the commerce of man; Article 1113 provides that all things within the
commerce of man are susceptible to prescription; and the same provision further provides that
patrimonial property of the State may be acquired by prescription.

Nonetheless, Article 422 of the Civil Code states that *p+roperty of public dominion, when no
longer intended for public use or for public service, shall form part of the patrimonial property of the
State. It is this provision that controls how public dominion property may be converted into
patrimonial property susceptible to acquisition by prescription. After all, Article 420 (2) makes clear that
those property which belong to the State, without being for public use, and are intended for some
public service or for the development of the national wealth are public dominion property. For as long
as the property belongs to the State, although already classified as alienable or disposable, it remains
property of the public dominion if when it is intended for some public service or for the development
of the national wealth.

Accordingly, there must be an express declaration by the State that the public dominion
property is no longer intended for public service or the development of the national wealth or that
the property has been converted into patrimonial. Without such express declaration, the property,
even if classified as alienable or disposable, remains property of the public dominion, pursuant to
Article 420(2), and thus incapable of acquisition by prescription. It is only when such alienable and
disposable lands are expressly declared by the State to be no longer intended for public service or for
the development of the national wealth that the period of acquisitive prescription can begin to run.
Such declaration shall be in the form of a law duly enacted by Congress or a Presidential Proclamation
in cases where the President is duly authorized by law.

It is comprehensible with ease that this reading of Section 14(2) of the Property Registration
Decree limits its scope and reach and thus affects the registrability even of lands already declared
alienable and disposable to the detriment of the bona fide possessors or occupants claiming title to the
lands. Yet this interpretation is in accord with the Regalian doctrine and its concomitant assumption that
all lands owned by the State, although declared alienable or disposable, remain as such and ought to be
used only by the Government.

Recourse does not lie with this Court in the matter. The duty of the Court is to apply the
Constitution and the laws in accordance with their language and intent. The remedy is to change the
law, which is the province of the legislative branch. Congress can very well be entreated to amend
Section 14(2) of the Property Registration Decree and pertinent provisions of the Civil Code to liberalize
the requirements for judicial confirmation of imperfect or incomplete titles.

The operation of the foregoing interpretation can be illustrated by an actual example. Republic
Act No. 7227, entitled An Act Accelerating The Conversion Of Military Reservations Into Other
Productive Uses, etc., is more commonly known as the BCDA law. Section 2 of the law authorizes the
sale of certain military reservations and portions of military camps in Metro Manila,
including Fort Bonifacio and Villamor Air Base. For purposes of effecting the sale of the military camps,
the law mandates the President to transfer such military lands to the Bases Conversion Development
Authority (BCDA)
[40]
which in turn is authorized to own, hold and/or administer them.
[41]
The President is
authorized to sell portions of the military camps, in whole or in part.
[42]
Accordingly, the BCDA law itself
declares that the military lands subject thereof are alienable and disposable pursuant to the provisions
of existing laws and regulations governing sales of government properties.
[43]


From the moment the BCDA law was enacted the subject military lands have become alienable
and disposable. However, said lands did not become patrimonial, as the BCDA law itself expressly makes
the reservation that these lands are to be sold in order to raise funds for the conversion of the former
American bases at Clark and Subic.
[44]
Such purpose can be tied to either public service or the
development of national wealth under Article 420(2). Thus, at that time, the lands remained property
of the public dominion under Article 420(2), notwithstanding their status as alienable and disposable. It
is upon their sale as authorized under the BCDA law to a private person or entity that such lands become
private property and cease to be property of the public dominion.


C.

Should public domain lands become patrimonial because they are declared as such in a duly
enacted law or duly promulgated proclamation that they are no longer intended for public service or for
the development of the national wealth, would the period of possession prior to the conversion of such
public dominion into patrimonial be reckoned in counting the prescriptive period in favor of the
possessors? We rule in the negative.

The limitation imposed by Article 1113 dissuades us from ruling that the period of possession
before the public domain land becomes patrimonial may be counted for the purpose of completing the
prescriptive period. Possession of public dominion property before it becomes patrimonial cannot be
the object of prescription according to the Civil Code. As the application for registration under Section
14(2) falls wholly within the framework of prescription under the Civil Code, there is no way that
possession during the time that the land was still classified as public dominion property can be counted
to meet the requisites of acquisitive prescription and justify registration.

Are we being inconsistent in applying divergent rules for Section 14(1) and Section 14(2)? There
is no inconsistency. Section 14(1) mandates registration on the basis of possession, while Section 14(2)
entitles registration on the basis of prescription. Registration under Section 14(1) is extended under
the aegis of the Property Registration Decree and the Public Land Act while registration under Section
14(2) is made available both by the Property Registration Decree and the Civil Code.

In the same manner, we can distinguish between the thirty-year period under Section 48(b) of the
Public Land Act, as amended by Rep. Act No. 1472, and the thirty-year period available through Section
14(2) of the Property Registration Decree in relation to Article 1137 of the Civil Code. The period under
the former speaks of a thirty-year period of possession, while the period under the latter concerns
a thirty-year period of extraordinary prescription. Registration under Section 48(b) of the Public Land
Act as amended by Rep. Act No. 1472 is based on thirty years of possession alone without regard to
the Civil Code, while the registration under Section 14(2) of the Property Registration Decree is
founded on extraordinary prescription under the Civil Code.

It may be asked why the principles of prescription under the Civil Code should not apply as well to
Section 14(1). Notwithstanding the vaunted status of the Civil Code, it ultimately is just one of numerous
statutes, neither superior nor inferior to other statutes such as the Property Registration Decree. The
legislative branch is not bound to adhere to the framework set forth by the Civil Code when it enacts
subsequent legislation. Section 14(2) manifests a clear intent to interrelate the registration allowed
under that provision with the Civil Code, but no such intent exists with respect to Section 14(1).

IV.

One of the keys to understanding the framework we set forth today is seeing how our land
registration procedures correlate with our law on prescription, which, under the Civil Code, is one of the
modes for acquiring ownership over property.

The Civil Code makes it clear that patrimonial property of the State may be acquired by private
persons through prescription. This is brought about by Article 1113, which states that *a+ll things which
are within the commerce of man are susceptible to prescription, and that *p+roperty of the State or any
of its subdivisions not patrimonial in character shall not be the object of prescription.

There are two modes of prescription through which immovables may be acquired under the Civil
Code. The first is ordinary acquisitive prescription, which, under Article 1117, requires possession in
good faith and with just title; and, under Article 1134, is completed through possession of ten (10) years.
There is nothing in the Civil Code that bars a person from acquiring patrimonial property of the State
through ordinary acquisitive prescription, nor is there any apparent reason to impose such a rule. At the
same time, there are indispensable requisitesgood faith and just title. The ascertainment of good faith
involves the application of Articles 526, 527, and 528, as well as Article 1127 of the Civil
Code,
[45]
provisions that more or less speak for themselves.

On the other hand, the concept of just title requires some clarification. Under Article 1129,
there is just title for the purposes of prescription when the adverse claimant came into possession of
the property through one of the modes recognized by law for the acquisition of ownership or other real
rights, but the grantor was not the owner or could not transmit any right. Dr. Tolentino explains:

Just title is an act which has for its purpose the transmission of ownership, and
which would have actually transferred ownership if the grantor had been the owner. This
vice or defect is the one cured by prescription. Examples: sale with delivery, exchange,
donation, succession, and dacion in payment.
[46]


The OSG submits that the requirement of just title necessarily precludes the applicability of
ordinary acquisitive prescription to patrimonial property. The major premise for the argument is that
the State, as the owner and grantor, could not transmit ownership to the possessor before the
completion of the required period of possession.
[47]
It is evident that the OSG erred when it assumed
that the grantor referred to in Article 1129 is the State. The grantor is the one from whom the person
invoking ordinary acquisitive prescription derived the title, whether by sale, exchange, donation,
succession or any other mode of the acquisition of ownership or other real rights.

Earlier, we made it clear that, whether under ordinary prescription or extraordinary
prescription, the period of possession preceding the classification of public dominion lands as
patrimonial cannot be counted for the purpose of computing prescription. But after the property has
been become patrimonial, the period of prescription begins to run in favor of the possessor. Once the
requisite period has been completed, two legal events ensue: (1) the patrimonial property is ipso
jure converted into private land; and (2) the person in possession for the periods prescribed under the
Civil Code acquires ownership of the property by operation of the Civil Code.

It is evident that once the possessor automatically becomes the owner of the converted
patrimonial property, the ideal next step is the registration of the property under the Torrens system. It
should be remembered that registration of property is not a mode of acquisition of ownership, but
merely a mode of confirmation of ownership.
[48]


Looking back at the registration regime prior to the adoption of the Property Registration
Decree in 1977, it is apparent that the registration system then did not fully accommodate the
acquisition of ownership of patrimonial property under the Civil Code. What the system accommodated
was the confirmation of imperfect title brought about by the completion of a period of possession
ordained under the Public Land Act (either 30 years following Rep. Act No. 1942, or since 12 June
1945 following P.D. No. 1073).

The Land Registration Act
[49]
was noticeably silent on the requisites for alienable public lands
acquired through ordinary prescription under the Civil Code, though it arguably did not preclude such
registration.
[50]
Still, the gap was lamentable, considering that the Civil Code, by itself, establishes
ownership over the patrimonial property of persons who have completed the prescriptive periods
ordained therein. The gap was finally closed with the adoption of the Property Registration Decree in
1977, with Section 14(2) thereof expressly authorizing original registration in favor of persons who have
acquired ownership over private lands by prescription under the provisions of existing laws, that is, the
Civil Code as of now.

V.

We synthesize the doctrines laid down in this case, as follows:

(1) In connection with Section 14(1) of the Property Registration Decree, Section 48(b) of the
Public Land Act recognizes and confirms that those who by themselves or through their predecessors in
interest have been in open, continuous, exclusive, and notorious possession and occupation of alienable
and disposable lands of the public domain, under a bona fide claim of acquisition of ownership, since
June 12, 1945 have acquired ownership of, and registrable title to, such lands based on the length and
quality of their possession.

(a) Since Section 48(b) merely requires possession since 12 June 1945 and does not
require that the lands should have been alienable and disposable during the entire period of
possession, the possessor is entitled to secure judicial confirmation of his title thereto as soon as
it is declared alienable and disposable, subject to the timeframe imposed by Section 47 of the
Public Land Act.
[51]


(b) The right to register granted under Section 48(b) of the Public Land Act is further
confirmed by Section 14(1) of the Property Registration Decree.

(2) In complying with Section 14(2) of the Property Registration Decree, consider that under the
Civil Code, prescription is recognized as a mode of acquiring ownership of patrimonial property.
However, public domain lands become only patrimonial property not only with a declaration that these
are alienable or disposable. There must also be an express government manifestation that the property
is already patrimonial or no longer retained for public service or the development of national wealth,
under Article 422 of the Civil Code. And only when the property has become patrimonial can the
prescriptive period for the acquisition of property of the public dominion begin to run.

(a) Patrimonial property is private property of the government. The person acquires
ownership of patrimonial property by prescription under the Civil Code is entitled to secure
registration thereof under Section 14(2) of the Property Registration Decree.

(b) There are two kinds of prescription by which patrimonial property may be
acquired, one ordinary and other extraordinary. Under ordinary acquisitive prescription, a
person acquires ownership of a patrimonial property through possession for at least ten (10)
years, in good faith and with just title. Under extraordinary acquisitive prescription, a persons
uninterrupted adverse possession of patrimonial property for at least thirty (30) years,
regardless of good faith or just title, ripens into ownership.

B.

We now apply the above-stated doctrines to the case at bar.

It is clear that the evidence of petitioners is insufficient to establish that Malabanan has acquired
ownership over the subject property under Section 48(b) of the Public Land Act. There is no substantive
evidence to establish that Malabanan or petitioners as his predecessors-in-interest have been in
possession of the property since 12 June 1945 or earlier. The earliest that petitioners can date back their
possession, according to their own evidencethe Tax Declarations they presented in particularis to
the year 1948. Thus, they cannot avail themselves of registration under Section 14(1) of the Property
Registration Decree.


Neither can petitioners properly invoke Section 14(2) as basis for registration. While the subject
property was declared as alienable or disposable in 1982, there is no competent evidence that is no
longer intended for public use service or for the development of the national evidence, conformably
with Article 422 of the Civil Code. The classification of the subject property as alienable and disposable
land of the public domain does not change its status as property of the public dominion under Article
420(2) of the Civil Code. Thus, it is insusceptible to acquisition by prescription.

VI.

A final word. The Court is comfortable with the correctness of the legal doctrines established in
this decision. Nonetheless, discomfiture over the implications of todays ruling cannot be discounted.
For, every untitled property that is occupied in the country will be affected by this ruling. The social
implications cannot be dismissed lightly, and the Court would be abdicating its social responsibility to
the Filipino people if we simply levied the law without comment.

The informal settlement of public lands, whether declared alienable or not, is a phenomenon tied
to long-standing habit and cultural acquiescence, and is common among the so-called Third World
countries. This paradigm powerfully evokes the disconnect between a legal system and the reality on
the ground. The law so far has been unable to bridge that gap. Alternative means of acquisition of these
public domain lands, such as through homestead or free patent, have

proven unattractive due to limitations imposed on the grantee in the encumbrance or alienation of said
properties.
[52]
Judicial confirmation of imperfect title has emerged as the most viable, if not the most
attractive means to regularize the informal settlement of alienable or disposable lands of the public
domain, yet even that system, as revealed in this decision, has considerable limits.

There are millions upon millions of Filipinos who have individually or exclusively held residential
lands on which they have lived and raised their families. Many more have tilled and made productive
idle lands of the State with their hands. They have been regarded for generation by their families and
their communities as common law owners. There is much to be said about the virtues of according
them legitimate states. Yet such virtues are not for the Court to translate into positive law, as the law
itself considered such lands as property of the public dominion. It could only be up to Congress to set
forth a new phase of land reform to sensibly regularize and formalize the settlement of such lands which
in legal theory are lands of the public domain before the problem becomes insoluble. This could be
accomplished, to cite two examples, by liberalizing the standards for judicial confirmation of imperfect
title, or amending the Civil Code itself to ease the requisites for the conversion of public dominion
property into patrimonial.

Ones sense of security over land rights infuses into every aspect of well-being not only of that
individual, but also to the persons family. Once that sense of security is deprived, life and livelihood are
put on stasis. It is for the political branches to bring welcome closure to the long pestering problem.

WHEREFORE, the Petition is DENIED. The Decision of the Court of Appeals dated 23 February
2007 and Resolution dated 2 October 2007are AFFIRMED. No pronouncement as to costs.

SO ORDERED.






DANTE O. TINGA
Associate Justice


WE CONCUR:







REYNATO S. PUNO
Chief Justice


LEONARDO A. QUISUMBING
Associate Justice

CONSUELO YNARES-SANTIAGO
Associate Justice




ANTONIO T. CARPIO
Associate Justice

MA. ALICIA AUSTRIA-MARTINEZ
Associate Justice




RENATO C. CORONA
Associate Justice

CONCHITA CARPIO MORALES
Associate Justice




MINITA V. CHICO-NAZARIO
Associate Justice

PRESBITERO J. VELASCO, JR.
Associate Justice




ANTONIO EDUARDO B. NACHURA
Associate Justice

TERESITA J. LEONARDO DE CASTO
Associate Justice




ARTURO D. BRION
Associate Justice

DIOSDADO M. PERALTA
Associate Justice



LUCAS P. BERSAMIN
Associate Justice


C E R T I F I C A T I O N


Pursuant to Article VIII, Section 13 of the Constitution, it is hereby certified that the conclusions in
the above Decision were reached in consultation before the case was assigned to the writer of the
opinion of the Court.


REYNATO S. PUNO
Chief Justice







[1]
Hernando de Soto Interview by Reason Magazine dated 30 November 1999,
at http://www.reason.com/news/show/32213.html (Last visited, 21 April 2009).

[2]
More particularly described and delineated in Plan CSD-04-017123. Records, p. 161.

[3]
But see note 5.

[4]
Id.

[5]
The trial court decision identified Eduardo Velazco as the vendor of the property,
notwithstanding the original allegation in the application that Malabanan purchased the same from
Virgilio Velazco. See note 3. In his subsequent pleadings, including those before this Court, Malabanan
or his heirs stated that the property was purchased from Eduardo Velazco, and not Virgilio. On this point,
the appellate court made this observation:

More importantly, Malabanan failed to prove his ownership over Lot 9864-A. In his application
for land registration, Malabanan alleged that he purchased the subject lot from Virgilio Velazco. During
the trial of the case, however, Malabanan testified that he purchased the subject lot from Eduardo
Velazco, which was corroborated by his witness, Aristedes Velazco, a son of Virgilio Velazco, who stated
that Eduardo was a brother of his grandfather. As aptly observed by the Republic, no copy of the deed of
sale covering Lot 9864-A, executed either by Virgilio or Eduardo Velazco, in favor of Malabanan was
marked and offered in evidence. In the appealed Decision, the court a quo mentioned of a deed of sale
executed in 1995 by Eduardo Velazco in favor of Malabanan which was allegedly marked as Exhibit
I. It appears, however, that what was provisionally marked as Exhibit I was a photocopy of the deed
of sale executed by Virgilio Velazco in favor of Leila Benitez and Benjamin Reyes. Section 34, Rule 132
of the Rules of Court provides that the court shall consider no evidence which has not been formally
offered. The offer is necessary because it is the duty of a judge to rest his findings of facts and his
judgment only and strictly upon the evidence offered by the parties at the trial. Thus, Malabanan has not
proved that Virgilio or Eduardo Velazco was his predecessor-in-interest. Rollo, pp. 39-40.

[6]
Rollo, p. 74.

[7]
Id. at 38. Emphasis supplied.

[8]
Penned by Associate Justice Marina Buzon of the Court of Appeals Fifth Division, and
concurred in by Associate Justices Edgardo Sundiam and Monina Arevalo-Zenarosa.

[9]
G.R. No. 156117, 26 May 2005, 459 SCRA 183.

[10]
See rollo, p. 11.

[11]
G.R. No. 144507, 17 January 2005, 448 SCRA 442.

[12]
Through a Resolution dated 5 December 2007. See rollo, p. 141.

[13]
Id. at 186-187.

[14]
G.R. No. 157466, 21 June 2007, 525 SCRA 268.

[15]
G.R. No. 166865, 2 March 2007, 459 SCRA 271.

[16]
G.R. No. 147359, 28 March 2008, 550 SCRA 92.

[17]
G.R. No. 173088, 25 June 2008, 555 SCRA 314.

[18]
G.R. No. 85322, 30 April 1991, 178 SCRA 708.

[19]
G.R. No. 154953, 16 June 2008.

[20]
Section 6, Com. Act No. 141, as amended.

[21]
Section 9, Com. Act No. 141, as amended.

[22]
Section 11, Com. Act No. 141, as amended.

[23]
OSG Memorandum, p. 13.

[24]
Section 47, Public Land Act, as amended by Rep. Act No. 9176.

[25]
R. AGPALO, STATUTORY CONSTRUCTION (3
rd
ed., 1995) at 182.

[26]
See note 3.

[27]
380 Phil. 156 (2000).

[28]
Also known as Republic v. Court of Appeals, 440 Phil. 697 (2002).

[29]
Id. at 710-712.

[30]
See CIVIL CODE, Art. 1113.

[31]
See e.g., Director of Lands v. IAC, G.R. No. 65663, 16 October 1992, 214 SCRA 604,
611; Republic v. Court of Appeals, G.R. No. 108998, 24 August 1994, 235 SCRA 567, 576; Group
Commander, Intelligence and Security Group v. Dr. Malvar, 438 Phil. 252, 275 (2002).

[32]
See Article 1134, CIVIL CODE.

[33]
See Article 1137, CIVIL CODE.

[34]
See Article 1117 in relation to Article 1128, Civil Code. See also Articles 526, 527, 528 &
529, Civil Code on the conditions of good faith required.

[35]
See Article 1117, in relation to Article 1129, Civil Code.

[36]
Citing Director of Lands v. IAC, G.R. No. 65663, 16 October 1992, 214 SCRA 604,
611; Republic v. Court of Appeals, G.R. No. 108998, 24 August 1994, 235 SCRA 567, 576; Group
Commander, Intelligence and Security Group v. Dr. Malvar, 438 Phil. 252, 275 (2002).

[37]
Section 48(b) of the Public Land Act, immediately before its amendment by Rep. Act No.
1942, reads as follows:

Those who by themselves or through their predecessors in interest have been in open,
continuous, exclusive and notorious possession and occupation of agricultural lands of the public domain,
under a bona fide claim of acquisition of ownership, except as against the Government, since July twenty-
sixth, eighteen hundred and ninety-four, except when prevented by war or force majeure. These shall be
conclusively presumed to have performed all the conditions essential to a Government grant and shall be
entitled to a certificate of title under the provisions of this Chapter.

[38]
Again, Section 48(b) of the Public Land Act, as amended by Rep. Act No. 1942, was
superseded by P.D. No. 1073, which imposed the 12 June 1945 reckoning point, and which was then
incorporated in Section 14(1) of the Property Registration Decree.

[39]
See Vllarico v. Sarmiento, G.R. No. 136438, 11 November 2004, 442 SCRA 110.

[40]
Rep. Act No. 7227, Sec.7.

[41]
Rep. Act No. 7227, Sec. 4(a).

[42]
Rep. Act No. 7227, Sec. 7.

[43]
Id.

[44]
Section 2, Rep. Act No. 7227.

[45]
See CIVIL CODE, Art. 1128.

[46]
A. TOLENTINO, IV CIVIL CODE OF THE PHILIPPINES (1991 ed.) at 26; citing 2 Castan
175.

[47]
Memorandum of the OSG, p. 21.

[48]
See Angeles v. Samia, 66 Phil. 44 (1938).

[49]
Act No. 496.

[50]
See Section 19, Land Registration Act, which allowed application for registration of title by
person or persons claiming, singly or collectively, to own the legal estate in fee simple.

[51]
See note 24.

[52]
See Section 118, Com. Act No. 141, as amended.

Except in favor of the Government or any of its branches, units, or institutions, lands acquired
under free patent or homestead provisions shall not be subject to encumbrance or alienation from the date
of the approval of the application and for a term of five years from and after the date of issuance of the
patent or grant, nor shall they become liable to the satisfaction of any debt contracted prior to the
expiration of said period, but the improvements or crops on the land may be mortgaged or pledged to
qualified persons, associations, or corporations.

No alienation, transfer, or conveyance of any homestead after five years and before twenty-five
years after issuance of title shall be valid without the approval of the Secretary of Agriculture and
Commerce, which approval shall not be denied except on constitutional and legal grounds.

FIRST DIVISION


REPUBLIC OF THE PHILIPPINES, G.R. No. 151910
Petitioner,
Present:

PUNO, C.J., Chairperson,
- versus - SANDOVAL-GUTIERREZ,
CORONA,
AZCUNA, and
GARCIA, JJ.
LUDOLFO V. MUOZ,
Respondent. Promulgated:

October 15, 2007
x ---------------------------------------------------------------------------------------- x

DECISION

AZCUNA, J.:

Before this Court is a Petition for Review on Certiorari, under Rule 45 of the 1997 Rules of Civil
Procedure, seeking to set aside the August 29, 2001 Decision
[1]
of the Court of Appeals (CA) in CA-G.R.
CV No. 58170, as well as its January 29, 2002 Resolution, which affirmed the October 3, 1997
Decision
[2]
of the Regional Trial Court (RTC) of Ligao, Albay, Branch 13, granting the application for
land registration of respondent Ludolfo V. Muoz.

The following facts prompted the present controversy.

On June 14, 1996, respondent filed an Application for Registration of Title of a parcel of
residential land before the RTC of Ligao, Albay containing an area of 1,986 square meters situated,
bounded, and described as follows:

A PARCEL OF LAND (Lot No. 2276 of the Cadastral Survey of Ligao) with
the building and improvements thereon, situated in the Barrio of Bagonbayan,
Municipality of Ligao, Province of Albay. Bounded on the S., along line 1-2, by Lot No.
2277, Ligao Cadastre; on the W., along Line 2-3, by Mabini Street; on the N., and E.,
along lines 3-4-5-6-4-7, by Lot 2284; and on the S., along line 7-8, by Lot 2281; and
along line 8-1, by Lot 2278 all of Ligao Cadastre, containing an area of ONE
THOUSAND NINE HUNDRED EIGHTY SIX (1,986) square meters.
[3]



In his application for registration, respondent averred that no mortgage or encumbrance of any
kind affects his property and that no other person has an interest, legal or equitable, on the subject lot.
Respondent further declared that the property was acquired by donation inter vivos, executed by the
spouses Apolonio R. Muoz and Anastacia Vitero on November 18, 1956, and that the spouses and their
predecessors-in-interest have been in possession thereof since time immemorial for more than 70 years.

On November 7, 1996, petitioner Republic of the Philippines, through the Office of the Solicitor
General (OSG), opposed the application on the following grounds:

(1) That neither the applicant nor his predecessors-in-interest have been
in open, continuous, exclusive and notorious possession and occupation of the land in
question since June 12, 1945 or prior thereto (Sec. 48[b], C.A. 141 as amended by P.D.
1073).

(2) That the muniment/s of title and/or the tax payment/s receipt/s of
application/s, if any, attached to or alleged in the application, do not constitute
competent and sufficient evidence of a bona fide acquisition of the lands acquired for or
his open, continuous, exclusive and notorious possession and occupation thereof in the
concept of owner since June 12, 1945 or prior thereto. Said muniment/s of title as well as
the title do not appear to be genuine and that the tax declaration/s and/or tax payment
receipt/s indicate the pretended possession of application to be of recent vintage.

(3) That the claim of ownership in fee simple on the basis of Spanish
title or grant can no longer be availed of by the applicant who has failed to file an
appropriate application for registration within the period of six (6) months from
February 16, 1976 as required by P.D. No. 892. From the records, it appears that the
instant application was recently filed.

(4) That the parcel applied for is part of the public domain belonging to
the Republic of the Philippines not subject to private appropriation.

(5) That this application was filed beyond December 31, 1987, the
period set forth under Sec. 2, P.D. No. 1073 and therefore, is filed out of time.
[4]



In respondents Answer to Opposition, he professed that the land in question is a residential lot
originally owned and possessed by Paulino Pulvinar and Geronimo Lozada. Sometime in April 1917,
Pulvinar sold his share of the unregistered land to the spouses Muoz and Vitero, respondents parents. In
June 1920, Lozada likewise sold his remaining part to the parents of respondent. Thereafter, the
ownership and possession of the property were consolidated by the spouses and declared for taxation
purposes in the name of Muoz in 1920. Furthermore, it was stated that during the cadastral survey
conducted in Ligao, Albay in 1928, the land was designated as Lot No. 2276, as per Survey Notification
Card issued to Muoz dated October 2, 1928. Finally, respondent contended that from 1920 up to 1996,
the time of application, the land taxes for the property had been fully paid.

On February 6, 1997, an Order of General Default
[5]
was entered by the trial court against the
whole world except for the government and a certain Alex Vasquez, who appeared during the scheduled
initial hearing stating that he would file an opposition to the application.
In the Opposition
[6]
filed by Vasquez dated February 19, 1997, he declared that he owns parcels
of land, Lot Nos. 2284-A-2 and 2275, adjoining that of the subject matter of the application. He added
that certain portions of his lands are included in the application as respondents concrete fence is found
within the area of his lots.

Respondent, in his answer to the opposition,
[7]
alleged that his property, Lot No. 2276, is
covered by a technical description, duly certified correct by the Bureau of Lands and approved for
registration by the Land Registration Authority (LRA), which specified the exact areas and boundaries of
Lot No. 2276. Granting that there is an encroachment to the oppositors adjoining land, respondent
reasoned that it is not for the court a quo, sitting as a Land Registration Court, to entertain the opposition
because the case should be ventilated in a separate proceeding as an ordinary civil case.

During the trial, respondent was presented as the sole witness. Respondent, who was 81 years old
at that time, testified that he acquired the property in 1956 when his parents donated the same to
him.
[8]
He presented as Exhibit H
[9]
Tax Declaration No. 048-0267, evidencing the payment of realty
taxes for Lot No. 2276 in 1997. A Certification from the Office of the Municipal Treasurer
[10]
was
likewise introduced by the respondent showing the payment of real estate taxes from 1956 up to the year
1997. He further declared that the property is a residential land with improvements such as a house made
of solid materials and fruit-bearing trees. In 1957, respondent told the court that he constructed a concrete
wall surrounding the entire property. Respondent also narrated that he grew up on the subject lot and
spent his childhood days in the area.
[11]


On cross-examination, respondent claimed that he has six brothers and sisters, none of whom are
claiming any interest over the property.
[12]


On June 16, 1997, the trial court noted
[13]
a Report
[14]
submitted by the Director of Lands, which
informed the court that as per records of the Land Management Bureau in Manila, Lot No. 2276, CAD-
239 is covered by Free Patent Application No. 10-2-664 of Anastacia Vitero.

The RTC rendered a Decision dated October 3, 1997 granting the application for registration.
The dispositive portion of the decision reads:

WHEREFORE, decision is hereby rendered finding the petitioner entitled to
registration. Accordingly, after the finality of this decision, let a decree and, thereafter
the corresponding certificate of title over Lot No. 2276 of the Ligao Cadastre as
delimited by the Technical Description, Annex A-2 of the application, together with the
improvements thereon, issue in the name of LUDOLFO Y. MUOZ, of legal age,
Filipino citizen, married to JOSEFINA PALENCIA, of Mabini Street, Barangay Tinago,
Municipality of Ligao, Province of Albay.

Conformably with the above findings, as prayed for by the Director,
Department of Registration, Land Registration Authority in his Report dated March 6,
1997, the application, if any, in Cad. Case No. 53, Cadastral Record No. 1404 is hereby
ordered dismissed.

The opposition of Alex Vasquez for lack of merit is hereby ordered dismissed.

Let copy of this Decision be furnished the Office of the Solicitor General,
Provincial Prosecutor of Albay, Oppositor Alez Vasquez and Petitioner.

SO ORDERED.
[15]



On appeal, petitioner argued that the trial court did not acquire jurisdiction over the subject lot
because: (1) the notice of initial hearing was not timely filed; (2) the applicant failed to present the
original tracing cloth plan of the property sought to be registered during the trial; and (3) the applicant
failed to present evidence that the land is alienable and disposable.

Subsequently, the CA affirmed the decision of the court a quo. The appellate court explained that
there was conclusive proof that the jurisdictional requirement of due notice had been complied with as
mandated under Section 24 of Presidential Decree No. 1529. Furthermore, the failure to present in
evidence the tracing cloth plan of the subject property did not deprive the lower court of its jurisdiction to
act on the application in question. Lastly, the CA ruled that respondent need not adduce documentary
proof that the disputed property had been declared alienable and disposable for the simple reason that the
lot had once been covered by free patent application; hence, this alone is conclusive evidence that the
property was already declared by the government as open for public disposition.

The petitioner, through the OSG, raises the following grounds for the petition:

I.
THE COURT OF APPEALS ERRED IN NOT FINDING THAT THE TRIAL COURT
HAS NOT ACQUIRED JURISDICTION OVER THE CASE.

II.
PRIVATE RESPONDENT HAS NOT PROVEN BY COMPETENT EVIDENCE
THAT THE PROPERTY IS ALIENABLE AND DISPOSABLE PROPERTY OF THE
PUBLIC DOMAIN.
[16]


Anent the first issue, petitioner maintains that the failure to present the original tracing cloth plan is
a fatal omission which necessarily affected the trial courts jurisdiction to proceed with the case.

It bears stressing that the constructive seizure of land accomplished by posting of notices and
processes upon all persons mentioned in notices by means of publication and sending copies to said
persons by registered mail in effect gives the court jurisdiction over the lands sought to be
registered.
[17]


While petitioner correctly contends that the submission in evidence of the original tracing cloth
plan is a mandatory and even a jurisdictional requirement, this Court has recognized instances of
substantial compliance with this rule.
[18]
It is true that the best evidence to identify a piece of land for
registration purposes is the original tracing cloth plan from the Bureau of Lands, but blueprint copies and
other evidence could also provide sufficient identification.
[19]
In the present application
for registration, respondent submitted, among other things, the following supporting documents: (1) a
blueprint copy of the survey plan
[20]
approved by the Bureau of Lands; and (2) the technical
descriptions
[21]
duly verified and approved by the Director of Lands.

The Court held in Recto v. Republic
[22]
that the blueprint copy of the cloth plan together with the
lots technical description duly certified as to their correctness by the Bureau of Lands are adequate to
identify the land applied for registration, thus

On the first challenge, the petitioner invokes the case of Director of Lands v.
Reyes, where it was held that the original tracing cloth plan of the land applied for
which must be approved by the Director of Lands was a statutory requirement of
mandatory character for the identification of the land sought to be registered. As what
was submitted was not the tracing cloth plan but only the blueprint copy of the survey
plan, the respondent court should have rejected the same as insufficient.

We disagree with this contention. The Court of Appeals was correct when it
observed that in that case the applicant in effect had not submitted anything at all to
identify the subject property because the blueprint presented lacked the approval of the
Director of Lands. By contrast

In the present case, there was considerable compliance with the
requirement of the law as the subject property was sufficiently
identified with the presentation of blueprint copy of Plan AS-06-
000002 (San Pedro v. Director of Lands, CA-G.R. No. 65332-R, May
28, 1981). It should be noted in this connection that the Bureau of
Lands has certified to the correctness of the blueprint copy of the plan
including the technical description that go with it. Hence, we cannot
ignore the fact, absent in the Reyes case, that applicant has provided
ample evidence to establish the identity of the subject
property. (Emphasis supplied)

x x x.
[23]


Moreover, if the survey plan is approved by the Director of Lands and its correctness has not been
overcome by clear, strong and convincing evidence, the presentation of the tracing cloth plan may be
dispensed with.
[24]
All the evidence on record sufficiently identified the property as the one applied for by
respondent, and containing the corresponding metes and bounds as well as area. Consequently, the
original tracing cloth plan need not be presented in evidence.
[25]


Anent the second issue, petitioner stresses that in proving the alienable and disposable nature of the
property, there has to be a certification from the Department of Environment and Natural Resources and
Community Environment and Natural Resources Office (CENRO).

The CA is of the opinion that respondent need not adduce documentary proofs that the disputed
property has been declared alienable and disposable because of the fact that it had once been covered by
Free Patent Application No. 10-2-664 in the name of respondents mother, which was unfortunately not
acted upon by the proper authorities. The CA declares that this is proof enough that the property was
declared by the government as open for public disposition. This contention was adopted by the respondent
both in his Comment and Memorandum filed before the Court.

Notwithstanding all the foregoing, the Court cannot sustain the argument of respondent that the
subject property was already declared alienable and disposable land.

Petitioner is correct when it remarked that it was erroneous for the appellate court to assume that
the property in question is alienable and disposable based only on the Report dated May 21, 1997 of the
Director of Lands indicating that the land involved in said case described as Lot 2276, CAD-239 is
covered by Free Patent Application No. 10-2-664 of Anastacia Vitero.

It must be pointed out that in its Report
[26]
dated March 6, 1997, the LRA stated that:

3. This Authority is not in a position to verify whether or not the parcel
of land subject of registration is already covered by land patent, previously approved
isolated survey and is within forest zone.

WHEREFORE, to avoid duplication in the issuance of titles covering the same
parcel of land and the issuance of titles for lands within the forest zone which have not
been released and classified as alienable, the foregoing is respectfully submitted to the
Honorable Court with the recommendation that the Lands Management Bureau,
Manila, Community Environment and Natural Resources Office, Lands
Management Sector and Forest Management Bureau, all in Legazpi City, be
ordered to submit a report to the Court on the status of the land applied for, to
determine whether or not said land or any portion thereof, is already covered by
land patent, previously approved isolated survey and is within the forest zone and
that should the instant application be given due course, the application in Cad. Case No.
53, Cadastral Record No. 1404 with respect to Lot 2276 be dismissed.
[27]


Noteworthy is the fact that neither the Director of Lands nor the LRA attested that the land
subject of this proceeding is alienable or disposable.

For clarity, applications for confirmation of imperfect title must be able to prove the following:
(1) that the land forms part of the alienable and disposable agricultural lands of the public domain; and
(2) that they have been in open, continuous, exclusive and notorious possession and occupation of the
same under a bona fide claim of ownership either since time immemorial or since June 12, 1945.
[28]


Commonwealth Act No. 141, also known as the Public Land Act, remains to this day the existing
general law governing the classification and disposition of lands of the public domain, other than timber
and mineral lands.
[29]
Section 6 of CA No. 141 empowers the President to classify lands of the public
domain into alienable and disposable lands of the public domain, which prior to such classification are
inalienable and outside the commerce of man. Section 7 of CA No. 141 authorizes the President to
declare what lands are open to disposition or concession. Section 8 of CA No. 141 states that the
government can declare open for disposition or concession only lands that are officially delimited and
classified.

Under the Regalian doctrine embodied in our Constitution, all lands of the public domain belong
to the State, which is the source of any asserted right to ownership of land. Therefore, all lands not
appearing to be clearly within private ownership are presumed to belong to the State. Accordingly, public
lands not shown to have been reclassified or released as alienable agricultural land or alienated to a
private person by the State remain part of the alienable public domain.
[30]


As already well-settled in jurisprudence, no public land can be acquired by private persons
without any grant, express or implied, from the government; and it is indispensable that the person
claiming title to public land should show that his title was acquired from the State or any other mode of
acquisition recognized by law.
[31]
To prove that the land subject of an application for registration is
alienable, the applicant must establish the existence of a positive act of the government such as a
presidential proclamation or an executive order; an administrative action; investigation reports of Bureau
of Lands investigators; and a legislative act or a statute.
[32]
The applicant may also secure a certification
from the Government that the land applied for is alienable and disposable.
[33]


In the present case, respondent failed to submit a certification from the proper government
agency to prove that the land subject for registration is indeed alienable and disposable. A CENRO
certificate, which respondent failed to secure, could have evidenced the alienability of the land involved.

Considering that respondent has failed to convince this Court of the alienable and disposable
character of the land applied for, the Court cannot approve the application for registration.

WHEREFORE, the instant petition is GRANTED. Accordingly, the decision dated August 29,
2001 of the Court of Appeals in CA-G.R. CV No. 58170, as reiterated in its resolution of January 29,
2002,
is REVERSED and SET ASIDE, and the application for registration filed by respondent Ludolfo
V. Muoz is DENIED.

No costs.

SO ORDERED.


ADOLFO S. AZCUNA
Associate Justice
WE CONCUR:




REYNATO S. PUNO
Chief Justice
Chairperson



ANGELINA SANDOVAL-GUTIERREZ RENATO C. CORONA
Associate Justice Associate Justice



CANCIO C. GARCIA
Associate Justice




CERTIFICATION


Pursuant to Section 13, Article VIII of the Constitution, it is hereby certified that the conclusions in
the above Decision had been reached in consultation before the case was assigned to the writer of the
opinion of the Courts Division.



REYNATO S. PUNO
Chief Justice




[1]
Penned by Associate Justice Cancio C. Garcia (now an Associate Justice of the Supreme
Court), with Associate Justices Hilarion L. Aquino and Jose L. Sabio, Jr. concurring, rollo, pp.
34-50.
[2]
Penned by Judge Jose S. Saez, id. at 62-69.
[3]
Records, p. 1.
[4]
Id. at 34-35.
[5]
Id. at 53.
[6]
Id. at 61-62.
[7]
Id. at 64.
[8]
TSN, May 23, 1997, p. 4.
[9]
Records, p. 76.
[10]
Exhibit I, id. at 77-79.
[11]
TSN, May 23, 1997, p. 6.
[12]
Id. at 7.
[13]
Records, p. 88.
[14]
Id. at 86.
[15]
Id. at 95-96.
[16]
Rollo, p. 14.
[17]
Registration of Land Titles and Deeds, Antonio H. Noblejas and Edilberto H. Noblejas, 1992
edition, p. 104 (underscoring supplied).
[18]
Republic v. Hubilla, G.R. No. 157683, February 11, 2005, 451 SCRA 181, 184.
[19]
Recto v. Republic, G.R. No. 160421, October 4, 2004, 440 SCRA 79, 87, citing Republic v.
Court of Appeals, G.R. No. L-62680, November 9, 1988, 167 SCRA 150, 154, Republic v.
Intermediate Appellate Court, 229 Phil. 20 (1986).
[20]
Annex A-1, records, p. 4.
[21]
Annex A-2, id. at 5.
[22]
Supra, note 19.
[23]
Id. at 87-88, citing Republic v. Court of Appeals, supra note 19, at 153-154.
[24]
Director of Lands v. Intermediate Appellate Court, G.R. No. 70825, March 11, 1991, 195
SCRA 38, 44, citing Director of Lands v. Court of Appeals, G.R. No. 56613, March 14, 1988,
158 SCRA 568, 671,Republic v. Intermediate Appellate Court, G.R. No. 70594, October 10,
1986, 144 SCRA 705.
[25]
Republic v. Enriquez, G.R. No. 160990, September 11, 2006, 501 SCRA 436, 447.
[26]
Records, pp. 67-68.
[27]
Id. at 67 (underscoring supplied).
[28]
Carlos v. Republic, G.R. No. 164823, August 31, 2005, 468 SCRA 709, 714-715.
[29]
Chavez v. Public Estates Authority, 433 Phil. 506, 545 (2002).
[30]
Republic v. Tri-Plus Corporation, G.R. No. 150000, September 26, 2006, 503 SCRA 91, 101-
102, citing Republic v. Naguiat, G.R. No. 134209, January 24, 2006, 479 SCRA 585, 590.
[31]
Republic v. Herbieto, G.R. No. 156117, May 26, 2005, 459 SCRA 183, 199-200,
citing Padilla v. Reyes, 60 Phil. 967, 969 (1934), Lee Hong Hok v. David, G.R. No. L-
30389, December 27, 1972, 48 SCRA 372, 379.
[32]
Republic v. Court of Appeals, G.R. No. 144057, January 17, 2005, 448 SCRA 442, 449.
[33]
Zarate v. Director of Lands, G.R. No. 131501, July 14, 2004, 434 SCRA 322, 332.
SECOND DIVISION
[G.R. No. 156117. May 26, 2005]
REPUBLIC OF THE PHILIPPINES, petitioner, vs. JEREMIAS AND DAVID
HERBIETO, respondents.
D E C I S I O N
CHICO-NAZARIO, J .:
Before this Court is a Petition for Review on Certiorari, under Rule 45 of the 1997 Rules of
Civil Procedure, seeking the reversal of the Decision of the Court of Appeals in CA-G.R. CV No.
67625, dated 22 November 2002,
[1]
which affirmed the Judgment of the Municipal Trial Court
(MTC) of Consolacion, Cebu, dated 21 December 1999,
[2]
granting the application for land
registration of the respondents.
Respondents in the present Petition are the Herbieto brothers, Jeremias and David, who
filed with the MTC, on 23 September 1998, a single application for registration of two parcels of
land, Lots No. 8422 and 8423, located in Cabangahan, Consolacion, Cebu (Subject Lots). They
claimed to be owners in fee simple of the Subject Lots, which they purchased from their parents,
spouses Gregorio Herbieto and Isabel Owatan, on 25 June 1976.
[3]
Together with their
application for registration, respondents submitted the following set of documents:
(a) Advance Survey Plan of Lot No. 8422, in the name of respondent Jeremias; and Advance
Survey Plan of Lot No. 8423, in the name of respondent David;
[4]

(b) The technical descriptions of the Subject Lots;
[5]

(c) Certifications by the Department of Environment and Natural Resources (DENR)
dispensing with the need for Surveyors Certificates for the Subject Lots;
[6]

(d) Certifications by the Register of Deeds of Cebu City on the absence of certificates of title
covering the Subject Lots;
[7]

(e) Certifications by the Community Environment and Natural Resources Office (CENRO) of
the DENR on its finding that the Subject Lots are alienable and disposable, by virtue of
Forestry Administrative Order No. 4-1063, dated 25 June 1963;
[8]

(f) Certified True Copies of Assessment of Real Property (ARP) No. 941800301831, in the
name of Jeremias, covering Lot No. 8422, issued in 1994; and ARP No. 941800301833,
in the name of David, covering Lot No. 8423, also issued in 1994;
[9]
and
(g) Deed of Definite Sale executed on 25 June 1976 by spouses Gregorio Herbieto and Isabel
Owatan selling the Subject Lots and the improvements thereon to their sons and
respondents herein, Jeremias and David, for P1,000. Lot No. 8422 was sold to Jeremias,
while Lot No. 8423 was sold to David.
[10]

On 11 December 1998, the petitioner Republic of the Philippines (Republic) filed an
Opposition to the respondents application for registration of the Subject Lots arguing that: (1)
Respondents failed to comply with the period of adverse possession of the Subject Lots
required by law; (2) Respondents muniments of title were not genuine and did not constitute
competent and sufficient evidence of bona fide acquisition of the Subject Lots; and (3) The
Subject Lots were part of the public domain belonging to the Republic and were not subject to
private appropriation.
[11]

The MTC set the initial hearing on 03 September 1999 at 8:30 a.m.
[12]
All owners of the land
adjoining the Subject Lots were sent copies of the Notice of Initial Hearing.
[13]
A copy of the
Notice was also posted on 27 July 1999 in a conspicuous place on the Subject Lots, as well as
on the bulletin board of the municipal building of Consolacion, Cebu, where the Subject Lots
were located.
[14]
Finally, the Notice was also published in the Official Gazette on 02 August
1999
[15]
and The Freeman Banat News on 19 December 1999.
[16]

During the initial hearing on 03 September 1999, the MTC issued an Order of Special
Default,
[17]
with only petitioner Republic opposing the application for registration of the Subject
Lots. The respondents, through their counsel, proceeded to offer and mark documentary
evidence to prove jurisdictional facts. The MTC commissioned the Clerk of Court to receive
further evidence from the respondents and to submit a Report to the MTC after 30 days.
On 21 December 1999, the MTC promulgated its Judgment ordering the registration and
confirmation of the title of respondent Jeremias over Lot No. 8422 and of respondent David over
Lot No. 8423. It subsequently issued an Order on 02 February 2000 declaring its Judgment,
dated 21 December 1999, final and executory, and directing the Administrator of the Land
Registration Authority (LRA) to issue a decree of registration for the Subject Lots.
[18]

Petitioner Republic appealed the MTC Judgment, dated 21 December 1999, to the Court of
Appeals.
[19]
The Court of Appeals, in its Decision, dated 22 November 2002, affirmed the
appealed MTC Judgment reasoning thus:
In the case at bar, there can be no question that the land sought to be registered has been classified as
within the alienable and disposable zone since June 25, 1963. Article 1113 in relation to Article 1137 of
the Civil Code, respectively provides that All things which are within the commerce of men are
susceptible of prescription, unless otherwise provided. Property of the State or any of its subdivisions of
patrimonial character shall not be the object of prescription and that Ownership and other real rights
over immovables also prescribe through uninterrupted adverse possession thereof for thirty years, without
need of title or of good faith.
As testified to by the appellees in the case at bench, their parents already acquired the subject parcels of
lands, subject matter of this application, since 1950 and that they cultivated the same and planted it with
jackfruits, bamboos, coconuts, and other trees (Judgment dated December 21, 1999, p. 6). In short, it is
undisputed that herein appellees or their predecessors-in-interest had occupied and possessed the subject
land openly, continuously, exclusively, and adversely since 1950. Consequently, even assuming
arguendo that appellees possession can be reckoned only from June 25, 1963 or from the time the subject
lots had been classified as within the alienable and disposable zone, still the argument of the appellant
does not hold water.
As earlier stressed, the subject property, being alienable since 1963 as shown by CENRO Report dated
June 23, 1963, may now be the object of prescription, thus susceptible of private ownership. By express
provision of Article 1137, appellees are, with much greater right, entitled to apply for its registration, as
provided by Section 14(4) of P.D. 1529 which allows individuals to own land in any manner provided by
law. Again, even considering that possession of appelless should only be reckoned from 1963, the year
when CENRO declared the subject lands alienable, herein appellees have been possessing the subject
parcels of land in open, continuous, and in the concept of an owner, for 35 years already when they filed
the instant application for registration of title to the land in 1998. As such, this court finds no reason to
disturb the finding of the court a quo.
[20]

The Republic filed the present Petition for the review and reversal of the Decision of the
Court of Appeals, dated 22 November 2002, on the basis of the following arguments:
First, respondents failed to establish that they and their predecessors-in-interest had been
in open, continuous, and adverse possession of the Subject Lots in the concept of owners since
12 June 1945 or earlier. According to the petitioner Republic, possession of the Subject Lots
prior to 25 June 1963 cannot be considered in determining compliance with the periods of
possession required by law. The Subject Lots were classified as alienable and disposable only
on 25 June 1963, per CENROs certification. It also alleges that the Court of Appeals, in
applying the 30-year acquisitive prescription period, had overlooked the ruling in Republic v.
Doldol,
[21]
where this Court declared that Commonwealth Act No. 141, otherwise known as the
Public Land Act, as amended and as it is presently phrased, requires that possession of land of
the public domain must be from 12 June 1945 or earlier, for the same to be acquired through
judicial confirmation of imperfect title.
Second, the application for registration suffers from fatal infirmity as the subject of the
application consisted of two parcels of land individually and separately owned by two
applicants. Petitioner Republic contends that it is implicit in the provisions of Presidential
Decree No. 1529, otherwise known as the Property Registration Decree, as amended, that the
application for registration of title to land shall be filed by a single applicant; multiple applicants
may file a single application only in case they are co-owners. While an application may cover
two parcels of land, it is allowed only when the subject parcels of land belong to the same
applicant or applicants (in case the subject parcels of land are co-owned) and are situated
within the same province. Where the authority of the courts to proceed is conferred by a statute
and when the manner of obtaining jurisdiction is mandatory, it must be strictly complied with or
the proceedings will be utterly void. Since the respondents failed to comply with the procedure
for land registration under the Property Registration Decree, the proceedings held before the
MTC is void, as the latter did not acquire jurisdiction over it.
I
Jurisdiction
Addressing first the issue of jurisdiction, this Court finds that the MTC had no jurisdiction to
proceed with and hear the application for registration filed by the respondents but for reasons
different from those presented by petitioner Republic.
A. The misjoinder of causes of action and parties does not affect the jurisdiction of the MTC
to hear and proceed with respondents application for registration.
Respondents filed a single application for registration of the Subject Lots even though they
were not co-owners. Respondents Jeremias and David were actually seeking the individual and
separate registration of Lots No. 8422 and 8423, respectively.
Petitioner Republic believes that the procedural irregularity committed by the respondents
was fatal to their case, depriving the MTC of jurisdiction to proceed with and hear their
application for registration of the Subject Lots, based on this Courts pronouncement in Director
of Lands v. Court of Appeals,
[22]
to wit:
. . . In view of these multiple omissions which constitute non-compliance with the above-cited sections of
the Act, We rule that said defects have not invested the Court with the authority or jurisdiction to proceed
with the case because the manner or mode of obtaining jurisdiction as prescribed by the statute which is
mandatory has not been strictly followed, thereby rendering all proceedings utterly null and void.
This Court, however, disagrees with petitioner Republic in this regard. This procedural
lapse committed by the respondents should not affect the jurisdiction of the MTC to proceed
with and hear their application for registration of the Subject Lots.
The Property Registration Decree
[23]
recognizes and expressly allows the following
situations: (1) the filing of a single application by several applicants for as long as they are co-
owners of the parcel of land sought to be registered;
[24]
and (2) the filing of a single application
for registration of several parcels of land provided that the same are located within the same
province.
[25]
The Property Registration Decree is silent, however, as to the present situation
wherein two applicants filed a single application for two parcels of land, but are seeking the
separate and individual registration of the parcels of land in their respective names.
Since the Property Registration Decree failed to provide for such a situation, then this Court
refers to the Rules of Court to determine the proper course of action. Section 34 of the Property
Registration Decree itself provides that, [t]he Rules of Court shall, insofar as not inconsistent
with the provisions of this Decree, be applicable to land registration and cadastral cases by
analogy or in a suppletory character and whenever practicable and convenient.
Considering every application for land registration filed in strict accordance with the
Property Registration Decree as a single cause of action, then the defect in the joint application
for registration filed by the respondents with the MTC constitutes a misjoinder of causes of
action and parties. Instead of a single or joint application for registration, respondents Jeremias
and David, more appropriately, should have filed separate applications for registration of Lots
No. 8422 and 8423, respectively.
Misjoinder of causes of action and parties do not involve a question of jurisdiction of the
court to hear and proceed with the case.
[26]
They are not even accepted grounds for dismissal
thereof.
[27]
Instead, under the Rules of Court, the misjoinder of causes of action and parties
involve an implied admission of the courts jurisdiction. It acknowledges the power of the court,
acting upon the motion of a party to the case or on its own initiative, to order the severance of
the misjoined cause of action, to be proceeded with separately (in case of misjoinder of causes
of action); and/or the dropping of a party and the severance of any claim against said misjoined
party, also to be proceeded with separately (in case of misjoinder of parties).
The misjoinder of causes of action and parties in the present Petition may have been
corrected by the MTC motu propio or on motion of the petitioner Republic. It is regrettable,
however, that the MTC failed to detect the misjoinder when the application for registration was
still pending before it; and more regrettable that the petitioner Republic did not call the attention
of the MTC to the fact by filing a motion for severance of the causes of action and parties,
raising the issue of misjoinder only before this Court.
B. Respondents, however, failed to comply with the publication requirements mandated by
the Property Registration Decree, thus, the MTC was not invested with jurisdiction as a
land registration court.
Although the misjoinder of causes of action and parties in the present Petition did not affect
the jurisdiction of the MTC over the land registration proceeding, this Court, nonetheless, has
discovered a defect in the publication of the Notice of Initial Hearing, which bars the MTC from
assuming jurisdiction to hear and proceed with respondents application for registration.
A land registration case is a proceeding in rem,
[28]
and jurisdiction in rem cannot be
acquired unless there be constructive seizure of the land through publication and service of
notice.
[29]

Section 23 of the Property Registration Decree requires that the public be given Notice of
the Initial Hearing of the application for land registration by means of (1) publication; (2) mailing;
and (3) posting. Publication of the Notice of Initial Hearing shall be made in the following
manner:
1. By publication.
Upon receipt of the order of the court setting the time for initial hearing, the Commissioner of Land
Registration shall cause a notice of initial hearing to be published once in the Official Gazette and once in
a newspaper of general circulation in the Philippines: Provided, however, that the publication in the
Official Gazette shall be sufficient to confer jurisdiction upon the court. Said notice shall be addressed to
all persons appearing to have an interest in the land involved including the adjoining owners so far as
known, and to all whom it may concern. Said notice shall also require all persons concerned to appear
in court at a certain date and time to show cause why the prayer of said application shall not be granted.
Even as this Court concedes that the aforequoted Section 23(1) of the Property
Registration Decree expressly provides that publication in the Official Gazette shall be sufficient
to confer jurisdiction upon the land registration court, it still affirms its declaration in Director of
Lands v. Court of Appeals
[30]
that publication in a newspaper of general circulation is mandatory
for the land registration court to validly confirm and register the title of the applicant or
applicants. That Section 23 of the Property Registration Decree enumerated and described in
detail the requirements of publication, mailing, and posting of the Notice of Initial Hearing, then
all such requirements, including publication of the Notice in a newspaper of general circulation,
is essential and imperative, and must be strictly complied with. In the same case, this Court
expounded on the reason behind the compulsory publication of the Notice of Initial Hearing in a
newspaper of general circulation, thus
It may be asked why publication in a newspaper of general circulation should be deemed mandatory when
the law already requires notice by publication in the Official Gazette as well as by mailing and posting, all
of which have already been complied with in the case at hand. The reason is due process and the reality
that the Official Gazette is not as widely read and circulated as newspaper and is oftentimes delayed in its
circulation, such that the notices published therein may not reach the interested parties on time, if at all.
Additionally, such parties may not be owners of neighboring properties, and may in fact not own any
other real estate. In sum, the all encompassing in rem nature of land registration cases, the consequences
of default orders issued against the whole world and the objective of disseminating the notice in as wide a
manner as possible demand a mandatory construction of the requirements for publication, mailing and
posting.
[31]

In the instant Petition, the initial hearing was set by the MTC, and was in fact held, on 03
September 1999 at 8:30 a.m. While the Notice thereof was printed in the issue of the Official
Gazette, dated 02 August 1999, and officially released on 10 August 1999, it was published
in The Freeman Banat News, a daily newspaper printed in Cebu City and circulated in the
province and cities of Cebu and in the rest of Visayas and Mindanao, only on 19 December
1999, more than three months after the initial hearing.
Indubitably, such publication of the Notice, way after the date of the initial hearing, would
already be worthless and ineffective. Whoever read the Notice as it was published in The
Freeman Banat News and had a claim to the Subject Lots was deprived of due process for it
was already too late for him to appear before the MTC on the day of the initial hearing to oppose
respondents application for registration, and to present his claim and evidence in support of
such claim. Worse, as the Notice itself states, should the claimant-oppositor fail to appear
before the MTC on the date of initial hearing, he would be in default and would forever be
barred from contesting respondents application for registration and even the registration decree
that may be issued pursuant thereto. In fact, the MTC did issue an Order of Special Default on
03 September 1999.
The late publication of the Notice of Initial Hearing in the newspaper of general circulation is
tantamount to no publication at all, having the same ultimate result. Owing to such defect in the
publication of the Notice, the MTC failed to constructively seize the Subject Lots and to acquire
jurisdiction over respondents application for registration thereof. Therefore, the MTC
Judgment, dated 21 December 1999, ordering the registration and confirmation of the title of
respondents Jeremias and David over Lots No. 8422 and 8423, respectively; as well as the
MTC Order, dated 02 February 2000, declaring its Judgment of 21 December 1999 final and
executory, and directing the LRA Administrator to issue a decree of registration for the Subject
Lots, are both null and void for having been issued by the MTC without jurisdiction.
II
Period of Possession
Respondents failed to comply with the required period of possession of the Subject Lots for the
judicial confirmation or legalization of imperfect or incomplete title.
While this Court has already found that the MTC did not have jurisdiction to hear and
proceed with respondents application for registration, this Court nevertheless deems it
necessary to resolve the legal issue on the required period of possession for acquiring title to
public land.
Respondents application filed with the MTC did not state the statutory basis for their title to
the Subject Lots. They only alleged therein that they obtained title to the Subject Lots by
purchase from their parents, spouses Gregorio Herbieto and Isabel Owatan, on 25 June 1976.
Respondent Jeremias, in his testimony, claimed that his parents had been in possession of the
Subject Lots in the concept of an owner since 1950.
[32]

Yet, according to the DENR-CENRO Certification, submitted by respondents themselves,
the Subject Lots are within Alienable and Disposable, Block I, Project No. 28 per LC Map No.
2545 of Consolacion, Cebu certified under Forestry Administrative Order No. 4-1063, dated
June 25, 1963. Likewise, it is outside Kotkot-Lusaran Mananga Watershed Forest Reservation
per Presidential Proclamation No. 932 dated June 29, 1992.
[33]
The Subject Lots are thus
clearly part of the public domain, classified as alienable and disposable as of 25 June 1963.
As already well-settled in jurisprudence, no public land can be acquired by private persons
without any grant, express or implied, from the government;
[34]
and it is indispensable that the
person claiming title to public land should show that his title was acquired from the State or any
other mode of acquisition recognized by law.
[35]

The Public Land Act, as amended, governs lands of the public domain, except timber and
mineral lands, friar lands, and privately-owned lands which reverted to the State.
[36]
It explicitly
enumerates the means by which public lands may be disposed, as follows:
(1) For homestead settlement;
(2) By sale;
(3) By lease;
(4) By confirmation of imperfect or incomplete titles;
(a) By judicial legalization; or
(b) By administrative legalization (free patent).
[37]

Each mode of disposition is appropriately covered by separate chapters of the Public Land Act
because there are specific requirements and application procedure for every mode.
[38]
Since
respondents herein filed their application before the MTC,
[39]
then it can be reasonably inferred
that they are seeking the judicial confirmation or legalization of their imperfect or incomplete title
over the Subject Lots.
Judicial confirmation or legalization of imperfect or incomplete title to land, not exceeding
144 hectares,
[40]
may be availed of by persons identified under Section 48 of the Public Land
Act, as amended by Presidential Decree No. 1073, which reads
Section 48. The following-described citizens of the Philippines, occupying lands of the public domain or
claiming to own any such lands or an interest therein, but whose titles have not been perfected or
completed, may apply to the Court of First Instance of the province where the land is located for
confirmation of their claims and the issuance of a certificate of title thereafter, under the Land
Registration Act, to wit:
(a) [Repealed by Presidential Decree No. 1073].
(b) Those who by themselves or through their predecessors-in-interest have been in open,
continuous, exclusive, and notorious possession and occupation of agricultural lands of the
public domain, under a bona fide claim of acquisition of ownership, since June 12, 1945, or
earlier, immediately preceding the filing of the applications for confirmation of title, except
when prevented by war or force majeure. These shall be conclusively presumed to have
performed all the conditions essential to a Government grant and shall be entitled to a
certificate of title under the provisions of this chapter.
(c) Members of the national cultural minorities who by themselves or through their
predecessors-in-interest have been in open, continuous, exclusive and notorious possession
and occupation of lands of the public domain suitable to agriculture whether disposable or
not, under a bona fide claim of ownership since June 12, 1945 shall be entitled to the rights
granted in subsection (b) hereof.
Not being members of any national cultural minorities, respondents may only be entitled to
judicial confirmation or legalization of their imperfect or incomplete title under Section 48(b) of
the Public Land Act, as amended. Section 48(b), as amended, now requires adverse
possession of the land since 12 June 1945 or earlier. In the present Petition, the Subject Lots
became alienable and disposable only on 25 June 1963. Any period of possession prior to the
date when the Subject Lots were classified as alienable and disposable is inconsequential and
should be excluded from the computation of the period of possession; such possession can
never ripen into ownership and unless the land had been classified as alienable and disposable,
the rules on confirmation of imperfect title shall not apply thereto.
[41]
It is very apparent then that
respondents could not have complied with the period of possession required by Section 48(b) of
the Public Land Act, as amended, to acquire imperfect or incomplete title to the Subject Lots
that may be judicially confirmed or legalized.
The confirmation of respondents title by the Court of Appeals was based on the erroneous
supposition that respondents were claiming title to the Subject Lots under the Property
Registration Decree. According to the Decision of the Court of Appeals, dated 22 November
2002, Section 14(4) of the Property Registration Decree allows individuals to own land in any
other manner provided by law. It then ruled that the respondents, having possessed the Subject
Lots, by themselves and through their predecessors-in-interest, since 25 June 1963 to 23
September 1998, when they filed their application, have acquired title to the Subject Lots by
extraordinary prescription under Article 1113, in relation to Article 1137, both of the Civil
Code.
[42]

The Court of Appeals overlooked the difference between the Property Registration Decree
and the Public Land Act. Under the Property Registration Decree, there already exists a title
which is confirmed by the court; while under the Public Land Act, the presumption always is that
the land applied for pertains to the State, and that the occupants and possessors only claim an
interest in the same by virtue of their imperfect title or continuous, open, and notorious
possession.
[43]
As established by this Court in the preceding paragraphs, the Subject Lots
respondents wish to register are undoubtedly alienable and disposable lands of the public
domain and respondents may have acquired title thereto only under the provisions of the Public
Land Act.
However, it must be clarified herein that even though respondents may acquire imperfect or
incomplete title to the Subject Lots under the Public Land Act, their application for judicial
confirmation or legalization thereof must be in accordance with the Property Registration
Decree, for Section 50 of the Public Land Act reads
SEC. 50. Any person or persons, or their legal representatives or successors in right, claiming any lands
or interest in lands under the provisions of this chapter, must in every case present an application to the
proper Court of First Instance, praying that the validity of the alleged title or claim be inquired into and
that a certificate of title be issued to them under the provisions of the Land Registration Act.
[44]

Hence, respondents application for registration of the Subject Lots must have complied
with the substantial requirements under Section 48(b) of the Public Land Act and the procedural
requirements under the Property Registration Decree.
Moreover, provisions of the Civil Code on prescription of ownership and other real rights
apply in general to all types of land, while the Public Land Act specifically governs lands of the
public domain. Relative to one another, the Public Land Act may be considered a special
law
[45]
that must take precedence over the Civil Code, a general law. It is an established rule of
statutory construction that between a general law and a special law, the special law prevails
Generalia specialibus non derogant.
[46]

WHEREFORE, based on the foregoing, the instant Petition is GRANTED. The Decision of
the Court of Appeals in CA-G.R. CV No. 67625, dated 22 November 2002, is REVERSED. The
Judgment of the MTC of Consolacion, Cebu in LRC Case No. N-75, dated 21 December 1999,
and its Order, dated 02 February 2000 are declared NULL AND VOID. Respondents
application for registration is DISMISSED.
SO ORDERED.
Puno, Acting C.J., (Chairman), Austria-Martinez, and Callejo, Sr., JJ., concur.
Tinga, J., out of the country.



[1]
Penned by Associate Justice Mercedes Gozo-Dadole with Associate Justices B.A. Adefuin-
De La Cruz and Mariano C. Del Castillo concurring, Rollo, pp. 52-58.
[2]
Penned by Judge Wilfredo A. Dagatan, Records, pp. 100-108.
[3]
Records, pp. 1-6.
[4]
Ibid., pp. 7-8.
[5]
Ibid., pp. 9-10.
[6]
Ibid., pp. 11-12.
[7]
Ibid., pp. 13-14.
[8]
Ibid., pp. 15-18.
[9]
Ibid., pp. 19-20.
[10]
Ibid., p. 21.
[11]
Ibid., pp. 27-29.
[12]
Order, dated 29 April 1999, penned by Judge Wilfredo A. Dagatan, Ibid., p. 41.
[13]
Ibid., p. 59.
[14]
Ibid., p. 52.
[15]
Ibid., p. 58.
[16]
Ibid., pp. 96-97.
[17]
Penned by Judge Wilfredo A. Dagatan, Ibid., 62-65.
[18]
Penned by Judge Wilfredo A. Dagatan, Records, p. 109.
[19]
CA Rollo, pp. 20-38.
[20]
Supra, note 1, pp. 57-58.
[21]
G.R. No. 132963, 10 September 1998, 295 SCRA 359.
[22]
G.R. No. L-45168, 27 January 1981, 102 SCRA 370, 438, also quoted and/or reiterated in
subsequent cases of Alabang Development Corporation v. Valenzuela, G.R. No. L-
54094, 30 August 1982, 116 SCRA 261, 271; Tahanan Development Corporation v.
Court of Appeals, G.R. No. L-55771, 15 November 1982, 118 SCRA 273, 309; Register
of Deeds of Malabon, G.R. No. 88623, 05 February 1990, 181 SCRA 788, 791;
Allama v. Republic, G.R. No. 88226, 26 February 1992, 206 SCRA 600, 605.
[23]
Presidential Decree No. 1529.
[24]
Section 14 of the Property Registration Decree provides that, Where the land is owned in
common, all the co-owners shall file the application jointly.
[25]
Section 18 of the Property Registration Decree reads
SEC. 18. Application covering two or more parcels. An application may include
two or more parcels of land belonging to the applicant/s provided they are situated within
the same province or city. The court may at any time order an application to be
amended by striking out one or more of the parcels or by a severance of the application.
[26]
Katipunan v. Zandueta, 60 Phil 220 (1934).
[27]
Significant provisions of the Rules of Court are quoted below
RULE 2, SEC. 6. Misjoinder of causes of action. Misjoinder of causes of action is not
a ground for dismissal of an action. A misjoined cause of action may, on motion of a
party or on the initiative of the court, be severed and proceeded with separately.
RULE 3, SEC. 11. Misjoinder and non-joinder of parties. Neither misjoinder nor non-
joinder of parties is ground for dismissal of action. Parties may be dropped or added by
order of the court on motion of any party or on its own initiative at any stage of the action
and on such terms as are just. Any claim against a misjoined party may be severed and
proceeded with separately.
[28]
Section 2 of the Property Registration Decree.
[29]
Republic v. Court of Appeals, G.R. No. 113549, 05 July 1996, 258 SCRA 223, 236.
[30]
G.R. No. 102858, 28 July 1997, 276 SCRA 276.
[31]
Ibid., p. 286.
[32]
TSN, 24 September 1999, p. 28.
[33]
Records, pp. 15, 17.
[34]
Padilla v. Reyes, 60 Phil 967, 969 (1934).
[35]
Lee Hong Hok v. David, G.R. No. L-30389, 27 December 1972, 48 SCRA 372, 379.
[36]
Section 2.
[37]
Section 11.
[38]
Del Rosario-Igtiben v. Rebublic, G.R. No. 158449, 22 October 2004, p. 11.
[39]
Section 34 of Batas Pambansa Blg. 129, otherwise known as the Judiciary Reorganization
Act of 1980, as amended, allows the inferior courts (i.e., Metropolitan Trial Courts,
Municipal Trial Courts, and Municipal Circuit Trial Courts), duly assigned by the
Supreme Court, to hear and determine cadastral and land registration cases covering
lots where there is no controversy or opposition, or contested lots with values not
exceeding P100,000. Decisions of the inferior courts in such cases shall be appealable
in the same manner as decisions of the Regional Trial Courts. Accordingly, the
Supreme Court issued Administrative Circular No. 6-93-A, dated 15 November 1995,
authorizing the inferior courts to hear and decide the cadastral or land registration cases
as provided for by the Judiciary Reorganization Act of 1980, as amended.
[40]
Section 47 of the Public Land Act, as amended.
[41]
Almeda v. Court of Appeals, G.R. No. 85322, 30 April 1991, 196 SCRA 476; Vallarta v.
Intermediate Appellate Court, G.R. No. L-74957, 30 June 1987, 151 SCRA 679;
Republic v. Court of Appeals, G.R. No. L-40402, 16 March 1987, 148 SCRA 480.
[42]
The complete text of these provisions are reproduced below, for reference
ART. 1113. All things which are within the commerce of men are susceptible of
prescription, unless otherwise provided. Property of the State or any of its subdivisions
not patrimonial in character shall not be the object of prescription.
ART. 1137. Ownership and other real rights over immovables also prescribe through
uninterrupted adverse possession thereof for thirty years, without need of title or of good
faith.
[43]
Aquino v. Director of Lands, 39 Phil 850, 858 (1919).
[44]
Now the provisions of the Property Registration Decree.
[45]
This Court is not unaware that there are decisions by this Court declaring the Public Land Act
as a general law [Republic v. Court of Appeals, G.R. No. 106673, 09 May 2001, 357
SCRA 608, 616; Oliva v. Lamadrid, 128 Phil 770, 775 (1967)]. These cases, however,
involve the Public Land Act in relation to statutes other than the Civil Code. The
pronouncement made in the present Petition is particular to the nature of the Public Land
Act vis--vis the Civil Code.
[46]
Manila Railroad Co. v. Rafferty, 40 Phil 224 (1919).
FIRST DIVISION
[G.R. No. 146527. January 31, 2005]
REPUBLIC OF THE PHILIPPINES, petitioner, vs. MANNA PROPERTIES, INC., Represented
by its President, JOSE TANYAO,respondent.
D E C I S I O N
CARPIO, J .:
The Case
This is a petition for review
[1]
seeking to set aside the Court of Appeals Decision
[2]
dated 20
December 2000. The Court of Appeals affirmed the Decision of the Regional Trial Court, Branch
26, San Fernando, La Union (trial court) dated 21 February 1996 in Land Registration Case
No. N-2352 (LRC No. N-2352) approving the application of respondent Manna Properties, Inc.
(Manna Properties) for the registration in its name of a parcel of land located in Barangay
Pagdaraoan, San Fernando, La Union.
Antecedent Facts
As culled by the Court of Appeals from the evidence, the facts of the case are as follows:
On September 29, 1994, applicant-appellee filed an Application for the registration of title of two (2)
parcels of land, specifically:
a) Lot No. 9515, Cad. 539-D of As-013314-001434; and
b) Lot No. 1006, Cad. 539-D of As-013314-001434, located in Barangay Pagdaraoan,
San Fernando, La Union measuring around 1,480 square meters.
Initial hearing was set on February 16, 1995 by the court a quo.
Copies of the application, postal money orders for publication purposes and record were forwarded to the
Land Registration Authority by the Court a quo on October 7, 1994.
However, per Report dated November 21, 1994 of the Land Registration Authority, the full names and
complete postal addresses of all adjoining lot owners were not stated for notification purposes. As a
result thereto, per Order dated December 5, 1994, the applicant was directed to submit the names and
complete postal addresses of the adjoining owners of Lots 9514 and 9516. On December 14, 1994, the
applicant filed its compliance, which was forwarded to the Land Registration Authority on December 22,
1994 together with the notice of the Initial Hearing, which was reset to April 13, 1995.
On January 31, 1995, the Land Registration Authority requested for the resetting of the initial hearing
since April 13, 1995 fell on Holy Thursday, a non-working day to a date consistent with LRC Circular
No. 353 or ninety (90) days from date of the Order to allow reasonable time for possible mail delays and
to enable them to cause the timely publication of the notice in the Official Gazette.
The initial hearing was, accordingly, reset to April 20, 1995 by the court a quo.
On March 14, 1995, the court a quo received a letter dated March 6, 1995 from the LRA with the
information that the notice can no longer be published in the Official Gazette for lack of material time
since the National Printing Office required submission of the printing materials 75 days before the date of
the hearing. It was again requested that the initial hearing be moved to a date consistent with LRC
Circular No. 353.
Per Order dated March 15, 1995, the initial hearing was reset to July 18, 1995.
The Opposition to the application stated, among others, that the applicant is a private corporation
disqualified under the new Philippine Constitution to hold alienable lands of public domain.
Per Certificate of Publication issued by the LRA and the National Printing Office, the Notice of Initial
Hearing was published in the June 12, 1995 issue of the Official Gazette officially released on June 19,
1995. The same notice was published in the July 12, 1995 issue of the The Ilocos Herald.
Applicant-appellee presented its president Jose [Tanyao], who testified on the acquisition of the subject
property as well as Manuel Sobrepea, co-owner of the subject property, who testified on the possession
of the applicant-appellees predecessors-in-interest.
The [documentary] evidence presented were:
1. Plan AS-013314-001434 of Lots No. 9515 and 1006;
2. Technical Description of Lot No. 9515;
3. Technical Description of Lot No. 1006;
4. Certificate in lieu of Lost Surveyors Certificate;
5. Certificate of Latest Assessment;
6. Notice of Initial Hearing;
7. Certificate of Publication of the Notice of Initial Hearing by the LRA;
8. Certificate of Publication of the Notice of Initial Hearing by the National Printing
Office;
9. Certificate of Publication of the Notice of Initial Hearing by the Circulation Manager
of the Ilocos Herald;
10. Clipping of the Notice of Initial Hearing;
11. Whole Issue of the Ilocos Herald dated July 12, 1995;
12. Page 3 of Ilocos Herald dated January 12, 1995;
13. Sheriffs Return of Posting;
14. Certificate of Notification of all adjoining owners of the Notice of Initial Hearing on
July 18, 1995.
Thereafter, the court a quo rendered a Decision dated February 21, 1996 granting the application. (sic)
[3]

The Office of the Solicitor General, appearing on behalf of petitioner Republic of the
Philippines (petitioner), promptly appealed the trial courts decision to the Court of Appeals.
On 20 December 2000, the Court of Appeals dismissed petitioners appeal.
Hence, this petition.
The Regional Trial Courts Ruling
The trial court found that Manna Properties has substantiated by clear and competent
evidence all its allegations in the application for original land registration. The Land Registration
Authority (LRA) did not present any evidence in opposition to the application. The trial court
ruled in this wise:
WHEREFORE, premises considered, the Court hereby approves the application, and orders that the
parcels of land identified as Lots 9515 and 1006 of Cad. 5[3]9-D San Fernando Cadastre with a total area
of One Thousand Four Hundred Eighty (1,480) square meters, situated in Barangay Pagdaraoan, San
Fernando, La Union and embraced in Plan AS-1331434 (Exh. A and the technical description described
in Exhibit B and B-1) shall be registered in accordance with Presidential Decree No. 1529,
otherwise known as the Property Registration Decree in the name of the applicant Manna Properties, Inc.,
represented by its President Jose [Tanyao], Filipino citizen, of legal age, married to Marry [Tanyao] with
residence and postal address at Jackivi Enterprises, Pagdaraoan, San Fernando, La Union, pursuant to the
provisions of Presidential Decree No. 1529.
[4]

The Court of Appeals Ruling
The Court of Appeals upheld the trial courts ruling and dismissed petitioners argument that
the applicant failed to comply with the jurisdictional requirements of Presidential Decree No.
1529
[5]
(PD 1529). The Court of Appeals pointed out that the 90-day period for setting the
initial hearing under Section 23 of PD 1529 is merely directory and that it is the publication of
the notice of hearing itself that confers jurisdiction. The Court of Appeals stated that the records
of the case reveal that the testimony of Manuel Sobrepea was not the sole basis for the trial
courts finding that Manna Propertiess predecessors-in-interest had been in possession of the
land in question as early as 1953. The Court of Appeals added that while tax declarations are
not conclusive proof of ownership, they are the best indicia of possession.
The Issues
Petitioner raises the following issues for resolution:
1. WHETHER MANNA PROPERTIES FAILED TO COMPLY WITH THE
JURISDICTIONAL REQUIREMENTS FOR ORIGINAL REGISTRATION; and
2. WHETHER MANNA PROPERTIES HAS SUFFICIENTLY PROVEN POSSESSION OF
THE PROPERTY FOR THE REQUISITE PERIOD.
The Ruling of the Court
On Whether Manna Properties Failed
to Comply with the J urisdictional
Requirements for Original Registration
Petitioner contends that PD 1529 sets a 90-day maximum period between the court order
setting the initial hearing date and the hearing itself. Petitioner points out that in this case, the
trial court issued the order setting the date of the initial hearing on 15 March 1995, but the trial
court set the hearing date itself on 18 July 1995. Considering that there are 125 days in
between the two dates, petitioner argues that the trial court exceeded the 90-day period set by
PD 1529. Thus, petitioner concludes the applicant [Manna Properties] failed to comply with the
jurisdictional requirements for original registration.
The petitioner is mistaken.
The pertinent portion of Section 23 of PD 1529 reads:
Sec. 23. Notice of initial hearing, publication etc. The court shall, within five days from filing of the
application, issue an order setting the date and hour of initial hearing which shall not be earlier than forty-
five days nor later than ninety days from the date of the order.
xxx
The duty and the power to set the hearing date lies with the land registration court. After an
applicant has filed his application, the law requires the issuance of a court order setting the
initial hearing date. The notice of initial hearing is a court document. The notice of initial hearing
is signed by the judge and copy of the notice is mailed by the clerk of court to the LRA. This
involves a process to which the party applicant absolutely has no participation.
Petitioner is correct that in land registration cases, the applicant must strictly comply with
the jurisdictional requirements. In this case, the applicant complied with the jurisdictional
requirements.
The facts reveal that Manna Properties was not at fault why the hearing date was set
beyond the 90-day maximum period. The records show that the Docket Division of the LRA
repeatedly requested the trial court to reset the initial hearing date because of printing problems
with the National Printing Office, which could affect the timely publication of the notice of hearing
in the Official Gazette. Indeed, nothing in the records indicates that Manna Properties failed to
perform the acts required of it by law.
We have held that a party to an action has no control over the Administrator or the Clerk of
Court acting as a land court; he has no right to meddle unduly with the business of such official
in the performance of his duties.
[6]
A party cannot intervene in matters within the exclusive
power of the trial court. No fault is attributable to such party if the trial court errs on matters
within its sole power. It is unfair to punish an applicant for an act or omission over which the
applicant has neither responsibility nor control, especially if the applicant has complied with all
the requirements of the law.
Petitioner limited itself to assailing the lapse of time between the issuance of the order
setting the date of initial hearing and the date of the initial hearing itself. Petitioner does not
raise any other issue with respect to the sufficiency of the application. Petitioner does not also
question the sufficiency of the publication of the required notice of hearing. Consequently,
petitioner does not dispute the real jurisdictional issue involved in land registration cases
compliance with the publication requirement under PD 1529. As the records show, the notice of
hearing was published both in the Official Gazette and a newspaper of general circulation well
ahead of the date of hearing. This complies with the legal requirement of serving the entire
world with sufficient notice of the registration proceedings.
On Whether Manna Properties Sufficiently
Established Possession of the Land
For the Period Required by Law
Petitioner asserts that Manna Properties has failed to prove its possession of the land for
the period of time required by law. Petitioner alleges that the trial court and the Court of
Appeals based their findings solely on their evaluation of the tax declarations presented by
Manna Properties.
The jurisdiction of this Court under Rule 45 of the 1997 Rules of Civil Procedure is limited to
the review and revision of errors of law.
[7]
This Court is not bound to analyze and weigh
evidence already considered in prior proceedings. Absent any of the established grounds for
exception, this Court is bound by the findings of fact of the trial and appellate courts.
The issue of whether Manna Properties has presented sufficient proof of the required
possession, under a bona fide claim of ownership, raises a question of fact.
[8]
It invites an
evaluation of the evidentiary record. Petitioner invites us to re-evaluate the evidence and
substitute our judgment for that of the trial and appellate courts. Generally, Rule 45 does not
allow this. Matters of proof and evidence are beyond the power of this Court to review under a
Rule 45 petition, except in the presence of some meritorious circumstances.
[9]
We find one such
circumstance in this case. The evidence on record does not support the conclusions of both the
trial court and the Court of Appeals.
Petitioner claimed in its opposition to the application of Manna Properties that, as a private
corporation, Manna Properties is disqualified from holding alienable lands of the public domain,
except by lease. Petitioner cites the constitutional prohibition in Section 3 of Article XII in the
1987 Constitution. Petitioner also claims that the land in question is still part of the public
domain.
On the other hand, Manna Properties claims that it has established that the land in question
has been in the open and exclusive possession of its predecessors-in-interest since the 1940s.
Thus, the land was already private land when Manna Properties acquired it from its
predecessors-in-interest.
The governing law is Commonwealth Act No. 141 (CA 141) otherwise known as the
Public Land Act. Section 48(b) of the said law, as amended by Presidential Decree No. 1073,
provides:
(b) Those who by themselves or through their predecessors-in-interest have been in open, continuous,
exclusive, and notorious possession and occupation of agricultural lands of the public domain, under
a bona fide claim of acquisition of ownership, since June 12, 1945 or earlier, immediately preceding the
filing of the application for confirmation of title except when prevented by war or force majeure. These
shall be conclusively presumed to have performed all the conditions essential to a Government grant and
shall be entitled to a certificate of title under the provisions of this chapter. (Emphasis supplied)
Lands that fall under Section 48 of CA 141 are effectively segregated from the public
domain by virtue of acquisitive prescription. We have held that open, exclusive and undisputed
possession of alienable public land for the period prescribed by CA 141 ipso jure converts such
land into private land.
[10]
Judicial confirmation in such cases is only a formality that merely
confirms the earlier conversion of the land into private land, the conversion having occurred in
law from the moment the required period of possession became complete.
[11]

Under CA 141, the reckoning point is June 12, 1945. If the predecessors-in-interest of
Manna Properties have been in possession of the land in question since this date, or earlier,
Manna Properties may rightfully apply for confirmation of title to the land. Following our ruling
in Director of Lands v. IAC,
[12]
Manna Properties, a private corporation, may apply for judicial
confirmation of the land without need of a separate confirmation proceeding for its
predecessors-in-interest first.
We rule, however, that the land in question has not become private land and remains part
of the public domain.
Under the Regalian doctrine, the State is the source of any asserted right to ownership of
land. This is premised on the basic doctrine that all lands not otherwise appearing to be clearly
within private ownership are presumed to belong to the State.
[13]
Any applicant for confirmation
of imperfect title bears the burden of proving that he is qualified to have the land titled in his
name.
[14]
Although Section 48 of CA 141 gives rise to a right that is only subject to formal
recognition, it is still incumbent upon any claimant to first prove open, continuous and adverse
possession for the requisite period of time.
[15]
It is only when the applicant complies with this
condition that he may invoke the rights given by CA 141.
The evidence submitted by Manna Properties to prove the required length of possession
consists of the testimony of one of its predecessors-in-interest, Manuel Sobrepea
(Manuel),
[16]
transferees affidavits, and several tax declarations covering the land in question.
We have ruled that while a tax declaration by itself is not sufficient to prove ownership, it
may serve as sufficient basis for inferring possession.
[17]
However, the tax declarations
presented by Manna Properties do not serve to prove their cause. Although Manna Properties
claimed during trial that they were presenting the tax declaration proving possession since 12
June 1945,
[18]
a scrutiny of the tax declaration reveals that it is not the tax declaration Manna
Properties claimed it to be.Exhibit Q-16 was in fact a substitute tax declaration allegedly
issued on 28 November 1950. The annotation at the back of this tax declaration indicates that
it was issued to replace the 1945 tax declaration covering the land in question. A substitute is
not enough.
The 1945 tax declaration must be presented considering that the date, 12 June 1945, is
material to this case. CA 141 specifically fixes the date to 12 June 1945 or earlier. A tax
declaration simply stating that it replaces a previous tax declaration issued in 1945 does not
meet this standard. It is unascertainable whether the 1945 tax declaration was issued on,
before or after 12 June 1945. Tax declarations are issued any time of the year. A tax
declaration issued in 1945 may have been issued in December 1945. Unless the date and
month of issuance in 1945 is stated, compliance with the reckoning date in CA 141
cannot be established.
There is another reason why the application for registration of Manna Properties must fail.
The tax declaration allegedly executed in 1950 and marked as Exhibit Q-16 bears several
irregularities. A small annotation found at the bottom of the back page of Exhibit Q-16 states it
cancels a previous tax declaration. Beyond stating that the cancelled tax declaration was issued
in 1945, Exhibit Q-16 does not provide any of the required information that will enable this Court
or any interested party to check whether the original 1945 tax declaration ever existed.
19
The
blanks left by Exhibit Q-16 render any attempt to trace the original tax declaration futile.
Moreover, on its face Exhibit Q-16 lacks any indication that it is only a substitute or reconstituted
tax declaration. The net effect is an attempt to pass off Exhibit Q-16 as the original tax
declaration.
The form used to prepare the tax declaration marked as Exhibit Q-16 states that it was
FILED UNDER SECTION 202 OF R.A. 7160. Republic Act No. 7160 is the Local Government
Code of 1991. The sworn undertaking by the Deputy Assessor who allegedly prepared the tax
declaration reads, Subscribed and sworn before me this 28 (sic) day of Nov. 1950 This
means that the tax declaration was issued more than forty (40) years before the form
used came into existence.Manna Properties gave no explanation why its tax declaration used
a form that did not exist at the time of the alleged issuance of the tax declaration. The totality of
these circumstances leads this Court to conclude that Exhibit Q-16 was fabricated for the sole
purpose of making it appear that Manna Properties predecessors-in-interest have been in
possession of the land in question since 12 June 1945.
The earliest of the un-cancelled tax declarations presented by Manna Properties is dated
1950. This is clearly insufficient to prove possession of the land since 12 June 1945. The same
can be said of the transferees affidavit, which was dated 1955. Manna Properties reliance on
Manuels testimony is similarly misplaced. Not only is such evidence insufficient and self-serving
on its own but, Manuel did not also specifically testify that he, or his parents or predecessors-in-
interest were in possession of the land since 12 June 1945 or earlier. The only clear assertion
of possession made by Manuel was that his family used to plant rice on that piece of land.
20

Other than the mentioned pieces of evidence, Manna Properties did not present sufficient
proof that its predecessors-in-interest have been in open, continuous and adverse possession
of the land in question since 12 June 1945. At best, Manna Properties can only prove
possession since 1952. Manna Properties relied on shaky secondary evidence like the
testimony of Manuel and substitute tax declarations. We have previously cautioned against the
reliance on such secondary evidence in cases involving the confirmation of an imperfect title
over public land.
21
Manna Properties evidence hardly constitutes the well-nigh incontrovertible
evidence necessary to acquire title through adverse occupation under CA 141.
22

WHEREFORE, we GRANT the instant petition. We REVERSE the Decision of the Court of
Appeals dated 20 December 2000 in CA-G.R. CV No. 52562. The Application for Registration
filed by Manna Properties, Inc. over Lots No. 9515 and 1006 of Cad. 539-D, with a total area of
One Thousand Four Hundred Eighty (1,480) square meters situated in Barangay Pagdaraoan,
San Fernando, La Union, is DENIED.
SO ORDERED.
Davide, Jr., C.J., (Chairman), Quisumbing, Ynares-Santiago, and Azcuna, JJ., concur.



[1]
Under Rule 45 of the 1997 Rules on Civil Procedure.
[2]
Penned by Justice Remedios A. Salazar-Fernando, with Associate Justices Quirino D. Abad
Santos, Jr. and Salvador J. Valdez, Jr., concurring.
[3]
Rollo, pp. 18-20.
[4]
Records, pp. 110-111.
[5]
Entitled Amending and Codifying the Laws Relative to Registration of Property and for other
Purposes.
[6]
Banco Espaol-Filipino v. Palanca, 37 Phil. 921 (1918).
[7]
Section 1 of Rule 45 states:
SECTION 1. Filing of petition with Supreme Court. A party desiring to appeal
by certiorari from a judgment or final order or resolution of the Court of Appeals, the
Sandiganbayan, the Regional Trial Court or other courts whenever authorized by law,
may file with the Supreme Court a verified petition for review on certiorari. The petition
shall raise only questions of law which must be distinctly set forth.
[8]
Republic v. Court of Appeals, G.R. No. 116372, 18 January 2001, 349 SCRA 451.
[9]
As laid out in Ramos, et al. v. Pepsi-Cola Bottling Co. of the Phils., et al., 125 Phil. 701
(1967):
1. when the conclusion is a finding grounded entirely on speculation,
surmises and conjectures;
2. when the inference made is manifestly mistaken, absurd or impossible;
3. where there is a grave abuse of discretion;
4. when the judgment is based on a misapprehension of facts;
5. when the findings of fact are conflicting;
6. when the Court of Appeals, in making its findings, went beyond the
issues of the case and the same is contrary to the admissions of both appellant
and appellee.
[10]
Cario v. Insular Government, 41 Phil. 935 (1909).
[11]
The Director of Lands v. IAC, 230 Phil. 590 (1986).
[12]
Ibid.
[13]
Pagkatipunan v. Court of Appeals, 429 Phil. 377 (2002).
[14]
Collado v. Court of Appeals, 439 Phil. 149 (2002).
[15]
Republic of the Philippines v. Court of Appeals, 440 Phil. 697 (2002).
[16]
TSN, 24 October 1995, p. 25.
[17]
Republic v. Court of Appeals, G.R. No. 108926, 12 July 1996, 258 SCRA 712.
[18]
Exhibit Q-16.
19
Exhibit Q-16 left the following items blank: (a) the year when the previous tax declaration
ceased; (b) the year the previous tax declaration was entered into the Real Property
Assessment Roll; (c) the name of the Assessor who executed the previous tax declaration; (d)
the previous owner of the land and its improvements; and (e) the prior assessed value of the
land.
20
TSN, 24 October 1995, pp. 25-29.
21
Alba Vda. De Raz v. Court of Appeals, 372 Phil. 710 (1999).
22
Abejaron v. Nabasa, 411 Phil. 552 (2001).
Republic of the Philippines
SUPREME COURT
Manila
FIRST DIVISION
G.R. No. 170724 January 29, 2007
REPUBLIC OF THE PHILIPPINES, Petitioner,
vs.
SAN LORENZO DEVELOPMENT CORPORATION, Respondent.
D E C I S I O N
GARCIA, J .:
In this petition for review under Rule 45 of the Rules of Court, petitioner Republic of the
Philippines seeks the reversal and setting aside of the Decision
1
dated May 23, 2005 of the
Court of Appeals (CA)-Cebu City in CA-G.R. CV No. 73996, as reiterated in its Resolution
2
of
December 7, 2005, dismissing the Republic's appeal from an earlier decision of the Municipal
Trial Court in Cities (MTCC), Danao City, which ordered the registration of title in the name of
herein respondent San Lorenzo Development Corporation over a 64,909-square meter parcel of
land in Barangay Maslog, City of Danao, Province of Cebu.
The facts:
On November 13, 1997, respondent San Lorenzo Development Corporation filed with the
MTCC of Danao City an application for registration of title to a parcel of land, described as Lot 1
of the Consolidation-Subdivision Plan, Ccn-07-000094, being a portion of Lots 3151, 3152,
3158, 3159, 3160 and 3161, Cad. 681-D, Danao Cadastre, situated in Barangay Maslog, City of
Danao, Province of Cebu, with a total area of 64,909 square meters, more or less. The
application was docketed in the MTCC as LRC No. 100.
On November 14, 1997, the MTCC issued an Order setting the application for initial hearing on
March 5, 1998. The Order required that a copy thereof be furnished the Commissioner, Land
Registration Authority, for notice and for the necessary publication to be made.
On December 11, 1997, the Solicitor General entered his appearance as counsel for petitioner
Republic and deputized the City Prosecutor of Danao City to appear in the case. On the same
date, the Republic filed its opposition to the application.
On February 24, 1998, another Order was issued by the MTCC resetting the initial hearing of
the application to June 15, 1998. This was followed by an Order of May 15, 1998, resetting
anew the initial hearing to September 23, 1998.
During the initial hearing, the respondent corporation, through counsel, offered in evidence the
following documents to prove or establish the jurisdictional facts of the case, to wit:
Exhibit "A" - The Petition for Registration containing seven (7)
pages and mandatory annexes designated as A-1 to
A-3;
Exhibit "A-1" - Lot Plan No. Ccn-07000094 of Lot 1 comprising
Cad. Lot Nos. 3151, 3152, 3158, 3159, 3160 and
3161;
Exhibit "A-2" - Technical Description of Lot No. 1;
Exhibit "A-3" - Certification of Non-requirement of Surveyor's
Certificate;
Exhibit "B" - Order resetting date of Initial Hearing to September
23, 1998;
Exhibit "B-1" - Newspaper Clipping;
Exhibit "C" - Affidavit of Publication issued by Banat News;
Exhibit "D" - Certificate of Publication issued by the Land
Registration Authority;
Exhibit "E" - Certificate of Posting issued by the Court Sheriff;
Exhibit "F" - Certificate of Publication issued by the NPO;
Exhibit "F-1" - Copy of Notice of Initial Hearing;
Exhibit "G" - Copy of the Indorsement addressed to the Clerk of
Court, MTCC, Danao City, from Salvador Oriel,
Chief, Docket Division, Land Registration Authority,
dated July 7, 1998; and
Exhibit "H" - Notice of Appearance of the Solicitor General.
Thereafter, the case was called aloud in open court to determine whether there were other
oppositors aside from the Republic. There being none, the court issued an Order of General
Default on September 23, 1998.
Respondent corporation, to prove that it and its predecessors-in-interest had been in
possession of the land applied for in the concept of an owner peacefully, continuously,
adversely and notoriously for a period required under the law, presented six (6) witnesses. The
six, who were predecessors-in-interest of composite portions of the subject parcel of land,
provided testimonies to the effect that they had been in possession of the land, and had
subsequently sold their respective parcels thereof to the respondent. Their testimonies were
supported by tax declarations and deeds of sale.
On October 12, 2001, the trial court rendered its decision
3
granting the respondents application
for registration of title, thus:
WHEREFORE, premises considered, Judgment is hereby rendered ordering the issuance of
title to Lot 1 of the Consolidation-Subdivision of Plan Ccn-07-000094, being a portion of Lot
3152, 3151, 3158, 3159, 3160 and 3161, Cad. 681-D, Danao Cadastre, situated in the
Barangay of Maslog, Danao City, Province of Cebu, Island of Cebu, containing an area of
SIXTY FOUR THOUSAND NINE HUNDRED NINE (64,909) square meters, for and in the name
of San Lorenzo Development Corporation, with principal office address at Ground Floor,
Stanford Tower Condominium, 1870 M.H. Del Pilar Street, Malate, Metro Manila.
Upon finality of this Decision, let a corresponding decree of registration be issued in favor of
applicant in accordance with Sec. 39 of PD 1529.
SO ORDERED.
On November 7, 2001, petitioner Republic filed a Notice of Appeal, therein making known that it
was elevating the case to the CA. In the CA, the Republics appellate recourse was docketed as
CA-G.R. CV No. 73996.
In the herein assailed decision
4
of May 23, 2005, the CA-Cebu City dismissed the Republics
appeal. Its motion for reconsideration having been denied by the same court in its equally
assailed resolution
5
of December 7, 2005, the Republic is now before this Court via the instant
petition raising the following issues:
1. Whether or not the defective and/or want of notice by publication of the initial
hearing(s) of the case a quo vested the trial court with jurisdiction to take cognizance
thereof; and
2. Whether or not deeds of sale and tax declarations/clearances constitute the "well-nigh
incontrovertible" evidence necessary to acquire title through adverse occupation under
C.A. No. 141.
In the matter of jurisdiction, petitioner Republic maintains that the MCTC never acquired
jurisdiction over the case on account of its failure to conduct the initial hearing thereof within the
period fixed in Section 23 of P.D. No. 1529, otherwise known as the Property Registration
Decree, which mandates that the date and hour of initial hearing shall not be earlier than 45
days nor later than 90 days from the date of the Order. In the Republics own words:
6

After a series of postponements, the trial court finally set the initial hearing of the case on
September 23, 1998 in an order issued on May 15, 1998 xxx. The notice of initial hearing,
however, was issued only on June 6, 1998.
Pursuant to Section 23, P.D. 1529, the initial hearing of the case must have to be not earlier
than forty-five (45) days and not later than ninety (90) days from the date of the order setting the
date and hour of the initial hearing. The Order having been issued on May 15, 1998, the initial
hearing should have been set not earlier than June 29, 1998 (45 days from May 15, 1998 and
not later than August 13, 1998 (90 days from May 15, 1998). Unfortunately, the initial hearing
was scheduled and actually held on September 23, 1998, some forty-one (41) days later than
the prescribed period.
Even if counted from June 8, 1998 (date of notice of hearing), still the hearing on September 23,
1998 is seventeen (17) days late than the prescribed period of ninety (90) days, the last day of
which fell on September 6, 1998.
It is noteworthy that both parties invoke the decision of the Court in Republic v. Manna
Properties, Inc.,[7] decided January 31, 2005, albeit each cites different portions thereof, and for
different purposes. The common reliance on said case is well-placed as it is, indeed, of a similar
factual setting. Furthermore, that case tackles the same two (2) issues presently raised:
compliance with the jurisdictional requirements for original registration, and proof of possession
for the requisite period.
A careful reading of Republic v. Manna Properties, Inc. will support a finding in favor of the
respondent but only as regards the issue of jurisdiction. Speaking on that issue, the Court in
Manna Properties, Inc., wrote:
The duty and the power to set the hearing date lie with the land registration court. After an
applicant has filed his application, the law requires the issuance of a court order setting the
initial hearing date. The notice of initial hearing is a court document. The notice of initial hearing
is signed by the judge and copy of the notice is mailed by the clerk of court to the LRA [Land
Registration Authority]. This involves a process to which the party applicant absolutely has no
participation.
Petitioner is correct that in land registration cases, the applicant must strictly comply with the
jurisdictional requirements. In this case, the applicant complied with the jurisdictional
requirements.
The facts reveal that Manna Properties was not at fault why the hearing date was set beyond
the 90-day maximum period. x x x.
We have held that "a party to an action has no control over the Administrator or the Clerk of
Court acting as a land court; he has no right to meddle unduly with the business of such official
in the performance of his duties."
8
A party cannot intervene in matters within the exclusive
power of the trial court. No fault is attributable to such party if the trial court errs on matters
within its sole power. It is unfair to punish an applicant for an act or omission over which the
applicant has neither responsibility nor control, especially if the applicant has complied with all
the requirements of the law.
Moreover, it is evident in Manner Properties, Inc. that what is more important than the date on
which the initial hearing is set is the giving of sufficient notice of the registration proceedings via
publication. In fact, in its memorandum,
9
petitioner Republic "concedes (a) that respondent
should not be faulted if the initial hearing that was conducted on September 23, 1995 was
outside the 90-day period set forth under Section 23 of Presidential Decree No. 1529, and (b)
that respondent might have substantially complied with the requirement thereunder relating to
the registration of the subject land."
10
Hence, on the issue of jurisdiction, we find for the
respondent, in that its application for registration was rightfully given due course by the MTCC.
However, on the more important issue of lack of evidence of possession on the part of the
respondent for the period required by law, the balance must tilt in favor of the petitioner.
Very evident from Republic v. Manna Properties, Inc. is that the reckoning date under the Public
Land Act
11
for the acquisition of ownership of public lands is J une 12, 1945 or earlier, and that
evidence of possession from that date or earlier is essential for a grant of an application for
judicial confirmation of imperfect title. Section 48(b) of the Public Land Act, as amended by P.D.
No. 1073, provides:
(b) Those who by themselves or through their predecessors-in-interest have been in open,
continuous, exclusive, and notorious possession and occupation of agricultural lands of the
public domain, under a bona fide claim of acquisition of ownership, since June 12, 1945 or
earlier, immediately preceding the filing of the application for confirmation of title except when
prevented by war or force majeure. Those shall be conclusively presumed to have performed all
the conditions essential to a Government grant and shall be entitled to a certificate of title under
the provisions of this chapter. (Emphasis supplied)
Similarly, Section 14 of P.D. No. 1529 the Property Registration Decree provides, inter alia,
as follows:
Section 14. Who may apply. - The following persons may file in the proper Court of First
Instance an application for registration of title to land, whether personally or through their duly
authorized representatives:
1. Those who by themselves or through their predecessors-in-interest have been in open,
continuous, exclusive and notorious possession and occupation of alienable and disposable
lands of the public domain under a bona fide claim of ownership since June 12, 1945, or earlier;
(Emphasis supplied)
Here, in support of its application for registration, the respondent corporation submitted a
certification from the Community Environment and Natural Resources Office (CENRO) that the
parcel of land sought to be registered forms part of the general area classified as alienable and
disposable public land under Forestry Administrative Order No. 4-467 dated June 7, 1938. It
also submitted tax declarations and/or clearances, the earliest of which is in the year 1964 for
Lots 3150 and 3160; 1963 for Lot 3151; and 1948 for Lots 3152, 3159 and 3161.
The respondent's application was granted by the two (2) courts below on the premise that,
reckoned to date, possession of the subject parcel of land since the declaration of alienability
and disposability on June 7, 1938 was more than fifty (50) years already. Adverse possession
for at least thirty (30) years had long been completed. This reasoning was fraught with errors.
First, the law, as mentioned earlier, requires that the possession of lands of the public domain
must be from at least June 12, 1945 for the same to be acquired through judicial confirmation of
imperfect title. Through the years, Section 48(b), supra, of the Public Land Act has been
amended several times. The case of Republic v. Doldol[12] provides a summary of these
amendments:
x x x. The original Section 48(b) of C.A. No. 141 provided for possession and occupation of
lands of the public domain since July 26, 1894. This was superseded by R.A. No. 1942, which
provided for a simple thirty-year prescriptive period of occupation by an applicant for judicial
confirmation of imperfect title. The same, however, has already been amended by Presidential
Decree 1073, approved on January 25, 1977. As amended, Section 48(b) now reads:
(b) Those who by themselves or through their predecessors-in-interest have been in open,
continuous, exclusive, and notorious possession and occupation of agricultural lands of the
public domain, under a bona fide claim of acquisition of ownership, since June 12, 1945 or
earlier, immediately preceding the filing of the application for confirmation of title except when
prevented by war or force majeure. These shall be conclusively presumed to have performed all
the conditions essential to a Government grant and shall be entitled to a certificate of title under
the provisions of this chapter.
Section 48(b) of the Public Land Act, as amended by PD No. 1073, presently requires, for
judicial confirmation of an imperfect or incomplete title, the possession and occupation of the
piece of land by the applicants, by themselves or through their predecessors-in-interest, since
12 June 1945 or earlier. This provision is in total conformity with Section 14(1) of the Property
Registration Decree heretofore cited.
As the law now stands, a mere showing of possession for thirty years or more is not sufficient. It
must be shown, too, that possession and occupation had started on June 12, 1945 or earlier.
Second, the acceptance by the courts a quo of the CENRO certificate of alienability and
disposability as evidence ofpossession since the date of said certificate is patently erroneous.
According to the CENRO certification, the subject land was alienable and disposable public land
since June 7, 1938. This certification does not in any way indicate that the respondent and its
predecessors-in-interest had been in possession of the property as far back as 1938.
The Public Land Act requires that the applicant must prove two things, to wit:
1. That the land is alienable public land; and
2. That his open, continuous, exclusive and notorious possession and occupation of the
same must either be since time immemorial or for the period prescribed in the Public
Land Act.
13

All that the CENRO certificate evidences is the alienability of the land involved, not the open,
continuous, exclusive and notorious possession and occupation thereof by the respondent or its
predecessors-in-interest for the period prescribed by law.
As in Manna Properties, Inc., while the Court acknowledges tax declarations as sufficient basis
for inferring possession, the tax declarations presented by the respondent in this case do not
serve to prove their cause:
The 1945 tax declaration must be presented considering that the date, 12 June 1945, is material
to this case. CA 141 specifically fixes the date to 12 June 1945 or earlier. x x x Unless the date
and month of issuance in 1945 is stated, compliance with the reckoning date in CA 141 cannot
be established.
14

Here, the earliest of the tax declarations presented by the respondent was in the year 1948 for
Lots 3152, 3159 and 3161;1963 for Lot 3151; and 1964 for Lots 3150 and 3160. At best, the
respondent corporation can only prove possession since said dates. This does not constitute
the evidence necessary to acquire title through adverse occupation under CA 141, as amended.
Lastly, while it is true that the issue of whether or not the respondent corporation has presented
sufficient proof or the required possession raises a question of fact, which ordinarily cannot be
entertained in a petition under Rule 45, one of the exceptions to that rule is when, as here, the
evidence on record does not support the conclusions of both the trial and the appellate courts.
On the whole, we find merit in the petition.
WHEREFORE, the instant petition is GRANTED. Accordingly, the decision dated May 23, 2005
of the Court of Appeals-Cebu City in CA-G.R. CV No. 73996, as reiterated in its resolution of
December 7, 2005, is REVERSED and SET ASIDE,and the application for registration filed by
respondent San Lorenzo Development Corporation is DENIED.
No costs.
SO ORDERED.
CANCIO C. GARCIA
Associate Justice
WE CONCUR:
REYNATO S. PUNO
Chief Justice
Chairperson
ANGELINA SANDOVAL-GUTIERREZ
Associate Justice
RENATO C. CORONA
Asscociate Justice
ADOLFO S. AZCUNA
Associate Justice
C E R T I F I C A T I O N
Pursuant to Article VIII, Section 13 of the Constitution, it is hereby certified that the conclusions
in the above decision had been reached in consultation before the case was assigned to the
writer of the opinion of the Courts Division.
REYNATO S. PUNO
Chief Justice


Footnotes
1
Penned by Associate Justice Isaias P. Dicdican, with Associate Justices Vicente L. Yap
and Enrico A. Lanzanas, concurring; Rollo, pp. 39-46.
2
Id. at 47-48.
3
Id. at 63-69.
4
Supra note 1.
5
Supra note 2.
6
Petition, pp. 19-20; Rollo, pp. 25-26.
7
G.R. No. 146527, 450 SCRA 247.
8
Banco Espanol-Filipino v. Palanca, 37 Phil. 921 (1918).
9
Rollo pp. 150-185.
10
Id. at 170.
11
Commonwealth Act No. 141, as amended.
12
G.R. No. 132963, September 10, 1998, 295 SCRA 359, 364.
13
Republic v. Court of Appeals, G.R. No. 108998, August 24, 1994, 235 SCRA 567.
14
Supra note 6 at 261.
FIRST DIVISION
[G.R. No. 155012. April 14, 2004]
REPUBLIC OF THE PHILIPPINES, petitioner, vs. CARMENCITA M. ALCONABA; LUISITO
B. MELENDEZ; CONCEPCION M. LAZARO; MAURICIO B. MELENDEZ, JR.; and
MYRNA M. GALVEZ, represented by CONCEPCION M. LAZARO,respondents.
D E C I S I O N
DAVIDE, JR., C.J .:
To serve the ends of social justice, which is the heart of the 1987 Constitution, the State
promotes an equitable distribution of alienable agricultural lands of the public domain to
deserving citizens, especially the underprivileged. A land registration court must, therefore,
exercise extreme caution and prudent care in deciding an application for judicial confirmation of
an imperfect title over such lands so that the public domain may not be raided by unscrupulous
land speculators.
[1]

At bar is a petition for review under Rule 45 of the Rules of Civil Procedure seeking to set
aside the decision
[2]
of the Court of Appeals of 26 August 2002 in CA-G.R. CV No. 64323, which
affirmed the decision
[3]
of the Municipal Trial Court (MTC) of Cabuyao, Laguna,
[4]
of 1
September 1998 in MTC LRC Case No. 06 ordering the registration in favor of the respondents
of parcels of land situated at Barangay Sala, Cabuyao, Laguna, designated as Lot 2111-A,
2111-B, 2111-C, 2111-D, and 2111-E.
The pertinent facts are as follows:
On 14 November 1996, the respondents filed before the MTC of Cabuyao, Laguna, an
application
[5]
for registration of title over five parcels of land, each with an area of 5,220 square
meters, situated in Barangay Sala, Cabuyao, Laguna. In their application, they stated, among
other things, that they are the sole heirs of Spouses Melencio E. Melendez, Sr., and Luz
Batallones Melendez, original owners of Lot 2111 of CAD-455, with an area of 2.6
hectares. Their parents had been in possession of the said property since 1949, more or
less. After the death of their mother and father on 19 February 1967 and 5 May 1976,
respectively, they partitioned the property among themselves and subdivided it into five lots,
namely, Lots 2111-A, 2111-B, 2111-C, 2111-D, and 2111-E. Since then they have been in
actual possession of the property in the concept of owners and in a public and peaceful manner.
Petitioner Republic of the Philippines, through the Office of the Solicitor General (OSG),
opposed the application on the following grounds: (a) neither the respondents nor their
predecessors-in-interest possess sufficient title to the property or have been in open,
continuous, exclusive, and notorious possession and occupation of the land in question since
1945 or prior thereto; (b) the muniments of title, i.e., tax declaration and tax receipts, presented
by the respondents do not constitute competent and sufficient evidence of a bona fide right to
registration of the land under Section 48(b), Commonwealth Act No. 141, otherwise known
as The Public Land Act,
[6]
as amended by Presidential Decree No. 1073; (c) the claim of
ownership in fee simple on the basis of a Spanish title or grant can no longer be availed of by
the respondents; and (d) the land is part of the public domain belonging to the Republic of the
Philippines.
[7]

At the trial on the merits, respondents Mauricio B. Melendez, Jr., and Carmencita M.
Alconaba testified to establish their claim over the subject lots. Mauricio claimed that he and his
co-respondents acquired by inheritance from their deceased parents Lot 2111 of Cad-455,
which is an agricultural land. Their parents had been in possession of the said land since 1949
and had been religiously paying the taxes due thereon. When their parents died, he and his
siblings immediately took possession of said property in the concept of an owner, paid taxes,
and continued to plant rice thereon. On 24 June 1996, he and his co-heirs executed an
Extrajudicial Settlement with Partition over the said lot and subdivided it into five lots.
[8]

For her part, Carmencita testified that Lot 2111 of Cad-455 had been in the possession of
their parents since 1940 and that after the death of their parents she and her siblings
immediately took possession of it and religiously paid the taxes thereon. The land is being
cultivated by Julia Garal, their tenant. She admitted that no improvements have been
introduced by their family on the lot. On cross examination, she admitted that plans to sell the
property were at hand.
[9]

In its decision of 1 September 1998, the trial court found that the respondents have
sufficiently established their familys actual, continuous, adverse, and notorious possession of
the subject property for more than fifty-seven years, commencing from the possession of their
predecessors-in-interest in 1940, and that such possession was in an adverse and public
manner. Likewise, it found that the land in question is alienable and disposable and is not within
any reservation or forest zone. Thus, it confirmed the title of the respondents over the said lots;
directed the Register of Deeds of Laguna, Calamba Branch, to cause the registration of said
parcels of land in the name of the respondents upon payment of fees; and ordered the issuance
of a Decree of Registration once the decision becomes final and executory.
Upon appeal
[10]
by the petitioner, the Court of Appeals affirmed the decision of the trial
court. Hence, this petition.
The OSG argues that both the trial court and the Court of Appeals erred in (a) giving weight
to the self-serving testimonies of Mauricio and Carmencita that the respondents and their
predecessors-in-interest had been in open, continuous, and adverse possession of the lots in
question in the concept of an owner for at least thirty years; and (b) holding that respondents
tax declaration is sufficient proof that they and their parents have been in possession of the
property for at least thirty years, despite the fact that the said tax declaration was only for the
year 1994 and the property tax receipts presented by the respondents were all of recent dates,
i.e., 1990, 1991,1992, 1994, 1996, and 1997. Finally, the OSG states that even granting for the
sake of argument that the respondents have been in possession of the property since 1940,
their adverse possession should be reckoned only from 28 September 1981 when the property
was declared to be within alienable and disposable zone.
The petition is meritorious.
While the rule is well settled that the findings of fact of appellate courts are conclusive upon
us,
[11]
there are recognized exceptions thereto, among which is where the findings of fact are not
supported by the record or are so glaringly erroneous as to constitute a serious abuse of
discretion.
[12]
This exception is present in this case.
Section 48(b) of C.A. No. 141, as amended by Republic Act No. 1942,
[13]
reads as follows:
Section 48. The following described citizens of the Philippines, occupying lands of the public domain or
claiming to own any such lands or an interest therein, but whose titles have not been perfected or
completed, may apply to the Court of First Instance of the province where the land is located for
confirmation of their claims and the issuance of a certificate of title therefor, under the Land Registration
Act, to wit:

(b) Those who by themselves or through their predecessors-in-interest have been in open, continuous,
exclusive, and notorious possession and occupation of agricultural lands of the public domain, under
a bona fide claim of acquisition of ownership, for at least thirty years immediately preceding the filing of
the application for confirmation of title except when prevented by war orforce majeure. These shall be
conclusively presumed to have performed all the conditions essential to a Government grant and shall be
entitled to a certificate of title under the provisions of this chapter.
This provision was further amended by P.D. No. 1073
[14]
by substituting the phrase for at least
thirty years with since June 12, 1945; thus:
SEC. 4. The provisions of Section 48(b) and Section 48(c), Chapter VIII, of the Public Land Act are
hereby amended in the sense that these provisions shall apply only to alienable and disposable lands of
the public domain which have been in open, continuous, exclusive and notorious possession and
occupation by the applicant himself or through his predecessor-in-interest, under a bona fide claim of
acquisition of ownership, since June 12, 1945.
The date 12 June 1945 was reiterated in Section 14(1) of P. D. No. 1529,
[15]
otherwise known
as the Property Registration Decree, provides:
SEC. 14. Who may apply. The following persons may file in the proper Court of First Instance [now
Regional Trial Court] an application for registration of title to land, whether personally or through
their duly authorized representatives:
(1) Those who by themselves or through their predecessors-in-interest have been in
open, continuous, exclusive and notorious possession and occupation of alienable
and disposable lands of the public domain under a bona fide claim of
ownership since June 12, 1945, or earlier. (Emphasis supplied).
Applicants for confirmation of imperfect title must, therefore, prove the following: (a) that the
land forms part of the disposable and alienable agricultural lands of the public domain; and (b)
that they have been in open, continuous, exclusive, and notorious possession and occupation of
the same under a bona fide claim of ownership either since time immemorial or since 12 June
1945.
There is no doubt that the subject property is part of the disposable and alienable
agricultural lands of the public domain. But it is not clear as to when it was classified as
alienable and disposable by proper authorities.
We do not find merit in OSGs claim that the subject property was classified as within the
alienable and disposable zone only on 28 September 1981, and hence, possession by
respondents predecessors-in-interest before that date cannot be considered. In support of this
claim, the OSG relies on a statement appearing in the survey plan marked as Exhibit Q, which
reads:
This survey is inside alienable and disposable area as per Project No. 23-A L.C. Map No. 004 certified
on September 28, 1981 and is outside any civil or military reservation.
As postulated by the respondents, the phrase certified on September 28, 1981 could not have
meant that Lot 2111 became alienable and disposable only on 28 September 1981. That date
obviously refers to the time that Project No. 23-A L.C. Map No. 004 was certified.
Neither can we give weight to the contention of the respondents that since Project No. 23-A
L.C. Map No. 004 of which Lot 2111 forms part was approved on 31 December 1925 by the
then Bureau of Forestry, Lot 2111 must have been disposable and alienable as early as of that
date. There is nothing to support their claim that31 December 1925 is the date of the approval
of such project or the date of the classification of the subject property as disposable and
alienable public land. It is settled that a person who seeks registration of title to a piece of land
must prove his claim by clear and convincing evidence.
[16]
The respondents have failed to
discharge the burden of showing that Lot 2111 was classified as part of the disposable and
alienable agricultural lands of public domain as of 12 June 1945 or earlier.
Likewise, the respondent have miserably failed to prove that they and their predecessors-
in-interest have been in open, continuous, exclusive, and notorious possession and occupation
of the subject property under a bona fide claim of ownership either since time immemorial or
since 12 June 1945.
The trial court and the Court of Appeals based the finding of fifty-seven years of possession
by the respondents and their predecessors-in-interest on the testimonies of Carmencita and
Mauricio. The two were aged 62
[17]
and 60,
[18]
respectively, when they testified in 1997. Thus,
they must have been born in 1935 and 1937, respectively. If the asserted possession lasted for
a period of fifty-seven years at the time they testified, the same must have commenced
sometime in 1940, or at the time that Carmencita was just 5 years old and Mauricio, about 3
years old. It is quite impossible that they could fully grasp, before coming to the age of reason,
the concept of possession of such a big tract of land and testify thereon nearly six decades
later. In short their testimonies could not be relied upon to prove the adverse possession of the
subject parcel of land by their parents.
In any case, respondents bare assertions of possession and occupation by their
predecessors-in-interest since 1940 (as testified to by Carmencita
[19]
) or since 1949 (as testified
to by Mauricio
[20]
and declared in respondents application for registration) are hardly the well-
nigh incontrovertible evidence required in cases of this nature. Proof of specific acts of
ownership must be presented to substantiate their claim. They cannot just offer general
statements which are mere conclusions of law than factual evidence of possession.
[21]
Even
granting that the possession by the respondents parents commenced in 1940, still they failed to
prove that their predecessors-in-interest had been in open, continuous, exclusive, and notorious
possession and occupation of the subject land under a bona fide claim of acquisition of
ownership.
The law speaks of possession and occupation. Since these words are separated by the
conjunction and, the clear intention of the law is not to make one synonymous with the
other. Possession is broader than occupation because it includes constructive
possession. When, therefore, the law adds the word occupation, it seeks to delimit the all
encompassing effect of constructive possession. Taken together with the words open,
continuous, exclusive and notorious, the word occupationserves to highlight the fact that for an
applicant to qualify, his possession must not be a mere fiction.
[22]
Actual possession of a land
consists in the manifestation of acts of dominion over it of such a nature as a party would
naturally exercise over his own property.
[23]

No evidence on record shows that Spouses Mauricio and Luz Melendez cultivated, had
control over, or used the whole or even a greater portion of the tract of land for agricultural
purposes.
[24]
Moreover, only one tenant worked on the land, and there is no evidence as to how
big was the portion occupied by the tenant. Moreover, there is no competent proof that the
Melendez Spouses declared the land in their name for taxation purposes or paid its
taxes. While tax receipts and declarations are not incontrovertible evidence of ownership, they
constitute, at the least, proof that the holder has a claim of title over the property.
[25]
The
voluntary declaration of a piece of property for taxation purposes not only manifests ones
sincere and honest desire to obtain title to the property, but also announces an adverse claim
against the State and all other interested parties with an intention to contribute needed revenues
to the government. Such an act strengthens ones bona fide claim of acquisition of
ownership.
[26]

The respondents claim that they immediately took possession of the subject land upon the
death of their parents, Mauricio and Luz Melendez, who died on 5 May 1976 and 19 February
1967, respectively, and that they had been religiously paying the taxes thereon. If that were so,
why had they not themselves introduced any improvement on the land?
[27]
We even find
unsubstantiated the claim of Carmencita that they had a tenant on the land. They did not
present any tenant. In any case, we wonder how one tenant could have cultivated such a vast
tract of land with an area of 2.6 hectares.
The records also reveal that the subject property was declared for taxation purposes by the
respondents only for the year 1994. They paid the taxes thereon only for the years 1990, 1991,
1992, 1994, 1996, and 1997. Being of recent dates, we cannot trust the assertion of the
respondents that they immediately took possession of the property in the concept of an owner
after the death of their parents. While belated declaration of a property for taxation purposes
does not necessarily negate the fact of possession,
[28]
tax declarations or realty tax payments of
property are, nevertheless, good indicia of possession in the concept of an owner, for no one in
his right mind would be paying taxes for a property that is not in his actual or, at least,
constructive possession.
[29]

Likewise, it is noteworthy that none of the respondents reside on the subject property.
Carmencita even admitted that plans of selling the property were at hand. Thus, it would be
rational to conclude that this move for registration is just but a camouflage by smart land
speculators who saw in the land applied for expected profits from its existence.
In a nutshell, the respondents did not have in their favor an imperfect title over the land
subject of the application at the time MTC LRC Case No. 06 was filed with the trial court. They
failed to prove that (1) Lot 2111 was classified as part of the disposable and alienable
agricultural lands of public domain as of 12 June 1945 or earlier; (2) they and their
predecessors-in-interest have been in continuous, exclusive, and adverse possession and
occupation thereof in the concept of owners from 12 June 1945 or earlier.
WHEREFORE, the petition is GRANTED, and the decisions of the Court of Appeals of 26
August 2002 in CA-G.R. CV No. 64323 and of the Municipal Trial Court of Cabuyao, Laguna, of
1 September 1998 in MTC LRC Case No. 06 are hereby REVERSED and SET ASIDE. The
land registration case MTC LRC Case No. 06 is hereby ordered DISMISSED.
Costs de oficio.
SO ORDERED.
Panganiban, Ynares-Santiago, Carpio, and Azcuna, JJ., concur.



[1]
Director of Lands v. IAC, G.R. No. 68946, 22 May 1992, 209 SCRA 214, 231.
[2]
Per Associate Justice Perlita J. Tria Tirona concurred in by Associate Justices Buenaventura
J. Guerrero and Rodrigo V. Cosico. Rollo, 11-16.
[3]
Per Judge Zenaida Lubriga Galvez. Rollo, 125-127.
[4]
In its capacity as Cadastral and Land Registration Court by virtue of Section 34 of the Batas
Pambansa Blg. 129, as amended by R.A. No. 7691, entitled An Act Expanding the
Jurisdiction of the Metropolitan Trial Courts, Municipal Trial Courts, and Municipal Circuit
Trial Courts, Amending for the Purpose Batas Pambansa Blg. 129, Otherwise Known as
the Judiciary Reorganization Act of 1980, which became effective on 15 April 1994 as
declared by SC Administrative Circular No. 09-94.
[5]
Rollo, 58-64.

[7]
Rollo, 97-98.
[8]
TSN, 17 July 1997, 5-10; Rollo, 105-110.
[9]
TSN, 20 November 1997, 2-7; Rollo, 115-120.
[10]
By virtue of Section 34 of B.P. Blg. 129, as amended by R.A. No. 7691, which provides that
decisions of Metropolitan Trial Courts, Municipal Trial Courts, and Municipal Circuit Trial
Courts in their capacity as cadastral and land registration courts are appealable in the
same manner as decisions of the Regional Trial Courts.
[11]
Chan v. Court of Appeals, No. L- 27488, 30 June 1970, 33 SCRA 737, 738; Director of
Lands v. IAC, supra note 1, at 221.
[12]
Samson v. Court of Appeals, No. L-40771, 29 January 1986, 141 SCRA 194,
199; Municipality of Meycauayan, Bulacan v. IAC, No. L- 72126, 29 January 1988, 157
SCRA 640, 642-643.
[13]
Effective on 22 June 1957.
[14]
Effective on 25 January 1977.
[15]
Effective on 11 June 1978.
[16]
Republic v. IAC, No. L-66069, 28 September 1984, 132 SCRA 395, 397; Edao v. Court of
Appeals, G.R. No. 83995, 4 September 1992, 213 SCRA 585, 592.
[17]
TSN, 20 November 1997, 1;Rollo, 114.
[18]
TSN, 17 July 1997, 1; Rollo,100.
[19]
TSN, 20 November 1997, 2; Rollo, 115.
[20]
TSN, 17 July 1997, 7; Rollo, 107.
[21]
Republic v. Court of Appeals, No. L-62680, 9 November 1988, 167 SCRA 150, 156.
[22]
Director of Lands v. IAC, supra note 1, at 222-223.
[23]
Ramos v. Director of Lands, 39 Phil. 175 (1918); Republic v. Court of Appeals, G.R. Nos.
115747 and 116658, 20 November 2000, 345 SCRA 104, 115.
[24]
Director of Lands v. Buyco, G.R. No. 91189, 27 November 1992, 216 SCRA 78, 96.
[25]
Director of Lands v. Reyes, No. L-27594, 28 November 1975, 68 SCRA 177, 194.
[26]
Director of Lands v. IAC, supra note 1, at 227.
[27]
Republic v. Court of Appeals, supra note 21, at 157.
[28]
Republic v. Court of Appeals, G.R. No. 116372, 18 January 2001, 349 SCRA 451, 462.
[29]
Republic v. Court of Appeals, G.R. No. 108926, 12 July 1996, 258 SCRA 712, 720.
SECOND DIVISION
[G.R. No. 117734. February 22, 2001]
VICENTE G. DIVINA, petitioner, vs. HON. COURT OF APPEALS and VILMA GAJO-
SY, respondents.
D E C I S I O N
QUISUMBING, J .:
Before us is a petition for review of the decision
[1]
dated October 27, 1994 of the Court of Appeals in
CA-GR CV No. 03068 reversing and setting aside the judgment dated July 7, 1979 of the Court of First
Instance of Sorsogon, Branch II, in LRC Case No. N-147.
The facts of this case are as follows:
Lot No. 1893 located at Gubat, Sorsogon, was originally owned by Antonio Berosa. On July 22,
1960, he sold it to Teotimo Berosa. The portion is particularly described as:
A parcel of land unirrigated situated in San Ignacio, Gubat, Sorsogon, Philippines, with an area of
TWENTY THOUSAND (20,000) square meters and bounded on the North by Lot #1464 - Fausto Ayson
and Lot #1888 - Gloria Fajardo: on the East, by Lot # 1446 - Silverio Garcia: on the South, by Lot #1891
- Antonio Escobedo and on the West, by Lot #1880 - Federico Faronas and Lot #1890 - Eugenia
Espedido. Cadastral concrete posts are the visible signs of boundary. It has no permanent improvement
thereon. Designated as Lot 1893 of Antonio Berosa. Declared under Tax No. 13038, valued at P760.00
for the current year in the name of ANTONIO BEROSA
[2]

On March 23, 1961, the Berosa spouses sold the same Lot 1893 to Jose P. Gamos. In the deed of
sale to Gamos, the lot was more particularly described as:
A parcel of RICE land situated in San Ignacio, Gubat, Sorsogon, Philippines, with an area of TWENTY
THOUSAND (20,000) square meters and bounded on the North, by Lot #1462 - Fausto Ayson and Lot
#1888 - Gloria F. Estonante: on the East, by Lot #1464 Zacarias Espadilla; and Lot #1466 - Felix
Arimado; on the South, by Lot #1898 - Silverio Garcia; and on the West, by Lot #1890 - Eugenia
Espedido and Lot #1892 - Antonio Escobedo. Concrete cements posts are the visible signs of
boundary. No permanent improvements thereon. Covered by Lot #1893 of Teotimo E. Berosa,
and declared under Tax No. 13039, valued at P760.00 for the present year in the name of TEOTIMO E.
BEROSA.
[3]

On April 26, 1960, Gamos acquired from the heirs of Felix Arimado, a boundary owner of Lot 1893,
a 20,687 sq. m. parcel of land identified as Lot 1466, also in Gubat. It adjoins Lot 1893. On March 28,
1961, Gamos had these two parcels of land under Tax Declaration No. 13237 and declared it had a total
area of 4.0867 hectares. He also had the property resurveyed by private land surveyor Antonio
Tiotangco. In 1967, Tax Declaration No. 13237 was cancelled by Tax Declaration No. 9032 in Gamos
name.
The re-survey plan (AP-9021), of Lots 1466 and 1893 conducted on June 16, 1961 for Gamos,
showed that the consolidated properties contained a total area of 100,034 sq. m.This plan was approved
on July 12, 1961 by the Acting Director of Lands.
On November 23, 1968, Tax Declaration No. 12927 which cancelled Tax Declaration No. 9032 was
secured by Gamos and declared therein that the area of the consolidated property was 10.0034 hectares
with 2500 sq. m. planted to coconut, 3.8187 irrigated for rice planting and 5.9347 were thickets.
On January 19, 1967, Teotimo Berosa conveyed to Vicente G. Divina, herein petitioner, a portion of
Lot 1893 referred to as Lot 1893-B. It is described as follows:
A parcel of dry and thicket land situated in San Ignacio, Gubat, Sorsogon, Philippines, having an area of
54,818 square meters and bounded on the N., by Lot 1888 (Inocencio Eroe); Lot 1887 (Jaime Enaje); and
Lot 1462 (Heirs of Zacarias Espadilla); on the (Illegible), by Lot 1466; on the SE, by Lot 1893-A; on the
S., by Lot 1898 (Heirs of Silverio Garcia); on the E., by Lot 1892 (Antonio Escobedo) and Lot 1890
(Eugenia Espedido); and on the NE, by Lot 1889 (Pedro Fajardo); all of Gubat Cadastre. NOTE: This lot
is designed as Lot 1893-B, a portion of Lot 1893, Cad 308-D.
[4]

On November 28, 1968, two years from the date of said sale and five (5) days after November 23,
1968, when Gamos secured Tax Declaration No. 12927 declaring the consolidated property as containing
100,034 sq. m., the deed of sale was registered. An undated Subdivision PLAN of Lot 1893, was
prepared for petitioner. The plan, without Bureau of Lands approval, showed that Lot 1893 was divided
into two, Lot 1893-A and Lot 1893-B.
On July 24, 1970 Gamos sold the consolidated property to private respondent Vilma Gajo-Sy, for
P20,000.00. The land was particularly described as follows:
A parcel of land located at San Ignacio, Gubat, Sorsogon, under Tax Declaration No. 12927 in the name
of Jose P. Gamos, covered by Lots No. 1466 and 1893 of the Gubat Cadastre, with an area of 100,034 sq.
m., more or less.
[5]

On July 29, 1970, Tax Declaration No. 13768 secured by private respondent, was cancelled by Tax
Declaration No. 12509.
On August 28, 1972, she filed an application for registration of title to the property at the then Court
of First Instance of Sorsogon, docketed as LRC Case No. N-147, GLRO Record No. 42920. The
application was amended on March 8, 1973, on order of Branch II of the said court to include therein the
postal address of Inocencio Erpe, adjoining owner of Lot No. 1893 described in Plan AP-9021.
The land registration court, by Decision of July 29, 1975, ordered the registration of private
respondents title over Lots Nos. 1466 and 1893.
On July 13, 1977, pending issuance of the final decree of registration petitioner filed before the same
court a Petition for Review of the July 29, 1975 judgment. He alleged that he is the owner of a portion of
Lot 1893 consisting of 54,818 sq. m. conveyed to him by Teotimo Berosa on January 19, 1967; that he
was unaware of the registration proceedings on Lot 1893 due to private respondents failure to give him
notice and post any notice in the subject lot; and that private respondent fraudulently misrepresented
herself as the owner of the disputed portion despite her knowledge that another person had acquired the
same.
Private respondent opposed the petition alleging that the registration case had long become final and
the court no longer had any jurisdiction thereon; and that lack of personal notice to the petitioner of the
registration proceedings did not constitute actual fraud.
The trial court, in its Decision
[6]
dated June 7, 1979, found that the petition for review was timely
filed. It also ruled that the failure of private respondent to include a known claimant in her application for
registration constituted deliberate misrepresentation that the lot subject of her application is not
contested when in fact it was. Private respondent, according to the trial court, should have included in her
application at least the person of petitioners cousin, Elena Domalaon who had, before respondent filed
her application for registration, made known to the latters sister her apprehension of their land being
included in respondents application for registration. This misrepresentation, according to the court,
amounted to fraud within the contemplation of Section 38 of Act 496.
[7]
The trial court in its decision
disposed as follows:
WHEREFORE, judgment is hereby rendered:
(a) Setting aside the Decision rendered in the land registration case and revoking the order for
the issuance of a Decree;
(b) Declaring petitioner Vicente G. Divina the owner of the portion of the land applied for
containing an area of 54,818 square meters which is described in paragraph 3 of the Petition
for Review of Judgment; and
(c) Allowing this land registration case to proceed as to the portion applied for which is outside
the limits of the portion herein awarded to the petitioner Vicente G. Divina; and
(d) Ordering a subdivision survey of the lots applied for, delimiting therein the area not
contested and which is registrable in favor of applicant Vilma Gajo-Sy, and the area herein
adjudicated to petitioner Vicente G. Divina, as to whom the land registration proceedings
shall likewise be allowed to proceed after he shall have adduced such other evidence as are
appropriate in land registration cases.
SO ORDERED.
[8]

Private respondent assailed the decision of the trial court before the Court of Appeals. It averred that
the trial court erred (1) in declaring petitioner-appellee owner of a portion of Lot 1893, in ordering a
subdivision survey, and allowing petitioner-appellee to proceed with registration after adducing evidence
as are appropriate; (2) in declaring respondent-appellant guilty of actual fraud in the land registration
case; (3) in taking cognizance of the petition for review of judgment, setting aside the decision dated June
29, 1975, and revoking the order of the issuance of the final decree in the land registration case; and (4) in
not dismissing the petition for review of judgment with cost.
[9]

The CA reversed the trial court and dismissed the petition. It ruled:
In the case at bar, petitioner-appellee did not indeed appear in the survey plan as an adjoining owner of
the subject property. Neither was he a known claimant or possessor of the questioned portion of Lot 1893
which was found by the court a quo to be untouched and thickly planted with bigaho. A fortiori, there
was no need to mention in the application for registration the apprehension or claim of at least
petitioner-appellees cousin Evelyn (sic) Domalaon in the application for registration, nor to personally
notify Elena about registration proceeding.
There could, therefore, have been no misrepresentation in any form on the part of respondent-appellee.
x x x
There being no extrinsic or collateral fraud attendant to the registration of the property in the name of
respondent-appellee, We find it unnecessary to discuss the rest of the assigned errors. Suffice it to state
that Lot 1893 bought by Teotimo Berosa which he sold to Jose P. Gamos who in turn sold it to
respondent-appellee in 1970 was designated by boundaries in such a manner as to put its identity beyond
doubt; that the total area of Lot 1893 lot was determined after a resurvey/relocation was conducted for
Gamos in 1961 the result of which is reflected in the Plan approved by the Bureau of Lands also in 1961;
that what really defines a piece of land is not the area but the boundaries therein laid down (Pea,
Registration of Land Titles and Deeds, 1988 Edition, p. 213); that the Lot 1893-B sold to petitioner-
appellee made no mention of any tax declaration covering it, unlike the different deeds of sale covering
Lot 1893, thereby raising the presumption that Lot 1893-B was really part of the Lot 1893 sold by
Gamos to respondent-appellant; and that the subdivision of Lot 1893 into Lots A and B, caused to be
made by petitioner-appellee who claims Lot 1893-B to have been the Lot 1893-B subject of the January
19, 1967 Deed of Sale in his favor which was registered on November 28, 1968, appears to have been
made in 1977, ten years from the date of said deed of sale, a confirmation that there was no such Lot
1893-B subject of his purchase in 1967; and in any event, as the subdivision survey prepared for
petitioner-appellee was not approved by the Bureau of Lands, it is not of much value (vide Flores vs.
Director of Lands, 17 Phil. 512 [1910]).
In fine, not all the basic elements for the allowance of the reopening or review of the judgment rendered
in the land registration case in respondent-appellants favor are present. The present appeal is thus
meritorious.
WHEREFORE, the assailed judgment is hereby REVERSED and set aside and another rendered
DISMISSING petitioner-appellees petition at the court a quo.
[10]

Hence, the present petition. Petitioner now assails the reversal of the Court of Appeals of the trial
court decision. In substance, he raises the primary issue of whether or not, there was deliberate
misrepresentation constituting actual fraud on private respondents part when she failed to give or post
notice to petitioner of her application for registration of the contested land, such that it was error for the
trial court to declare private respondent owner of the disputed land.
Prefatorily, on the timeliness of the petition for review of judgment, we have repeatedly said that the
adjudication in a registration of a cadastral case does not become final and incontrovertible until the
expiration of one year after the entry of then final decree. As long as the final decree is not issued, and
the one year within which it may be revised had not elapsed, the decision remains under the control and
sound discretion of the court rendering the decree, which court after hearing may set aside the decision or
decree or adjudicate the land to another party.
[11]
In the present case, a certification was issued by the
Land Registration Commission that no final decree of registration had yet been issued and by the order of
the trial court dated September 28, 1977, it restrained the Commission from issuing such a
decree. Clearly, the tolling of the one year period has not even began. Thus, the trial court did not err
when it entertained the petition.
Now, we consider the crux of the petition. Both the trial and appellate courts found that petitioners
name did not appear in the survey plan as an adjacent owner, nor claimant nor possessor. However, the
trial and appellate courts differed in their conclusion on whether or not there was deliberate
misrepresentation constituting fraud in private respondents part when it failed to give notice or post
notice to potential claimant and include their names in the application for registration. The trial court said
there was, but the appellate court disagreed.
Section 15 of P.D. 1529
[12]
is explicit in requiring that in the application for registration of land
titles, the application shall also state the full names and addresses of all occupants of the land and those
of the adjoining owners if known, and if not known, it shall state the extent of the search made to find
them. As early as Francisco vs. Court of Appeals, 97 SCRA 22 [1980] we emphasized that a mere
statement of the lack of knowledge of the names of the occupants and adjoining owners is not sufficient
but what search has been made to find them is necessary. The trial court was correct when it took notice
that respondents sister Lydia Gajo-Anonuevo admitted that she had a conversation with petitioners
cousin Elena Dumalaon about the latters apprehension that their land may have been included in
respondents application for registration of the disputed land.
[13]
Respondents omission of this material
information prevented petitioner from having his day in court. The trial court in its decision more than
amply supported its conclusion with jurisprudence to the effect that it is fraud to knowingly omit or
conceal a fact upon which benefit is obtained to the prejudice of a third person.
[14]
Such omission can not
but be deliberate misrepresentation constituting fraud, a basis for allowing a petition for review of
judgment under Section 38 of Act No. 496, The Land Registration Act.
Additionally, it should be noted that petitioner acquired the bigger portion of Lot 1893 long after the
initial survey of Barrio San Ignacio. Teotimo Berosa sold Lot 1893 to Jose P. Gamos who in turn sold it
to respondent in 1970. Clearly, going by the records, petitioners name would not be found on the said
survey plan approved by the Bureau of Lands in 1961, years before his purchase of the portion of Lot
1893. Petitioners claim is clearly meritorious.
WHEREFORE, the petition is GRANTED. The assailed decision of the Court of Appeals dated
October 24, 1994 is REVERSED and SET ASIDE. The judgment in LRC Case No. N-147 of the then
Court of First Instance, Branch II in Gubat, Sorsogon is REINSTATED. Costs against private
respondent.
SO ORDERED.
Bellosillo (Chairman), Mendoza, Buena, and De Leon, Jr., JJ., concur.



[1]
Rollo, pp. 21-32.
[2]
Id. at 21-22; Records, p. 17.
[3]
Id. at 22 only.
[4]
Id. at 23-24.
[5]
Id. at 24 only.
[6]
Id. at 36-44.
[7]
Sec. 38. If the Court after hearing finds that the applicant or adverse claimant has title as stated in his
application or adverse claim and proper for registration, a decree of confirmation and registration shall be
entered. Every decree of registration shall bind the land, and quiet title thereto, subject only to the
exceptions stated in the following section. It shall be conclusive upon and against all persons, including
the Insular Government and all the branches thereof, whether mentioned by name in the application,
notice, or citation, or included in the general description To all whom it may concern. Such decree
shall not be opened by reason of the absence, infancy, or other disability of any person affected thereby,
nor by any proceeding in any court for reversing judgments or decrees; subject, however, to the right of
any person deprived of land or of any estate or interest therein by decree of registration obtained
by fraud to file in the competent Court of First Instance a petition for review within one year after
the entry of the decree,provided no innocent purchaser for value has acquired an interest. Upon the
expiration of said term of one year, every decree or certificate of title issued in accordance with this
section shall be incontrovertible. If there is any such purchaser, the decree of registration shall not be
opened, but shall remain in full force and effect forever, subject only to the right of appeal herein before
provided: Provided, however, That no decree or certificate of title issued to persons not parties to the
appeal shall be cancelled or annulled. But any person aggrieved by such decree in any case may pursue
his remedy by action for damages against the applicant or any other person for fraud in procuring the
decree. Whenever the phrase innocent purchaser for value or an equivalent phrase occurs in this Act, it
shall be deemed to include an innocent lessee, mortgagee, or other encumbrancer for value. (As amended
by Section 3, Act 3621; and Sec. 1, Act No. 3630). [Emphasis ours.]
[8]
Rollo, p. 44.
[9]
Id. at 12.
[10]
Rollo, pp. 29-32.
[11]
Laburada vs. Land Registration Authority, 287 SCRA 333, 341 (1998) citing Gomez vs. Court of
Appeals, 168 SCRA 503 (1988); Republic vs. Associacion Benevola de Cebu, 178 SCRA 692, 699 (1989)
citing Afalla and Pinaroc vs. Rosauro, 60 Phil 622 (1934); Capio vs. Capio, 94 Phil 113 (1953).
[12]
SEC. 15. Form and contents.The application for land registration shall be in writing, signed by the
applicant or the person duly authorized in his behalf, and sworn to before any officer authorized to
administer oaths for the province or city where the application was actually signed. If there is more than
one applicant, the application shall be signed and sworn to by and in behalf of each. The application shall
contain a description of the land shall state the citizenship and civil status of the applicant, whether single
or married, and, if married, the name of the wife or husband, and, if the marriage has been legally
dissolved, when and how the marriage relation terminated. It shall also state the full names and
addresses of all occupants of the land and those of the adjoining owners, if known, and if not known, it
shall state the extent of the search made to find them.
[13]
Rollo, pp. 40-41.
[14]
Varela vs. Villanueva, 95 Phil 248 (1954); Labayen vs. Talisay-Silay Milling Co., 68 Phil 376 (1939);
and Palanca vs. American Food Manufacturing Co., 24 SCRA 819, 827 (1968).
Republic of the Philippines
SUPREME COURT
Manila
SECOND DIVISION
G.R. No. 139588 March 4, 2004
REPUBLIC OF THE PHILIPPINES, represented by the DIRECTOR OF LANDS, petitioner,
vs.
JOSEFINA B. VDA. DE NERI, SPS. GRACIANO B. NERI, JR. and VICTORIA BABIERA,
SPS. VICTORIA NERI and MARIO FERNANDEZ, RAMON NERI, SPS. TERESA NERI and
ALBERTO YRASTORZA and the REGISTER OF DEEDS OF CAGAYAN DE ORO
CITY, respondents.
D E C I S I O N
CALLEJO, SR., J .:
Before the Court is the petition for review on certiorari filed by the Republic of the Philippines,
represented by the Director of the Bureau of Lands, seeking to reverse and set aside the
Decision
1
of the Court of Appeals (CA) in CA-G.R. CV No. 50139 affirming the decision of the
Regional Trial Court of Misamis Oriental, Branch 20, Cagayan de Oro City, which, in turn,
dismissed the petitioners complaint for the annulment of Original Certificate of Title (OCT) No.
0662 and reversion. Likewise sought to be reversed and set aside is the appellate courts
Resolution dated August 4, 1999, denying the petitioners motion for reconsideration.
The antecedent facts are summarized as follows:
Lot 2821, Plan (LRC) SWO-150, approved by the Land Registration Commission, is a parcel of
land with an area of 105.568 hectares located along the Cagayan de Oro River in Sitio
Taguanao, Indahag, Cagayan de Oro City. On September 3, 1973, the Bureau of Forest
Development certified that the property was alienable and disposable.
2
On July 24, 1974, the
heirs of Graciano Neri, Sr.
3
filed an application with the then Court of First Instance of Misamis
Oriental for judicial confirmation of imperfect or incomplete title, docketed as Land Registration
Case No. N-531, LRC Record No. 46236. They alleged, inter alia, thus:
1. That applicants GRACIANO B. NERI, JR. is of legal age, married to VICTORIA
BABIERA, Filipino and residing at 833 Recto Avenue, Cagayan de Oro City; JOSEFINA
B. VDA. [DE] NERI is of legal age, widow of GRACIANO A. NERI, Filipino and residing
at 833 Recto Avenue, Cagayan de Oro City; VICTORIA N. FERNANDEZ is of legal age,
married to MARIO FERNANDEZ, Filipino and residing at 832 Recto Avenue, Cagayan
de Oro City; RAMON B. NERI is of legal age, single, Filipino and residing at 833 Recto
Avenue, Cagayan de Oro City; and MA. TERESA N. YRASTORZA is of legal age,
married to ALBERT YRASTORZA, Filipino and residing at 833-Y Recto Avenue,
Cagayan de Oro City, hereby apply to have the land hereinafter described brought under
the operation of the Land Registration Act, and to have the title thereto registered and
confirmed;
2. That the applicants are the owners in fee simple of a certain parcel of land situated at
Barrio Indahag in the City of Cagayan de Oro, the said land bounded and described on
the plan and technical description, attached hereto and made a part hereof;
3. That said land at the last assessment for taxation was assessed for P33,820.00;
4. That applicants do not know of any mortgage or encumbrances affecting the said
land, or that any other person has any estate or interest therein legal or equitable
possession, remainder, reversion or expectance;
5. That applicants obtained title on said property by virtue of intestate succession from
the late GRACIANO A. NERI who died on December 20, 1971 in the City of Cagayan de
Oro;
6. That applicants herein together with their predecessors-in-interest have been in open,
public, peaceful, adverse, and continuous possession in the concept of owners and have
been paying taxes thereon;
7. That the said land is legally occupied by the following persons together with their
families in the concept of Tenants, namely:
a) PEDRO CABALUNA residing at Indahag, Tibasak, Cagayan de Oro City
b) CERELO BADETAH residing at Indahag, Tibasak, Cagayan de Oro City
8. That the names and addresses so far as known to applicants of the owners of all land
adjoining the land above-mentioned are as follows:
a) JOSE F. MARFORI of Cagayan de Oro City
b) INSULAR GOVERNMENT of the PHILIPPINES
c) CAGAYAN DE ORO CITY (Cagayan River)
9. This application is accompanied by the following documents:
a) Tracing plan of the lot together with a blue print copy thereof;
b) Technical description of the land;
c) Tax Declarations:
1. T.D. No. 1096
2. T.D. No. 22280
3. T.D. No. 10964
4. T.D. No. 81439
10. That by the application of Sections: 47, 48, 49, 50, and 51 of Com. Act 141 as
amended by Rep. Acts Nos. 107, 1942, 2061 and 2036, this application is in order and
the jurisdiction of this Honorable Court exist and can be exercised in connection with this
instant application.
4

The applicants prayed that, after due proceedings, judgment be rendered in their favor in this
wise:
WHEREFORE, premises considered, it is most respectfully prayed that after due notice, the
validity of the alleged title or claim be inquired into and after due hearing an order be issued
directing the Land Registration Commission to issue the corresponding decree over the said
parcel of land so that a Certificate of Title be issued in favor of the applicants under the
provisions of the Land Registration Act.
Prays for other relief in the premises.
5

The applicants thereafter filed an amended application in the same case on December 17,
1974. On January 27, 1975, the Court, through the Land Registration Commission, issued a
notice of initial hearing addressed, among others, to the Director of the Bureau of Lands, the
Solicitor General, and the Bureau of Forest Development.
6
The notice was published in the
February 17 and 24, 1975 issues of the Official Gazette. Copies thereof were sent by ordinary
mail to the persons named therein; the copies intended for the Director of the Bureau of Lands
and the Office of the Solicitor General were sent by special messenger.
7
The Solicitor General
and the Director of the Bureau of Lands failed to file any opposition thereto.
On the designated time and date of the hearing, no representative from the Office of the
Solicitor General and the Bureau of Lands appeared in court. The court granted the motion of
the applicants for an order of general default "against the whole world," except those who filed
their opposition or appeared during the hearing. The court thereafter issued an order allowing
the applicants to adduce evidence ex parte.
On February 5, 1976, the court rendered judgment granting the application. The Office of the
Solicitor General, as well as the Director of the Bureau of Lands, failed to appeal the same.
Thus, the court issued Decree No. N-361749, on the basis of which OCT No. 0662 was issued
by the Register of Deeds of Cagayan de Oro City on September 26, 1976.
On January 5, 1981, the Office of the Solicitor General, for and in behalf of the petitioner
Republic of the Philippines, through the Director of the Bureau of Lands, filed with the court a
quo a complaint for annulment of OCT No. 0662 and the reversion of the property covered by
the said title against herein respondents.
8
The case was docketed as Civil Case No. 7514.
In its complaint, the petitioner alleged inter alia that it is the true owner of a parcel of land of the
public domain surveyed as Lot No. 2821 (subject lot) containing an area of 1,055,684 square
meters or 105.5684 hectares situated in Cagayan de Oro City.
9
The petitioner also alleged that
the Bureau of Forest and Development had classified the subject lot as alienable and
disposable; as such, it was under the direct executive control, administration and disposition of
the Director of the Bureau of Lands. Despite the fact that the Solicitor General and the Director
of the Bureau of Lands were not served copies of the respondents application for judicial
confirmation of imperfect title in LRC Case No. N-531, in violation of Section 51 of
Commonwealth Act No. 141, the said case was set for initial hearing on June 18, 1975. The
petitioner also averred that although the survey plan of the subject lot, Plan (LRC) SWO-150,
was processed and approved by the Land Registration Commission, it was not submitted to the
Director of the Bureau of Lands for re-verification and approval as required by Sections 2 and 3
of Presidential Decree No. 239. As such, according to the petitioner, the court failed to acquire
jurisdiction over the property.
The petitioner prayed that after due proceedings, judgment be rendered nullifying the
proceedings in LRC No. N-531, as well as the courts decision therein, for lack of jurisdiction
over the person of the petitioner as well as the property subject matter of the case. According to
the petitioner, this is in view of the courts failure to comply with Section 51 of Commonwealth
Act No. 141, which mandates that a copy of an application for judicial confirmation of imperfect
title should be duly served on the Director of the Bureau of Lands; and the private respondents
failure to comply with Sections 2 and 3 of P.D. No. 239, which requires the plan to be re-verified
and approved by the Director of the Bureau of Lands, in this case, Plan (LRC) SWO-150. The
petitioner prayed OCT No. 0662 issued in favor of the private respondents be declared null and
void.
In their answer, the private respondents averred that the subject lot had been theirs and their
predecessors private and exclusive property for more than fifty years, and that OCT No. 0662
covering the same was issued in their favor on September 20, 1976. The private respondents
contended that the duty to comply with the requirement under Section 51 of CA No. 141, that
the Solicitor General through the Director of the Bureau of Lands be served a copy of the
application for the judicial confirmation of imperfect or incomplete title, devolved upon the clerk
of court of the land registration court. They also averred that a report on the pre-verification and
approval of Plan (LRC) SWO-150 had been forwarded by the Regional Director of the Bureau of
Lands to the Director of the Bureau of Lands. Moreover, Plan (LRC) SWO-150 covering the
subject lot had been duly processed and approved by the Land Registration Commission.
According to the respondents, even if there were deficiencies on the part of the administrative
officials in complying with the procedures relative to land registration, the same was not
jurisdictional, but merely a procedural flaw. As such, the failure of the Bureau of Lands and the
Land Registration Commission to comply with the law did not result in nullifying the proceedings
in LRC Case No. N-531. Finally, the respondents countered, the action of the petitioner had
long since prescribed.
In its reply, the petitioner alleged that Plan (LRC) SWO-150 was never submitted, much less
approved, to the Director of the Bureau of Lands. The report required by Section 2 of P.D. No.
239 is one which the Director of the Bureau of Lands actually submitted to the court. The
petitioner alleged that no such report was submitted in this case.
On October 16, 1976, the private respondents filed a motion in LRC Case No. N-531 for the
issuance of a writ of possession and the demolition of the houses of the occupants. The court
granted the motion. The Director of the Bureau of Lands, likewise, prayed for the suspension of
the enforcement of the writ pending final resolution of Civil Case No. 7514. The court refused to
suspend the enforcement of its decision.
On November 19, 1981, the Republic of the Philippines, through the Director of the Bureau of
Lands, filed a petition for certiorari and prohibition with this Court, docketed as G.R. No. 58823,
with the following prayer:
1. Declaring as null and void the decision rendered by the court in Land Registration
Case as plan (LRC) Swo-150, Cagayan de Oro City, in favor of the private defendants;
2. Declaring as null and void the corresponding Decree No. N-361749 and Original
Certificate of Title No. 0662 issued by the same court in favor of the private defendants;
3. Ordering the Register of Deeds of Cagayan de Oro City to cancel Original Certificate
of Title No. 0662 issued in the name of the private defendants;
4. Ordering the reversion of the land covered by Original Certificate of Title No. 0662 to
the State and declaring the same as owned and belonging to the latter; and
5. Awarding such further reliefs and remedies as may be just and equitable in the
premises.
10

On March 18, 1985, this Court rendered a decision dismissing the petition without prejudice to
the outcome of the petitioners action against the private respondents in Civil Case No. 7514.
The decision of the Court became final and executory.
On July 17, 1985, Leonel Valdehuesa and 22 others filed a motion for leave to intervene, and
alleged, as follows: (a) They were members of the Cagayan de Oro Green Revolution
Movement Association (COGREMA); (b) They had been occupying the property even before
1969 and commenced working extensive improvements thereon in 1969; (c) They filed a
petition with the Presidential Action Committee on Land Problems (PACLAP) in 1973, and
sought the subdivision of the property and the distribution of the lots to the occupants thereof;
and, (d) As occupants, they were never informed of Graciano Neris application in LRC Case
No. N-531. The intervenors prayed that OCT No. 0662 be nullified.
In an Order dated September 6, 1985, the court a quo dismissed the complaint and the
complaint-in-intervention for lack of jurisdiction to annul the judgment of the CFI in LRC No. N-
531, in view of the promulgation of Batas Pambansa Blg. 129. However, on petition for certiorari
filed by the petitioner, docketed as G.R. No. 72218, this Court, in its Resolution dated July 21,
1986, set aside the said order and directed the court a quo to proceed with the hearing of the
case.
11

In the meantime, the court issued an Order on April 6, 1988 denying the said motion for leave to
intervene. The court later denied the motion for the reconsideration of the said order. Thereafter,
Undersecretary and Officer-in-Charge of the Bureau of Lands Rolleo Ignacio executed a Special
Power of Attorney authorizing Atty. Vicente Seria of the Office of the Regional Director of the
Bureau of Lands to represent the petitioner during the pre-trial.
12

During the hearing of March 27, 1989, the parties agreed to forego a full-blown trial and to
instead file their respective "Memorandum of Authority" and to submit evidence in support of
their respective contentions. The court issued an order on the said date, giving the parties thirty
days to submit their respective memoranda and evidence.
The petitioner submitted its memorandum, appending thereto the documents marked
respectively as follows:
1) Exhibit A The Certification made by the Officer-in-Charge of the Regional Directors
Office for Forestry that the property had been certified on September 3, 1973 as
alienable and disposable.
13

2) Exhibit B The letter of the representatives of the Philippine Constabulary Provincial
Commander and those of the Bureau of Lands, Bureau of Forest Development, and
Department of Agrarian Reform to the Provincial Commander that there were 73 bona
fide tillers on the property seeking to have the property subdivided and distributed to
them.
14

3) Exhibit C The Letter dated October 3, 1977 from the Chief, Regional Director of the
Bureau of Lands directing the District Land Officer of Cagayan de Oro to submit a report
within two days from notice thereof on the letter of the occupants requesting for the
subdivision of the property, viz.:
Considering the urgency of the case, you are hereby directed to submit the report within
five (5) days from receipt hereof. In your report you should state whether the copy of the
petition for registration was received by that Office considering that the case has come
into your knowledge and if so, the date of its receipt thereof, and why the same has not
been forwarded to us immediately. You should also secure a certification from the
District Forest Office regarding the date of release of the area from the forest zone, as it
appears that the land was only released on September 13, 1973. This information is
necessary to determine whether the applicant has acquired a registerable title to the
land.
15

4) Exhibit D The Report dated July 15, 1975 of the Chief Surveyor of the Land
Registration Commission and Acting Chief, Division of Original Registration of the Land
Registration Commission to the Court in LRC Case No. N-531. Thus:
That a certain parcel of land described on plan LRC Swo-1507, Lot 2821,
Cagayan Cadastre 237, Case 1, is applied for registration of title in the above-
entitled land registration case;
That upon verification of our Record of Cadastral Lots, Book No. 64, under Cad.
Court Case No. 17, LRC (GLRO) Cadastral Record No. 1561, Cagayan
Cadastre, Province of Misamis Oriental, it was found that said Lot 2821 is subject
of the following annotations, to quote:
"Lot 2821 (129-1) Pte. de Subd. Parte Public Land."
That it is gleaned from the aforequoted annotations that a decision has been
rendered for said Lot 2821 in cadastral proceedings under Cad. Court Case No.
17, LRC (GLRO) Cadastral Record No. 1561, Cagayan Cadastre, Province of
Misamis Oriental, but no decree of registration has been issued for said lot
pursuant to the decision rendered in the aforementioned cadastral case. Copy,
however, of said decision is not among our salvaged records. It likewise appears
in the above annotations that Lot 2821 is pending subdivision and that portion of
the same was declared public land;
That it is further informed that this Commission is not in a position to verify
whether the land described on plan Swo-1507, Lot 2821, Cagayan Cadastre 237,
Case 1, subject of this application for registration is already covered by a patent
previously issued or within the forest zone.
16

5) Exhibit E The application in LRC Case No. N-531.
17

6) Exhibit F Photographs showing the Taguanao District Elementary School which
stood on a portion of the subject property, to prove that more than 300 hundred families
resided on the property and that its occupants had made extensive improvements
thereon.
For their part, the private respondents appended to their memorandum documentary evidence
marked as follows:
Exhibit
No.
Brief Description
1 Tax Declaration No. 10994 covering 1948 Lot
2821-C-1 with an area of 101.56
18

2 to 2-
G
Tax receipts over lots from 1938
19

3 Letter of Commissioner of Land Registration to
the Court dated October 31, 1974 acknowledging
receipts of the duplicate records of the application
and its answer in LRC Case No. N-531
20

4 Notice of Initial Hearing in LRC Case No. N-531
dated January 27, 1975
21

5 Certificate of Publication from Land Registration
Commission dated March 24, 1975
22

6 Letter of Land Registration Commission to
counsel of applicants dated March 12, 1975
23

7 Report of Land Registration Commission
24

8 Letter of the Geodetic Engineer dated July 20,
1973, to the respondents
25

9 Letter of the Administrative Assistant of the
District Lands Office to the Bureau of Lands
through the District Land Office dated June 3,
1975.
On January 31, 1995, the court rendered judgment dismissing the complaint on the ground that
the petitioner failed to prove the factual averments therein. The dispositive portion of the
decision reads:
WHEREFORE, in the light of the above, judgment is hereby rendered in favor of the defendants
and against the Republic of the Philippines:
1. Dismissing the complaint in the above-entitled case and no pronouncement will be
made on the civil aspect herein as the answer did not claim any damages, etc.;
2. Declaring as valid and legal all the proceedings taken by the Court of First Instance of
Misamis Oriental, Branch 1, regarding OCT No. 0662;
3. Upholding the validity and indefeasibility of Original Certificate of Title No. 0662 issued
on September 20, 1976 involved herein;
4. Declaring the order for issuance of a decree to issue Original Certificate of Title No.
0662 as valid and lawful; and
5. Costs against plaintiff.
SO ORDERED.
26

The trial court declared that the requisite copies of the notice of initial hearing had been
transmitted to the Office of the Solicitor General and the Bureau of Lands as confirmed by
Josefina Bacarusso, the incumbent Branch Clerk of Court when LRC Case No. N-531 was
being heard. The court further stated that the petitioner failed to adduce at least prima facie
evidence to prove the material allegations of the complaint. The presumption, then, the trial
court declared, was that the personnel of Branch 1 of the CFI and the officers of the Bureau of
Lands performed their duties in accordance with law, there being no evidence to the contrary.
The court even noted the fact that the petitioner, through counsel, agreed to dispense with the
trial and to submit the case for decision after the submission of the parties respective
memoranda and documentary evidence.
The petitioner appealed to the Court of Appeals contending that:
I. THE LOWER COURT GRAVELY ERRED IN RELYING SOLELY ON
PRESUMPTIONS AS THE BASIS OF ITS DECISION.
II. THE LOWER COURT GRAVELY ERRED IN RENDERING A DECISION WITHOUT
HOLDING A TRIAL AND GIVING AN OPPORTUNITY TO APPELLANT TO PRESENT
EVIDENCE.
27

The Office of the Solicitor General alleged that the trial court acted arbitrarily when it rendered
judgment based on the pleadings notwithstanding the following factual issues that were raised
by the parties:
FACTUAL ISSUES
APPELLANTS
POSITION
APPELLEES POSITION
1. Service of copy of
application and its annexes
to the Director of Lands
No compliance The Clerk of Court complied or is
supposed to comply therewith
2. Service of copy of
application and its annexes
to the Solicitor General
No compliance The Clerk of Court complied or is
supposed to comply therewith
3. Submission of the survey
plan to the Director of Lands
for reverification and
approval
No compliance The survey plan has been duly
processed and approved by the Land
Registration Commission
4. Submission of the report
by the Director of Lands
No compliance A Report has been made to the
Director of Lands, Manila by the
Regional Office of the Bureau of
Lands in Cagayan de Oro
28

The Office of the Solicitor General contends that the trial court should have conducted a full-
blown trial instead of allowing the parties to forego with the same. The private respondents, for
their part, admitted that the Land Registration Commissions survey plan had not been re-
evaluated much less approved, by the Director of the Bureau of Lands. They averred, however,
that the same was merely a procedural defect.
29

On April 29, 1999, the CA rendered a decision affirming the ruling of the CFI holding that: (a) the
petitioner failed to prove the material allegations of its complaint; and, (b) the personnel of the
CFI and the Land Registration Commission are presumed to have performed their duties as the
law mandated.
30
The CA denied the petitioners motion for reconsideration of the said decision.
The petitioner filed its petition for review on certiorari praying that the court resolve the following
issues:
A. WHETHER OR NOT THE COURT OF APPEALS, IN AFFIRMING THE APPEALED
JUDGMENT DATED JANUARY 31, 1995, COMMITTED GRAVE ERROR WHEN IT
AFFIRMED THE JURISDICTION OF THE COURT OF FIRST INSTANCE OF MISAMIS
ORIENTAL OVER THE SUBJECT MATTER AND THE PARTIES IN LAND
REGISTRATION CASE NO. 531, COURT (SIC) ON THE BASIS OF THE DISPUTABLE
PRESUMPTION OF REGULARITY OF PERFORMANCE OF OFFICIAL ACTS
(SECTION 3[m], Rule 131 OF THE RULES OF COURT).
B. WHETHER OR NOT THE COURT OF APPEALS COMMITTED GRAVE ERROR
WHEN IT AFFIRMED THE APPEALED JUDGMENT DATED JANUARY 31, 1995
NOTWITHSTANDING THAT IT WAS RENDERED WITHOUT A TRIAL.
31

In their comment on the petition, the private respondents assert that the issues raised by the
petitioner pertain merely to factual matters and not to questions of law. Furthermore, as shown
by the records of Branch 1 of the CFI in LRC Case No. N-531, the petitioner received a copy of
the application, the amended application, as well as the notice of the initial hearing of the case.
The threshold issues for resolution are as follows: (a) whether or not the Court may review the
decision of the appellate court on the issues raised herein; (b) whether the RTC erred in
rendering the decision without a full-blown trial, based solely on the pleadings of the parties and
the documents appended to their memorandum; and (c) whether the decision of the trial court
was made in accordance with law.
On the first issue, the rule is that only questions of law may be reviewed in this Court on a
petition for review on certiorari under Rule 45 of the Rules of Court. However, it has also been
held that the finding of facts of the appellate court may be questioned in this Court, where as in
this case, the latters judgment is based on a misapprehension of the facts, or such findings are
contrary to the admissions of the parties, or when certain relevant facts are overlooked, which, if
property considered, would justify a different conclusion.
32

In this case, the petitioner avers that the trial court erred when it rendered a decision without
conducting a full-blown trial, and based its ruling merely on the pleadings of the parties and the
documents appended to their respective memoranda. The petitioner asserts that under Rule 34
of the Rules of Court, the court may render judgment on the pleadings only when the
respondents answer fails to tender an issue or otherwise admits the material allegations of the
adverse partys pleadings. Furthermore, it was not proper for the trial court to render summary
judgment under Rule 35 of the Rules of Court, for the simple reason that the private
respondents, in their answer, tendered genuine issues of fact which called for the presentation
of evidence.
We do not agree with the petitioner. The trial court dispensed with a full-blown trial because,
precisely, the parties themselves agreed thereto, on the claim that the issues raised may be
resolved on the basis of the pleadings, the memoranda and the appended documents, without
need of presenting witnesses thereon. A party may waive its right to present testimonial
evidence and opt to adduce documentary evidence and thereafter, submit the case for
resolution based solely on their pleadings and documentary evidence. In this case, no less than
the petitioner, represented by the Office of the Solicitor General through Special Attorney
Vicente Seria, agreed to dispense with a full-blown trial.
On the second issue, we agree with the petitioner that the trial court erred in rendering judgment
in favor of the private respondents and that the CA committed a reversible error in affirming the
same.
The CA ruled that the petitioner was burdened to prove that the issuance of OCT-0662 was
marred by irregularities. It further held that a title issued under the torrens system of registration
is presumed valid, and unless and until the petitioner adduced competent and strong evidence
to prove otherwise, government officials such as the personnel of the CFI and the Land
Registration Commission and the Director of the Bureau of Lands are presumed to have
performed their duties in accordance with law. According to the CA, the petitioner failed to
adduce such evidence. The appellate court took note that the private respondents even
appended documentary evidence to their memorandum showing compliance with the statutory
requirement.
For its part, the petitioner contends that as defendants in the trial court, it was the burden of the
private respondents to prove the existence of a fact that the land registration court had
acquired jurisdiction over the subject matter of the petition and over the persons of the
respondent therein; conversely, the private respondents, as applicants therein, were obliged to
adduce in evidence the survey plan approved by the Director of the Bureau of Lands as
required by P.D. Nos. 239 and 1529. According to the petitioner, there is no presumption in
favor of the jurisdiction of a court of limited jurisdiction, such as a land registration court. It
contends that where the jurisdiction of a court depends upon the existence of facts, it has no
right or power to proceed or act upon a pleading which does not substantially set forth such
facts.
We find for the petitioner. As applicants in LRC Case No. N-531, the private respondents had
the burden of complying with the statutory requirement of serving the Director of the Bureau of
Lands with a copy of their application and amended application, and to show proof of their
compliance thereon. However, we also agree with the CA that it was the burden of the petitioner
in the trial court to prove the material allegations of its complaint. This is provided in Section 1,
Rule 131 of the Rules of Court which reads:
Burden of proof. Burden of proof is the duty of a party to present evidence on the facts in
issue necessary to establish his claim or defense by the amount of evidence required by law.
Obviously, the burden of proof is, in the first instance, with the plaintiff who initiated the action.
But in the final analysis, the party upon whom the ultimate burden lies is to be determined by the
pleadings, not by who is the plaintiff or the defendant. The test for determining where the burden
of proof lies is to ask which party to an action or suit will fail if he offers no evidence competent
to show the facts averred as the basis for the relief he seeks to obtain,
33
and based on the result
of an inquiry, which party would be successful if he offers no evidence.
34

In ordinary civil cases, the plaintiff has the burden of proving the material allegations of the
complaint which are denied by the defendant, and the defendant has the burden of proving the
material allegations in his case where he sets up a new matter. All facts in issue and relevant
facts must, as a general rule, be proven by evidence except the following:
(1) Allegations contained in the complaint or answer immaterial to the issues.
(2) Facts which are admitted or which are not denied in the answer, provided they have
been sufficiently alleged.
(3) Those which are the subject of an agreed statement of facts between the parties; as
well as those admitted by the party in the course of the proceedings in the same case.
(4) Facts which are the subject of judicial notice.
(5) Facts which are legally presumed.
(6) Facts peculiarly within the knowledge of the opposite party.
35

The effect of a presumption upon the burden of proof is to create the need of presenting
evidence to overcome the prima facie case created thereby which if no proof to the contrary is
offered will prevail; it does not shift the burden of proof.
36
In this case, the personnel of the Land
Registration Commission and the CFI in LRC Case No. N-531 are presumed to have performed
their duty of serving a copy of the application and its appendages to the petitioner. It was thus
the burden of the petitioner to prove that: (a) it was not served with a copy of the application of
the private respondents and its annexes; (b) the private respondents failed to append to their
application the survey plan of Lot No. 2821, duly approved by the Director of the Bureau of
Lands as required by P.D. Nos. 1529 and 239. Unless the same were admitted by the
respondents, the petitioner should have adduced in evidence the relevant portions of the
records of LRC Case No. N-531, including the decision of the trial court, to prove that the
Director of the Bureau of Lands was not served with a copy of the application and the amended
application.
37

Section 13, Rule 13 of the Rules of Court provides as follows:
SEC. 13. Proof of service. Proof of personal service shall consist of a written admission of the
party served, or the official return of the server, or the affidavit of the party serving, containing a
full statement of the date, place and manner of service. If the service is by ordinary mail, proof
thereof shall consist of an affidavit of the person mailing of facts showing compliance with
section 7 of this Rule. If service is made by registered mail, proof shall be made by such
affidavit and the registry receipt issued by the mailing office. The registry return card shall be
filed immediately upon its receipt by the sender, or in lieu thereof the unclaimed letter together
with the certified or sworn copy of the notice given by the postmaster to the addressee.
Such proof of service should be found in the records of the case in which the
application/amended application was filed, in this case, LRC Case No. N-531. The same
records will also show whether or not the private respondents appended the survey plan duly
approved by the Director of the Bureau of Lands to their application, as mandated by P.D. Nos.
1529 and 239, and whether the private respondents adduced the said plan in evidence.
The petitioner should have moved for the issuance of a subpoena duces tecum for the Clerk of
Court of Branch 1 of the RTC to bring to the court the records of LRC Case No. N-531 to prove
the material allegations of its complaint. The petitioner did not.
The question that comes to fore then is whether or not the petitioner was burdened to prove its
allegation that the Director of the Bureau of Lands had approved Plan (LRC) SWO-150. The
answer to the question is dependent on the resolution of the issue of whether or not the private
respondents admitted the same, impliedly or expressly, in their answer to the complaint and in
their pleadings.
A careful perusal of the records reveals that in paragraph 8 of its complaint, the petitioner
alleged that the survey plan, Plan (LRC) SWO-150 was not submitted to the Director of the
Bureau of Lands for re-verification and approval as required by law, notwithstanding which the
trial court rendered judgment in favor of the applicants. Hence, the petitioner concluded, the
said plan is void:
8. That the survey plan of the land applied for in said registration case, plan (LRC) SWO-150, is
a plan processed and approved by the Land Registration Commission, but the same plan was
not submitted to the Director of Lands for re-verification and approval as required by the
provision of Section 2, Presidential Decree No. 239, series of 1973. Hence, the plan (LRC) Swo-
150 submitted with the application should be considered as void and non-existing. Furthermore,
on February 5, 1976, the said land registration court, after receiving the evidence of the
applicants ex parte, rendered its decision in the land registration case without requiring the
Director of Lands to submit his report as required by the provision of Section 3 of the same
Presidential Decree No. 239;
38

In paragraph 4 of their answer to the complaint, the private respondents denied the foregoing
allegation in paragraph 8, but alleged as follows:
4. That defendants DENY the allegations in paragraph 8 of the Complaint, the truth of the matter
being that the survey plan has been duly processed and approved by the Land Registration
Commission. As regards the Report required by the provisions of Section 3 of PD 239, the
records show that a Report has been made to the Director of Lands, Manila, by the Regional
Office of the Bureau of Lands in Cagayan de Oro City. Moreover, the deficiencies of the
administrative officials of the government in following procedures or rules and implementing
circulars relative to land registration cases, if any, is not a ground for voiding the title already
issued since the defect, if any, is not jurisdictional but merely procedural in nature. Besides, the
fault or omission, if any, is that of the Land Registration Commission and the Director of Lands
and not that of the defendants;
39

The private respondents failed to specifically deny the petitioners averment in its complaint that
LRC Plan SWO-150 had not been approved by the Director of the Bureau of Lands. The private
respondents thereby impliedly admitted that the Director of the Bureau of Lands had not
approved any survey plan as required by Sections 2 and 3 of P.D. No. 239.
40

In light of the private respondents admission, the petitioner was relieved of its burden of still
proving that the Director of the Bureau of Lands had not approved any survey plan of Lot 2821
before the trial court rendered its decision.
We reject the contention of the private respondents that the reevaluation and approval of the
Director of the Bureau of Lands of the survey plan are not mandatory requirements and that the
lack thereof did not render Original Certificate of Title No. 0662 void. Case law has it that it is
not the function of the Land Registration Commission to approve original plans.
41
The duty
devolved upon the Director of the Bureau of Lands, as mandated by Section 17 of P.D. No.
1529. Applicants for land registration are required to append a survey plan to their application,
duly approved by the Bureau of Lands, thus:
Sec. 17. What and where to file. The application for land registration shall be filed with the
Court of First Instance of the province or city where the land is situated. The applicant shall file
together with the application all original muniments of titles or copies thereof and a survey plan
approved by the Bureau of Lands.
The submission of the plan approved by the Director of the Bureau of Lands is a statutory
requirement which is mandatory in nature. The plan approved by the Land Registration
Commission is of no value.
42
It behooved the trial court not to take cognizance of any
application for land registration in the absence of a survey plan duly approved by the Director of
the Bureau of Lands appended thereto. The private respondents admitted that the Director of
the Bureau of Lands had not approved any survey plan for Lot No. 2821. Consequently, the title
issued by the Register of Deeds in favor of the private respondents is null and void. Such title
cannot ripen into private ownership. As we held in a recent case:
43

No plan or survey may be admitted in land registration proceedings until approved by the
Director of Lands. The submission of the plan is a statutory requirement of mandatory character.
Unless a plan and its technical description are duly approved by the Director of Lands, the same
are of no value.
Thus, the allegation that the signature approval for the survey plan was nowhere to be found is
an important jurisdictional fact that must be ventilated before the trial court. In Republic vs.
Intermediate Appellate Court, this Court stated that "void ab initio land titles issued cannot ripen
into private ownership." Thus, as OCT No. 17 is void and Segundina traces her rights to OCT
No. 17, her claim would have no basis as a spring cannot rise higher than its source.
We also reject the private respondents submission that they should not be faulted for the failure
of the Director of the Bureau of Lands to act on and resolve the recommendation of its Regional
Director. The private respondents were mandated to comply with the requirements of P.D. No.
1529 with fealty before they filed their application for judicial confirmation of imperfect title in the
court a quo.
Parenthetically, the evidence of the petitioner shows that the private respondents failed to
append a survey plan duly approved by the Director of the Bureau of Lands to their
application.
44

The records show that on October 3, 1977, or after the CFI had rendered judgment in LRC
Case No. N-531, the Regional Director of the Bureau of Lands directed the District Land Officer
to report if the applicants in LRC Case No. N-531 had already furnished a copy to his office and,
if so, to explain why the same had not been forwarded to the Regional Office.
45
In fine, as of
October 3, 1977, the copy of the application of the private respondents in LRC No. N-531 had
not been forwarded to the Regional Office of the Bureau of Lands. Indeed, it appears, based on
the evidence of the parties, that the trial court even ignored the Report of the Land Registration
Commission dated July 15, 1975 in LRC Case No. N-531 requiring the Directors of the Bureau
of Lands and Forestry to submit a status report of Lot No. 2821 before setting the case for
hearing:
WHEREFORE, this matter is brought to the attention of this Honorable Court for its information
and guidance in the disposition of the instant land registration case. Further, to avoid duplication
in the issuance of titles covering the same parcel of land and the issuance of titles for lands
within the forest zone which have not been released and classified as alienable, it is respectfully
recommended that the Director of Lands and the Director of Forestry, respectively, be required
to submit a report on the status of the land applied for, before the hearing of the case, to
determine whether said land or any portion thereof is comprised in any patent or forest zone.
46

In recapitulation, then, the CFI committed a reversible error in dismissing the petitioners
complaint and in not rendering judgment in favor of the petitioner. In turn, the Court of Appeals
erred in affirming the decision of the CFI.
IN LIGHT OF ALL THE FOREGOING, the Decision of the Court of Appeals affirming the
Decision of the Court of First Instance in Civil Case No. 7514 is SET ASIDE AND REVERSED.
The Decision of the Court of First Instance appealed from is also SET ASIDE AND REVERSED.
The Court hereby nullifies Original Certificate of Title No. 6662 under the names of the private
respondents and orders the reversion of the property covered by the said title to the petitioner.
SO ORDERED.
Quisumbing, (Acting Chairman), Austria-Martinez, and Tinga, JJ., concur.
Puno, J., (Chairman), on leave.


Footnotes
1
Penned by Associate Justice B.A. Adefuin-de la Cruz with Associate Justices Eugenio
S. Labitoria (Acting Chairman) and Presbitero J. Velasco, Jr. concurring.
2
Records, p. 449.
3
Josefina B. Vda. de Neri, Spouses Graciano and Victoria Neri, Spouses Mario and
Victoria Neri Fernandez, Spouses Albert and Ma. Teresa Neri Yrastorza, and Ramon
Neri.
4
Records, pp. 463-464.
5
Id. at 465.
6
Id. at 418.
7
Id. at 419-420.
8
Josefina B. Vda. de Neri, Spouses Graciano B. Neri, Jr. and Victoria Babiera, Spouses
Victoria Neri and Mario Fernandez, Ramon Neri, Spouses Ma. Teresa Neri and Albert
Yrastorza, and the Register of Deeds of Cagayan de Oro City.
9
Cagayan De Oro City has a total land area of 48,885 hectares.
Respondents informed this Court that a portion (12 hectares) of the subject lot is subject
of the expropriation case entitled The City of Cagayan de Oro v. Josefina Vda. de Neri,
et al., Civil Case No. 8092, pending before the Regional Trial Court of Cagayan de Oro
City, Branch 19. (Rollo, pp. 170-172.)
10
Rollo, p. 96.
11
Records, pp. 170-172.
12
Id. at 378.
13
Id. at 449.
14
Id. at 450.
15
Id. at 459-460.
16
Id. at 461. (Emphasis supplied.)
17
Id. at 463-466.
18
Id. at 408.
19
Id. at 409-416.
20
Id. at 417.
21
Id. at 418.
22
Id. at 419.
23
Id. at 420.
24
Id. at 421-422.
25
Id. at 424.
26
Id. at 537-538.
27
Rollo, p. 29.
28
Id. at 78.
29
Records, pp. 137-138.
30
Rollo, pp. 24-36.
31
Id. at 15.
32
See Tansipek v. Philippine Bank of Communications, 372 SCRA 456 (2001).
33
Di Baco v. Bendetto, 95 SE 601.
34
Francisco, The Revised Rules of Court in the Philippines, Vol. VII, Part II, pp. 5-6.
35
Francisco, Revised Rules of Court, Vol. VII, Part II, 1997 ed., pp. 8-9.
36
Id. at 7.
37
Arcelona v. Court of Appeals, 280 SCRA 20 (1997).
38
Records, p. 8.
39
Id. at 20. (Underscoring supplied).
40
Sec. 2. The Land Registration Commissioner shall within thirty (30) days from the date
hereof turn over all survey returns submitted to his Office to the Bureau of Lands for
verification and appropriate action in accordance with Section 1858 of the Revised
Administrative Code and the rules and regulations promulgated hereunder; and furnish
the latter Office with copies of all plans that it had already approved as of the date of
issuance of this Decree for re-verification and appropriate action in accordance with law
and regulations.
Sec. 3. If the land covered by any survey approved by the Land Registration
Commission has already been brought to court for registration purposes under
Act 496 or under Section 48 of Commonwealth Act No. 141, as amended,
otherwise known as the Public Land Act, no decision shall be rendered thereon
until the Director of Lands shall have submitted his report and recommendation
thereon.
41
Director of Lands v. Judge Reyes, 68 SCRA 177 (1975).
42
Ibid.
43
University of the Philippines v. Rosario, 355 SCRA 591 (2001).
44
Exhibit "D."
45
Exhibit "C."
46
Records, pp. 461-462.


THIRD DIVISION


JOSE CAOIBES, JR., MELENCIO
CAOIBES and LOIDA CAOIBES,
Petitioners,


- versus -



CORAZON CAOIBES-PANTOJA, assisted
by her husband CONRADO PANTOJA,
Respondents.


G.R. No. 162873

Present:

QUISUMBING, J., Chairperson,
CARPIO,
CARPIO MORALES,
TINGA, and
VELASCO, JR., JJ.


Promulgated:

July 21, 2006

x - - - - - - - - - - - - - - - - - - - - - - - - - - - - - - - - - - - - - - - - - - - - - - - - - - -x




D E C I S I O N


CARPIO MORALES, J .:

Petitioners Jose Caoibes, Jr., Melencio Caoibes and Loida Caoibes, as FIRST PARTY, and
respondent Corazon Caoibes-Pantoja, as SECOND PARTY, forged on May 10, 1982 an agreement
entitled RENUNCIATION AND TRANSFER OF CLAIMS, RIGHTS, AND INTERESTS (the
agreement) covering a parcel of land, Lot 2 of plan Psd-162069 (Lot 2), situated in Calaca, Batangas
containing an area of 54,665 sq. m., the pertinent portions of which agreement read:


x x x x

THAT under and by virtue of a court approved document entitled Compromise
Agreement entered into by the parties in Special Proceeding No. 857 and Civil Case No.
861 of the Court of First Instance of Batangas, Branch VII, in particular Paragraph 4 (b)
of aforesaid document, the FIRST PARTY are to receive, among others, in full
ownership pro indiviso, and free from all liens and encumbrances, the following
described real property, to wit:

A parcel of land (Lot 2 of plan Psd-162069), situated in the sitio
of Taklang-Anak, Barrio of
Calantas, Municipality of Calaca, Province ofBatangas. Bounded on the
NW., along line 1-2, by center of Creek and property of Felimon Las
Herras (Lot 1 of plan Psu-101302); on the SE., along lines 2, 3, 4 and 5,
by Lot 1 of plan Psu-162069; on the S., along lines 5, 6, 7, 8 and 9, by
Creek; on the NW., along lines 9, 10, 11, 12, 13 and 1, by center of
Creek and property of Felimon Las Herras (Lot 1 of plan Psu-101302). x
x x containing an area of FIFTY-FOUR THOUSAND SIX HUNDRED
SIXTY-FIVE (54,665) square meters.

THAT issuance to the FIRST PARTY of the proper title to the aforesaid property
is presently the subject of a land registration proceeding LRC No. N-411pending before
the Court of First Instance of Batangas, Branch VII, acting as a land registration court.

THAT for and in consideration of the payment by the SECOND PARTY[-herein
respondent Corazon Caoibes-Pantoja] of the loan secured by a real estate
mortgage constituted on the property described and delineated in Transfer Certificate of
Title No. P-189 of the Registry of Deeds of Batangas, said loan in the principal amount
of NINETEEN THOUSAND PESOS (P19,000.00) exclusive of accrued interest being
presently outstanding in the name of GUILLERMO C. JAVIER with the LEMERY
SAVINGS AND LOAN ASSOCIATION, Balayan Branch, and the further undertaking
of the SECOND PARTY to forthwith deliver upon release to the FIRST PARTY
aforesaid TCT No. P-189 free from all liens and encumbrances, the FIRST
PARTY hereby RENOUNCE, RELINQUISH and ABANDON whatever rights,
interests, or claims said FIRST PARTY may have over the real property in
paragraph 1 hereof x x x [illegible] hereby TRANSFER, CEDE, and CONVEY said
rights x x x [illegible] and claims, in a manner absolute and irrevocable, unto and in
favor of the SECOND PARTY, her heirs, successors and assigns;








THAT by virtue of aforestated renunciation and transfer, the SECOND PARTY
is hereby subrogated and/or substituted to whatever rights, interests or
representations the FIRST PARTY may have in the prosecution of the proper land
registration proceeding mentioned elsewhere in this instrument.
[1]


x x x x (Emphasis and underscoring supplied)

As reflected in the abovequoted agreement of the parties, petitioners, as FIRST PARTY,
renounced, relinquished, abandoned and transferred, ceded and conveyed whatever rights [they] may
have over Lot 2 in favor of respondent, as SECOND PARTY, and on account of the renunciation and
transfer, petitioners transferred whatever rights . . . [they] may have in the prosecution of the land
registration proceeding, LRC No. N-411.

About 14 years after the execution of the parties above-said agreement or in 1996, respondent
filed a motion to intervene and be substituted as applicant in LRC Case No. N-411. The motion was
opposed by petitioners who denied the authenticity and due execution of the agreement, they claiming
that the same was without the consent and conformity of their mother, the usufructuary owner [sic] of
the land. The land registration court, finding for petitioners, denied respondents motion by Order
of March 2, 1999.

Respondent thus filed on March 16, 2000 a Complaint for Specific Performance and Damages
against petitioners before the Regional Trial Court (RTC) of Balayan, Batangas, docketed as Civil Case
No. 3705, for the
enforcement of petitioners obligation under the agreement. To the complaint, petitioners filed a motion
to dismiss anchored on prescription, laches and prematurity of action on account of respondents failure to
refer the case to the barangay lupon for conciliation.

On their defense of prescription, petitioners argued:

It was clearly alleged in the complaint that the purported RENUNCIATION
AND TRANSFER OF CLAIMS, RIGHTS AND INTERESTS was . . . entered into on or
about May 10, 1982 a period of almost 18 LONG YEARS [BEFORE] THE PRESENT
ACTION. Under Article 1144 (1) of the New Civil Code, it is required that an action
founded upon a written contract must be brought WITHIN TEN (10) YEARS FROM
THE TIME THE RIGHT OF ACTION ACCRUES.
[2]
(Underscoring supplied)


Branch 9 of the Balayan RTC, by Resolution
[3]
dated July 12, 2000, granted petitioners motion in
this wise:

The Court is of the view that immediately after the execution of the
RENUNCIATION contract, herein defendants were deemed to have renounced and
transferred their rights or whatever claim they may have on the subject property and the
latter should have at once acted to make the renunciation effective by having herself
substituted to petitioner in the land registration proceedings. Her failure to make
immediately effective the terms of the said RENUNCIATION was constitutive of what is
referred to as the requisite cause of action on the part of the plaintiff.

A cause of action arises when that which should have been done is not done, or
that which should not have been done is done, and in cases where there is no special
provision for such computation, recourse must be had to the rule that the period must be
counted from the day on which the corresponding action could have been instituted
(Central Philippine University vs. CA, 246 SCRA 511).


The fact, that, from the day immediately following the execution of the
RENUNCIATION contract up to the present, with the defendants still continuing the land
registration proceedings without any substitution of plaintiff, could only be interpreted as
a clear manifestation of defendants willful violation of the claimed RENUNCIATION
contract. It is quite incorrect, therefore, to say that the violation happened only when the
defendants objected that they be substituted by plaintiff in an intervention proceedings
filed by the latter.

The added fact that plaintiff did not raise this glaring violation earlier is
something that eludes the comprehension of this Court. What separates the execution of
the contract and the filing of this case is a period of almost EIGHTEEN (18) long years
way beyond the prescriptive period set by law.
[4]
(Underscoring supplied)


On appeal by respondent, the Court of Appeals, by Decision
[5]
of December 4, 2003 subject of the
present petition for review on certiorari, reversed the trial courts Resolution, it holding that prescription
had not yet set in. The Court of Appeals reasoned:

x x x It is not from the date of the instrument but from the date of the breach that the
period of prescription of action starts. Since, it was only in 1996 when plaintiff-appellant
moved to intervene and be substituted as the applicant in the land registration proceeding
involving the subject property that defendants-appellees raised the issue of genuineness
and due execution of the instrument, it is only from this date that the cause of action of
plaintiff-appellant accrued. The period should not be made to retroact to the date of the
execution of the instrument on May 10, 1982 as claimed by the defendants-appellees for
at that time, there would be no way for the plaintiff-appellant to know of the violation of
her rights.
[6]
(Underscoring supplied)

The appellate court thus ordered the remand of the case to the trial court for further proceedings.

Petitioners motion for reconsideration of the decision of the appellate court having been denied,
the present petition for review on certiorari was filed, faulting said court to have

I. . . . ERRED IN REVERSING THE TRIAL COURT AND LABOR[ING]
UNDER A GROSS MISAPPREHENSION OF FACTS IN HOLDING THAT THE
ACTION OF RESPONDENT HAS NOT YET PRESCRIBED.

II. . . . ERRED IN RULING THAT RESPONDENTS CAUSE OF ACTION
ACCRUED ONLY IN 1996 WHEN SHE MOVED TO INTERVENE AND BE
SUBSTITUTED AS AN APPLICANT, IN LIEU OF PETITIONERS IN THE
LAND REGISTRATION PROCEEDING (LRC N-411) BEFORE THE
REGIONAL TRIAL COURT, BRANCH 11 OF BALAYAN, BATANGAS.

III. . . . COMMITTED REVERSIBLE ERROR IN HOLDING THAT THE
PERIOD OF PRESCRIPTION SHOULD NOT BE MADE TO RETROACT TO
THE DATE OF THE EXECUTION OF THE INSTRUMENT ON MAY 10, 1982.

IV. . . . ERRED IN NOT DISMISSING THE COMPLAINT JUST THE SAME BY
NOT FINDING THAT LACHES HAD ALREADY SET IN.
[7]



By the earlier-quoted pertinent portions of the agreement, petitioners renounced and transferred
whatever rights, interests, or claims they had overLot 2 in favor of respondent for and in consideration of
her payment of the therein mentioned loan in the principal amount of P19,000 which was outstanding in
the name of one Guillermo C. Javier.

Articles 1458, 1498 and 1307 of the Civil Code which are pertinent to the resolution of the
petition provide:

Art. 1458. By the contract of sale one of the contracting parties obligates himself
to transfer the ownership of and to deliver a determinate thing, and the other to pay
therefor a price certain in money or its equivalent.



x x x x

Art. 1498. When the sale is made through a public instrument, the execution
thereof shall be equivalent to the delivery of the thing which is the object of the contract,
if from the deed the contrary does not appear or cannot clearly be inferred.

x x x x

Art. 1307. Innominate contracts shall be regulated by the stipulations of the
parties, by the provisions of Title I and II of this Book, by the rules governing the most
analogous nominate contracts, and by the customs of the place.

(Emphasis and underscoring supplied)


The agreement of the parties is analogous to a deed of sale in favor of respondent, it having
transferred ownership for and in consideration of her payment of the loan in the principal amount
of P19,000 outstanding in the name of one Guillermo C. Javier. The agreement having been made through
a public instrument, the execution was equivalent to the delivery of the property to respondent.
[8]


In respondents complaint for specific performance, she seeks to enforce the agreement for her to
be subrogated and/or substituted as applicant in the land registration proceeding over Lot 2. The
agreement is of course in consonance with Sec. 22 of P.D. 1529 (Property Registration Decree which
became effective on June 11, 1978) reading:








SEC. 22. Dealings with land pending original registration. After the filing of
the application and before the issuance of the decree of registration, the land therein
described may still be the subject of dealings in whole or in part, in which case the
interested party shall present to the court the pertinent instruments together with the
subdivision plan approved by the Director of Lands in case of transfer of portions thereof,
and the court, after notice to the parties, shall order such land registered subject to the
conveyance or encumbrance created by said instruments, or order that the decree of
registration be issued in the name of the person to whom the property has been conveyed
by said instruments. (Underscoring supplied)


In Mendoza v. Court of Appeals,
[9]
this Court, passing on Sec. 29 of Art. No. 496, as amended
(Land Registration Act), which is substantially incorporated in the immediately-quoted Sec. 22 of the
Property Registration Decree, held:

The law does not require that the application for registration be amended by
substituting the buyer or the person to whom the property has been conveyed for the
applicant. Neither does it require that the buyer or the person to whom the
property has been conveyed be a party to the case. He may thus be a total stranger to
the land registration proceedings. The only requirements of the law are: (1) that the
instrument be presented to the court by the interested party together with a motion that
the same be considered in relation with the application; and (2) that prior notice be given
to the parties to the case. x x x (Emphasis supplied)


In light of the law and jurisprudence, the substitution by respondent of petitioners as applicant in
the land registration case over Lot 2 is not even necessary. All respondent has to do is to comply with the
requirements under the above-quoted Sec. 22 of the Property Registration Decree. Ergo, it was
unnecessary for respondent to file the case for specific performance subject of the present petition against
petitioners to honor their agreement allowing her to be substituted in their stead as applicant in the land
registration proceeding.

WHEREFORE, the assailed decision of the Court of Appeals is REVERSED and SET
ASIDE. The complaint of respondent, docketed by the Regional Trial Court of Balayan, Batangas as Civil
Case No. 3705, Corazon Caoibes-Pantoja is, in light of the foregoing ratiocination,DISMISSED.

SO ORDERED.


CONCHITA CARPIO MORALES
Associate Justice


WE CONCUR:





LEONARDO A. QUISUMBING
Associate Justice
Chairperson





ANTONIO T. CARPIO
Associate Justice

DANTE O. TINGA
Associate Justice




PRESBITERO J. VELASCO, JR.
Associate Justice





ATTESTATION

I attest that the conclusions in the above Decision were reached in consultation before the case was
assigned to the writer of the opinion of the Courts Division.



LEONARDO A. QUISUMBING
Associate Justice
Chairperson






CERTIFICATION

Pursuant to Article VIII, Section 13 of the Constitution, and the Division Chairmans Attestation, it
is hereby certified that the conclusions in the above Decision were reached in consultation before the case
was assigned to the writer of the opinion of the Court.



ARTEMIO V. PANGANIBAN
Chief Justice



[1]
RTC records at 6-7.
[2]
Id. at 15.
[3]
Id. at 36-43.
[4]
Id. at 41-42.
[5]
Penned by Justice Remedios A. Salazar-Fernando and concurred in by Justices Eubolo G. Verzola
and Edgardo F. Sundiam, CA rollo at 57-64.
[6]
Id. at 63.
[7]
Rollo, pp. 7-8.
[8]
Art. 1496 of the Civil Code provides:
The ownership of the thing is acquired by the vendee from the moment it is delivered to him in
any of the ways specified in Articles 1497 to 1501, or in any other manner signifying an agreement
that the possession is transferred from the vendor to the vendee.
[9]
G.R. No. L-36637, July 14, 1978, 84 SCRA 67, 77.




FIRST DIVISION


REPUBLIC OF THE PHILIPPINES, G.R. No. 151910
Petitioner,
Present:

PUNO, C.J., Chairperson,
- versus - SANDOVAL-GUTIERREZ,
CORONA,
AZCUNA, and
GARCIA, JJ.
LUDOLFO V. MUOZ,
Respondent. Promulgated:

October 15, 2007
x ---------------------------------------------------------------------------------------- x

DECISION

AZCUNA, J.:

Before this Court is a Petition for Review on Certiorari, under Rule 45 of the 1997 Rules of Civil
Procedure, seeking to set aside the August 29, 2001 Decision
[1]
of the Court of Appeals (CA) in CA-G.R.
CV No. 58170, as well as its January 29, 2002 Resolution, which affirmed the October 3, 1997
Decision
[2]
of the Regional Trial Court (RTC) of Ligao, Albay, Branch 13, granting the application for
land registration of respondent Ludolfo V. Muoz.

The following facts prompted the present controversy.

On June 14, 1996, respondent filed an Application for Registration of Title of a parcel of
residential land before the RTC of Ligao, Albay containing an area of 1,986 square meters situated,
bounded, and described as follows:

A PARCEL OF LAND (Lot No. 2276 of the Cadastral Survey of Ligao) with
the building and improvements thereon, situated in the Barrio of Bagonbayan,
Municipality of Ligao, Province of Albay. Bounded on the S., along line 1-2, by Lot No.
2277, Ligao Cadastre; on the W., along Line 2-3, by Mabini Street; on the N., and E.,
along lines 3-4-5-6-4-7, by Lot 2284; and on the S., along line 7-8, by Lot 2281; and
along line 8-1, by Lot 2278 all of Ligao Cadastre, containing an area of ONE
THOUSAND NINE HUNDRED EIGHTY SIX (1,986) square meters.
[3]



In his application for registration, respondent averred that no mortgage or encumbrance of any
kind affects his property and that no other person has an interest, legal or equitable, on the subject lot.
Respondent further declared that the property was acquired by donation inter vivos, executed by the
spouses Apolonio R. Muoz and Anastacia Vitero on November 18, 1956, and that the spouses and their
predecessors-in-interest have been in possession thereof since time immemorial for more than 70 years.

On November 7, 1996, petitioner Republic of the Philippines, through the Office of the Solicitor
General (OSG), opposed the application on the following grounds:

(1) That neither the applicant nor his predecessors-in-interest have been
in open, continuous, exclusive and notorious possession and occupation of the land in
question since June 12, 1945 or prior thereto (Sec. 48[b], C.A. 141 as amended by P.D.
1073).

(2) That the muniment/s of title and/or the tax payment/s receipt/s of
application/s, if any, attached to or alleged in the application, do not constitute
competent and sufficient evidence of a bona fide acquisition of the lands acquired for or
his open, continuous, exclusive and notorious possession and occupation thereof in the
concept of owner since June 12, 1945 or prior thereto. Said muniment/s of title as well as
the title do not appear to be genuine and that the tax declaration/s and/or tax payment
receipt/s indicate the pretended possession of application to be of recent vintage.

(3) That the claim of ownership in fee simple on the basis of Spanish
title or grant can no longer be availed of by the applicant who has failed to file an
appropriate application for registration within the period of six (6) months from
February 16, 1976 as required by P.D. No. 892. From the records, it appears that the
instant application was recently filed.

(4) That the parcel applied for is part of the public domain belonging to
the Republic of the Philippines not subject to private appropriation.

(5) That this application was filed beyond December 31, 1987, the
period set forth under Sec. 2, P.D. No. 1073 and therefore, is filed out of time.
[4]



In respondents Answer to Opposition, he professed that the land in question is a residential lot
originally owned and possessed by Paulino Pulvinar and Geronimo Lozada. Sometime in April 1917,
Pulvinar sold his share of the unregistered land to the spouses Muoz and Vitero, respondents parents. In
June 1920, Lozada likewise sold his remaining part to the parents of respondent. Thereafter, the
ownership and possession of the property were consolidated by the spouses and declared for taxation
purposes in the name of Muoz in 1920. Furthermore, it was stated that during the cadastral survey
conducted in Ligao, Albay in 1928, the land was designated as Lot No. 2276, as per Survey Notification
Card issued to Muoz dated October 2, 1928. Finally, respondent contended that from 1920 up to 1996,
the time of application, the land taxes for the property had been fully paid.

On February 6, 1997, an Order of General Default
[5]
was entered by the trial court against the
whole world except for the government and a certain Alex Vasquez, who appeared during the scheduled
initial hearing stating that he would file an opposition to the application.
In the Opposition
[6]
filed by Vasquez dated February 19, 1997, he declared that he owns parcels
of land, Lot Nos. 2284-A-2 and 2275, adjoining that of the subject matter of the application. He added
that certain portions of his lands are included in the application as respondents concrete fence is found
within the area of his lots.

Respondent, in his answer to the opposition,
[7]
alleged that his property, Lot No. 2276, is
covered by a technical description, duly certified correct by the Bureau of Lands and approved for
registration by the Land Registration Authority (LRA), which specified the exact areas and boundaries of
Lot No. 2276. Granting that there is an encroachment to the oppositors adjoining land, respondent
reasoned that it is not for the court a quo, sitting as a Land Registration Court, to entertain the opposition
because the case should be ventilated in a separate proceeding as an ordinary civil case.

During the trial, respondent was presented as the sole witness. Respondent, who was 81 years old
at that time, testified that he acquired the property in 1956 when his parents donated the same to
him.
[8]
He presented as Exhibit H
[9]
Tax Declaration No. 048-0267, evidencing the payment of realty
taxes for Lot No. 2276 in 1997. A Certification from the Office of the Municipal Treasurer
[10]
was
likewise introduced by the respondent showing the payment of real estate taxes from 1956 up to the year
1997. He further declared that the property is a residential land with improvements such as a house made
of solid materials and fruit-bearing trees. In 1957, respondent told the court that he constructed a concrete
wall surrounding the entire property. Respondent also narrated that he grew up on the subject lot and
spent his childhood days in the area.
[11]


On cross-examination, respondent claimed that he has six brothers and sisters, none of whom are
claiming any interest over the property.
[12]


On June 16, 1997, the trial court noted
[13]
a Report
[14]
submitted by the Director of Lands, which
informed the court that as per records of the Land Management Bureau in Manila, Lot No. 2276, CAD-
239 is covered by Free Patent Application No. 10-2-664 of Anastacia Vitero.

The RTC rendered a Decision dated October 3, 1997 granting the application for registration.
The dispositive portion of the decision reads:

WHEREFORE, decision is hereby rendered finding the petitioner entitled to
registration. Accordingly, after the finality of this decision, let a decree and, thereafter
the corresponding certificate of title over Lot No. 2276 of the Ligao Cadastre as
delimited by the Technical Description, Annex A-2 of the application, together with the
improvements thereon, issue in the name of LUDOLFO Y. MUOZ, of legal age,
Filipino citizen, married to JOSEFINA PALENCIA, of Mabini Street, Barangay Tinago,
Municipality of Ligao, Province of Albay.

Conformably with the above findings, as prayed for by the Director,
Department of Registration, Land Registration Authority in his Report dated March 6,
1997, the application, if any, in Cad. Case No. 53, Cadastral Record No. 1404 is hereby
ordered dismissed.

The opposition of Alex Vasquez for lack of merit is hereby ordered dismissed.

Let copy of this Decision be furnished the Office of the Solicitor General,
Provincial Prosecutor of Albay, Oppositor Alez Vasquez and Petitioner.

SO ORDERED.
[15]



On appeal, petitioner argued that the trial court did not acquire jurisdiction over the subject lot
because: (1) the notice of initial hearing was not timely filed; (2) the applicant failed to present the
original tracing cloth plan of the property sought to be registered during the trial; and (3) the applicant
failed to present evidence that the land is alienable and disposable.

Subsequently, the CA affirmed the decision of the court a quo. The appellate court explained that
there was conclusive proof that the jurisdictional requirement of due notice had been complied with as
mandated under Section 24 of Presidential Decree No. 1529. Furthermore, the failure to present in
evidence the tracing cloth plan of the subject property did not deprive the lower court of its jurisdiction to
act on the application in question. Lastly, the CA ruled that respondent need not adduce documentary
proof that the disputed property had been declared alienable and disposable for the simple reason that the
lot had once been covered by free patent application; hence, this alone is conclusive evidence that the
property was already declared by the government as open for public disposition.

The petitioner, through the OSG, raises the following grounds for the petition:

I.
THE COURT OF APPEALS ERRED IN NOT FINDING THAT THE TRIAL COURT
HAS NOT ACQUIRED JURISDICTION OVER THE CASE.

II.
PRIVATE RESPONDENT HAS NOT PROVEN BY COMPETENT EVIDENCE
THAT THE PROPERTY IS ALIENABLE AND DISPOSABLE PROPERTY OF THE
PUBLIC DOMAIN.
[16]


Anent the first issue, petitioner maintains that the failure to present the original tracing cloth plan is
a fatal omission which necessarily affected the trial courts jurisdiction to proceed with the case.

It bears stressing that the constructive seizure of land accomplished by posting of notices and
processes upon all persons mentioned in notices by means of publication and sending copies to said
persons by registered mail in effect gives the court jurisdiction over the lands sought to be
registered.
[17]


While petitioner correctly contends that the submission in evidence of the original tracing cloth
plan is a mandatory and even a jurisdictional requirement, this Court has recognized instances of
substantial compliance with this rule.
[18]
It is true that the best evidence to identify a piece of land for
registration purposes is the original tracing cloth plan from the Bureau of Lands, but blueprint copies and
other evidence could also provide sufficient identification.
[19]
In the present application
for registration, respondent submitted, among other things, the following supporting documents: (1) a
blueprint copy of the survey plan
[20]
approved by the Bureau of Lands; and (2) the technical
descriptions
[21]
duly verified and approved by the Director of Lands.

The Court held in Recto v. Republic
[22]
that the blueprint copy of the cloth plan together with the
lots technical description duly certified as to their correctness by the Bureau of Lands are adequate to
identify the land applied for registration, thus

On the first challenge, the petitioner invokes the case of Director of Lands v.
Reyes, where it was held that the original tracing cloth plan of the land applied for
which must be approved by the Director of Lands was a statutory requirement of
mandatory character for the identification of the land sought to be registered. As what
was submitted was not the tracing cloth plan but only the blueprint copy of the survey
plan, the respondent court should have rejected the same as insufficient.

We disagree with this contention. The Court of Appeals was correct when it
observed that in that case the applicant in effect had not submitted anything at all to
identify the subject property because the blueprint presented lacked the approval of the
Director of Lands. By contrast

In the present case, there was considerable compliance with the
requirement of the law as the subject property was sufficiently
identified with the presentation of blueprint copy of Plan AS-06-
000002 (San Pedro v. Director of Lands, CA-G.R. No. 65332-R, May
28, 1981). It should be noted in this connection that the Bureau of
Lands has certified to the correctness of the blueprint copy of the plan
including the technical description that go with it. Hence, we cannot
ignore the fact, absent in the Reyes case, that applicant has provided
ample evidence to establish the identity of the subject
property. (Emphasis supplied)

x x x.
[23]


Moreover, if the survey plan is approved by the Director of Lands and its correctness has not been
overcome by clear, strong and convincing evidence, the presentation of the tracing cloth plan may be
dispensed with.
[24]
All the evidence on record sufficiently identified the property as the one applied for by
respondent, and containing the corresponding metes and bounds as well as area. Consequently, the
original tracing cloth plan need not be presented in evidence.
[25]


Anent the second issue, petitioner stresses that in proving the alienable and disposable nature of the
property, there has to be a certification from the Department of Environment and Natural Resources and
Community Environment and Natural Resources Office (CENRO).

The CA is of the opinion that respondent need not adduce documentary proofs that the disputed
property has been declared alienable and disposable because of the fact that it had once been covered by
Free Patent Application No. 10-2-664 in the name of respondents mother, which was unfortunately not
acted upon by the proper authorities. The CA declares that this is proof enough that the property was
declared by the government as open for public disposition. This contention was adopted by the respondent
both in his Comment and Memorandum filed before the Court.

Notwithstanding all the foregoing, the Court cannot sustain the argument of respondent that the
subject property was already declared alienable and disposable land.

Petitioner is correct when it remarked that it was erroneous for the appellate court to assume that
the property in question is alienable and disposable based only on the Report dated May 21, 1997 of the
Director of Lands indicating that the land involved in said case described as Lot 2276, CAD-239 is
covered by Free Patent Application No. 10-2-664 of Anastacia Vitero.

It must be pointed out that in its Report
[26]
dated March 6, 1997, the LRA stated that:

3. This Authority is not in a position to verify whether or not the parcel
of land subject of registration is already covered by land patent,previously approved
isolated survey and is within forest zone.

WHEREFORE, to avoid duplication in the issuance of titles covering the same
parcel of land and the issuance of titles for lands within the forest zone which have not
been released and classified as alienable, the foregoing is respectfully submitted to the
Honorable Court with the recommendation that the Lands Management Bureau,
Manila, Community Environment and Natural Resources Office, Lands
Management Sector and Forest Management Bureau, all in Legazpi City, be
ordered to submit a report to the Court on the status of the land applied for, to
determine whether or not said land or any portion thereof, is already covered by
land patent, previously approved isolated survey and is within the forest zone and
that should the instant application be given due course, the application in Cad. Case No.
53, Cadastral Record No. 1404 with respect to Lot 2276 be dismissed.
[27]


Noteworthy is the fact that neither the Director of Lands nor the LRA attested that the land
subject of this proceeding is alienable or disposable.

For clarity, applications for confirmation of imperfect title must be able to prove the following:
(1) that the land forms part of the alienable and disposable agricultural lands of the public domain; and
(2) that they have been in open, continuous, exclusive and notorious possession and occupation of the
same under a bona fide claim of ownership either since time immemorial or since June 12, 1945.
[28]


Commonwealth Act No. 141, also known as the Public Land Act, remains to this day the existing
general law governing the classification and disposition of lands of the public domain, other than timber
and mineral lands.
[29]
Section 6 of CA No. 141 empowers the President to classify lands of the public
domain into alienable and disposable lands of the public domain, which prior to such classification are
inalienable and outside the commerce of man. Section 7 of CA No. 141 authorizes the President to
declare what lands are open to disposition or concession. Section 8 of CA No. 141 states that the
government can declare open for disposition or concession only lands that are officially delimited and
classified.

Under the Regalian doctrine embodied in our Constitution, all lands of the public domain belong
to the State, which is the source of any asserted right to ownership of land. Therefore, all lands not
appearing to be clearly within private ownership are presumed to belong to the State. Accordingly, public
lands not shown to have been reclassified or released as alienable agricultural land or alienated to a
private person by the State remain part of the alienable public domain.
[30]


As already well-settled in jurisprudence, no public land can be acquired by private persons
without any grant, express or implied, from the government; and it is indispensable that the person
claiming title to public land should show that his title was acquired from the State or any other mode of
acquisition recognized by law.
[31]
To prove that the land subject of an application for registration is
alienable, the applicant must establish the existence of a positive act of the government such as a
presidential proclamation or an executive order; an administrative action; investigation reports of Bureau
of Lands investigators; and a legislative act or a statute.
[32]
The applicant may also secure a certification
from the Government that the land applied for is alienable and disposable.
[33]


In the present case, respondent failed to submit a certification from the proper government
agency to prove that the land subject for registration is indeed alienable and disposable. A CENRO
certificate, which respondent failed to secure, could have evidenced the alienability of the land involved.

Considering that respondent has failed to convince this Court of the alienable and disposable
character of the land applied for, the Court cannot approve the application for registration.

WHEREFORE, the instant petition is GRANTED. Accordingly, the decision dated August 29,
2001 of the Court of Appeals in CA-G.R. CV No. 58170, as reiterated in its resolution of January 29,
2002,
is REVERSED and SET ASIDE, and the application for registration filed by respondent Ludolfo
V. Muoz is DENIED.

No costs.

SO ORDERED.


ADOLFO S. AZCUNA
Associate Justice
WE CONCUR:




REYNATO S. PUNO
Chief Justice
Chairperson



ANGELINA SANDOVAL-GUTIERREZ RENATO C. CORONA
Associate Justice Associate Justice



CANCIO C. GARCIA
Associate Justice




CERTIFICATION


Pursuant to Section 13, Article VIII of the Constitution, it is hereby certified that the conclusions in
the above Decision had been reached in consultation before the case was assigned to the writer of the
opinion of the Courts Division.



REYNATO S. PUNO
Chief Justice




[1]
Penned by Associate Justice Cancio C. Garcia (now an Associate Justice of the Supreme
Court), with Associate Justices Hilarion L. Aquino and Jose L. Sabio, Jr. concurring, rollo, pp.
34-50.
[2]
Penned by Judge Jose S. Saez, id. at 62-69.
[3]
Records, p. 1.
[4]
Id. at 34-35.
[5]
Id. at 53.
[6]
Id. at 61-62.
[7]
Id. at 64.
[8]
TSN, May 23, 1997, p. 4.
[9]
Records, p. 76.
[10]
Exhibit I, id. at 77-79.
[11]
TSN, May 23, 1997, p. 6.
[12]
Id. at 7.
[13]
Records, p. 88.
[14]
Id. at 86.
[15]
Id. at 95-96.
[16]
Rollo, p. 14.
[17]
Registration of Land Titles and Deeds, Antonio H. Noblejas and Edilberto H. Noblejas, 1992
edition, p. 104 (underscoring supplied).
[18]
Republic v. Hubilla, G.R. No. 157683, February 11, 2005, 451 SCRA 181, 184.
[19]
Recto v. Republic, G.R. No. 160421, October 4, 2004, 440 SCRA 79, 87, citing Republic v.
Court of Appeals, G.R. No. L-62680, November 9, 1988, 167 SCRA 150, 154, Republic v.
Intermediate Appellate Court, 229 Phil. 20 (1986).
[20]
Annex A-1, records, p. 4.
[21]
Annex A-2, id. at 5.
[22]
Supra, note 19.
[23]
Id. at 87-88, citing Republic v. Court of Appeals, supra note 19, at 153-154.
[24]
Director of Lands v. Intermediate Appellate Court, G.R. No. 70825, March 11, 1991, 195
SCRA 38, 44, citing Director of Lands v. Court of Appeals, G.R. No. 56613, March 14, 1988,
158 SCRA 568, 671,Republic v. Intermediate Appellate Court, G.R. No. 70594, October 10,
1986, 144 SCRA 705.
[25]
Republic v. Enriquez, G.R. No. 160990, September 11, 2006, 501 SCRA 436, 447.
[26]
Records, pp. 67-68.
[27]
Id. at 67 (underscoring supplied).
[28]
Carlos v. Republic, G.R. No. 164823, August 31, 2005, 468 SCRA 709, 714-715.
[29]
Chavez v. Public Estates Authority, 433 Phil. 506, 545 (2002).
[30]
Republic v. Tri-Plus Corporation, G.R. No. 150000, September 26, 2006, 503 SCRA 91, 101-
102, citing Republic v. Naguiat, G.R. No. 134209, January 24, 2006, 479 SCRA 585, 590.
[31]
Republic v. Herbieto, G.R. No. 156117, May 26, 2005, 459 SCRA 183, 199-200,
citing Padilla v. Reyes, 60 Phil. 967, 969 (1934), Lee Hong Hok v. David, G.R. No. L-
30389, December 27, 1972, 48 SCRA 372, 379.
[32]
Republic v. Court of Appeals, G.R. No. 144057, January 17, 2005, 448 SCRA 442, 449.
[33]
Zarate v. Director of Lands, G.R. No. 131501, July 14, 2004, 434 SCRA 322, 332.
G.R. No. 170724 January 29, 2007
REPUBLIC OF THE PHILIPPINES, Petitioner,
vs.
SAN LORENZO DEVELOPMENT CORPORATION, Respondent.
D E C I S I O N
GARCIA, J .:
In this petition for review under Rule 45 of the Rules of Court, petitioner Republic of the
Philippines seeks the reversal and setting aside of the Decision
1
dated May 23, 2005 of the
Court of Appeals (CA)-Cebu City in CA-G.R. CV No. 73996, as reiterated in its Resolution
2
of
December 7, 2005, dismissing the Republic's appeal from an earlier decision of the Municipal
Trial Court in Cities (MTCC), Danao City, which ordered the registration of title in the name of
herein respondent San Lorenzo Development Corporation over a 64,909-square meter parcel of
land in Barangay Maslog, City of Danao, Province of Cebu.
The facts:
On November 13, 1997, respondent San Lorenzo Development Corporation filed with the
MTCC of Danao City an application for registration of title to a parcel of land, described as Lot 1
of the Consolidation-Subdivision Plan, Ccn-07-000094, being a portion of Lots 3151, 3152,
3158, 3159, 3160 and 3161, Cad. 681-D, Danao Cadastre, situated in Barangay Maslog, City of
Danao, Province of Cebu, with a total area of 64,909 square meters, more or less. The
application was docketed in the MTCC as LRC No. 100.
On November 14, 1997, the MTCC issued an Order setting the application for initial hearing on
March 5, 1998. The Order required that a copy thereof be furnished the Commissioner, Land
Registration Authority, for notice and for the necessary publication to be made.
On December 11, 1997, the Solicitor General entered his appearance as counsel for petitioner
Republic and deputized the City Prosecutor of Danao City to appear in the case. On the same
date, the Republic filed its opposition to the application.
On February 24, 1998, another Order was issued by the MTCC resetting the initial hearing of
the application to June 15, 1998. This was followed by an Order of May 15, 1998, resetting
anew the initial hearing to September 23, 1998.
During the initial hearing, the respondent corporation, through counsel, offered in evidence the
following documents to prove or establish the jurisdictional facts of the case, to wit:
Exhibit "A" - The Petition for Registration containing seven (7)
pages and mandatory annexes designated as A-1 to
A-3;
Exhibit "A-1" - Lot Plan No. Ccn-07000094 of Lot 1 comprising
Cad. Lot Nos. 3151, 3152, 3158, 3159, 3160 and
3161;
Exhibit "A-2" - Technical Description of Lot No. 1;
Exhibit "A-3" - Certification of Non-requirement of Surveyor's
Certificate;
Exhibit "B" - Order resetting date of Initial Hearing to September
23, 1998;
Exhibit "B-1" - Newspaper Clipping;
Exhibit "C" - Affidavit of Publication issued by Banat News;
Exhibit "D" - Certificate of Publication issued by the Land
Registration Authority;
Exhibit "E" - Certificate of Posting issued by the Court Sheriff;
Exhibit "F" - Certificate of Publication issued by the NPO;
Exhibit "F-1" - Copy of Notice of Initial Hearing;
Exhibit "G" - Copy of the Indorsement addressed to the Clerk of
Court, MTCC, Danao City, from Salvador Oriel,
Chief, Docket Division, Land Registration Authority,
dated July 7, 1998; and
Exhibit "H" - Notice of Appearance of the Solicitor General.
Thereafter, the case was called aloud in open court to determine whether there were other
oppositors aside from the Republic. There being none, the court issued an Order of General
Default on September 23, 1998.
Respondent corporation, to prove that it and its predecessors-in-interest had been in
possession of the land applied for in the concept of an owner peacefully, continuously,
adversely and notoriously for a period required under the law, presented six (6) witnesses. The
six, who were predecessors-in-interest of composite portions of the subject parcel of land,
provided testimonies to the effect that they had been in possession of the land, and had
subsequently sold their respective parcels thereof to the respondent. Their testimonies were
supported by tax declarations and deeds of sale.
On October 12, 2001, the trial court rendered its decision
3
granting the respondents application
for registration of title, thus:
WHEREFORE, premises considered, Judgment is hereby rendered ordering the issuance of
title to Lot 1 of the Consolidation-Subdivision of Plan Ccn-07-000094, being a portion of Lot
3152, 3151, 3158, 3159, 3160 and 3161, Cad. 681-D, Danao Cadastre, situated in the
Barangay of Maslog, Danao City, Province of Cebu, Island of Cebu, containing an area of
SIXTY FOUR THOUSAND NINE HUNDRED NINE (64,909) square meters, for and in the name
of San Lorenzo Development Corporation, with principal office address at Ground Floor,
Stanford Tower Condominium, 1870 M.H. Del Pilar Street, Malate, Metro Manila.
Upon finality of this Decision, let a corresponding decree of registration be issued in favor of
applicant in accordance with Sec. 39 of PD 1529.
SO ORDERED.
On November 7, 2001, petitioner Republic filed a Notice of Appeal, therein making known that it
was elevating the case to the CA. In the CA, the Republics appellate recourse was docketed as
CA-G.R. CV No. 73996.
In the herein assailed decision
4
of May 23, 2005, the CA-Cebu City dismissed the Republics
appeal. Its motion for reconsideration having been denied by the same court in its equally
assailed resolution
5
of December 7, 2005, the Republic is now before this Court via the instant
petition raising the following issues:
1. Whether or not the defective and/or want of notice by publication of the initial
hearing(s) of the case a quo vested the trial court with jurisdiction to take cognizance
thereof; and
2. Whether or not deeds of sale and tax declarations/clearances constitute the "well-nigh
incontrovertible" evidence necessary to acquire title through adverse occupation under
C.A. No. 141.
In the matter of jurisdiction, petitioner Republic maintains that the MCTC never acquired
jurisdiction over the case on account of its failure to conduct the initial hearing thereof within the
period fixed in Section 23 of P.D. No. 1529, otherwise known as the Property Registration
Decree, which mandates that the date and hour of initial hearing shall not be earlier than 45
days nor later than 90 days from the date of the Order. In the Republics own words:
6

After a series of postponements, the trial court finally set the initial hearing of the case on
September 23, 1998 in an order issued on May 15, 1998 xxx. The notice of initial hearing,
however, was issued only on June 6, 1998.
Pursuant to Section 23, P.D. 1529, the initial hearing of the case must have to be not earlier
than forty-five (45) days and not later than ninety (90) days from the date of the order setting the
date and hour of the initial hearing. The Order having been issued on May 15, 1998, the initial
hearing should have been set not earlier than June 29, 1998 (45 days from May 15, 1998 and
not later than August 13, 1998 (90 days from May 15, 1998). Unfortunately, the initial hearing
was scheduled and actually held on September 23, 1998, some forty-one (41) days later than
the prescribed period.
Even if counted from June 8, 1998 (date of notice of hearing), still the hearing on September 23,
1998 is seventeen (17) days late than the prescribed period of ninety (90) days, the last day of
which fell on September 6, 1998.
It is noteworthy that both parties invoke the decision of the Court in Republic v. Manna
Properties, Inc.,[7] decided January 31, 2005, albeit each cites different portions thereof, and for
different purposes. The common reliance on said case is well-placed as it is, indeed, of a similar
factual setting. Furthermore, that case tackles the same two (2) issues presently raised:
compliance with the jurisdictional requirements for original registration, and proof of possession
for the requisite period.
A careful reading of Republic v. Manna Properties, Inc. will support a finding in favor of the
respondent but only as regards the issue of jurisdiction. Speaking on that issue, the Court in
Manna Properties, Inc., wrote:
The duty and the power to set the hearing date lie with the land registration court. After an
applicant has filed his application, the law requires the issuance of a court order setting the
initial hearing date. The notice of initial hearing is a court document. The notice of initial hearing
is signed by the judge and copy of the notice is mailed by the clerk of court to the LRA [Land
Registration Authority]. This involves a process to which the party applicant absolutely has no
participation.
Petitioner is correct that in land registration cases, the applicant must strictly comply with the
jurisdictional requirements. In this case, the applicant complied with the jurisdictional
requirements.
The facts reveal that Manna Properties was not at fault why the hearing date was set beyond
the 90-day maximum period. x x x.
We have held that "a party to an action has no control over the Administrator or the Clerk of
Court acting as a land court; he has no right to meddle unduly with the business of such official
in the performance of his duties."
8
A party cannot intervene in matters within the exclusive
power of the trial court. No fault is attributable to such party if the trial court errs on matters
within its sole power. It is unfair to punish an applicant for an act or omission over which the
applicant has neither responsibility nor control, especially if the applicant has complied with all
the requirements of the law.
Moreover, it is evident in Manner Properties, Inc. that what is more important than the date on
which the initial hearing is set is the giving of sufficient notice of the registration proceedings via
publication. In fact, in its memorandum,
9
petitioner Republic "concedes (a) that respondent
should not be faulted if the initial hearing that was conducted on September 23, 1995 was
outside the 90-day period set forth under Section 23 of Presidential Decree No. 1529, and (b)
that respondent might have substantially complied with the requirement thereunder relating to
the registration of the subject land."
10
Hence, on the issue of jurisdiction, we find for the
respondent, in that its application for registration was rightfully given due course by the MTCC.
However, on the more important issue of lack of evidence of possession on the part of the
respondent for the period required by law, the balance must tilt in favor of the petitioner.
Very evident from Republic v. Manna Properties, Inc. is that the reckoning date under the Public
Land Act
11
for the acquisition of ownership of public lands is J une 12, 1945 or earlier, and that
evidence of possession from that date or earlier is essential for a grant of an application for
judicial confirmation of imperfect title. Section 48(b) of the Public Land Act, as amended by P.D.
No. 1073, provides:
(b) Those who by themselves or through their predecessors-in-interest have been in open,
continuous, exclusive, and notorious possession and occupation of agricultural lands of the
public domain, under a bona fide claim of acquisition of ownership, since June 12, 1945 or
earlier, immediately preceding the filing of the application for confirmation of title except when
prevented by war or force majeure. Those shall be conclusively presumed to have performed all
the conditions essential to a Government grant and shall be entitled to a certificate of title under
the provisions of this chapter. (Emphasis supplied)
Similarly, Section 14 of P.D. No. 1529 the Property Registration Decree provides, inter alia,
as follows:
Section 14. Who may apply. - The following persons may file in the proper Court of First
Instance an application for registration of title to land, whether personally or through their duly
authorized representatives:
1. Those who by themselves or through their predecessors-in-interest have been in open,
continuous, exclusive and notorious possession and occupation of alienable and disposable
lands of the public domain under a bona fide claim of ownership since June 12, 1945, or earlier;
(Emphasis supplied)
Here, in support of its application for registration, the respondent corporation submitted a
certification from the Community Environment and Natural Resources Office (CENRO) that the
parcel of land sought to be registered forms part of the general area classified as alienable and
disposable public land under Forestry Administrative Order No. 4-467 dated June 7, 1938. It
also submitted tax declarations and/or clearances, the earliest of which is in the year 1964 for
Lots 3150 and 3160; 1963 for Lot 3151; and 1948 for Lots 3152, 3159 and 3161.
The respondent's application was granted by the two (2) courts below on the premise that,
reckoned to date, possession of the subject parcel of land since the declaration of alienability
and disposability on June 7, 1938 was more than fifty (50) years already. Adverse possession
for at least thirty (30) years had long been completed. This reasoning was fraught with errors.
First, the law, as mentioned earlier, requires that the possession of lands of the public domain
must be from at least June 12, 1945 for the same to be acquired through judicial confirmation of
imperfect title. Through the years, Section 48(b), supra, of the Public Land Act has been
amended several times. The case of Republic v. Doldol[12] provides a summary of these
amendments:
x x x. The original Section 48(b) of C.A. No. 141 provided for possession and occupation of
lands of the public domain since July 26, 1894. This was superseded by R.A. No. 1942, which
provided for a simple thirty-year prescriptive period of occupation by an applicant for judicial
confirmation of imperfect title. The same, however, has already been amended by Presidential
Decree 1073, approved on January 25, 1977. As amended, Section 48(b) now reads:
(b) Those who by themselves or through their predecessors-in-interest have been in open,
continuous, exclusive, and notorious possession and occupation of agricultural lands of the
public domain, under a bona fide claim of acquisition of ownership, since June 12, 1945 or
earlier, immediately preceding the filing of the application for confirmation of title except when
prevented by war or force majeure. These shall be conclusively presumed to have performed all
the conditions essential to a Government grant and shall be entitled to a certificate of title under
the provisions of this chapter.
Section 48(b) of the Public Land Act, as amended by PD No. 1073, presently requires, for
judicial confirmation of an imperfect or incomplete title, the possession and occupation of the
piece of land by the applicants, by themselves or through their predecessors-in-interest, since
12 June 1945 or earlier. This provision is in total conformity with Section 14(1) of the Property
Registration Decree heretofore cited.
As the law now stands, a mere showing of possession for thirty years or more is not sufficient. It
must be shown, too, that possession and occupation had started on June 12, 1945 or earlier.
Second, the acceptance by the courts a quo of the CENRO certificate of alienability and
disposability as evidence ofpossession since the date of said certificate is patently erroneous.
According to the CENRO certification, the subject land was alienable and disposable public land
since June 7, 1938. This certification does not in any way indicate that the respondent and its
predecessors-in-interest had been in possession of the property as far back as 1938.
The Public Land Act requires that the applicant must prove two things, to wit:
1. That the land is alienable public land; and
2. That his open, continuous, exclusive and notorious possession and occupation of the
same must either be since time immemorial or for the period prescribed in the Public
Land Act.
13

All that the CENRO certificate evidences is the alienability of the land involved, not the open,
continuous, exclusive and notorious possession and occupation thereof by the respondent or its
predecessors-in-interest for the period prescribed by law.
As in Manna Properties, Inc., while the Court acknowledges tax declarations as sufficient basis
for inferring possession, the tax declarations presented by the respondent in this case do not
serve to prove their cause:
The 1945 tax declaration must be presented considering that the date, 12 June 1945, is material
to this case. CA 141 specifically fixes the date to 12 June 1945 or earlier. x x x Unless the date
and month of issuance in 1945 is stated, compliance with the reckoning date in CA 141 cannot
be established.
14

Here, the earliest of the tax declarations presented by the respondent was in the year 1948 for
Lots 3152, 3159 and 3161;1963 for Lot 3151; and 1964 for Lots 3150 and 3160. At best, the
respondent corporation can only prove possession since said dates. This does not constitute
the evidence necessary to acquire title through adverse occupation under CA 141, as amended.
Lastly, while it is true that the issue of whether or not the respondent corporation has presented
sufficient proof or the required possession raises a question of fact, which ordinarily cannot be
entertained in a petition under Rule 45, one of the exceptions to that rule is when, as here, the
evidence on record does not support the conclusions of both the trial and the appellate courts.
On the whole, we find merit in the petition.
WHEREFORE, the instant petition is GRANTED. Accordingly, the decision dated May 23, 2005
of the Court of Appeals-Cebu City in CA-G.R. CV No. 73996, as reiterated in its resolution of
December 7, 2005, is REVERSED and SET ASIDE,and the application for registration filed by
respondent San Lorenzo Development Corporation is DENIED.
No costs.
SO ORDERED.
CANCIO C. GARCIA
Associate Justice
WE CONCUR:
REYNATO S. PUNO
Chief Justice
Chairperson
ANGELINA SANDOVAL-GUTIERREZ
Associate Justice
RENATO C. CORONA
Asscociate Justice
ADOLFO S. AZCUNA
Associate Justice
C E R T I F I C A T I O N
Pursuant to Article VIII, Section 13 of the Constitution, it is hereby certified that the conclusions
in the above decision had been reached in consultation before the case was assigned to the
writer of the opinion of the Courts Division.
REYNATO S. PUNO
Chief Justice


Footnotes
1
Penned by Associate Justice Isaias P. Dicdican, with Associate Justices Vicente L. Yap
and Enrico A. Lanzanas, concurring; Rollo, pp. 39-46.
2
Id. at 47-48.
3
Id. at 63-69.
4
Supra note 1.
5
Supra note 2.
6
Petition, pp. 19-20; Rollo, pp. 25-26.
7
G.R. No. 146527, 450 SCRA 247.
8
Banco Espanol-Filipino v. Palanca, 37 Phil. 921 (1918).
9
Rollo pp. 150-185.
10
Id. at 170.
11
Commonwealth Act No. 141, as amended.
12
G.R. No. 132963, September 10, 1998, 295 SCRA 359, 364.
13
Republic v. Court of Appeals, G.R. No. 108998, August 24, 1994, 235 SCRA 567.
14
Supra note 6 at 261.
SECOND DIVISION
[G.R. No. 156117. May 26, 2005]
REPUBLIC OF THE PHILIPPINES, petitioner, vs. JEREMIAS AND DAVID
HERBIETO, respondents.
D E C I S I O N
CHICO-NAZARIO, J .:
Before this Court is a Petition for Review on Certiorari, under Rule 45 of the 1997 Rules of
Civil Procedure, seeking the reversal of the Decision of the Court of Appeals in CA-G.R. CV No.
67625, dated 22 November 2002,
[1]
which affirmed the Judgment of the Municipal Trial Court
(MTC) of Consolacion, Cebu, dated 21 December 1999,
[2]
granting the application for land
registration of the respondents.
Respondents in the present Petition are the Herbieto brothers, Jeremias and David, who
filed with the MTC, on 23 September 1998, a single application for registration of two parcels of
land, Lots No. 8422 and 8423, located in Cabangahan, Consolacion, Cebu (Subject Lots). They
claimed to be owners in fee simple of the Subject Lots, which they purchased from their parents,
spouses Gregorio Herbieto and Isabel Owatan, on 25 June 1976.
[3]
Together with their
application for registration, respondents submitted the following set of documents:
(a) Advance Survey Plan of Lot No. 8422, in the name of respondent Jeremias; and Advance
Survey Plan of Lot No. 8423, in the name of respondent David;
[4]

(b) The technical descriptions of the Subject Lots;
[5]

(c) Certifications by the Department of Environment and Natural Resources (DENR)
dispensing with the need for Surveyors Certificates for the Subject Lots;
[6]

(d) Certifications by the Register of Deeds of Cebu City on the absence of certificates of title
covering the Subject Lots;
[7]

(e) Certifications by the Community Environment and Natural Resources Office (CENRO) of
the DENR on its finding that the Subject Lots are alienable and disposable, by virtue of
Forestry Administrative Order No. 4-1063, dated 25 June 1963;
[8]

(f) Certified True Copies of Assessment of Real Property (ARP) No. 941800301831, in the
name of Jeremias, covering Lot No. 8422, issued in 1994; and ARP No. 941800301833,
in the name of David, covering Lot No. 8423, also issued in 1994;
[9]
and
(g) Deed of Definite Sale executed on 25 June 1976 by spouses Gregorio Herbieto and Isabel
Owatan selling the Subject Lots and the improvements thereon to their sons and
respondents herein, Jeremias and David, for P1,000. Lot No. 8422 was sold to Jeremias,
while Lot No. 8423 was sold to David.
[10]

On 11 December 1998, the petitioner Republic of the Philippines (Republic) filed an
Opposition to the respondents application for registration of the Subject Lots arguing that: (1)
Respondents failed to comply with the period of adverse possession of the Subject Lots
required by law; (2) Respondents muniments of title were not genuine and did not constitute
competent and sufficient evidence of bona fide acquisition of the Subject Lots; and (3) The
Subject Lots were part of the public domain belonging to the Republic and were not subject to
private appropriation.
[11]

The MTC set the initial hearing on 03 September 1999 at 8:30 a.m.
[12]
All owners of the land
adjoining the Subject Lots were sent copies of the Notice of Initial Hearing.
[13]
A copy of the
Notice was also posted on 27 July 1999 in a conspicuous place on the Subject Lots, as well as
on the bulletin board of the municipal building of Consolacion, Cebu, where the Subject Lots
were located.
[14]
Finally, the Notice was also published in the Official Gazette on 02 August
1999
[15]
and The Freeman Banat News on 19 December 1999.
[16]

During the initial hearing on 03 September 1999, the MTC issued an Order of Special
Default,
[17]
with only petitioner Republic opposing the application for registration of the Subject
Lots. The respondents, through their counsel, proceeded to offer and mark documentary
evidence to prove jurisdictional facts. The MTC commissioned the Clerk of Court to receive
further evidence from the respondents and to submit a Report to the MTC after 30 days.
On 21 December 1999, the MTC promulgated its Judgment ordering the registration and
confirmation of the title of respondent Jeremias over Lot No. 8422 and of respondent David over
Lot No. 8423. It subsequently issued an Order on 02 February 2000 declaring its Judgment,
dated 21 December 1999, final and executory, and directing the Administrator of the Land
Registration Authority (LRA) to issue a decree of registration for the Subject Lots.
[18]

Petitioner Republic appealed the MTC Judgment, dated 21 December 1999, to the Court of
Appeals.
[19]
The Court of Appeals, in its Decision, dated 22 November 2002, affirmed the
appealed MTC Judgment reasoning thus:
In the case at bar, there can be no question that the land sought to be registered has been classified as
within the alienable and disposable zone since June 25, 1963. Article 1113 in relation to Article 1137 of
the Civil Code, respectively provides that All things which are within the commerce of men are
susceptible of prescription, unless otherwise provided. Property of the State or any of its subdivisions of
patrimonial character shall not be the object of prescription and that Ownership and other real rights
over immovables also prescribe through uninterrupted adverse possession thereof for thirty years, without
need of title or of good faith.
As testified to by the appellees in the case at bench, their parents already acquired the subject parcels of
lands, subject matter of this application, since 1950 and that they cultivated the same and planted it with
jackfruits, bamboos, coconuts, and other trees (Judgment dated December 21, 1999, p. 6). In short, it is
undisputed that herein appellees or their predecessors-in-interest had occupied and possessed the subject
land openly, continuously, exclusively, and adversely since 1950. Consequently, even assuming
arguendo that appellees possession can be reckoned only from June 25, 1963 or from the time the subject
lots had been classified as within the alienable and disposable zone, still the argument of the appellant
does not hold water.
As earlier stressed, the subject property, being alienable since 1963 as shown by CENRO Report dated
June 23, 1963, may now be the object of prescription, thus susceptible of private ownership. By express
provision of Article 1137, appellees are, with much greater right, entitled to apply for its registration, as
provided by Section 14(4) of P.D. 1529 which allows individuals to own land in any manner provided by
law. Again, even considering that possession of appelless should only be reckoned from 1963, the year
when CENRO declared the subject lands alienable, herein appellees have been possessing the subject
parcels of land in open, continuous, and in the concept of an owner, for 35 years already when they filed
the instant application for registration of title to the land in 1998. As such, this court finds no reason to
disturb the finding of the court a quo.
[20]

The Republic filed the present Petition for the review and reversal of the Decision of the
Court of Appeals, dated 22 November 2002, on the basis of the following arguments:
First, respondents failed to establish that they and their predecessors-in-interest had been
in open, continuous, and adverse possession of the Subject Lots in the concept of owners since
12 June 1945 or earlier. According to the petitioner Republic, possession of the Subject Lots
prior to 25 June 1963 cannot be considered in determining compliance with the periods of
possession required by law. The Subject Lots were classified as alienable and disposable only
on 25 June 1963, per CENROs certification. It also alleges that the Court of Appeals, in
applying the 30-year acquisitive prescription period, had overlooked the ruling inRepublic v.
Doldol,
[21]
where this Court declared that Commonwealth Act No. 141, otherwise known as the
Public Land Act, as amended and as it is presently phrased, requires that possession of land of
the public domain must be from 12 June 1945 or earlier, for the same to be acquired through
judicial confirmation of imperfect title.
Second, the application for registration suffers from fatal infirmity as the subject of the
application consisted of two parcels of land individually and separately owned by two
applicants. Petitioner Republic contends that it is implicit in the provisions of Presidential
Decree No. 1529, otherwise known as the Property Registration Decree, as amended, that the
application for registration of title to land shall be filed by a single applicant; multiple applicants
may file a single application only in case they are co-owners. While an application may cover
two parcels of land, it is allowed only when the subject parcels of land belong to the same
applicant or applicants (in case the subject parcels of land are co-owned) and are situated
within the same province. Where the authority of the courts to proceed is conferred by a statute
and when the manner of obtaining jurisdiction is mandatory, it must be strictly complied with or
the proceedings will be utterly void. Since the respondents failed to comply with the procedure
for land registration under the Property Registration Decree, the proceedings held before the
MTC is void, as the latter did not acquire jurisdiction over it.
I
Jurisdiction
Addressing first the issue of jurisdiction, this Court finds that the MTC had no jurisdiction to
proceed with and hear the application for registration filed by the respondents but for reasons
different from those presented by petitioner Republic.
A. The misjoinder of causes of action and parties does not affect the jurisdiction of the MTC
to hear and proceed with respondents application for registration.
Respondents filed a single application for registration of the Subject Lots even though they
were not co-owners. Respondents Jeremias and David were actually seeking the individual and
separate registration of Lots No. 8422 and 8423, respectively.
Petitioner Republic believes that the procedural irregularity committed by the respondents
was fatal to their case, depriving the MTC of jurisdiction to proceed with and hear their
application for registration of the Subject Lots, based on this Courts pronouncement in Director
of Lands v. Court of Appeals,
[22]
to wit:
. . . In view of these multiple omissions which constitute non-compliance with the above-cited sections of
the Act, We rule that said defects have not invested the Court with the authority or jurisdiction to proceed
with the case because the manner or mode of obtaining jurisdiction as prescribed by the statute which is
mandatory has not been strictly followed, thereby rendering all proceedings utterly null and void.
This Court, however, disagrees with petitioner Republic in this regard. This procedural
lapse committed by the respondents should not affect the jurisdiction of the MTC to proceed
with and hear their application for registration of the Subject Lots.
The Property Registration Decree
[23]
recognizes and expressly allows the following
situations: (1) the filing of a single application by several applicants for as long as they are co-
owners of the parcel of land sought to be registered;
[24]
and (2) the filing of a single application
for registration of several parcels of land provided that the same are located within the same
province.
[25]
The Property Registration Decree is silent, however, as to the present situation
wherein two applicants filed a single application for two parcels of land, but are seeking the
separate and individual registration of the parcels of land in their respective names.
Since the Property Registration Decree failed to provide for such a situation, then this Court
refers to the Rules of Court to determine the proper course of action. Section 34 of the Property
Registration Decree itself provides that, [t]he Rules of Court shall, insofar as not inconsistent
with the provisions of this Decree, be applicable to land registration and cadastral cases by
analogy or in a suppletory character and whenever practicable and convenient.
Considering every application for land registration filed in strict accordance with the
Property Registration Decree as a single cause of action, then the defect in the joint application
for registration filed by the respondents with the MTC constitutes a misjoinder of causes of
action and parties. Instead of a single or joint application for registration, respondents Jeremias
and David, more appropriately, should have filed separate applications for registration of Lots
No. 8422 and 8423, respectively.
Misjoinder of causes of action and parties do not involve a question of jurisdiction of the
court to hear and proceed with the case.
[26]
They are not even accepted grounds for dismissal
thereof.
[27]
Instead, under the Rules of Court, the misjoinder of causes of action and parties
involve an implied admission of the courts jurisdiction. It acknowledges the power of the court,
acting upon the motion of a party to the case or on its own initiative, to order the severance of
the misjoined cause of action, to be proceeded with separately (in case of misjoinder of causes
of action); and/or the dropping of a party and the severance of any claim against said misjoined
party, also to be proceeded with separately (in case of misjoinder of parties).
The misjoinder of causes of action and parties in the present Petition may have been
corrected by the MTC motu propio or on motion of the petitioner Republic. It is regrettable,
however, that the MTC failed to detect the misjoinder when the application for registration was
still pending before it; and more regrettable that the petitioner Republic did not call the attention
of the MTC to the fact by filing a motion for severance of the causes of action and parties,
raising the issue of misjoinder only before this Court.
B. Respondents, however, failed to comply with the publication requirements mandated by
the Property Registration Decree, thus, the MTC was not invested with jurisdiction as a
land registration court.
Although the misjoinder of causes of action and parties in the present Petition did not affect
the jurisdiction of the MTC over the land registration proceeding, this Court, nonetheless, has
discovered a defect in the publication of the Notice of Initial Hearing, which bars the MTC from
assuming jurisdiction to hear and proceed with respondents application for registration.
A land registration case is a proceeding in rem,
[28]
and jurisdiction in rem cannot be
acquired unless there be constructive seizure of the land through publication and service of
notice.
[29]

Section 23 of the Property Registration Decree requires that the public be given Notice of
the Initial Hearing of the application for land registration by means of (1) publication; (2) mailing;
and (3) posting. Publication of the Notice of Initial Hearing shall be made in the following
manner:
1. By publication.
Upon receipt of the order of the court setting the time for initial hearing, the Commissioner of Land
Registration shall cause a notice of initial hearing to be published once in the Official Gazette and once in
a newspaper of general circulation in the Philippines: Provided, however, that the publication in the
Official Gazette shall be sufficient to confer jurisdiction upon the court. Said notice shall be addressed to
all persons appearing to have an interest in the land involved including the adjoining owners so far as
known, and to all whom it may concern. Said notice shall also require all persons concerned to appear
in court at a certain date and time to show cause why the prayer of said application shall not be granted.
Even as this Court concedes that the aforequoted Section 23(1) of the Property
Registration Decree expressly provides that publication in the Official Gazette shall be sufficient
to confer jurisdiction upon the land registration court, it still affirms its declaration in Director of
Lands v. Court of Appeals
[30]
that publication in a newspaper of general circulation is mandatory
for the land registration court to validly confirm and register the title of the applicant or
applicants. That Section 23 of the Property Registration Decree enumerated and described in
detail the requirements of publication, mailing, and posting of the Notice of Initial Hearing, then
all such requirements, including publication of the Notice in a newspaper of general circulation,
is essential and imperative, and must be strictly complied with. In the same case, this Court
expounded on the reason behind the compulsory publication of the Notice of Initial Hearing in a
newspaper of general circulation, thus
It may be asked why publication in a newspaper of general circulation should be deemed mandatory when
the law already requires notice by publication in the Official Gazette as well as by mailing and posting, all
of which have already been complied with in the case at hand. The reason is due process and the reality
that the Official Gazette is not as widely read and circulated as newspaper and is oftentimes delayed in its
circulation, such that the notices published therein may not reach the interested parties on time, if at all.
Additionally, such parties may not be owners of neighboring properties, and may in fact not own any
other real estate. In sum, the all encompassing in rem nature of land registration cases, the consequences
of default orders issued against the whole world and the objective of disseminating the notice in as wide a
manner as possible demand a mandatory construction of the requirements for publication, mailing and
posting.
[31]

In the instant Petition, the initial hearing was set by the MTC, and was in fact held, on 03
September 1999 at 8:30 a.m. While the Notice thereof was printed in the issue of the Official
Gazette, dated 02 August 1999, and officially released on 10 August 1999, it was published
in The Freeman Banat News, a daily newspaper printed in Cebu City and circulated in the
province and cities of Cebu and in the rest of Visayas and Mindanao, only on 19 December
1999, more than three months after the initial hearing.
Indubitably, such publication of the Notice, way after the date of the initial hearing, would
already be worthless and ineffective. Whoever read the Notice as it was published in The
Freeman Banat News and had a claim to the Subject Lots was deprived of due process for it
was already too late for him to appear before the MTC on the day of the initial hearing to oppose
respondents application for registration, and to present his claim and evidence in support of
such claim. Worse, as the Notice itself states, should the claimant-oppositor fail to appear
before the MTC on the date of initial hearing, he would be in default and would forever be
barred from contesting respondents application for registration and even the registration decree
that may be issued pursuant thereto. In fact, the MTC did issue an Order of Special Default on
03 September 1999.
The late publication of the Notice of Initial Hearing in the newspaper of general circulation is
tantamount to no publication at all, having the same ultimate result. Owing to such defect in the
publication of the Notice, the MTC failed to constructively seize the Subject Lots and to acquire
jurisdiction over respondents application for registration thereof. Therefore, the MTC
Judgment, dated 21 December 1999, ordering the registration and confirmation of the title of
respondents Jeremias and David over Lots No. 8422 and 8423, respectively; as well as the
MTC Order, dated 02 February 2000, declaring its Judgment of 21 December 1999 final and
executory, and directing the LRA Administrator to issue a decree of registration for the Subject
Lots, are both null and void for having been issued by the MTC without jurisdiction.
II
Period of Possession
Respondents failed to comply with the required period of possession of the Subject Lots for the
judicial confirmation or legalization of imperfect or incomplete title.
While this Court has already found that the MTC did not have jurisdiction to hear and
proceed with respondents application for registration, this Court nevertheless deems it
necessary to resolve the legal issue on the required period of possession for acquiring title to
public land.
Respondents application filed with the MTC did not state the statutory basis for their title to
the Subject Lots. They only alleged therein that they obtained title to the Subject Lots by
purchase from their parents, spouses Gregorio Herbieto and Isabel Owatan, on 25 June 1976.
Respondent Jeremias, in his testimony, claimed that his parents had been in possession of the
Subject Lots in the concept of an owner since 1950.
[32]

Yet, according to the DENR-CENRO Certification, submitted by respondents themselves,
the Subject Lots are within Alienable and Disposable, Block I, Project No. 28 per LC Map No.
2545 of Consolacion, Cebu certified under Forestry Administrative Order No. 4-1063, dated
June 25, 1963. Likewise, it is outside Kotkot-Lusaran Mananga Watershed Forest Reservation
per Presidential Proclamation No. 932 dated June 29, 1992.
[33]
The Subject Lots are thus
clearly part of the public domain, classified as alienable and disposable as of 25 June 1963.
As already well-settled in jurisprudence, no public land can be acquired by private persons
without any grant, express or implied, from the government;
[34]
and it is indispensable that the
person claiming title to public land should show that his title was acquired from the State or any
other mode of acquisition recognized by law.
[35]

The Public Land Act, as amended, governs lands of the public domain, except timber and
mineral lands, friar lands, and privately-owned lands which reverted to the State.
[36]
It explicitly
enumerates the means by which public lands may be disposed, as follows:
(1) For homestead settlement;
(2) By sale;
(3) By lease;
(4) By confirmation of imperfect or incomplete titles;
(a) By judicial legalization; or
(b) By administrative legalization (free patent).
[37]

Each mode of disposition is appropriately covered by separate chapters of the Public Land Act
because there are specific requirements and application procedure for every mode.
[38]
Since
respondents herein filed their application before the MTC,
[39]
then it can be reasonably inferred
that they are seeking the judicial confirmation or legalization of their imperfect or incomplete title
over the Subject Lots.
Judicial confirmation or legalization of imperfect or incomplete title to land, not exceeding
144 hectares,
[40]
may be availed of by persons identified under Section 48 of the Public Land
Act, as amended by Presidential Decree No. 1073, which reads
Section 48. The following-described citizens of the Philippines, occupying lands of the public domain or
claiming to own any such lands or an interest therein, but whose titles have not been perfected or
completed, may apply to the Court of First Instance of the province where the land is located for
confirmation of their claims and the issuance of a certificate of title thereafter, under the Land
Registration Act, to wit:
(a) [Repealed by Presidential Decree No. 1073].
(b) Those who by themselves or through their predecessors-in-interest have been in open,
continuous, exclusive, and notorious possession and occupation of agricultural lands of the
public domain, under a bona fide claim of acquisition of ownership, since June 12, 1945, or
earlier, immediately preceding the filing of the applications for confirmation of title, except
when prevented by war or force majeure. These shall be conclusively presumed to have
performed all the conditions essential to a Government grant and shall be entitled to a
certificate of title under the provisions of this chapter.
(c) Members of the national cultural minorities who by themselves or through their
predecessors-in-interest have been in open, continuous, exclusive and notorious possession
and occupation of lands of the public domain suitable to agriculture whether disposable or
not, under a bona fide claim of ownership since June 12, 1945 shall be entitled to the rights
granted in subsection (b) hereof.
Not being members of any national cultural minorities, respondents may only be entitled to
judicial confirmation or legalization of their imperfect or incomplete title under Section 48(b) of
the Public Land Act, as amended. Section 48(b), as amended, now requires adverse
possession of the land since 12 June 1945 or earlier. In the present Petition, the Subject Lots
became alienable and disposable only on 25 June 1963. Any period of possession prior to the
date when the Subject Lots were classified as alienable and disposable is inconsequential and
should be excluded from the computation of the period of possession; such possession can
never ripen into ownership and unless the land had been classified as alienable and disposable,
the rules on confirmation of imperfect title shall not apply thereto.
[41]
It is very apparent then that
respondents could not have complied with the period of possession required by Section 48(b) of
the Public Land Act, as amended, to acquire imperfect or incomplete title to the Subject Lots
that may be judicially confirmed or legalized.
The confirmation of respondents title by the Court of Appeals was based on the erroneous
supposition that respondents were claiming title to the Subject Lots under the Property
Registration Decree. According to the Decision of the Court of Appeals, dated 22 November
2002, Section 14(4) of the Property Registration Decree allows individuals to own land in any
other manner provided by law. It then ruled that the respondents, having possessed the Subject
Lots, by themselves and through their predecessors-in-interest, since 25 June 1963 to 23
September 1998, when they filed their application, have acquired title to the Subject Lots by
extraordinary prescription under Article 1113, in relation to Article 1137, both of the Civil
Code.
[42]

The Court of Appeals overlooked the difference between the Property Registration Decree
and the Public Land Act. Under the Property Registration Decree, there already exists a title
which is confirmed by the court; while under the Public Land Act, the presumption always is that
the land applied for pertains to the State, and that the occupants and possessors only claim an
interest in the same by virtue of their imperfect title or continuous, open, and notorious
possession.
[43]
As established by this Court in the preceding paragraphs, the Subject Lots
respondents wish to register are undoubtedly alienable and disposable lands of the public
domain and respondents may have acquired title thereto only under the provisions of the Public
Land Act.
However, it must be clarified herein that even though respondents may acquire imperfect or
incomplete title to the Subject Lots under the Public Land Act, their application for judicial
confirmation or legalization thereof must be in accordance with the Property Registration
Decree, for Section 50 of the Public Land Act reads
SEC. 50. Any person or persons, or their legal representatives or successors in right, claiming any lands
or interest in lands under the provisions of this chapter, must in every case present an application to the
proper Court of First Instance, praying that the validity of the alleged title or claim be inquired into and
that a certificate of title be issued to them under the provisions of the Land Registration Act.
[44]

Hence, respondents application for registration of the Subject Lots must have complied
with the substantial requirements under Section 48(b) of the Public Land Act and the procedural
requirements under the Property Registration Decree.
Moreover, provisions of the Civil Code on prescription of ownership and other real rights
apply in general to all types of land, while the Public Land Act specifically governs lands of the
public domain. Relative to one another, the Public Land Act may be considered a special
law
[45]
that must take precedence over the Civil Code, a general law. It is an established rule of
statutory construction that between a general law and a special law, the special law prevails
Generalia specialibus non derogant.
[46]

WHEREFORE, based on the foregoing, the instant Petition is GRANTED. The Decision of
the Court of Appeals in CA-G.R. CV No. 67625, dated 22 November 2002, is REVERSED. The
Judgment of the MTC of Consolacion, Cebu in LRC Case No. N-75, dated 21 December 1999,
and its Order, dated 02 February 2000 are declared NULL AND VOID. Respondents
application for registration is DISMISSED.
SO ORDERED.
Puno, Acting C.J., (Chairman), Austria-Martinez, and Callejo, Sr., JJ., concur.
Tinga, J., out of the country.



[1]
Penned by Associate Justice Mercedes Gozo-Dadole with Associate Justices B.A. Adefuin-
De La Cruz and Mariano C. Del Castillo concurring, Rollo, pp. 52-58.
[2]
Penned by Judge Wilfredo A. Dagatan, Records, pp. 100-108.
[3]
Records, pp. 1-6.
[4]
Ibid., pp. 7-8.
[5]
Ibid., pp. 9-10.
[6]
Ibid., pp. 11-12.
[7]
Ibid., pp. 13-14.
[8]
Ibid., pp. 15-18.
[9]
Ibid., pp. 19-20.
[10]
Ibid., p. 21.
[11]
Ibid., pp. 27-29.
[12]
Order, dated 29 April 1999, penned by Judge Wilfredo A. Dagatan, Ibid., p. 41.
[13]
Ibid., p. 59.
[14]
Ibid., p. 52.
[15]
Ibid., p. 58.
[16]
Ibid., pp. 96-97.
[17]
Penned by Judge Wilfredo A. Dagatan, Ibid., 62-65.
[18]
Penned by Judge Wilfredo A. Dagatan, Records, p. 109.
[19]
CA Rollo, pp. 20-38.
[20]
Supra, note 1, pp. 57-58.
[21]
G.R. No. 132963, 10 September 1998, 295 SCRA 359.
[22]
G.R. No. L-45168, 27 January 1981, 102 SCRA 370, 438, also quoted and/or reiterated in
subsequent cases of Alabang Development Corporation v. Valenzuela, G.R. No. L-
54094, 30 August 1982, 116 SCRA 261, 271; Tahanan Development Corporation v.
Court of Appeals, G.R. No. L-55771, 15 November 1982, 118 SCRA 273, 309; Register
of Deeds of Malabon, G.R. No. 88623, 05 February 1990, 181 SCRA 788, 791;
Allama v. Republic, G.R. No. 88226, 26 February 1992, 206 SCRA 600, 605.
[23]
Presidential Decree No. 1529.
[24]
Section 14 of the Property Registration Decree provides that, Where the land is owned in
common, all the co-owners shall file the application jointly.
[25]
Section 18 of the Property Registration Decree reads
SEC. 18. Application covering two or more parcels. An application may include
two or more parcels of land belonging to the applicant/s provided they are situated within
the same province or city. The court may at any time order an application to be
amended by striking out one or more of the parcels or by a severance of the application.
[26]
Katipunan v. Zandueta, 60 Phil 220 (1934).
[27]
Significant provisions of the Rules of Court are quoted below
RULE 2, SEC. 6. Misjoinder of causes of action. Misjoinder of causes of action is not
a ground for dismissal of an action. A misjoined cause of action may, on motion of a
party or on the initiative of the court, be severed and proceeded with separately.
RULE 3, SEC. 11. Misjoinder and non-joinder of parties. Neither misjoinder nor non-
joinder of parties is ground for dismissal of action. Parties may be dropped or added by
order of the court on motion of any party or on its own initiative at any stage of the action
and on such terms as are just. Any claim against a misjoined party may be severed and
proceeded with separately.
[28]
Section 2 of the Property Registration Decree.
[29]
Republic v. Court of Appeals, G.R. No. 113549, 05 July 1996, 258 SCRA 223, 236.
[30]
G.R. No. 102858, 28 July 1997, 276 SCRA 276.
[31]
Ibid., p. 286.
[32]
TSN, 24 September 1999, p. 28.
[33]
Records, pp. 15, 17.
[34]
Padilla v. Reyes, 60 Phil 967, 969 (1934).
[35]
Lee Hong Hok v. David, G.R. No. L-30389, 27 December 1972, 48 SCRA 372, 379.
[36]
Section 2.
[37]
Section 11.
[38]
Del Rosario-Igtiben v. Rebublic, G.R. No. 158449, 22 October 2004, p. 11.
[39]
Section 34 of Batas Pambansa Blg. 129, otherwise known as the Judiciary Reorganization
Act of 1980, as amended, allows the inferior courts (i.e., Metropolitan Trial Courts,
Municipal Trial Courts, and Municipal Circuit Trial Courts), duly assigned by the
Supreme Court, to hear and determine cadastral and land registration cases covering
lots where there is no controversy or opposition, or contested lots with values not
exceeding P100,000. Decisions of the inferior courts in such cases shall be appealable
in the same manner as decisions of the Regional Trial Courts. Accordingly, the
Supreme Court issued Administrative Circular No. 6-93-A, dated 15 November 1995,
authorizing the inferior courts to hear and decide the cadastral or land registration cases
as provided for by the Judiciary Reorganization Act of 1980, as amended.
[40]
Section 47 of the Public Land Act, as amended.
[41]
Almeda v. Court of Appeals, G.R. No. 85322, 30 April 1991, 196 SCRA 476; Vallarta v.
Intermediate Appellate Court, G.R. No. L-74957, 30 June 1987, 151 SCRA 679;
Republic v. Court of Appeals, G.R. No. L-40402, 16 March 1987, 148 SCRA 480.
[42]
The complete text of these provisions are reproduced below, for reference
ART. 1113. All things which are within the commerce of men are susceptible of
prescription, unless otherwise provided. Property of the State or any of its subdivisions
not patrimonial in character shall not be the object of prescription.
ART. 1137. Ownership and other real rights over immovables also prescribe through
uninterrupted adverse possession thereof for thirty years, without need of title or of good
faith.
[43]
Aquino v. Director of Lands, 39 Phil 850, 858 (1919).
[44]
Now the provisions of the Property Registration Decree.
[45]
This Court is not unaware that there are decisions by this Court declaring the Public Land Act
as a general law [Republic v. Court of Appeals, G.R. No. 106673, 09 May 2001, 357
SCRA 608, 616; Oliva v. Lamadrid, 128 Phil 770, 775 (1967)]. These cases, however,
involve the Public Land Act in relation to statutes other than the Civil Code. The
pronouncement made in the present Petition is particular to the nature of the Public Land
Act vis--vis the Civil Code.
[46]
Manila Railroad Co. v. Rafferty, 40 Phil 224 (1919).
THIRD DIVISION
[G.R. No. 102858. July 28, 1997]
THE DIRECTOR OF LANDS, petitioner, vs. COURT OF APPEALS and TEODORO
ABISTADO, substituted by MARGARITA, MARISSA, MARIBEL, ARNOLD and
MARY ANN, all surnamed ABISTADO, respondents.
D E C I S I O N
PANGANIBAN, J .:
Is newspaper publication of the notice of initial hearing in an original land registration case
mandatory or directory?
Statement of the Case
The Court of Appeals ruled that it was merely procedural and that the failure to cause such
publication did not deprive the trial court of its authority to grant the application. But the Solicitor
General disagreed and thus filed this petition to set aside the Decision
[1]
promulgated on July 3,
1991 and the subsequent Resolution
[2]
promulgated on November 19, 1991 by Respondent
Court of Appeals
[3]
in CA-G.R. CV No. 23719. The dispositive portion of the challenged
Decision reads:
[4]

"WHEREFORE, premises considered, the judgment of dismissal appealed from is hereby set aside, and a
new one entered confirming the registration and title of applicant, Teodoro Abistado, Filipino, a resident
of Barangay 7, Poblacion Mamburao, Occidental Mindoro, now deceased and substituted by Margarita,
Marissa, Maribel, Arnold and Mary Ann, all surnamed Abistado, represented by their aunt, Miss Josefa
Abistado, Filipinos, residents of Poblacion Mamburao, Occidental Mindoro, to the parcel of land covered
under MSI (IV-A-8) 315-D located in Poblacion Mamburao, Occidental Mindoro.
The oppositions filed by the Republic of the Philippines and private oppositor are hereby dismissed for
want of evidence.
Upon the finality of this decision and payment of the corresponding taxes due on this land, let an order for
the issuance of a decree be issued."
The Facts
On December 8, 1986, Private Respondent Teodoro Abistado filed a petition for original
registration of his title over 648 square meters of land under Presidential Decree (PD) No.
1529.
[5]
The application was docketed as Land Registration Case (LRC) No. 86 and assigned to
Branch 44 of the Regional Trial Court of Mamburao, Occidental Mindoro.
[6]
However, during the
pendency of his petition, applicant died. Hence, his heirs -- Margarita, Marissa, Maribel, Arnold
and Mary Ann, all surnamed Abistado -- represented by their aunt Josefa Abistado, who was
appointed their guardian ad litem, were substituted as applicants.
The land registration court in its decision dated June 13, 1989 dismissed the petition for
want of jurisdiction. However, it found that the applicants through their predecessors-in-interest
had been in open, continuous, exclusive and peaceful possession of the subject land since
1938.
In dismissing the petition, the trial court reasoned:
[7]

"x x x. However, the Court noted that applicants failed to comply with the provisions of Section 23 (1) of
PD 1529, requiring the Applicants to publish the notice of Initial Hearing (Exh. `E') in a newspaper of
general circulation in the Philippines. Exhibit `E' was only published in the Official Gazette (Exhibits `F'
and `G'). Consequently, the Court is of the well considered view that it has not legally acquired
jurisdiction over the instant application for want of compliance with the mandatory provision requiring
publication of the notice of initial hearing in a newspaper of general circulation."
The trial court also cited Ministry of Justice Opinion No. 48, Series of 1982, which in its
pertinent portion provides:
[8]

It bears emphasis that the publication requirement under Section 23 [of PD 1529] has a two-fold
purpose; the first, which is mentioned in the provision of the aforequoted provision refers to publication
in the Official Gazette, and is jurisdictional; while the second, which is mentioned in the opening clause
of the same paragraph, refers to publication not only in the Official Gazette but also in a newspaper of
general circulation, and is procedural. Neither one nor the other is dispensable. As to the first,
publication in the Official Gazette is indispensably necessary because without it, the court would be
powerless to assume jurisdiction over a particular land registration case. As to the second, publication of
the notice of initial hearing also in a newspaper of general circulation is indispensably necessary as a
requirement of procedural due process; otherwise, any decision that the court may promulgate in the case
would be legally infirm.
Unsatisfied, private respondents appealed to Respondent Court of Appeals which, as
earlier explained, set aside the decision of the trial court and ordered the registration of the title
in the name of Teodoro Abistado.
The subsequent motion for reconsideration was denied in the challenged CA Resolution
dated November 19, 1991.
The Director of Lands represented by the Solicitor General thus elevated this recourse to
us. This Court notes that the petitioners counsel anchored his petition on Rule 65. This is an
error. His remedy should be based on Rule 45 because he is appealing a final disposition of the
Court of Appeals. Hence, we shall treat his petition as one for review under Rule 45, and not for
certiorari under Rule 65.
[9]

The Issue
Petitioner alleges that Respondent Court of Appeals committed grave abuse of
discretion
[10]
in holding
x x x that publication of the petition for registration of title in LRC Case No. 86 need not be published in
a newspaper of general circulation, and in not dismissing LRC Case No. 86 for want of such publication.
Petitioner points out that under Section 23 of PD 1529, the notice of initial hearing shall be
published both in the Official Gazette and in a newspaper of general circulation. According to
petitioner, publication in the Official Gazette is necessary to confer jurisdiction upon the trial
court, and xxx in xxx a newspaper of general circulation to comply with the notice requirement of
due process.
[11]

Private respondents, on the other hand, contend that failure to comply with the requirement
of publication in a newspaper of general circulation is a mere procedural defect. They add that
publication in the Official Gazette is sufficient to confer jurisdiction.
[12]

In reversing the decision of the trial court, Respondent Court of Appeals ruled:
[13]

x x x although the requirement of publication in the Official Gazette and in a newspaper of general
circulation is couched in mandatory terms, it cannot be gainsaid that the law also mandates with equal
force that publication in the Official Gazette shall be sufficient to confer jurisdiction upon the court.
Further, Respondent Court found that the oppositors were afforded the opportunity to
explain matters fully and present their side. Thus, it justified its disposition in this wise:
[14]

x x x We do not see how the lack of compliance with the required procedure prejudiced them in any
way. Moreover, the other requirements of: publication in the Official Gazette, personal notice by mailing,
and posting at the site and other conspicuous places, were complied with and these are sufficient to notify
any party who is minded to make any objection of the application for registration.
The Courts Ruling
We find for petitioner.
Newspaper Publication Mandatory
The pertinent part of Section 23 of Presidential Decree No. 1529 requiring publication of the
notice of initial hearing reads as follows:
Sec. 23. Notice of initial hearing, publication, etc. -- The court shall, within five days from filing of the
application, issue an order setting the date and hour of the initial hearing which shall not be earlier than
forty-five days nor later than ninety days from the date of the order.
The public shall be given notice of initial hearing of the application for land registration by means of (1)
publication; (2) mailing; and (3) posting.
1. By publication. --
Upon receipt of the order of the court setting the time for initial hearing, the Commissioner of Land
Registration shall cause a notice of initial hearing to be published once in the Official Gazette and once in
a newspaper of general circulation in the Philippines: Provided, however, that the publication in the
Official Gazette shall be sufficient to confer jurisdiction upon the court. Said notice shall be addressed to
all persons appearing to have an interest in the land involved including the adjoining owners so far as
known, and `to all whom it may concern.' Said notice shall also require all persons concerned to appear in
court at a certain date and time to show cause why the prayer of said application shall not be granted.
xxx xxx xxx
Admittedly, the above provision provides in clear and categorical terms that publication in
the Official Gazette suffices to confer jurisdiction upon the land registration court. However, the
question boils down to whether, absent any publication in a newspaper of general circulation,
the land registration court can validly confirm and register the title of private respondents.
We answer this query in the negative. This answer is impelled by the demands of statutory
construction and the due process rationale behind the publication requirement.
The law used the term shall in prescribing the work to be done by the Commissioner of
Land Registration upon the latters receipt of the court order setting the time for initial
hearing. The said word denotes an imperative and thus indicates the mandatory character of a
statute.
[15]
While concededly such literal mandate is not an absolute rule in statutory
construction, as its import ultimately depends upon its context in the entire provision, we hold
that in the present case the term must be understood in its normal mandatory
meaning. In Republic vs. Marasigan,
[16]
the Court through Mr. Justice Hilario G. Davide, Jr. held
that Section 23 of PD 1529 requires notice of the initial hearing by means of (1) publication, (2)
mailing and (3) posting, all of which must be complied with. If the intention of the law were
otherwise, said section would not have stressed in detail the requirements of mailing of notices
to all persons named in the petition who, per Section 15 of the Decree, include owners of
adjoining properties, and occupants of the land. Indeed, if mailing of notices is essential, then
by parity of reasoning, publication in a newspaper of general circulation is likewise imperative
since the law included such requirement in its detailed provision.
It should be noted further that land registration is a proceeding in rem.
[17]
Being in rem, such
proceeding requires constructive seizure of the land as against allpersons, including the state,
who have rights to or interests in the property. An in rem proceeding is validated essentially
through publication. This being so, the process must strictly be complied with. Otherwise,
persons who may be interested or whose rights may be adversely affected would be barred
from contesting an application which they had no knowledge of. As has been ruled, a party as
an owner seeking the inscription of realty in the land registration court must prove by
satisfactory and conclusive evidence not only his ownership thereof but the identity of the same,
for he is in the same situation as one who institutes an action for recovery of realty.
[18]
He must
prove his title against the whole world. This task, which rests upon the applicant, can best be
achieved when all persons concerned -- nay, the whole world -- who have rights to or interests
in the subject property are notified and effectively invited to come to court and show cause why
the application should not be granted. The elementary norms of due process require that before
the claimed property is taken from concerned parties and registered in the name of the
applicant, said parties must be given notice and opportunity to oppose.
It may be asked why publication in a newspaper of general circulation should be deemed
mandatory when the law already requires notice by publication in the Official Gazette as well as
by mailing and posting, all of which have already been complied with in the case at hand. The
reason is due process and the reality that the Official Gazette is not as widely read and
circulated as newspapers and is oftentimes delayed in its circulation, such that the notices
published therein may not reach the interested parties on time, if at all. Additionally, such
parties may not be owners of neighboring properties, and may in fact not own any other real
estate. In sum, the all-encompassing in rem nature of land registration cases, the consequences
of default orders issued against the whole world and the objective of disseminating the notice in
as wide a manner as possible demand a mandatory construction of the requirements for
publication, mailing and posting.
Admittedly, there was failure to comply with the explicit publication requirement of the
law. Private respondents did not proffer any excuse; even if they had, it would not have
mattered because the statute itself allows no excuses. Ineludibly, this Court has no authority to
dispense with such mandatory requirement. The law is unambiguous and its rationale
clear. Time and again, this Court has declared that where the law speaks in clear and
categorical language, there is no room for interpretation, vacillation or equivocation; there is
room only for application.
[19]
There is no alternative. Thus, the application for land registration
filed by private respondents must be dismissed without prejudice to reapplication in the future,
after all the legal requisites shall have been duly complied with.
WHEREFORE, the petition is GRANTED and the assailed Decision and Resolution
are REVERSED and SET ASIDE. The application of private respondent for land registration
is DISMISSED without prejudice. No costs.
SO ORDERED.
Davide, Jr., Melo, and Francisco, JJ., concur.
Narvasa, C.J., (Chairman), on leave.



[1]
Rollo, pp. 29-36.
[2]
Ibid., p. 37.
[3]
Seventh Division composed of Justice Celso L. Magsino, ponente, and Justices Serafin E.
Camilon, Chairman, and Artemon D. Luna, concurring.
[4]
Ibid., p. 35.
[5]
Known as the Property Registration Decree.
[6]
Presided by Judge Niovady M. Marin.
[7]
Rollo, p. 41.
[8]
Ibid., pp. 41-42.
[9]
The Solicitor General asked for and was granted an extension of 30 days within which to file a
petition for review on certiorari. It is thus strange why the OSG described its petition as
one forcertiorari under Rule 65 of the Rules of Court. In any event, the Court, in its
Resolution dated March 9, 1992 admitted the OSGs petition for review on certiorari,
clearly ruling that the petition was one for review, and not one for certiorari.
[10]
Ibid., p. 21. This should really read reversible error since as already explained, the petition
should be treated as one for review under Rule 45.
[11]
Ibid., pp. 22-23.
[12]
Ibid., pp. 56-57.
[13]
Ibid., p. 34; Decision, p. 6.
[14]
Ibid.
[15]
Bersabal vs. Salvador, 84 SCRA 176, 179-180, July 21, 1978, citing
Dizon vs. Encarnacion, 9 SCRA 714, 716-717, December 24, 1963.
[16]
198 SCRA 219, 227-228, June 6, 1991.
[17]
Grey Alba vs. De la Cruz, 17 Phil. 49, September 16, 1910.
[18]
Archbishop of Manila vs. Arnedo, 30 Phil. 593, March 31, 1915.
[19]
Cebu Portland Cement Company vs. Municipality of Naga, Cebu, 24 SCRA 708, 712, August
22, 1968 citing Lizarraga Hermanos vs. Yap Tico, 24 Phil. 504, 1913; People vs. Mapa,
L-22301, August 30, 1967; Pacific Oxygen and Acetylene Co. vs. Central Bank, L-
21881, March 1, 1968; Dequito vs. Lopez, L-27757, March 28, 1968.
Republic of the Philippines
SUPREME COURT
Manila
EN BANC

G.R. No. 100643 December 12, 1995
ADEZ REALTY, INCORPORATED, petitioner,
vs.
HONORABLE COURT OF APPEALS, THE PRESIDING JUDGE, REGIONAL TRIAL COURT,
Br. 79, Morong Rizal, THE REGISTER OF DEEDS FOR QUEZON CITY, and AGUEDO
EUGENIO, respondents.
R E S O L U T I O N

BELLOSILLO, J .:
On 30 October 1992 the Court found movant, Atty. Benjamin M. Dacanay, guilty of intercalating
a material fact in a decision of the Court of Appeals, which he appealed to this Court
on certiorari, thereby altering the factual findings of the Court of Appeals with the apparent
purpose of misleading this Court in order to obtain a favorable judgment. Consequently, Atty.
Dacanay was disbarred from the practice of law.
1

On 20 November 1992 movant filed a Motion for Reconsideration and Leave to Offer Evidence
Re Charge of Unauthorized Intercalation in a Judicial Record dated 18 November 1992. He
claimed that the inserted words were written by his client, the President of Adez Realty, Inc., in
the draft of the petition to be filed before the Supreme Court and unwittingly adopted by
movant's secretary when the latter formalized the petition. He manifested that he would not risk
committing the act for which he was found guilty considering that he was a nominee of the
Judicial and Bar Council to the President for appointment as regional trial judge.
2
But the Court
on 3 December 1992 denied the motion for want of a compelling reason to justify a reversal of
the questioned resolution.
3

On 23 February 1994 movant Dacanay filed a Motion to Lift (Disbarment) stating that he was
already 62 years old, has learned his lesson from his mistake, was terribly sorry for what he had
done, and in all candor promised that if given another chance he would live up to the exacting
demands of the legal profession. He appended to his motion certifications of good moral
character from: Fr. Celso Fernando, Parochial Vicar, Parish of St. Michael Archangel, Marilao,
Bulacan; Fr. Lauro V. Larlar, OAR, Rector, San Sebastian College-Recoletos; Sis. Aniceta B.
Abion, EMM, Chairperson, Center for Housing and Ecology Development Foundation, Inc.;
Dean Rufus B. Rodriquez, College of Law, San Sebastian College-Recoletos; Judge Pedro T.
Santiago, Executive Judge, RTC, Quezon City; Judge Teodoro P. Regino, RTC-Br. 84, Quezon
City; Judge Antonio P. Solano, RTC-Br. 86, Quezon City; and Judge Gregorio D. Dayrit, MTC-
Br. 35, Quezon City.
4
However, on 11 August 1994 the Court denied the motion.
5

On 1 December 1994 movant again filed an Ex-Parte Motion to Lift Disbarment alleging among
others that he had been deprived of his means to life; he had pursued civic, religious and
community work, especially for the poor and the underprivileged short of extending legal
assistance because of his incapacity; he had admitted "with profound regret and with utmost
humility his commission of an unpardonable mistake and ask(ed) that he be given another
chance;" and, he was "remorseful for what he has done and comes to this Honorable Court with
a contrite heart."
6

His wife Norma M. Dacanay likewise wrote the Court on 1 December 1994 saying that while she
did not condone what her husband had done, it had been her fervent wish that the Court took a
second look into its decision disbarring her husband as her entire family had been traumatized
by his disbarment.
7

On 6 March 1995 movant sent a letter addressed to the Chief Justice and the Associate
Justices of the Court reiterating his Ex-Parte Motion to Lift Disbarment of 1 December 1994.
Thus
I am truly penitent for the serious offense I committed and admit full responsibility
for it. I realize it was dishonest and unfair to pass the blame to my secretary who
was merely following my instructions. The intercalation was my own act and I am
justly punished for it.
Your Honors, I do not question your decision but I only beg for your mercy. I have
a wife and children to support but my only means of livelihood has been
withdrawn from me. I am destitute and desperate and can only turn to you for
relief . . . .
Looking back, I cannot imagine how I could have even thought of blackening the
law profession, to which I owe so much. Please let me redeem myself by
admitting me back to its precincts, where I swear to live strictly according to its
canons . . . .
8

On 21 March 1995 the Court noted the letter of 6 March 1995 of movant Dacanay.
On 4 August 1995 movant again prayed for his reinstatement
It has been 33 long months since my disbarment, during which time I have been
struggling to make both ends meet to provide for my wife and three children.
Please give me the chance to prove that I am a reformed offender who will
henceforth do nothing whatsoever to dishonor the legal profession.
9

On 12 September 1995 the Court noted respondent's 4 August 1995 letter.
10

On 17 November, 1995 movant once more wrote the Court
I humbly acknowledge again that I committed a grievous offense for which I was
justly punished at the time with the extreme sanction of disbarment.
I have been suffering much since my disbarment more than 36 months ago, but it
is my wife and children who have suffered more for my transgression. Although
innocent, they bear with me the stigma and burden of my punishment.
11

The disbarment of movant Benjamin M. Dacanay for three (3) years has, quite apparently, given
him sufficient time and occasion to soul-search and reflect on his professional conduct, redeem
himself and prove once more that he is worthy to practice law and be capable of upholding the
dignity of the legal profession. His admission of guilt and repeated pleas for compassion and
reinstatement show that he is ready once more to meet the exacting standards the legal
profession demands from its practitioners. Accordingly, the Court lifts the disbarment of
Benjamin M. Dacanay. However he should be sternly warned that
[T]he practice of law is a privilege burdened with conditions. Adherence to the
rigid standards of mental fitness, maintenance of the highest degree of morality
and faithful compliance with the rules of the legal profession are the conditions
required for remaining a member of good standing of the bar and for enjoying the
privilege to practice law. The Supreme Court, as guardian of the legal profession,
has ultimate disciplinary power over attorneys. This authority to discipline its
members is not only a right, but a bounden duty as well . . . That is why respect
and fidelity to the Court is demanded of its members . . .
12

WHEREFORE, the disbarment of BENJAMIN M. DACANAY from the practice of law is LIFTED
and he is therefore allowed to resume the practice of law upon payment of the required legal
fees. This resolution is effective immediately.
SO ORDERED.
Narvasa, C.J., Padilla, Regalado, Davide, Jr., Romero, Melo, Puno, Vitug, Kapunan, Mendoza,
Francisco, Hermosisima, Jr. and Panganiban, JJ., concur.
Feliciano, J., is on leave.
Footnotes
1 Rollo, pp. 101-111.
2 Id., pp. 135-144.
3 Id., pp. 171-172.
4 Id., pp. 173-185.
5 Id., pp. 186-187.
6 Id., pp. 189-192.
7 Id., p. 193.
8 Id., p. 196.
9 Id., p. 198.
10 Id., p. 199.
11 Id., p. 200.
12 Zaldivar v. Sandiganbayan, G.R. Nos. 79690-707 and 80578, 7 April 1993,
221 SCRA 132.
G.R. No. 168913 March 14, 2007
ROLANDO TING, Petitioner,
vs.
HEIRS OF DIEGO LIRIO, namely: FLORA A. LIRIO, AMELIA L. ROSKA, AURORA L.
ABEJO, ALICIA L. DUNQUE, ADELAIDA L. DAVID, EFREN A. LIRIO and JOCELYN
ANABELLE L. ALCOVER, Respondents.
D E C I S I O N
CARPIO MORALES, J .:
In a Decision of December 10, 1976 in Land Registration Case (LRC) No. N-983, then Judge
Alfredo Marigomen of the then Court of First Instance of Cebu, Branch 7, granted the
application filed by the Spouses Diego Lirio and Flora Atienza for registration of title to Lot No.
18281 (the lot) of the Cebu Cadastral 12 Extension, Plan Rs-07-000787.
The decision in LRC No. N-983 became final and executory on January 29, 1977. Judge
Marigomen thereafter issued an order of November 10, 1982 directing the Land Registration
Commission to issue the corresponding decree of registration and the certificate of title in favor
of the spouses Lirio.
On February 12, 1997, Rolando Ting (petitioner) filed with the Regional Trial Court (RTC) of
Cebu an application for registration of title to the same lot. The application was docketed as
LRC No. 1437-N.
1

The herein respondents, heirs of Diego Lirio, namely: Flora A. Lirio, Amelia L. Roska, Aurora L.
Abejo, Alicia L. Dunque, Adelaida L. David, Efren A. Lirio and Jocelyn Anabelle L. Alcover, who
were afforded the opportunity to file an opposition to petitioners application by Branch 21 of the
Cebu RTC, filed their Answer
2
calling attention to the December 10, 1976 decision in LRC No.
N-983 which had become final and executory on January 29, 1977 and which, they argued,
barred the filing of petitioners application on the ground of res judicata.
After hearing the respective sides of the parties, Branch 21 of the Cebu RTC, on motion of
respondents, dismissed petitioners application on the ground of res judicata.
3
1vvphi1.nt
Hence, the present petition for review on certiorari which raises the sole issue of whether the
decision in LRC No. N-983 constitutes res judicata in LRC No. 1437-N.
Petitioner argues that although the decision in LRC No. N-983 had become final and executory
on January 29, 1977, no decree of registration has been issued by the Land Registration
Authority (LRA);
4
it was only on July 26, 2003 that the "extinct" decision belatedly surfaced as
basis of respondents motion to dismiss LRC No. 1437-N;
5
and as no action for revival of the
said decision was filed by respondents after the lapse of the ten-year prescriptive period, "the
cause of action in the dormant judgment pass[d] into extinction."
6

Petitioner thus concludes that an "extinct" judgment cannot be the basis of res judicata.
7

The petition fails.
Section 30 of Presidential Decree No. 1529 or the Property Registration Decree provides:
SEC. 30. When judgment becomes final; duty to cause issuance of decree. The judgment
rendered in a land registration proceeding becomes final upon the expiration of thirty days
8
to
be counted from the date of receipt of notice of the judgment. An appeal may be taken from the
judgment of the court as in ordinary civil cases.
After judgment has become final and executory, it shall devolve upon the court to forthwith issue
an order in accordance with Section 39 of this Decree to the Commissioner for the issuance of
the decree of registration and the corresponding certificate of title in favor of the person
adjudged entitled to registration. (Emphasis supplied)
In a registration proceeding instituted for the registration of a private land, with or without
opposition, the judgment of the court confirming the title of the applicant or oppositor, as the
case may be, and ordering its registration in his name constitutes, when final, res
judicata against the whole world.
9
It becomes final when no appeal within the reglementary
period is taken from a judgment of confirmation and registration.
10

The land registration proceedings being in rem, the land registration courts approval in LRC No.
N-983 of spouses Diego Lirio and Flora Atienzas application for registration of the lot settled its
ownership, and is binding on the whole world including petitioner.
Explaining his position that the December 10, 1976 Decision in LRC No. N-983 had become
"extinct," petitioner advances that the LRA has not issued the decree of registration, a certain
Engr. Rafaela Belleza, Chief of the Survey Assistance Section, Land Management Services,
Department of Environment and Natural Resources (DENR), Region 7, Cebu City having
claimed that the survey of the Cebu Cadastral Extension is erroneous and all resurvey within
the Cebu Cadastral extension must first be approved by the Land Management Services of
the DENR, Region 7, Cebu City before said resurvey may be used in court; and that the
spouses Lirio did not comply with the said requirement for they instead submitted to the court a
mere special work order.
11

There is, however, no showing that the LRA credited the alleged claim of Engineer Belleza and
that it reported such claim to the land registration court for appropriate action or reconsideration
of the decision which was its duty.
Petitioners insist that the duty of the respondent land registration officials to issue the decree is
purely ministerial. It is ministerial in the sense that they act under the orders of the court and the
decree must be in conformity with the decision of the court and with the data found in the
record, and they have no discretion in the matter. However, if they are in doubt upon any
point in relation to the preparation and issuance of the decree, it is their duty to refer the
matter to the court. They act, in this respect, as officials of the court and not as
administrative officials, and their act is the act of the court. They are specifically called
upon to "extend assistance to courts in ordinary and cadastral land registration
proceedings."
12
(Emphasis supplied)
As for petitioners claim that under Section 6, Rule 39 of the Rules of Court reading:
SEC. 6. Execution by motion or by independent action. A final and executory judgment or
order may be executed on motion within five (5) years from the date of its entry. After the lapse
of such time, and before it is barred by the statute of limitations, a judgment may be enforced by
action. The revived judgment may also be enforced by motion within five (5) years from the date
of its entry and thereafter by action before it is barred by the statute of limitations[,]
the December 10, 1976 decision became "extinct" in light of the failure of respondents and/or of
their predecessors-in-interest to execute the same within the prescriptive period, the same does
not lie.
Sta. Ana v. Menla, et al.
13
enunciates the raison detre why Section 6, Rule 39 does not apply in
land registration proceedings, viz:
THAT THE LOWER COURT ERRED IN ORDERING THAT THE DECISION RENDERED IN
THIS LAND REGISTRATION CASE ON NOVEMBER 28, 1931 OR TWENTY SIX YEARS
AGO, HAS NOT YET BECOME FINAL AND UNENFORCEABLE.
We fail to understand the arguments of the appellant in support of the above assignment,
except in so far as it supports his theory that after a decision in a land registration case has
become final, it may not be enforced after the lapse of a period of 10 years, except by another
proceeding to enforce the judgment or decision. Authority for this theory is the provision in the
Rules of Court to the effect that judgment may be enforced within 5 years by motion, and after
five years but within 10 years, by an action (Sec. 6, Rule 39.) This provision of the Rules
refers to civil actions and is not applicable to special proceedings, such as a land
registration case. This is so because a party in a civil action must immediately enforce a
judgment that is secured as against the adverse party, and his failure to act to enforce
the same within a reasonable time as provided in the Rules makes the decision
unenforceable against the losing party. In special proceedings the purpose is to
establish a status, condition or fact; in land registration proceedings, the
ownership by a person of a parcel of land is sought to be established. After the
ownership has been proved and confirmed
by judicial declaration, no further proceeding to enforce said ownership is necessary,
except when the adverse or losing party had been in possession of the land and the
winning party desires to oust him therefrom.
Furthermore, there is no provision in the Land Registration Act similar to Sec. 6, Rule 39,
regarding the execution of a judgment in a civil action, except the proceedings to place the
winner in possession by virtue of a writ of possession. The decision in a land registration case,
unless the adverse or losing party is in possession, becomes final without any further action,
upon the expiration of the period for perfecting an appeal.
x x x x (Emphasis and underscoring supplied)
WHEREFORE, the petition is, in light of the foregoing discussions, DENIED.
Costs against petitioner, Rolando Ting.
SO ORDERED.
CONCHITA CARPIO MORALES
Associate Justice
WE CONCUR:
LEONARDO A. QUISUMBING
Associate Justice
Chairperson
ANTONIO T. CARPIO
Associate Justice
DANTE O. TINGA
Asscociate Justice
PRESBITERO J. VELASCO, JR.
Associate Justice
A T T E S T A T I O N
I attest that the conclusions in the above Decision were reached in consultation before the case
was assigned to the writer of the opinion of the Courts Division.
LEONARDO A. QUISUMBING
Associate Justice
Chairperson
C E R T I F I C A T I O N
Pursuant to Article VIII, Section 13 of the Constitution, and the Division Chairpersons
Attestation, it is hereby certified that the conclusions in the above Decision were reached in
consultation before the case was assigned to the writer of the Courts Division.
REYNATO S. PUNO
Chief Justice


Foonotes
1
Rollo, p. 8.
2
Records, pp. 219-220.
3
Rollo, p. 23.
4
Id. at 12.
5
Ibid.
6
Ibid.
7
Id. at 13.
8
The first paragraph of Section 39 of B.P. Blg. 129 or The Judiciary Reorganization Act
OF 1980 provides:
SEC. 39. Appeals. The period for appeal from final orders, resolutions, awards,
judgments, or decisions of any court in all cases shall be fifteen (15) days
counted from the notice of the final order, resolution , award, judgment, or
decision appealed from: Provided, however, That in habeas corpus cases, the
period for appeal shall be forty-eight (48) hours from the notice of the judgment
appealed from.
9
Noblejas and Noblejas, Registration Of Land Titles And Deeds 136 (1992 ed.)
10
Id. at 162
11
Rollo, p. 13.
12
Gomez v. Court of Appeals, No. L-77770, December 15, 1988, 168 SCRA 503,
510. Vide also Ramos v. Rodriguez, G.R. No. 94033, May 29, 1995, 244 SCRA 418 and
P.D. No. 1529, Section 6(2)(b).
13
111 Phil. 947, 951 (1961); vide also Cacho v. Court of Appeals, G.R. No. 123361,
March 3, 1997, 269 SCRA 159, 170-171
SECOND DIVISION
[G.R. No. 120747. September 21, 2000]
VICENTE GOMEZ, as successor-in-interest of awardee LUISA GOMEZ, petitioner, vs.
COURT OF APPEALS, City of MANILA acting thru the City Tenants Security
Committee now the Urban Settlement Office, Register of Deeds of
Manila,respondents.
D E C I S I O N
BUENA, J .:
Sought to be reversed in this petition for review on certiorari under Rule 45 of the Rules of
Court is the decision
[1]
of the Court of Appeals in C.A. G.R. Sp. No. 32101 promulgated on 22
February 1995 which annulled and set aside the decision of the Regional Trial Court of Manila,
Branch 12 in Civil Case No. 51930.
Impugned similarly is the resolution
[2]
of the Court of Appeals dated 29 June 1995 denying
petitioners motion for reconsideration.
From the records, we find the following antecedents:
Pursuant to the Land for the Landless Program of the City of Manila and in accordance with
City Ordinance No. 6880, the Office of City Mayor issued Resolution No. 16-A,
[3]
Series of 1978,
dated 17 May 1978, which effectively set guidelines and criteria for the award of city home lots
to qualified and deserving applicants. Attached to said resolution and made as integral part
thereof was a Contract to Sell
[4]
that further laid down terms and conditions which the lot
awardee must comply with.
On 30 June 1978, the City of Manila, through the City Tenants Security Committee (CTSC)
presently known as the Urban Settlement Office (URBAN), passed Resolution 17-78
[5]
which in
effect awarded to 46 applicants, 37 homelots in the former Ampil-Gorospe estate located in
Tondo, Manila. Luisa Gomez, predecessor-in-interest of herein petitioner Vicente Gomez, was
awarded Lot 4, Block 1, subject to the provisions of Resolution No. 3-78 of the CTSC and
building, subdivision and zoning rules and regulations.
Consequently, a certificate of award
[6]
dated 02 July 1978 was granted by the CTSC in favor
of Luisa Gomez, who paid the purchase price of the lot in the amount of P3,556.00 on
installment basis,
[7]
said payments being duly covered by official receipts.
In 1979, Luisa Gomez traveled to the Unites States of America but returned to the
Philippines in the same year.
On 18 January 1980, Luisa Gomez finally paid in full the P 3,556.00 purchase price of the
lot. Despite the full payment, Luisa still paid in installment an amount of P8,244.00, in excess of
the purchase price, which the City of Manila, through the CTSC, accepted. Additionally, the lot
was declared for taxation purposes and the corresponding real estate taxes thereon paid from
1980-1988. In 1982, Luisa, together with her spouse Daniel, left again for the United States of
America where she died
[8]
on 09 January 1983. She is survived by her husband and four
children, namely, Ramona G. Takorda, Edgardo Gomez, Erlinda G. Pena, and Rebecca G.
Dizon.
[9]

Subsequently, in a memorandum dated 07 February 1984, the Urban Settlements Officer
and Member-Executive Secretary of the CTSC directed the Western Police District, City Hall
Detachment, to conduct an investigation regarding reported violations of the terms and
conditions of the award committed by the lot awardees.
Thus, on 23 November 1984, a team headed by Pfc. Reynaldo Cristobal of the Western
Police District, proceeded to the former Ampil-Gorospe estate where the subject lots are
located, and conducted an investigation of alleged violations thereat.
On 19 December 1984, team leader Pfc. Reynaldo Cristobal rendered an investigation
report
[10]
addressed to the City Mayor of Manila, as Chairman of the CTSC, stating, among
others, the following findings:
X X X After the said operation, it was found out that of all the lot awardees in the said estate, the
following were confirmed to have violated the terms and conditions of their respective awards as
indicated opposite their names, to wit:
X X X 2. Name of awardee : Daniel Gomez
Address : No. 2557-C Juan Luna St. Tondo, Manila
Violation: The place was found actually occupied by Mrs. Erlinda Perez and her family together
with Mr. Mignony Lorghas and family, who are paying monthly rentals of P210.00 each to
Vicente Gomez, brother of awardee. Daniel Gomez is now presently residing in the United
States of America and only returns for vacation once in a while as a Balikbayan X X X.
Thus, on 01 July 1986, the CTSC, headed by then City Mayor Gemiliano Lopez, Jr. as
Chairman, issued Resolution No. 015-86,
[11]
adopting the findings of the investigation report
submitted by Pfc. Cristobal, and ordering the cancellation of the lot awards of Daniel Gomez
and other awardees who were found to have committed violations, and further declaring the
forfeiture of payments made by said awardees as reasonable compensation for the use of the
homelots.
In a letter
[12]
dated 04 August 1986, herein petitioner Vicente Gomez, acting as attorney-in-
fact
[13]
of his brother Daniel Gomez (spouse of Luisa Gomez) asked for reconsideration of the
CTSC resolution revoking the award of the lot.
On 28 June 1988, Daniel Gomez, spouse of awardee Luisa Gomez, died in the United
States of America. Eventually, on 01 February 1989, the surviving children of the deceased
spouses, who were American citizens and residents of the United States of America, executed
an affidavit of adjudication with deed of donation
[14]
disposing gratuitously Lot No. 1, Block 4, in
favor of their uncle Vicente Gomez.
On 20 February 1989, petitioner Vicente Gomez filed a memorandum
[15]
before the CTSC
praying that Resolution 15-86 be set aside and that the award of the lot be restored to Luisa
Gomez, or her heirs or successor-in-interest , preferably Vicente Gomez.
Thereafter, two supplemental memoranda, dated 26 July 1989
[16]
and 10 January
1990,
[17]
were submitted by petitioner before the CTSC reiterating the prayer in the initial
memorandum.
On 05 February 1990, herein petitioner filed before the Regional Trial Court (RTC) of
Manila, Branch 12, a petition for certiorari, prohibition and mandamusdocketed as Civil Case
No. 90-51930, entitled Vicente Gomez, as successor-in-interest of Awardee, Luisa Gomez,
petitioner, versus City Tenants Security Committee (now Urban Settlement Office) and Register
of Deeds of Manila, respondents.
In an order
[18]
dated 24 April 1990, the lower court directed the petitioner to amend its
petition so as to implead the proper government agency.
Hence, petitioner filed an amended petition
[19]
impleading the City of Manila as respondent,
to which the latter submitted an answer.
[20]

Accordingly, after the presentation of evidence, the lower court promulgated its
decision
[21]
dated 20 January 1993, the decretal portion of which reads:
Wherefore, the petition is hereby granted :
1. Ordering the City of Manila through its agency the City Tenants Security Committee (now Urban
Settlement Office) to set aside the order of cancellation of the award for Lot No. 4, Block 1 (formerly of
the Ampil-Gorospe estate) in favor of Luisa Gomez, her heirs and successor-in-interest, the herein
petitioner;
2. Prohibiting the City of Manila through its agency including the Register of Deeds of Manila from
awarding the same lot and issuing the corresponding certificate of title therefor to any other person;
3. Ordering the City of Manila through its agency the City Tenants Security Committee (now Urban
Settlement Office) to execute a Deed of Absolute Sale over the aforementioned lot in favor of the
petitioner as successor-in-interest of the awardee and further ordering them to stop and/or refrain from
disturbing the peaceful physical possession thereof of (sic) the petitioner; and
4. Ordering the City of Manila through its agency the City Tenants Security Committee (now Urban
Settlement Office) to refund to the petitioner his overpayments amounting toP8,244.00 and to pay the
costs of suit.
On appeal, the Court of Appeals reversed the lower courts decision prompting petitioner to
file a motion for reconsideration which the appellate court deniedvia its assailed resolution dated
29 June 1995.
Hence, the instant appeal where the core of controversy revolves around the propriety of
CTSCs act of canceling the lot award, through Resolution No. 015-86, and further declaring the
forfeiture of amounts paid by the awardee, as reasonable compensation for the use of the home
lot.
The petition is unmeritorious.
A thorough scrutiny of the records and an even more exhaustive perusal of the evidence,
both documentary and testimonial, would lead to the inevitable conclusion that the fact of
cancellation of the award covering Lot 4, Block 1, by the City of Manila, acting through the
CTSC, was properly exercised within the bounds of law and contractual stipulation between the
parties.
Viewed broadly, petitioner anchors his case on the premise, albeit erroneous, that upon full
payment of the purchase price of the lot in January 1980, Luisa Gomez, actual awardee,
already acquired a vested right over the real property subject of the present controversy. Thus,
according to petitioner, upon the death of Luisa Gomez on 09 January 1983, the alleged vested
right was transmitted by operation of law to her lawful heirs, pursuant to Article 777 of the Civil
Code.Additionally, petitioner submits that by virtue of the affidavit of adjudication with Deed of
Donation executed on 01 February 1989 in his favor by the surviving children of Luisa, he, in
effect, became the successor-in-interest of Luisa and thus entitled to whatever rights enjoyed by
the latter over the property.
In the light of existing law and jurisprudence and based on the evidence adduced, this
Court finds difficulty giving credence and weight to petitioners submissions. We therefore rule
that the cancellation of the award of Lot 4, Block 1, through the expediency of Resolution No.
015-86, was proper.
Primarily, it must be stressed that the contract entered into between the City of Manila and
awardee Luisa Gomez was not one of sale but a contract to sell, which, under both statutory
and case law, has its own attributes, peculiarities and effects.
Speaking through Mr. Justice Florenz Regalado, this Court in Adelfa Properties, Inc. vs.
Court of Appeals,
[22]
mapped out the bold distinctions between these species of contracts, to
wit:
In a contract of sale, the title passes to the vendee upon the delivery of the thing sold; whereas in a
contract to sell, by agreement, the ownership is reserved in the vendor and is not to pass until the full
payment of the price. In a contract of sale, the vendor has lost and cannot recover ownership until and
unless the contract is resolved or rescinded; whereas in a contract to sell, title is retained by the vendor
until the full payment of the purchase price, such payment being a positive suspensive condition and
failure of which is not a breach but an event that prevents the obligation of the vendor to convey title from
being effective. Thus, a deed of sale is considered absolute in nature where there is neither a stipulation in
the deed that title to the property sold is reserved in the seller until the full payment of the price, nor one
giving the vendor the right to unilaterally resolve the contract the moment the buyer fails to pay within a
fixed period.
To our mind, however, this pronouncement should not curtail the right of the parties in a
contract to sell to provide additional stipulations, nor bar them from imposing conditions relative
to the transfer of ownership.
To be sure, a contract of sale may either be absolute or conditional. One form of conditional
sales is what is now popularly termed as a Contract to Sell, where ownership or title is retained
until the fulfillment of a positive suspensive condition normally the payment of the purchase
price in the manner agreed upon.
[23]
(Emphasis ours)
From the above disquisition in Galang and applying Article 1306 of the Civil Code, the
contracting parties are accorded the liberality and freedom to establish such stipulations,
clauses, terms and conditions as they may deem convenient, provided the same are not
contrary to law, morals, good custom, public order or public policy. In the law on contracts, such
fundamental principle is known as the autonomy of contracts.
Under the present circumstances, we see no hindrance that prohibits the parties from
stipulating other lawful conditions, aside from full payment of the purchase price, which they
pledge to bind themselves and upon which transfer of ownership depends.
In the instant case, we uphold the Contract to Sell, duly annexed and attached to
Resolution 16-A, which explicitly provides for additional terms and conditions upon which the lot
awardees are bound. Although unsigned, the Contract to Sell, in addition to the provisions of
Resolution 16-A, constitutes the law between the contracting parties. After all, under the law
there exists a binding contract between the parties whose minds have met on a certain matter
notwithstanding that they did not affix their signatures to its written form.
[24]

For a contract, like a contract to sell, involves a meeting of minds between two persons
whereby one binds himself, with respect to the other, to give something or to render some
service. Contracts, in general, are perfected by mere consent, which is manifested by the
meeting of the offer and the acceptance upon the thing and the cause which are to constitute
the contract. The offer must be certain and the acceptance absolute.
[25]

As to the matter of acceptance, the same may be evidenced by some acts, or conduct,
communicated to the offeror, either in a formal or an informal manner, that clearly manifest the
intention or determination to accept the offer to buy or sell.
[26]

In the case at bar, acceptance on the part of the vendee was manifested through a plethora
of acts, such as payment of the purchase price, declaration of the property for taxation
purposes, and payment of real estate taxes thereon, and similar acts showing vendee's assent
to the contract.
Verily, Resolution 16-A and the Contract to Sell which was annexed, attached and made to
form part of said resolution, clearly laid down the terms and conditions which the awardee-
vendee must comply with. Accordingly, as an awardee, Luisa Gomez, her heirs and successors-
in-interest alike, are duty-bound to perform the correlative obligations embodied in Resolution
16-A and the Contract to Sell.
Resolution 16-A, Series of 1978, explicitly provides that aside from the requirement of
Filipino citizenship and legal age, the basic criteria for award of the lot pursuant to the Land for
the Landless Program of the City of Manila shall be the following:
a) Occupancy - The applicant must be the legal and actual or physical occupant of the lot in question at
the time of its acquisition by the City. He must be the owner of the house and lot, must be using the same
for his residential purposes, and must have had a lessee-lessor relationship with the previous owner of the
land or landed estate of which the subject lot is a part.
b) Non-ownership of land - The applicant and/or his spouse, if he is married, must not be an owner of
any parcel of land in Manila, Metropolitan Manila or elsewhere in the Philippines.Neither must he and/or
his spouse be a prospective owner or a buyer on installment basis of any lot other than that which he is
occupying and for which he is applying for award from the City.
c) Capacity to pay- The applicant must have such financial means and/or support as will enable him to
make regular payments of amortizations or installments for the lot if the same is awarded to him.
Of equal importance are the essential terms and conditions embraced in the Contract to
Sell, which awardee Luisa Gomez, her heirs and successors-in-interest, violated, to wit:
X X X Par.(3). The vendee shall occupy and use the lot exclusively for his/her residential purpose . X
X X
X X X Par. (5). The vendee hereby warrants and declares under oath that he/she is a bonafide and actual
occupant and tenant of the lot; X X X and that he/she fully understands that any false statement or
misrepresentation hereof (sic) shall be sufficient cause for the automatic cancellation of his/her rights
under this agreement as well as ground for criminal prosecution.
Par. (6). Until complete payment of the purchase price and compliance with all the vendees
obligations herein, title to the lot remains in the name of the owner. During the effectivity of this
agreement, however, the owner may transfer its title or assign its rights and interest under this agreement
to any person, corporation, bank or financial institution.
Title shall pass to the vendee upon execution of a final deed of sale in his/her favor. X X X
Par. (8). In order not to defeat the purpose of this social land reform program of the City of
Manila, and to prevent real estate speculations within twenty years from complete payment of the
purchase price and execution of the final deed of sale, the lot and residential house or improvement
thereon shall not be sold, transferred, mortgaged, leased or otherwise alienated or encumbered
without the written consent of the City Mayor.
Par. (9). During the effectivity of this agreement, the residential house or improvement thereon
shall not be leased, sold, transferred or otherwise alienated by the vendee without the written
consent of the owner. X X X
Par. (14). In the event that the vendee dies before full payment of the purchase price of the lot, his/her
surviving spouse, children heirs and/or successors-in-interest shall succeed in all his/her rights and
interest, as well as assume all/his/her obligations under this agreement.
Par. (15). This agreement shall be binding upon the heirs, executors and administrators of the
vendee. (emphasis ours)
Petitioner urges that awardee Luisa Gomez did not commit any violation of the lot award.
On the contrary, the records would indubitably show that Luisa Gomez, including her heirs and
successors-in-interest, have performed acts that constitute gross, if not brazen, violation of the
aforementioned terms and conditions of the award, as evidenced by the investigation report
submitted by Pfc. Cristobal, dated 19 December 1984.
Results of the investigation conducted on 23 November 1984, reveal that the lot was
actually occupied and leased by a certain Erlinda Perez and Mignony Lorghas, together with
their respective families, who were paying rentals to petitioner Vicente Gomez for the lease of
the subject premises.
Moreover, in a conference held on 13 January 1989 at the Office of the Acting Urban
Settlement Officer, Lorghas admitted that she has been leasing the property and paying rent to
petitioner Vicente Gomez, thus:
[27]

Atty. Bernardo: Mrs. Lorghas, how long have you been renting the property?
Mrs. Lorghas: I was living there since 1960 until today. I was renting a small room downfloor
(sic). When the family of Mr. Gomez died, kami na ang tumira sa itaas until now.
Atty. Bernardo : Magkano ang upa mo?
Mrs. Lorghas: P300 a month.
Atty. Bernardo: Kanino?
Mrs. Lorghas: Kay Vicente Gomez.
Atty. Bernardo: Meron bang resibo?
Mrs. Lorghas: Wala po.
Atty. Bernardo: Noong 1973, kayo na rin ang nakatira sa lugar ni Gomez.
Mrs. Lorghas: Opo.
Certainly, said acts constitute a brazen transgression of Resolution 16-A and clear
contravention of the Contract to Sell, specifically par. (3), (8) and (9) thereof.
The contract provides in no uncertain terms, that the abovementioned terms and conditions
shall bind the heirs, executors and administrators of the vendee. The contract further states that
breach thereof would result to the automatic cancellation of the vendees rights thereunder.
Thus, par.(10) (b) (a) of the Contract to Sell, which reads:
X X X any violation of the terms and conditions of this agreement shall automatically cause the
cancellation of the vendees rights under this agreement without necessity of prior notice or judicial
declaration X X X.
Such kind of stipulation was upheld by this Court in the Adelfa case where we categorically
declared that Article 1592 of the Civil Code, which requires rescission either by judicial action, or
notarial act, does not apply to a contract to sell.
[28]

Moreover, judicial action for rescission of a contract is not necessary where the contract
provides for automatic rescission in case of breach,
[29]
as in the contract involved in the present
controversy.
Likewise, this Court sustains the forfeiture of the payments made by awardee as
reasonable compensation for the use of the lot. At this juncture, par. (1) of the Contract to Sell
furnishes support to this conclusion:
X X X In case of the cancellation of the vendees rights under this agreement as hereinafter stipulated,
all payments made by him/her shall be forfeited and considered as rentals for the use of the lot X X X.
Further, Article 1486 of the Civil Code provides that a stipulation that the installments or
rents paid shall not be returned to the vendee or lessee shall be valid insofar as the same may
not be unconscionable under the circumstances.
[30]

Applying the foregoing, we are of the considered view that the payment of the purchase
price of P3,556.00, constitutes fair and reasonable rental for the period in which said property
was under the control of awardee Luisa Gomez, her heirs and successors-in-interest.
Undeniably, the awardee together with her heirs and successors-in-interest, have gained
benefits, financial or otherwise, for a period of eight years - from the time of actual award of the
lot to the time of cancellation thereof (1978-1986).
Nonetheless, we ought to stress that in the present case, forfeiture of the installments paid
as rentals, only applies to the purchase price of P3,556.00 and not to the overpayment of the
amount of P8,244.00.
Under these circumstances, the vendor should refund the amount of P8,244.00
representing the overpayment made, plus interest, to be computed in accordance with the rule
of thumb enunciated in the landmark case of Eastern Shipping Lines, Inc. vs. Court of
Appeals
[31]
and reiterated in the case ofPhilippine National Bank vs. Court of Appeals.
[32]

For us to uphold the forfeiture of the amount representing the overpayment would be to
revolt against the dictates of justice and fairness. A contrary ruling would unjustly enrich the
vendor to the prejudice of the vendee.
In the same vein, the provisions of Article 777 of the Civil Code notwithstanding, we hold
that the surviving children of awardee Luisa Gomez are not qualified transferees of Lot 4, Block
1 for failure to conform with the prerequisites set by Resolution 16-A, to wit, Filipino citizenship
and actual occupancy, which in the present case, are basic criteria for the award of the lot,
pursuant to the Land for the Landless Program of the City of Manila.
The records reveal that the children of Luisa Gomez are American citizens and permanent
residents of the United States of America. Notably, Resolution 16-A specifically enumerates
Filipino citizenship and actual occupancy of the lot for residential purposes, as qualifications for
entitlement to the lot award. For this court to consider said surviving children as qualified
awardee-transferees would render illusory the purposes for which Resolution 16-A and the
Land for the Landless Program of the City of Manila were adopted.
Even assuming arguendo that the surviving children of Luisa Gomez are entitled to the lot
by virtue of Article 777 of the Civil Code, said heirs nevertheless abandoned their right when
they violated the terms and conditions of the award by donating the subject property to
petitioner Vicente Gomez.
As paragraph (15) of the agreement provides that the heirs of the vendee shall be bound
thereby, it is then incumbent upon said heirs to render strict compliance with the provisions
thereof.
In particular, paragraph (8) of the Contract proscribes the sale, transfer, mortgage, lease,
alienation or encumbrance of the lot, residential house, or improvement thereon, without the
written consent of the City Mayor, within a period of twenty (20) years from complete payment of
the purchase price and execution of the final deed of sale. The execution of the Deed of
Donation by the surviving children of Luisa Gomez on February 1, 1989, in favor of Vicente
Gomez, was clearly within the prohibited period of 20 years from the full payment of the
purchase price on January 18, 1980.
Without doubt, the prohibition applies to them.
Furthermore, the subject lot and residential house were occupied by, and leased to, third
persons, in crystalline and evident derogation of the terms of the award.
WHEREFORE, premises considered, the instant petition is DISMISSED for lack of merit,
and the assailed decision of the Court of Appeals with respect to the cancellation of the award
of Lot 4, Block 1, is AFFIRMED SUBJECT TO MODIFICATION as to the forfeiture of amounts
paid by the vendee.
As modified, the City of Manila, is hereby ordered to refund with dispatch the amount
of P8,244.00 representing the overpayment made by petitioner plus interest.
SO ORDERED.
Bellosillo, (Chairman), Mendoza, Quisumbing, and De Leon, Jr., JJ., concur.



[1]
Penned by Justice Ramon Mabutas, Jr. and concurred in by Justice Nathanael P. De Pano,
Jr. and Justice Artemon D. Luna ; Rollo, pp. 30-42.
[2]
Rollo, p.53.
[3]
Original Records, pp. 108-122.
[4]
Ibid, pp. 118-122.
[5]
Ibid, pp. 104-105.
[6]
Ibid, p. 10.
[7]
Ibid, p.149; TSN, 18 July 1990.
[8]
Certificate of Death, O.R., p..32.
[9]
Stipulation of Facts, dated 07 June 1990; O.R., p.86.
[10]
O.R., pp. 123-124.
[11]
Ibid, pp. 11-12.
[12]
Ibid, p. 13.
[13]
Evidenced by Special Power of Attorney, dated 21 October 1987; O.R. pp. 6-8.
[14]
O.R., p. 9.
[15]
Ibid, pp. 14-17.
[16]
Ibid, pp. 19-20.
[17]
Ibid, pp. 27-31.
[18]
Ibid, p. 59.
[19]
Ibid, pp. 61-66.
[20]
Ibid, pp. 72-74.
[21]
Ibid, pp. 149-157.
[22]
240 SCRA 565 [1995]; Pingol et al. vs. Court of Appeals, 226 SCRA 118 [1993].
[23]
Galang vs. Court of Appeals, 225 SCRA 37 [1993].
[24]
Peoples Industrial and Commercial Corporation vs. Court of Appeals, 281 SCRA 207 [1997].
[25]
Adelfa Properties, Inc. vs. Court of Appeals, 240 SCRA 565 [1995].
[26]
Ibid.
[27]
TSN, 13 January 1989.
[28]
Peoples Industrial and Commercial Corporation vs. Court of Appeals, 281 SCRA 206 [1997];
Albea vs. Inquimboy, et al., 86 Phil. 477 [1950]; Alfonso, et al. vs. Court of Appeals, et al., 186
SCRA 400 [1990].
[29]
Palay Inc. et al. vs. Jacobo C. Clave, et al., 124 SCRA 638 [1983].
[30]
Peoples Industrial and Commercial Corporation vs. Court of Appeals, 281 SCRA 207[1997];
Delta Sales Corporation vs. Niu Kim Duan, 213 SCRA 259 [1992].
[31]
234 SCRA 78 [1994].
[32]
263 SCRA 766 [1996].

Potrebbero piacerti anche